Você está na página 1de 268

O PR

MATEMÁTICA
L D O
IA FE
R

S
TE

SO
MA

R
R
MA

SO
T

R
E

IA F
L DO PRO

Sistema Prodígio de Ensino


Kernel Editora Ltda
Brasil, Rio de Janeiro, 2022

PM_BOOK05_MAT.indb 1 01/11/2021 13:40:06


SUMÁRIO
MATEMÁTICA
CONJUNTOS 3
FUNÇÕES 13
FUNÇÃO AFIM 21
FUNÇÃO QUADRÁTICA 29
FUNÇÃO MODULAR
L DO PRO 35

IA FE
FUNÇÃO EXPONENCIAL 45
FUNÇÃO LOGARÍTMICA
R
ANÁLISE COMBINATÓRIA: CONTAGEM, PERMUTAÇÃO E ARRANJO
51
57

S
TE

ANÁLISE COMBINATÓRIA: COMBINAÇÃO E BINÔMIO DE NEWTON 61

SO
PROBABILIDADE 69
MA

NOÇÕES DE ESTATÍSTICA 75

R
PROGRESSÃO ARITMÉTICA 85
PROGRESSÃO GEOMÉTRICA 91
MATRIZES 95
DETERMINANTES R 103
SISTEMA LINEAR 111
MA

SO

GEOMETRIA PLANA: PRINCIPAIS TEOREMAS ANGULARES E MÉTRICOS 119


GEOMETRIA PLANA: POTÊNCIA DE PONTO, TRIGONOMETRIA NO TRIÂNGULO
RETÂNGULO, LEIS DOS SENOS E DOS COSSENOS E ÁREAS 131
T

R
TRIGONOMETRIA: CÍRCULO TRIGONOMÉTRICO E FÓRMULAS DE ADIÇÃO,
E

ARCO DOBRO E METADE


IA
TRIGONOMETRIA: FUNÇÕES TRIGONOMÉTRICAS, EQUAÇÕES O
F
E INEQUAÇÕES
145

TRIGONOMÉTRICAS L DO PR 155
NÚMEROS COMPLEXOS: FORMA ALGÉBRICA E MÓDULO 169
NÚMEROS COMPLEXOS: FORMA TRIGONOMÉTRICA 175
POLINÔMIOS 183
EQUAÇÕES POLINOMIAIS 189
GEOMETRIA ANALÍTICA: COORDENADAS NO PLANO E DISTÂNCIA 195
GEOMETRIA ANALÍTICA: RETA 201
GEOMETRIA ANALÍTICA: CIRCUNFERÊNCIA 209
GEOMETRIA ANALÍTICA: CÔNICAS 215
GEOMETRIA ESPACIAL: CONCEITOS, PARALELEPÍPEDO E CUBO 229
GEOMETRIA ESPACIAL: PRISMAS, PIRÂMIDES E CILINDRO 243
GEOMETRIA ESPACIAL: CONE, ESFERA, INSCRIÇÃO E CIRCUNSCRIÇÃO DE SÓLIDOS 251

PM_BOOK05_MAT.indb 2 01/11/2021 13:40:06


CONJUNTOS

NOÇÃO DE CONJUNTO, ELEMENTO E CONJUNTO VAZIO


PERTINÊNCIA Chama-se de conjunto vazio todo conjunto que não possui
elementos. Representa-se por { } ou ∅.
Em teoria dos conjuntos adotamos que elemento, conjunto e
relação de pertinência não possuem definição, ou seja, são conceitos O conjunto representado por {∅} é unitário e não vazio.
primitivos. Exemplo:
Se um elemento x pertence a um conjunto P, diz-se que x ∈ P. P = conjunto dos divisores de 21 e pares
Consequentemente, se um elemento x não pertence a P, diz-se que
D ( 21) : 1,3,7,21
x ∉ P. Os símbolos ∈ e ∉ são utilizados para a relação de pertinência
Logo, P = ∅
De O P R O
entre elemento e conjunto.

L
É coerente adotar letra maiúscula para nomear um conjunto

I Apar de chaves SUBCONJUNTOS FE


letras minúsculas para seus elementos. Os conjuntos podem ser
representados com os elementos dispostos entre um
ou com os elementos dispostos no interior deRuma linha fechada Um conjunto A é subconjunto de um conjunto B se, e somente
(diagrama de Venn). se, todo elemento de A é também elemento de B. Denota-se por

S
TE

Exemplo: A ⊂ B (lê-se A está contido em B) esta relação de inclusão do conjunto

SO
A em B ou por B ⊃ A (lê-se B contém A). Não havendo a inclusão nos
MA

Representação extensa: V = {a,e,i,o,u} conjuntos, usa-se ⊄ (não está contido) e ⊃/ (não contém).
ou
A ⊂ B ⇔ ( ∀x ) ( x ∈ A ⇒ x ∈ B)

R
Representação gráfica (Diagrama de Venn):

ou
V .a .e
.i
R
.o .u
MA

SO

Exemplo:
T

Descrição do conjunto pela citação dos elementos:


S

R Exemplo:
E

A = conjunto dos númerosprimos


E

A = { 2,3,5,7,11,13,17, } IA =
A {1,2,3} e=
BF {1,2,3,4,5} , logo A ⊂ B ou B ⊃ A.
Exemplo: R} O
L D O CP= {a,b e D= {b,c} , logo C ⊂/ D ou D ⊃/ C.
Descrição do conjunto por uma propriedade:
B = { x x é divisor de 6}
Propriedades da inclusão:
B = { −6, −3, −2, −1,1,2,3,6} Considere os conjuntos A, B e C conjuntos quaisquer, logo:
Como se pode observar nos exemplos, um conjunto pode ser • ∅ ⊂ A. O conjunto vazio é subconjunto de qualquer conjunto.
finito, ou seja, com um número limitado de elementos ou infinito, ou • A ⊂ A. O subconjunto mais amplo que um conjunto pode ter
seja, com um número ilimitado de elementos. é ele próprio.
• A ⊂ B e B ⊂ C ⇒ A ⊂ C. Propriedade transitiva.
CONJUNTO UNITÁRIO • A ⊂ B e B ⊂ A ⇒ A = B ou A = B ⇔ ( ∀x ) ( x ∈ A ⇔ x ∈ B) .
Chama-se de conjunto unitário todo conjunto que possui um Igualdade de conjuntos.
único elemento.
Observação
Exemplo:
P = conjunto dos números primos e pares Subconjunto próprio de um conjunto A é todo subconjunto
diferente do próprio conjunto A.
P = {2}

PROMILITARES.COM.BR 3

PM_BOOK05_MAT.indb 3 01/11/2021 13:40:07


CONJUNTOS

CONJUNTO DAS PARTES OU Atenção: dois conjuntos são chamados de disjuntos quando sua
interseção é vazia, ou seja, A ∩ B =∅ .
CONJUNTO POTÊNCIA Propriedades:
Conjunto das partes de um conjunto A (denotado por P(A)) é o
Considere A, B e C conjuntos quaisquer, logo:
conjunto formado por todos os subconjuntos de A.
• A∩A=
A
Exemplo:
• A ∩B = B∩ A
B= {∅, {1} , {2} , {3} , {1,2} , {1,3} , {2,3} , {1,2,3}}
{1,2,3} ⇒ P (B) =
• A∩∅ = ∅∩A = ∅
O número de elementos de um conjunto A pode ser representado
• A ∩ (B ∩ C) = ( A ∩ B) ∩ C
por n(A) ou #(A). Um conjunto A de n(A) elementos possui 2n(A)
subconjuntos.
Portanto, o número de elementos do conjunto das partes de um DIFERENÇA DE CONJUNTOS
conjunto A é 2n(A) , ou seja, n (P ( A ) ) = 2 ( ) .
nA
Dados dois conjuntos A e B, a diferença desses dois conjuntos
(nota-se por A – B) é o conjunto formado por todos os elementos de
A que não pertençam a B.
CONJUNTO UNIVERSO
É o conjunto mais amplo que contém todos os conjuntos em um
determinado contexto. O conjunto com essa característica é chamado
de conjunto universo e é comumente nomeado pela letra U.
A − B= {x x ∈ A / B}
e x∈ ou

OPERAÇÕES COM CONJUNTOS


L DO PRO
UNIÃO DE CONJUNTOS
I A Exemplo:
FE
R
Dados dois conjuntos A e B, a união desses dois conjuntos (nota-
se por A ∪ B) é o conjunto formado por todos os elementos que =
A {1,2,3} e=
B {3,4} ⇒ A−
= B {1,2}

S
TE

pertençam a A ou a B.
Propriedades:

SO
Considere A e B conjuntos quaisquer, logo:
MA

• A–A=∅
{x x ∈ A ou x ∈ B}

R
A ∪ B= ou • A–∅=A
• ∅–A=∅
• A – B ≠ B – A.
Exemplo:
=
A {1,2,3} e=
B {3,4} ⇒ A∪
= B {1,2,3,4} Observação R
Se A – B = B – A então A = B, logo A – B = B – A = ∅.
MA

Propriedades:
SO

Diferença Simétrica de dois conjuntos A e B (nota-se por A ∆ B)


Considere A, B e C conjuntos quaisquer, logo:
é o conjunto formado pelos elementos que pertencem à união
• A∪A=
A dos dois conjuntos, mas não pertencem à interseção dos dois
T

conjuntos, ou seja,
S

• A ∪B = B∪ A
R
E

• A∪∅ = ∅∪A = A
IA F
L DO PRO
• A ∪ (B ∪ C) = ( A ∪ B) ∪ C A ∆ B = ( A − B) ∪ (B − A ) ou A ∆ B = ( A ∪ B) − ( A ∩ B) ou

ou
INTERSEÇÃO DE CONJUNTOS
Dados dois conjuntos A e B, a interseção desses dois conjuntos
(nota-se por A ∩ B) é o conjunto formado por todos os elementos que
pertençam a A e a B.

A ∩ B= {x x ∈ A e x ∈ B}
ou

COMPLEMENTAR DE B EM A
Dados dois conjuntos A e B, tais que B ⊂ A, o complementar do
conjunto B no conjunto A é o próprio conjunto A – B, ou seja, é o
Exemplo:
conjunto formado todos os elementos de A que não pertençam a B.
=
A {1,2,3} e=
B {3,4} ⇒ A∩
= B {3}
CBA = A − B = {x x ∈ A / B}
e x∈

ou

4 PROMILITARES.COM.BR

PM_BOOK05_MAT.indb 4 01/11/2021 13:40:08


CONJUNTOS

PRINCÍPIO DA INCLUSÃO-EXCLUSÃO
Esse princípio permite calcular o número de elementos existentes
no conjunto união de dois ou mais conjuntos.
Para dois conjuntos:
) n ( A ) + n (B) − n ( A ∩ B)
n ( A ∪ B=

Para três conjuntos:


) n ( A ) + n (B) + n (C) − n ( A ∩ B) − n ( A ∩ C) − n (B ∩ C) + n ( A ∩ B ∩ C)
n ( A ∪ B ∪ C=

Para n conjuntos:
Exemplo: ∪ Ai )
n ( A1 ∪ A 2 ∪ ...= ∑ n( A ) − ∑ n( A ∩ A ) +
i i j

A = {1,2,3,4,5} e B = {3,4} ⇒ CBA = A − B = {1,2,5} i i< j

Propriedades:
+ ∑ n( A ∩ A ∩ A ) − ... + ( −1)
i< j< k
i j k
n +1
.n ( A1 ∩ A 2 ∩ ... ∩ Ai )

Considere A, B e C conjuntos quaisquer, logo: .


Exemplo:
 CAA =A−A=∅
 C∅A= A − ∅= A
=A {3,4,5,6,7
= } e B {6,7,8,9} ⇒
⇒ n ( A ∪ B) =n ( A ) + n (B) − n ( A ∩ B) =5 + 4 − 2 =7
 C ∩ B =( A − B) ∩ B =∅
D O PObserve
B
A

 CBA ∪ B = ( A − B) ∪ B = A L R Oque o conjunto A ∪ B ={3,4,5,6,7,8,9} tem 7 elementos.


 CA
(CBA ) = A − ( A − B) = B I A FE
R
 C(AB ∩ C) = A − (B ∩ C) = ( A − B) ∪ ( A − C) = CBA ∪ CCA
PRODUTO CARTESIANO
Dados A e B dois conjuntos não vazios, o produto cartesiano de

S
TE

 C(AB ∪ C) = A − (B ∪ C) = ( A − B) ∩ ( A − C) = CBA ∩ CCA A por B (nota-se por A × B) é o conjunto formado por todos os pares

SO
ordenados (x,y) em que x é elemento de A e y é elemento de B, ou
MA

Observação A × B {( x,y ) / x ∈ A e y ∈ B} .
seja, =

R
Exemplo:
Negação de um conjunto, ou seja, todos os elementos que estão
fora do conjunto A. =A {=
2,3,4} e B {1,2}
A ×B={(2,1) , (3,1) , ( 4,1) , (2,2) , (3,2) , ( 4,2)} e
A = U − A = A C = CA = { x x ∉ A} ou {(1,2) , (1,3) , (1,4) , (2,2) , (2,3) , (2,4)}
B×A =
.
R
MA

Propriedades:
SO

Considere A, B e C conjuntos quaisquer, logo:


• A ×B ≠ B× A .
T

Atenção: Se A × B = B × A então A = B.
R
E

IA

F A×∅=∅

O •
L D O •P ∅R× ∅ = ∅
∅×A=∅

• n ( A × B) =
n ( A ) .n (B) em que n(A) representa o número de
elementos do conjunto A.
• A × A = A².
Propriedades finais: • A × (B ∪ C) = ( A × B) ∪ ( A × C) .
Considere A, B e C conjuntos quaisquer, logo: • A × (B ∩ C) = ( A × B) ∩ ( A × C) .
• A ∪ (B ∩ C) = ( A ∪ B) ∩ ( A ∪ C) . Propriedade distributiva da • A × (B − C) = ( A × B) − ( A × C)
união em relação à interseção.
• A ∩ (B ∪ C) = ( A ∩ B) ∪ ( A ∩ C) . Propriedade distributiva da
interseção em relação à união. CONJUNTOS NUMÉRICOS
Para entendimento dos conceitos e propriedades de conjuntos
A ∩ B = A ∪ B ou ( A ∩ B) = A C ∪ BC . Lei de Morgan.
C
• numéricos, é importante o entendimento da propriedade do
A ∪ B = A ∩ B ou ( A ∪ B) = A C ∩ BC . Lei de Morgan. fechamento em alguma operação matemática, isto é, um conjunto
C

é fechado em relação a uma determinada operação se, e somente
• ( A ∪ B) − C = ( A − C) ∪ (B − C) . se, quaisquer que sejam os elementos do conjunto a ser realizada
• ( A ∩ B) − C = ( A − C) ∩ (B − C) a operação matemática, o resultado pertence ao próprio conjunto.
Como exemplo, tem-se que a soma de dois números naturais é sempre
• ( A ∆ B) ∪ C = ( A ∆ C) ∩ (B ∆ C) um número natural, ou seja, diz-se que o conjunto dos naturais é
• ( A ∆ B) ∩ C = ( A ∆ C) ∪ (B ∆ C) fechado na operação adição.

PROMILITARES.COM.BR 5

PM_BOOK05_MAT.indb 5 01/11/2021 13:40:11


CONJUNTOS

CONJUNTO DOS NATURAIS Inteiros não-nulos: *= {..., −3, −2, −1,1,2,3,}
 = {0,1,2,3,4,5,6,} Inteiros não-negativos:  = = {0,1,2,3,}
+

O conjunto dos naturais é fechado nas operações de adição e Inteiros não-positivos:  −= {..., −3, −2, −1,0}
multiplicação, ou seja, se a,b ∈ então a + b ∈  e a.b ∈ . Inteiros positivos:  + = {1,2,3,}
*

O conjunto dos naturais não é fechado nas operações de


subtração e divisão.
Inteiros negativos: *−= {..., −3, −2, −1}
Exemplo:
2 Propriedades:
2 e 3 ∈ , porém 2 − 3 =−1 ∉  e =0,6666... ∉  .
• Simétrico ou Oposto de um número:
3
Se a ∈ Z, então –a ∈ Z tal que a + (–a) = (–a) + a = 0.
Atenção: O conjunto dos naturais positivos, denotado por N*, é
o conjunto dos naturais excluindo o zero, ou seja, N* – {0}.
Atenção: Valor Absoluto ou Módulo de um Número Inteiro.
N* = {1,2,3,4,5,6,...} a , se a ≥ 0
Definição: a = 
 −a , se a < 0
Propriedades:
Exemplo:
Sejam a,b,c ∈ , tem-se que:
2 =2
• Tricotomia: é sempre verdade que a = b ou a < b ou a > b.
−2 = − ( −2) = 2
• Transitividade:
Se a = b e b = c, então a = c L DO PRO
Se a < b e b < c, então a < c
IA PropriedadesF Edo módulo:
• Associatividade:
R  x ≥ 0, ∀x ∈ .
 − x ≤ x ≤ x , ∀x ∈ .

S
TE

Adição: (a + b) + c = a ⋅ (b + c) = x . y , ∀x,y ∈ .

SO
 x.y
Multiplicação: (a ⋅ b) ⋅ c = a ⋅ (b ⋅ c)
MA

x x
 = , ∀x,y ∈  e y ≠ 0.
y y

R
• Comutatividade:
 x 2= x , ∀x ∈ .
Adição: a + b = b + a
 x =c ⇔ x =± c, ∀x ∈  e c ∈ *+ .
Multiplicação: a ⋅ b = b ⋅ a
 x < c ⇔ − c < x < c, ∀x ∈  e c ∈ *+ .

• Elemento neutro:  x > c ⇔ x < −c ou x > c, ∀x ∈  e c ∈ *+ .


R
2
 x = x 2= x 2 , ∀x ∈ .
MA

Adição: a + 0 = 0 + a = a
SO

Multiplicação: a ⋅ 1 = 1 ⋅ a = a 2n
x = x 2n= x 2n , ∀x ∈  e n ∈ .
 x + y ≤ x + y , ∀x,y ∈ . (Desigualdade Triangular)
• Distributividade: a ⋅ (b + c) = a ⋅ b + a ⋅ c
T

 x − y ≤ x + y , ∀x,y ∈ .
a ⋅ (b + c) = ab + ac
R
E

IA F
 x − y ≤ x − y , ∀x,y ∈ .

O
L D OCONJUNTO
P R DOS RACIONAIS
• Lei do corte:
Adição: se a + b = a + c, então b = c
p
Multiplicação: se a ⋅ b = a ⋅ c e a ≠ 0, então b = c Número racional é o número que pode ser colocado na forma ,
q
em que p e q são números inteiros, q é diferente de zero e
p 
CONJUNTO DOS INTEIROS mdc(p,q) = 1, ou seja,  =  p ∈ , q ∈ * e mdc (p,q) = 1 .
 q 
= {..., −3, −2, −1,0,1,2,3,}
Atenção: os racionais podem ser representados por
O conjunto dos números inteiros é fechado nas operações de  números int eiros
adição, subtração e multiplicação, ou seja, se a,b ∈ N então a + b ∈ N, 
a – n ∈ N e a ⋅ b ∈ N.  números decimais exatos
 
O conjunto dos números inteiros não é fechado na operação  frações   simples
  dízimas periódicas composta
divisão.   
Exemplo:
Os números inteiros são também números racionais, pois podem
−2 ser considerados frações de denominador 1.
–2 e 3 ∈ Z, porém =
−0,6666... ∉ .
3
O conjunto dos números racionais é fechado nas quatro operações,
Subconjuntos importantes do conjunto dos números inteiros: a
ou seja, se a,b ∈ Q então a + b ∈ Q, a – b ∈ Q, a ⋅ b ∈ Q e ∈ .
b

6 PROMILITARES.COM.BR

PM_BOOK05_MAT.indb 6 01/11/2021 13:40:12


CONJUNTOS

Exemplo:
2
Número inteiro: 2 = ∈ , pois 2 ∈ .
1
2 1
=
Número decimal exato: 0,02 = ∈.
100 50
2
Dízima periódica simples: 0,22222...= ∈.
9
2 1
=
Dízima periódica composta: 0,002222... = ∈.
900 450 O conjunto R pode ser representado por uma reta orientada que
recebe o nome de reta real.
Operações com números racionais:
a c
• Igualdade: = ⇔ ad =bc, tal que b,d ≠ 0.
b d

a c ad + bc
• =
Adição: + , tal que b,d ≠ 0.
b d bd
O conjunto dos números irracionais não é fechado nas quatro
a c ac operações, pois, a,b ∈ I então a + b ∈ I ou a + b ∈ Q, a – b ∈ I ou a – b
• =
Multiplicação: . , tal que b,d ≠ 0.
b d bd a a
∈ Q, a ⋅ b ∈ I ou a ⋅ b ∈ Q e ∈ I ou ∈ .

• Divisão:
a c a d ad
÷ = . = , tal que c,b,d ≠ 0. L DO PRO Exemplo:
b b

IA 2 + 3 ∈ IF
b d b c bc
e (2 − ) ( )
2 + 2 + 2 = 4 ∈

R E
2 − 3 ∈I e (2 + ) ( )
2 − 3 + 2 =−1 ∈ 
Fração geratriz:

S
TE

Operação para obtenção da fração geratriz de uma dízima 2. 3 =6 ∈ I e ( 2 + 2 ) . ( 2 − 2 ) =∈


2 

SO
periódica:
2 6 2 2
MA

– toma-se o número até o final do primeiro período =∈ I e =


2 ∈
(ignorar a vírgula); 3 3 2

R
– diminui do número até o início do primeiro período;
– divide por tantos noves de acordo com a quantidade de INTERVALOS REAIS
algarismo do período; Dados dois números reais tal que a < b, define-se:
– acrescenta-se zero ao denominador de acordo com a • [a,b] = {x ∈  / a ≤ x ≤ b} ou
quantidade de algarismos do ante período.
Exemplo:
R
MA

4−0 4
SO

=
0,4444... = , e
9 9
16 − 0 16
= =
0,161616... • [a,b[ = [a,b) = {x ∈  / a ≤ x < b} ou
T

99 99
S

216 − 2 214
R
E

= =
0,2161616...
990 990
IA F
= =
2,1444...
214 − 21 193
90 90 L DO PR O
• ]a,b] = ( a,b] = { x ∈  / a < x ≤ b} ou

CONJUNTOS DOS REAIS


O conjunto dos números reais (R) é resultado da união do
conjuntos dos números racionais (Q) com o conjunto dos números
irracionais (I), ou seja, R = Q ∪ I. O conjunto dos números irracionais
é formado por números infinitos, decimais e não-periódicos, ou seja, • ]a,b[ = {x ∈  / a < x < b} ou
números que não podem ser representados na forma de fração.
Exemplo:
3 4
Raízes inexatas: 2, 3, 4 e 5
=
Dízimas não periódicas: 0,123456..., π 3,1416...,
= e 2,7182... • [a, +∞=[ [a, +∞=) {x ∈  / x ≥ a} ou

O conjunto dos números reais é fechado nas quatro operações, ou


a
seja, se a,b ∈ R então a + b ∈ R, a – b ∈ R, a ⋅ b ∈ R e ∈R .
b
=  ∪ I ou  =  − I ou I = −.
• ]a, +∞=[ {x ∈  / x > a} ou

 ⊂  ⊂  ⊂  , ou seja,

PROMILITARES.COM.BR 7

PM_BOOK05_MAT.indb 7 01/11/2021 13:40:14


CONJUNTOS

• ]−∞,a] = ( −∞,a] = {x ∈  / x ≤ a} ou Considere a seguinte figura que estes conjuntos formam

• ]−∞,a=[ {x ∈  / x < a} ou

Exercício Resolvido
A região hachurada pode ser representada por:
01. (UFF 2000) Com relação aos conjuntos a) M ∪ (N ∩ P)
P = { x ∈ ℤ | |x | ≤ 7 } e Q = { x ∈ ℤ | x 2 ≤ 0,333} afirma-se: b) M – (N ∪ P)
I. P∪Q=P c) M ∪ (N – P)
II. Q – P = {0} d) N – (M ∪ P)
III. P ⊂ Q e) N ∪ (P ∩ M)
IV. P ∩ Q = Q

D O OsP elementos
Resolução: B
Somente são verdadeiras as afirmativas:
L R Oa N ∪daP. região hachurada pertencem a M e não
a) I e III
b) I e IV I A pertencem
FE
c) II e III R

S
TE

d) II e IV Exercício Resolvido

SO
e) III e IV
04. (UFF 2004) Os muçulmanos sequer se limitam aos países
MA

de etnia árabe, como muitos imaginam. Por exemplo, a maior


Resolução: B
concentração de muçulmanos do mundo encontra-se na

R
P = {–2, –1, 0, 1, 2}; Q = {0}. Indonésia, que não é um país de etnia árabe.
Verdadeira Adaptado de Superinteressante. Ed. 169 – out. 2001.
Falsa: Q – P = ϕ
Considere T o conjunto de todas as pessoas do mundo; M o
Falsa: 1 ∈ P e 1 ∉ Q conjunto de todas aquelas que são muçulmanas e A o conjunto de
Verdadeira todas aquelas que são árabes. Sabendo que nem toda pessoa que
R
é muçulmana é árabe, pode-se representar o conjunto de pessoas
MA

do mundo que não são muçulmanas nem árabes por:


SO

Exercício Resolvido a) T – (A ∪ M)
b) T – A
02. (UFF 2001) O número π − 2 pertence ao intervalo:
T

c) T – (A ∩ M)
3
R
E

a) [1,
2
]
IA F
d) (A – M) ∪ (M – A)

L DO PRO
e) M – A
b) ( 1, 1]
2
Resolução: A
c) [ 3 , 2]
2 T
d) (–1, 1)
e) [– 3 , 0) M
2 A
Resolução: C
π − 2 ≅ 3,14 − 1,41 =1,73

Exercício Resolvido Exercício Resolvido

03. (UFF 2001) Os conjuntos não-vazios M, N e P estão, 05. (UERJ 2001) Um grupo de alunos de uma escola deveria visitar
isoladamente, representados abaixo. o Museu de Ciência e o Museu de História da cidade. Quarenta e
oito alunos foram visitar pelo menos um desses museus. 20% dos
que foram ao de Ciência visitaram o de História e 25% dos que
foram ao de História visitaram também o de Ciência.
Calcule o número de alunos que visitaram os dois museus.

8 PROMILITARES.COM.BR

PM_BOOK05_MAT.indb 8 01/11/2021 13:40:15


CONJUNTOS

06. Numa Universidade são lidos apenas os jornais X e Y, 80% dos


Resolução: alunos leem o jornal X e 60% o jornal Y. sabendo-se que todo aluno
48 alunos visitarem pelo menos um museu. é leitor de pelo menos um dos dois jornais, assinale a alternativa que
C = {alunos que visitaram Ciências} corresponde ao percentual de alunos que leem ambos.
H = {alunos que visitaram História} a) 80% c) 40% e) 48%
b) 14% d) 60%
C H
y 07. Após um jantar, foram servidas as sobremesas X e Y. Sabe-se que
x z das 10 pessoas presentes, 5 comeram a sobremesa X, 7 comeram a
sobremesa Y e 3 comeram as duas. Quantas não comeram nenhuma?
a) 1 c) 3 e) 0
x, y e z representam as quantidades de elementos em cada região. b) 2 d) 4
=x 0,2(x + y) ⇒=
y 4x
 08. (ESA) Se A e B são conjuntos quaisquer, não vazios, podemos
=
x 0,25(x + z) ⇒=z 3x afirmar que a única opção falsa é
x + y + z =48
 a) A – B = ∅ ⇒ B ⊂ A
Então, x + 4x + 3x = 48 ⇒ x = 6 . b) A ∩ B = A ⇒ A ∪ B = B
c) a∈Aea∈B⇒a∈A∩B
d) a ∈ A e A ⊂ B ⇒ a ∈ B

D O e) Pa ∈RA ∪ B ⇒ a ∈ A ou a ∈ B
L O
EXERCÍCIOS DE

FIXAÇÃO I A 09. Sendo M(0)Fo conjunto dos múltiplos de zero e D(0) o conjunto
E M(0) e D(0) são, respectivamente conjuntos:
R
dos divisores de zero,
a) unitário e infinito d) vazio e infinito

S
TE

b) unitário e vazio e) infinito e vazio


01. (ESA) Os números naturais eram inicialmente utilizados para

SO
facilitar a contagem. Identifique a alternativa que apresenta um c) vazio e unitário
MA

número natural.
a) – 4 c) −7 e) 5 10. Dados os conjuntos C = {15, 25, 30, 35} e D = {15, 25, 40, 50},

R
obtenha o n(A ∪ B):
8
b) 8 d) − a) 4 c) 6 e) 8
3
b) 5 d) 7
02. Se A = ∅ e B = {∅}, então:
a) A ⊂ B R
EXERCÍCIOS DE

TREINAMENTO
b) A ∪ B = ∅
MA

SO

c) A=B
d) A ∩ B = B
e) B ⊂ A
T

01. (EEAR) Considere os seguintes conjuntos numéricos: P = {x ∈ Z / x


R
E

é divisor de – 18}, Q = {x ∈ Z / x é múltiplo de – 4}, R = {x ∈ Z / x é


E

IA
03. Em relação aos principais conjuntos numéricos, é correto afirmar que:
F
divisor de 16 e x é divisor de – 24}, S = {x ∈ Z / x é múltiplo de 6}.
a) Todo número inteiro é natural, mas nem todo número natural é
inteiro.
O
L D OT = (PP∩ S)RU (R ∩ Q)?
Quantos números naturais pertencem ao conjunto:
b) Todo número real é natural, mas nem todo número natural é real.
c) Todo número irracional é real. a) 4 b) 2 c) 8 d) 6
d) Todo número racional é natural, mas nem todo número natural
02. (EEAR) Classifique em Verdadeiro (V) ou Falso (F)
é racional.
( ) + ⊂ 
e) Todo número racional é inteiro, mas nem todo número inteiro é
racional. ( ) + ≠ 
( )  − − =
*+
04. Se um conjunto A possui 1024 subconjuntos, então o cardinal de
A é igual a:
( ) (  + ∩  − ) ∪ * = 
( )  − + =−
a) 5 d) 9
A opção que contém a sequência correta é
b) 6 e) 10
a) V – V – F – F – F c) V–F–V–V–F
c) 7
b) F – V – F – F – V d) V – V – F – F – V
05. Os conjuntos A, B e A ∪ B tem respectivamente, 10, 9 e 15
elementos. O número de elementos de A ∩ B é 03. (EEAR) Os elementos de um conjunto A são tais que 10 deles são
múltiplos de 4; 9 são múltiplos de 6; 8 são múltiplos de 12; e 4 são
a) 2 d) 6 números ímpares. Se A ⊂ Ν (N = conjunto dos números naturais),
b) 3 e) 8 então o número de elementos de A é
c) 4 a) 31. b) 25. c) 21. d) 15.

PROMILITARES.COM.BR 9

PM_BOOK05_MAT.indb 9 01/11/2021 13:40:16


CONJUNTOS

04. (EEAR) Seja P o conjunto dos retângulos, Q o conjunto dos 02. (ESPCEX) Sejam os conjuntos A com 2 elementos, B com 3
quadrados e L o conjunto dos losangos. É correto afirmar que elementos e C com 4 elementos. O número de elementos do conjunto
a) L ∩ P = L – P c) L∩Q=P C – [(A ∩ B) ∩ C] pode variar entre:
b) L ∩ Q = L – Q d) L ∩ P = Q a) 2 e 4 b) 2 e 3 c) 0e4 d) 0 e 3 e) 0 e 2

05. (EEAR) “N” é o conjunto dos números naturais, K = {3x | x ∈ N}, 03. (AFA) Entrevistando 100 oficiais da AFA, descobriu-se que 20
L = {5x | x ∈ N} e M = {15x | x ∈ N}. A afirmativa correta é: deles pilotam a aeronave TUCANO, 40 pilotam o helicóptero ESQUILO
e 50 não são pilotos. Dos oficiais entrevistados, quantos pilotam o
a) K ∪ L = M TUCANO e o ESQUILO?
b) K ⊂ L a) 5 b) 10 c) 15 d) 20
c) K–L=M
d) K ∩ L = M 04. (AFA) Em um grupo de n cadetes da Aeronáutica, 17 nadam, 19
jogam basquetebol, 21 jogam voleibol, 5 nadam e jogam basquetebol,
06. (EEAR) Sejam os conjuntos A = {x ∈  / x é múltiplo de 2}, 2 nadam e jogam voleibol, 5 jogam basquetebol e voleibol e 2 fazem
B = {x ∈  / – 2 < x ≤ 9} e C = {x ∈  / x ≥ 5}. A soma dos elementos os três esportes. Qual o valor de n, sabendo-se que todos os cadetes
que formam o conjunto (A ∩ B) – C é desse grupo praticam pelo menos um desses esportes?
a) 9. b) 6. c) 3. d) 1. a) 31 b) 37 c) 47 d) 51

07. (CFOE) Em uma entrevista a um grupo de 979 sargentos da FAB, 05. Em uma entrevista 52 pessoas discutem a preferência por dois
indagou-se sobre o(s) idioma(s) que cada um fala: inglês e espanhol. produtos A e B, entre outros e conclui-se que o número de pessoas
que gostavam de B era:
D O I. PO quádruplo
Constatou-se que 527 falam inglês, 251 espanhol e 321 não falam
nenhum desses idiomas. Com base nessas informações, é INCORRETO
L R O do número de pessoas que gostavam de A e B;
IA F E de pessoas que não gostavam de A nem de B.
afirmar que dos sargentos entrevistados, II. O dobro do número de pessoas que gostavam de A;
a) 120 falam os dois idiomas.
b) mais de 600 falam inglês ou espanhol. R III. A metade do número
Nestas condições, o número de pessoas que não gostavam dos dois

S
TE

c) 452 não falam inglês. produtos é igual a:

SO
d) exatamente 130 falam somente espanhol. a) 48 b) 35 c) 36 d) 47 e) 37
MA

08. (PETROBRÁS) Conversando com os 45 alunos da primeira série 06. (CN) Considere os diagramas onde A, B, C, e U são conjuntos. A

R
de um colégio, o professor de educação física verificou que 36 alunos região sombreada pode ser representada por:
jogam futebol, e 14 jogam vôlei, sendo que 4 alunos não jogam nem
futebol nem vôlei. O número de alunos que jogam tanto futebol
quanto vôlei é
a) 5 c) 9 e) 13
b) 7 d) 11 R
MA

SO

09. (ESPCEX) Sendo:


• +, o conjunto dos números reais não negativos,
• , o conjunto dos números racionais,
T

• , o conjunto dos números inteiros,


R
E

• , o conjunto dos números naturais,


IA F
a intersecção dos conjuntos +,  ∪ ( ∩ ) e ( ∩ ) ∪  é igual a
L D Oa) P R O
( A ∩ B) ∪ ( A ∩ C) − (B ∩ C)
a) ∅ c) * e) 
b) *+ d)  b) ( A ∩ B) ∪ ( A ∩ C) − (B ∪ C)
c) ( A ∪ B) ∪ ( A ∩ C) ∪ (B ∩ C)
10. (EFOMM) Sejam os conjuntos U = {1,2,3,4} e A = {1,2}. O conjunto
B tal que B ∩ A = {1} e B ∪ A = U é d) ( A ∪ B) − ( A ∪ C) ∩ (B ∩ C)
a) 0 c) {1,2} e) U e) ( A − B) ∩ ( A − C) ∩ (B − C)
b) {1} d) {1, 3, 4}
07. (EPCAR 3° ANO) No cursinho , onde se ensina somente
Matemática, Física e Química, há um corpo docente composto de 24
EXERCÍCIOS DE professores, sendo que

COMBATE • 12 ensinam Matemática;


• 10 ensinam Física;
• 9 ensinam Química;
• 4 ensinam Matemática e Física;
01. (EFOMM) Numa companhia de 496 alunos, 210 fazem natação,
260 musculação e 94 estão impossibilitados de fazer esportes. Neste • e nenhum deles ensina Matemática e Química.
caso, o número de alunos que fazem só natação é Se o número de professores que ensinam apenas Física é igual ao
a) 116 c) 166 e) 194 número de professores que ensinam Física e Química, é INCORRETO
b) 142 d) 176 afirmar que:

10 PROMILITARES.COM.BR

PM_BOOK05_MAT.indb 10 01/11/2021 13:40:17


CONJUNTOS

a) 25% do corpo docente ensina apenas Química.


b) o número de professores que ensinam apenas Matemática é 1/3 RESOLUÇÃO EM VÍDEO
do número de professores que compõe o corpo docente.
Abra o ProApp, leia o QR Code, assista à resolução
c) o número de professores que ensinam apenas Física é a metade de cada exercício e AVANCE NOS ESTUDOS!
dos que ensinam apenas Química.
d) existem nesta Escola menos professores ensinando Matemática
ou Física do que professores ensinando Física ou Química.
GABARITO
08. (EPCAR 3° ANO) Para uma turma de 80 alunos do CPCAR, foi
aplicada uma prova de matemática valendo 9,0 pontos distribuídos EXERCÍCIOS DE FIXAÇÃO
igualmente em 3 questões sobre:
01. B 04. E 07. A 10. C
1ª. FUNÇÃO 02. A 05. C 08. A
2ª. GEOMETRIA 03. C 06. C 09. A
3ª. POLINÔMIOS EXERCÍCIOS DE TREINAMENTO
Sabe-se que: 01. A 04. D 07. D 10. D
• apesar de 70% dos alunos terem acertado a questão sobre 02. C 05. D 08. C
FUNÇÃO, apenas 1/10 da turma conseguiu nota 9,0; 03. D 06. B 09. D
• 20 alunos acertaram as questões sobre FUNÇÃO e GEOMETRIA; EXERCÍCIOS DE COMBATE
• 22 acertaram as questões sobre GEOMETRIA e POLINÔMIOS; e
D O 01.
P B
R
04. C 07. D 10. C

A L
• 18 acertaram as questões sobre FUNÇÃO e POLINÔMIOS. 02. A
O 05. A 08. C

I
A turma estava completa nessa avaliação, ninguém tirou nota zero, 03. B F EA
06. 09. D

R
no critério de correção não houve questões com acertos parciais e o
número de acertos apenas em GEOMETRIA é o mesmo que o número ANOTAÇÕES

S
TE

de acertos apenas em POLINÔMIOS.

SO
Nessas condições, é correto afirmar que
MA

a) o número de alunos que só acertaram a 2ª questão é o dobro do


número de alunos que acertaram todas as questões.

R
b) metade da turma só acertou uma questão.
c) mais de 50% da turma errou a terceira questão.
d) apenas 3/4 da turma atingiu a média maior ou igual a 5,0.

09. Sendo  o conjunto dos números reais,  o dos racionais,  o dos R


inteiros e  o dos complexos, considere os conjuntos
MA

SO

A =  – , B =  –  e P =  - ( – )
É correto afirmar que
T

a) B ∩ P ≠ B
S

R
E

b) não existe k ∈  tal que k 5 17 ∈ B


IA F
L DO PRO
c) 7 ∈B
d) (A ∪ B) ∩ P = B

10. (EPCAR 3° ANO) Em uma pesquisa realizada num grupo de 100


alunos do CPCAR, constatou-se que 42 falam inglês, 12 falam inglês
e francês, 18 falam espanhol e inglês e 16 falam espanhol e francês.
O número de alunos que falam espanhol é, precisamente, 50% maior
que o número daqueles que falam francês.
Com base nessas informações, julgue os itens abaixo, classificando-os
em (V) verdadeiros ou (F) falsos.
( ) O número de alunos que falam francês é igual ao do número dos
que falam espanhol.
( ) Se 9 dos alunos consultados falam as três línguas e 5 não falam
nenhuma dessas línguas, então mais da metade dos alunos
falam francês.
( ) Se 9 dos alunos consultados falam as três línguas, espanhol, inglês
e francês, enquanto 5 deles não falam nenhuma dessas línguas,
então exatamente 24 desses alunos falam apenas inglês.
A sequência correta é:
a) V – V – F c) V–F–F
b) F – V – V d) F – F – V

PROMILITARES.COM.BR 11

PM_BOOK05_MAT.indb 11 01/11/2021 13:40:17


CONJUNTOS

ANOTAÇÕES

L DO PRO
I A FE
R

S
TE

SO
MA

R
R
MA

SO
T

R
E

IA F
L DO PRO

12 PROMILITARES.COM.BR

PM_BOOK05_MAT.indb 12 01/11/2021 13:40:17


FUNÇÕES

RELAÇÕES Os exemplos abaixo não se tratam de uma função.


Par ordenado: conjunto ordenado de dois elementos,
representado pelo símbolo (x,y) onde x e y são números reais,
denominados respectivamente de abscissa e ordenada.
Exemplo: Par ordenado (6,–3) abscissas = 6 e ordenada = – 3.
A B
Propriedade: dois pares ordenados são iguais, quando são
respectivamente iguais as abscissas e as ordenadas. Em termos simbólicos: f não é função
(x;y) = (w;z) ↔ x = w e y = z

L DO PRO
PLANO CARTESIANO I A FE
R
Também conhecido como sistema de coordenadas retangulares,
consiste basicamente de dois eixos orientados que se interceptam segundo

S
TE

A B
um angulo reto, num ponto denominado origem. O eixo horizontal é

SO
denominado eixo das abscissas e o eixo vertical é denominado eixo das f não é função
MA

ordenadas. Denominamos o ponto O de origem do plano cartesiano,


sendo nulas a sua abscissa e a sua ordenada, ou seja, O(0;0). No primeiro está sobrando um elemento no conjunto A (domínio)

R
sem se relacionar com algum elemento de B (contradomínio). E no
segundo dois elementos do conjunto A estão se relacionando com um
PRODUTO CARTESIANO único elemento de B.
Dados dois conjuntos A e B, definimos o produto cartesiano de
A por B, que indicamos pelo símbolo A × B, ao conjunto de todos DOMÍNIO E IMAGEM
os pares ordenados (x;y) onde x ∈ A e y ∈ B. Em termos simbólicos,
podemos escrever: R
R 
MA

A × B = {(x;y); x ∈ A e y ∈ B}
SO

0 1
Exemplo: {0;2;3} x {5;7} = {(0;5), (0;7), (2;5), (2;7), (3;5), (3;7)}
-2 -3
T

a. A × B ≠ B × A (o produto cartesiano é uma operação não


S

comutativa)
R 1 2
E

b. A × ∅ = ∅
IA F 2
c. n(A × B) = n(A) ⋅ n(B), onde n(A) e n(B) representam os
números de elementos de A e de B, respectivamente. L DO PR O 2
2 2 +1

2,4
0,7
2x + 1
FUNÇÕES x
Para definir uma função, necessitamos de dois conjuntos
(Domínio e Contradomínio) e de uma fórmula ou uma lei que
relacione cada elemento do domínio a um e somente um elemento No exemplo acima temos dois conjuntos e uma regra que
do contradomínio. relaciona os elementos desses conjuntos. Nesse exemplo se trata de
uma função, pois no conjunto A não sobra nenhum elemento e dois
Nota: Então para uma relação ser uma função, devemos ter o elementos de A não possuem um mesmo correspondente em B.
seguinte: que não sobre nenhum elemento no conjunto A ou
conjunto de partida (domínio) sem se relacionar com um elemento A B
do conjunto B e que um mesmo elemento do conjunto A se f Im (f)
relacione com dois elementos do conjunto B. a
d
b
-1 e
0
0 c
1 g
1
2
2
3
3
A B
Chamamos o conjunto de partida de domínio da função, o
conjunto de chegada das setas de contradomínio, e cada elemento
Como no exemplo acima.
do contradomínio de imagem. Então por exemplo, 1 é a imagem do

PROMILITARES.COM.BR 13

PM_BOOK05_MAT.indb 13 01/11/2021 13:40:18


FUNÇÕES

elemento 0, 2 é a imagem do elemento ½. E temos que a regra dessa A função f : A →  será considerada par, em um gráfico cartesiano
função é 2x + 1, ou seja, cada elemento de A é multiplicado por 2 e se ele for simétrico em relação ao eixo Oy.
somado uma unidade para resultar na sua imagem que pertence a B
ou ao contradomínio. y
Em algumas situações, o domínio e o contradomínio não estarão
explícitos, sendo apresentada somente a lei de formação. Nesses
casos, consideremos que o domínio seja o maior subconjunto possível
dos números reais (D ⊂ R) para o qual a lei faça sentido.
Exemplo: f(x) = x , como não existe raiz quadrada de um
número negativo no conjuntos dos números reais, consideremos que
o domínio seja o conjunto dos números reais não negativos, para
–3 –x –1 1 x 3 x
poder haver a operação da raiz quadrada.
D(f) = R+ ou D(f) = {x ∈ R | x ≥ 0}
y
Exemplos:

1o)

–5 –x x 5
x

L DO PRO
-2 0 1 x I A FE
R FUNÇÃO ÍMPAR
D = {x ∈  | −2 ≤ x ≤ 1} A função f : A →  será considerada ímpar se f(– x) = – f(x) for

S
TE

Im = {y ∈  | 0 ≤ y ≤ 4} para todos x de A.

SO
MA

f : A →  éímpar ⇔ f( −x) =−f(x), ∀ x ∈ 


2o)

R
A função f : A →  será considerada ímpar em um gráfico
cartesiano se ele for simétrico em relação à origem.

y
3
-2 -1 x R 5
a
MA

D = {x ∈  | −2 ≤ x ≤ 3}
SO

3
Im = {y ∈  | −1 ≤ y ≤ 4}
–6 –x –1
T

y x
R
1 x 6
E

IA F
L DO PRO
r –3
d y = f(x) –a
–5

P 4

c
b
0 a b x x 5
Domínio = [a, b] –5 –x x
Conjunto imagem = [c, d]
–b

FUNÇÃO PAR E FUNÇÃO ÍMPAR


–4

FUNÇÃO PAR
A função f : A →  será considerada par, se f(– x) = f(x) for para FUNÇÃO COMPOSTA
todo x de A.
Considere as funções f : A → B e g : B → C, estas são funções
f : A →  épar ⇔ f( −x)
= f(x), ∀ x ∈  compotas das funções de g e f à função h : A → C tal que h (x) = g [f(x)].

14 PROMILITARES.COM.BR

PM_BOOK05_MAT.indb 14 01/11/2021 13:40:20


FUNÇÕES

B Uma função é considerada injetora no diagrama de flechas se


cada elemento de B for atingido por no máximo uma flecha.
f(x) A B A B
A C f f

f g

x
g[f(x)]
h

f é não injetora f é não injetora


A função h : A → C, composta de g e f, é indicada por gof ou
por g(f(x)). Uma função é considerada injetora no gráfico cartesiano se
Interpretado por: g bola f. qualquer reta horizontal interrompe o gráfico, no máximo, uma vez.
Desse modo: y
h(x) = (go f)(x) = g[f(x)]
Exemplos:
Considere os conjuntos A = {2,3,5}, B = {5,7,8} e C = {4,6,9} e as
funções
f : A → B e g: B → C definidas por f(x) = x + 1 e g(x) = 5x – 3.
f(2) = 5 e g(5) = 4
L DO PRO
f(3) = 7 e g(7) = 6
I A FE x
f(5) = 8 e g(8) = 9
R f é injetora

S
TE

A função h : A → C, composta de g e f, onde h(x) = (gof) é tal que:

SO
h(2) = (gof) (2) = g[f(2)] = g(5) = 4
MA

h(3) = (gof) (3) = g[f(3)] = g(7) = 6

R
h(5) = (gof) (5) = g[f(5)] = g(8) = 9
B

2
3 x
4
R
f não é injetora
MA

f g
SO

A C FUNÇÃO SOBREJETORA
Uma função f : A → B somente é considerada sobrejetora quando
T

1 7
S

o seu conjunto-imagem for igual ao contradomínio (B).


R
E

2 12
E

3 I17A F
f : A → B é sobrejetora ⇔ Imf =
CDf
h
L DO PRO Uma função somente será sobrejetora em um diagrama de flechas
se todos os elementos B forem atingidos por pelo menos uma flecha.

A B A B
Observação
f f
A imagem de um determinado elemento x de A através da função
composta gof é definida em duas partes:
A transformação do elemento x de A no elemento f(x) de B.
A transformação do elemento f(x) de B no elemento
g[f (x)] = (gof)(x) de C.
O contradomínio de f é idêntico ao domínio de g, porém, para
existir gof é preciso que Im (f) ⊂ D (g). f é sobrejetora f não é sobrejetora

A B A B
f f
FUNÇÕES INJETORA, SOBREJETORA E BIJETORA

FUNÇÃO INJETORA
Uma função f : A → B é considerada injetora se os elementos
distintos de A tiverem imagens distintas em B.
f : A → B éinjetora ⇔ ( x1 ≠ x 2 ⇒ f(x1) ≠ f(x 2 ) )
f é sobrejetora f não é sobrejetora

PROMILITARES.COM.BR 15

PM_BOOK05_MAT.indb 15 01/11/2021 13:40:22


FUNÇÕES

Uma função somente será sobrejetora em um plano cartesiano se b. f : + → +


a projeção do gráfico sobre o eixo Oy for contradomínio. f(x) = x2
y y y = x2

Se cada uma das retas cortar em um ou mais pontos, será


A sobrejetora.
x
f é sobrejetora a. f :  → 
pois lm(f ) = B f(x) = x – 1
y
y

L DO PRO y=x–1

I A FE
B
R

S
TE

SO
MA

A
f não é sobrejetora
x
R
pois lm(f ) B R
MA

SO

ProBizu
FUNÇÃO BIJETORA
Uma função f : A → B será considerada bijetora se f for sobrejetora Para a função f(x) = x2 se tivermos f :  → + ela será sobrejetora
T

e injetora. mas para f :  →  ela não será, pois para o caso f :  →  todos
S

R
Se cada uma das retas cortar o gráfico em um só ponto ou não os números reais negativos estarão disponíveis no contra domínio
E

cortar o Gráfico, será injetora. IA F


mas não serão relacionados, logo sobrarão.

a. f :  →  L D O b.P f :R →O +


f(x) = x
f(x) = x 2

y y=x y

x
x

16 PROMILITARES.COM.BR

PM_BOOK05_MAT.indb 16 01/11/2021 13:40:26


FUNÇÕES

Se cada uma das retas cortar em um único ponto, será bijetora. A função inversa f–1 é formada pelos pares {(1,1), (3,2), (5,3), (7,4)}
a. f :  →  onde D(f–1) = B e Im(f–1) = A.
Observemos que a função f é definida pela sentença y = 2x – 1, e
f(X) = 2x
y
f–1 pela sentença x = y + 1 isto é
2
a. f leva cada elemento x ∈ A até o y ∈ B tal que y = 2x – 1.
y +1
b. f–1 leva cada elemento y ∈ B até o x ∈ A tal que x = .
2
x B
A
f

b. f :  → 
f(x) = x · |x|
y
D(f–1) = B = lm(f)

B A

L DO PRO f–1

IA FE
x

S
TE

SO
1. Se nenhuma reta corta o gráfico mais de uma vez f é injetora.
MA

2. Se toda reta corta o gráfico, f é sobrejetora. lm(f–1) = A = D(f)

R
3. Se toda reta corta em um só ponto, f é bijetora.
Obter a função inversa utilizando a regra prática.
FUNÇÃO INVERSA
REGRA PRÁTICA EXEMPLO
Dados os conjuntos A = {1, 2, 3, 4} e B = {1, 3, 5, 7} consideremos
a função f de A em B definida por f(x) = 2x – 1.
Substituir f(x) por y R y = 2x + 3
B
MA

A
f
SO

Trocar x pory e y por x x = 2y + 3


1 x −3
1 Isolar y x = 2y + 3 ⇔ 2y = x = −3 ⇔ y =
2
T

3
S

2
R5 f −1 ( x ) =
x −3
E

Substituir y por f–1(x)


E

3
IA F 2

4 7 L DO PR O
GRÁFICOS DE F E f –1

B A Os gráficos de f e f–1 são simétricos em relação a bissetriz dos


f–1
quadrantes ímpares (1° e 3°), cuja equção é y = x.
1 1
y
3 2
f–1
5 3 y=x

7 4

Notemos que f é bijetora formada pelos pares ordenados


f = {(1,1), (2,3), (3,5), (4,7)} onde D(f) = A e Im(f) = B.
A relação f–1 = {(y,x) | (x,y) ∈ f}, inversa de f , é também uma
função, pois para todo y ∈ B existe x ∈ A.

PROMILITARES.COM.BR 17

PM_BOOK05_MAT.indb 17 01/11/2021 13:40:29


FUNÇÕES

Exercício Resolvido Exercício Resolvido


01. Na figura abaixo, estão representados os gráficos de duas 03. Considere a função real g, cuja representação gráfica está
funções reais, f e g, com domínios reais. Para cada x ∈  a função parcialmente ilustrada na figura a seguir. Sendo g ° g a função
h é definida por h(x) = f(x) · g(x). composta de g com g, então, o valor de (g ° g)(–2) é:
y
y
f 4
g

2
–5 –4 –2 0 2 3 x

0 1 4 5

a) 0
Nessas condições, o valor de h(5) é igual a b) 4
a) 0. c) 2
b) 4.
L D O d)P –2R O
IA FE
c) 10. e) –5
d) 25.
R Resolução: B

S
TE

Resolução: B De acordo com o gráfico, temos g (–2) – 0. Logo, segue que

SO
Desde que f(5) = 2 e g(5) = 2, tem-se que: (g ° g)(–2) = g(g(–2)) = g(0) = 4.
MA

h(5) = f(5) · g(5) = 2 · 2 = 4.

R
Exercício Resolvido
Exercício Resolvido
04. Dada f(x) = x² + 2x + 5, o valor de f(f(–1)) é:
02. Sejam a) – 56
x−2
I. f(x) = 2 b) 85
x +2
R
c) – 29
MA

1
f(x) = , x ≠ 0.
SO

II. d) 29
x2
e) – 85
2
III. f(x) = , x ≠ 0.
T

x
S

Resolução: D
IV. f(x) = (x + 1) + (x - 1)
R
E

IA F
Como f( −1) =( −1)2 + 2 ⋅ ( −1) + 5 =4, segue que

L DO PRO
Classificando cada uma das funções reais acima em par, ímpar ou f(f( −1)) = f(4) = 42 + 2 ⋅ 4 + 5 = 29.
nem par nem ímpar, temos, respectivamente:
a) par, par, ímpar, ímpar
b) nem par nem ímpar, par, ímpar, ímpar
c) par, ímpar, par, ímpar Exercício Resolvido
d) ímpar, par, ímpar, ímpar
x+3
e) par, par, ímpar, nem par nem ímpar 05. Sabe-se que a função f(x) = é invertível. Assim, f-1(3) é
5
a) 3
Resolução: B b) 4
[I] f não é par nem ímpar. De fato, como c) 6
−x − 2 x + 2 segue-se que f(–x) ≠ f(x) e f(–x) ≠ –f(x). d) 12
f( −x) = 2 =− 2 ,
( −x) + 2 x +2
Portanto, f não é par nem ímpar. Resolução: D
1 1 Se f possui inversa, então queremos calcular x tal que f(x) = 3.
[II] f é par. Com efeito, f( − x) = = = f(x). Por conseguinte,
f é par. ( −x)2 x 2 x+3
Assim, vem = 3 ⇔ x = 12.
5
2
[III] f é ímpar. De fato, f( − x) = =
2
− =−f(x). Portanto, f é
ímpar. −x x

[IV] f é ímpar. De fato, f(-x) = 2 ⋅ (-x) = -2x = -f(x). Por conseguinte,


f é ímpar.

18 PROMILITARES.COM.BR

PM_BOOK05_MAT.indb 18 01/11/2021 13:40:30


FUNÇÕES

EXERCÍCIOS DE 04. (EEAR) Considerando que o domínio de uma função é o maior

FIXAÇÃO
subconjunto de  constituído por todos os valores que podem ser
atribuídos à variável independente, o domínio da função h(x) = x + 4 é
a) *. c) {x ∈  / x < 4}.
b)  - {4}. d) {x ∈  / x ≥ –4}.
01. (EEAR) Considerando D = [0, 10] o domínio de uma função
y = f(x), um gráfico que poderia representá-la é 05. (EEAR) Seja f uma função definida no conjunto dos números
a) naturais, tal que f(x + 1) = 2f(x) + 3. Se f(0) = 0, então f(2) é igual a
a) 9. b) 10. c) 11. d) 12.

06. (EEAR) Seja f :  →  uma função. Essa função pode ser


a) f(x) = x
b) b) f(x) = |x|
c) f(x) = 1/x

d) f(x) = 1
1+ x
x −1
07. (EEAR) O domínio da função real g(x) = é
c) 3
x2 − 4
D = {x ∈  / ________________ }.

L D O a)b) Pxx >≥R12 eeO


x≠2 c) –1 ≤ x ≤1

I A FE
x≠4 d) –2 ≤ x ≤ 2 e x ≠ 0

d)
R 08. (EEAR) Considere a função ƒ : * →  definida por ƒ(x) =
2x + 2
x

S
TE

. Se ƒ(2a) = 0, então o valor de a é

SO
a) –1/2 b) 1/2 c) –1 d) 1
MA

x +5
09. (EEAR) Seja a função real ƒ(x) = . A sentença que

R
x −1
completa corretamente a expressão do conjunto domínio
02. (EEAR) O conjunto imagem da função representada pelo gráfico é D = {x ∈  / ______________} dessa função é
a) x > 1. b) x ≠ 1. c) x > 0. d) x ≠ 0.

(2x − 3)(4x + 1)
R
10. (EEAR) Seja ƒ(x) = uma função. Um valor que não
(x + 2)(x − 5)
MA

SO

pode estar no domínio de ƒ é


a) 1. b) 2. c) 3. d) 5.
T

R EXERCÍCIOS DE
E

IA F
TREINAMENTO
a) ]–5, –2] ∪ [0, 10]
b) ]–2, 0] ∪ [4, 10] L DO P R O
c) [–5, –2[ ∪ [0, 4]
x −1 3x
d) [–2, 0] ∪ [0, 4[ 01. (EEAR) Se ƒ(x) = + é uma função, seu domínio é
x +1 x+4
D = {x ∈  / ______________}.
03. (EEAR) Analisando o gráfico da função f da figura, percebe-se que,
nos intervalos [–5, –2] e [–1, 2] de seu domínio, ela é, respectivamente, a) x > 4 e x ≠ 1 c) x<–4ex≠–1
b) x < 4 e x ≠ ± 1 d) x > – 4 e x ≠ – 1

02. (EEAR) Seja a função ƒ(x) = x + 1 + −2x + 1. Os valores inteiros


do domínio de f são tais que seu produto é igual a
a) 0. b) 1. c) 2. d) 3.

03. (EEAR) Para que uma função seja invertível, é necessário que ela
seja
a) sobrejetora e positiva.
a) crescente e crescente. b) bijetora e positiva.
b) crescente e decrescente. c) apenas bijetora.
c) decrescente e crescente. d) apenas injetora.
d) decrescente e decrescente.

PROMILITARES.COM.BR 19

PM_BOOK05_MAT.indb 19 01/11/2021 13:40:31


FUNÇÕES

04. (EAM) Sejam os conjuntos A = {x ∈ ;1 ≤ x ≤ 4}, 04. (EEAR) A função g : [–5, 5] → B tem como imagem o conjunto
B = {y ∈ ;3 ≤ x ≤ 7}. Considerando o conjunto A × B, (A cartesiano I = [20, 30]. Para que ela seja sobrejetora é necessário que B seja igual
B) pode-se afirmar que a diagonal do pollgono formado por esse ao intervalo
conjunto é representada numericamente por a) [5, 20]. c) [–5, 30].
a) 2 d) 5 b) [–5, 20]. d) [20, 30].
b) 3 e) 6
c) 4 05. (EEAR) Sejam f e g duas funções reais inversas entre si. Se
f(x) = 3x – 2, então g(1) é igual a
05. (EEAR) Para que a função f :  ⟶ A; f(x) = (x + 1)(x − 3) seja a) 0. b) 1. c) 2. d) 3.
sobrejetora, é necessário ter o conjunto A igual a
a)  c) {x ∈  / x ≥ − 4} 06. (EEAR) A função f :  → , definida por f(x) = 3x + 2,
b) + + d) {x ∈  / x ≠ − 1 e x ≠ − 3} a) é apenas injetora.
b) é apenas sobrejetora.
1 + 3x c) é injetora e sobrejetora.
06. (EEAR) Se ƒ(x) = , com x ∈  e x ≠ –3, é uma função
x+3 d) não é injetora e nem sobrejetora.
invertível, o valor de ƒ (2) é
-1

a) –2 b) –1 c) 3 d) 5 1 1
07. (ESPCEX) O domínio da função =
real y − é
x+3 5−x
07. (EEAR) Dada a funçãoƒ(x – 1) = x² + 3x –2, considerando os valores
a) ]–3, 5[ c) ]–5, 3[
DO PRO
de ƒ(1) e ƒ(2), pode-se afirmar corretamente que
b) ]–3, +∞[ d) ]–∞, 3[ ∪ ]5, +∞[
a) ƒ(1) = ƒ(2) + 4 c) ƒ(2) = 2ƒ(1)
A L
b) ƒ(2) = ƒ(1) - 1 d)
I
ƒ(1) = 2ƒ(2) FE
08. (ESPCEX) Na função ƒ(x) = 3x – 2, sabemos que ƒ(a) = b – 2 e

08. (ESA) Sejam as funções reais dadas por ƒ(x) = R


ƒ(b) = 2b + a. O valor de ƒ(ƒ(a)) é:
5x + 1 e g(x) = 3x – 2.
a) 2 c) 0 e) -2

S
TE

Se m = ƒ(n), então g(m) vale:


b) 1 d) –1

SO
a) 15n + 1 c) 3n – 2 e) 14n – 2
MA

b) 14n – 1 d) 15n – 15
09. (AFA) Se f e g são funções de  em  definidas por
3x − 2

R
09. (ESA) Funções bijetoras possuem função inversa porque elas são f(3x + 2) = e g(x – 3) = 5x – 2, então f(g(x)) é
5
invertíveis, mas devemos tomar cuidado com o domínio da nova x−4
função obtida. Identifique a alternativa que apresenta a função a) c) 5x + 13
5
inversa de f(x) = x + 3.
5x + 9 5x + 11
a) ƒ(x)-1 = x – 3. b) d)
5 5
b) ƒ(x) = x + 3.
-1 R
MA

c) ƒ(x)-1 = – x – 3. 10. (AFA) Seja f : [1,∞) → [–3,∞) a função definida por f(x) = 3x2 – 6x.
SO

d) ƒ(x)-1 = – x + 3. Se g : [–3,∞) → [1,∞) é a função inversa de f, então [g(6) – g(3)]2 é


e) ƒ(x)-1 = 3x. a) 5 c) 5–2 6
T

b) 2 6 d) –5 + 2 6
S

x+3
R
E

10. (EEAR) Sabe-se que a função ƒ(x) = é invertível. Assim, ƒ-1(3) é


E

5
IA F
R O EM VÍDEO
a) 3 b) 4 c) 6 d) 12
L DO P RESOLUÇÃO
Abra o ProApp, leia o QR Code, assista à resolução
de cada exercício e AVANCE NOS ESTUDOS!
EXERCÍCIOS DE

COMBATE GABARITO
EXERCÍCIOS DE FIXAÇÃO
01. (EEAR) Seja ƒ(x) = 4x + 3 uma função inversível. A fórmula que
define a função inversa ƒ-1(x) 01. B 04. D 07. A 10. D
x−4 x−3 2x + 3 2x + 4 02. C 05. A 08. A
a) b) c) d)
3 4 4 3 03. B 06. B 09. A
EXERCÍCIOS DE TREINAMENTO
02. (EEAR) Seja a função ƒ :  →  definida por ƒ(x) = 4x – 3. Se ƒ-1 é 01. D 04. D 07. C 10. D
a função inversa de ƒ, então ƒ-1(5) é
02. A 05. C 08. A
a) 17. b) 1/17. c) 2. d) 1/2.
03. C 06. D 09. A
03. (EEAR) O conjunto imagem da função ƒ :  →  definida por EXERCÍCIOS DE COMBATE
1 01. B 04. D 07. A 10. C
ƒ(x) = , contém o elemento
1 + x² 02. C 05. B 08. B
a) 0. b) 2. c) 1/2. d) –1.
03. C 06. A 09. B

20 PROMILITARES.COM.BR

PM_BOOK05_MAT.indb 20 01/11/2021 13:40:32


FUNÇÃO AFIM

FUNÇÃO AFIM FUNÇÃO LINEAR


Um representante comercial recebe, mensalmente, um salário Uma aplicação f de R em R recebe o nome de função linear
composto de duas partes: uma parte fixa, no valor de R$ 2500,00 e quando a cada elemento x ∈ R associa elemento ax ∈ R, onde a ≠ 0 é
uma parte variável, que corresponde a uma comissão de 6% (0,06) um número real dado.
sobre o total das vendas que ele faz durante o mês. Observe que o f:R→R
salário desse vendedor é dado em função do total de vendas que ele x → ax, a ≠ 0
faz durante o mês.
Representando o total de venda por x, temos:

DO PRO
s(x) = 2500,00 + 0,06x
ou s(x) = 0,06x + 2500,00
A L
ou y = 0,06x + 2500,00
I FE
Esse é um exemplo de função afim.
R

S
TE

FUNÇÃO CONSTANTE

SO
Uma aplicação f de R em R recebe o nome de função constante
MA

quando a cada elemento x ∈ R associa sempre o mesmo elemento c ∈ R.


f:R→R O gráfico da função linear é uma reta que sempre passa pela

R
x→c origem.

FUNÇÃO AFIM
Uma aplicação f de R em R recebe o nome de função afim quando
a cada elemento x ∈ R associa (ax + b) ∈ R com a ≠ 0.
R
f:R→R
MA

SO

x → ax + b, a ≠ 0
Exemplos:
y = 3x + 2, onde a = 3 e b = 2
T

R y = –2x + 1, onde a = –2 e b = 1
E

IA F
y = x – 3, onde a = 1e b = –3

L D O Repare O
y = 4x, onde a = 4 e b = 0

FUNÇÃO IDENTIDADE
P Rque para b = 0 teremos, f(x) = ax que é uma função linear,
então, a função linear é um caso particular da função afim.
Uma aplicação f de R em R recebe o nome de função identidade
quando a cada elemento x ∈ R associa o próprio x. GRÁFICO
f:R→R O gráfico cartesiano da função f(x) = ax + b, com a ≠ 0, é uma reta.
x→x f(x) = 2x + 1

PROMILITARES.COM.BR 21

PM_BOOK05_MAT.indb 21 01/11/2021 13:40:32


FUNÇÃO AFIM

f(x) = –x + 3 FUNÇÕES CRESCENTES OU


DECRESCENTES
• f se diz uma função crescente se num intervalo x1 a x2, com
x1 < x2, temos f(x1) < f(x2).
Ou seja, se num intervalo x cresce, as imagens f(x) crescem
também.
• f se diz uma função decrescente se num intervalo x1 a x2, com
x1 < x2, temos f(x1) > f(x2).
Ou seja, se num intervalo x cresce, as imagens f(x) decrescem.

Teorema: Uma função afim é crescente se o seu coeficiente


angular (a) é positivo e é decrescente se o coeficiente angular é
IMAGEM negativo.
O conjunto imagem de f de R em R definida por ax + b, com a ≠ 0, é R.

=
De fato qualquer que seja y ∈ R, existe x
y −b
∈ R , (isolando x SINAL DA FUNÇÃO AFIM
na equação y = ax + b). a −b
Como já vimos o zero da função é x = .
a
Caso 1 – Se a > 0 (função crescente).
COEFICIENTES DA FUNÇÃO AFIM
L DO PRO
O coeficiente a da função f(x) = ax + b, é denominado coeficiente
A
angular ou declividade da reta representada no plano cartesiano.
I FE
R
O coeficiente b da função f(x) = ax + b, recebe o nome de
coeficiente linear.

S
TE

Exemplo:

SO
Na função f(x) = 2x + 1 o coeficiente angular é 2 e o coeficiente
MA

linear é 1. observe que se x = 0, temos, f(0) = 2 ⋅ 0 + 1, f(0) = 1,


portanto para x = 0, y = 1.

R
Portanto, o coeficiente linear é a ordenada do ponto em que a
reta corta o eixo y.
O coeficiente angular (a) é tangente do ângulo formado entre a

reta e o eixo Ox , vamos chamar esse ângulo de α.
yB − y A ∆y
a= tg α= = R
xB − x A ∆x
MA

SO

Caso 2 – Se a < 0, (função decrescente).


ZERO DA FUNÇÃO AFIM
Zero de uma função é todo número x cuja imagem é nula, ou
T

seja, f(x) = 0.
R
E

x é zero de y = f(x) ⇔ f(x) = 0


IA F
L DO PRO
Assim para determinarmos o zero da função afim, basta resolver
a equação do primeiro grau
ax + b = 0
b
isolando x temos, ax = –b, passando a dividindo, x = − .
a
Podemos interpretar graficamente o zero da função afim, como a
abscissa do ponto onde o gráfico corta o eixo x.

INEQUAÇÕES SIMULTÂNEAS
A dupla desigualdade f(x) < g(x) < h(x), se decompõem em 2
inequações simultâneas.

 f(x) < g(x) I


f(x) < g(x) < h(x) ⇔ 
g(x) < h(x) II
1
O zero desta função é , pois para x = 1/2 temos y = 0, (1/2, 0).
2

22 PROMILITARES.COM.BR

PM_BOOK05_MAT.indb 22 01/11/2021 13:40:33


FUNÇÃO AFIM

Exemplo: Exercício Resolvido


3x + 2 < −x + 3 I
3x + 2 < − x + 3 ≤ x + 4 ⇔  02. A função f está definida da seguinte maneira: para cada inteiro
−x + 3 ≤ x + 4 II
ímpar n,
Temos que resolver as 2 inequações:
x − (n − 1) , se n − 1 ≤ x ≤ n
1 f(x) = 
1) 3x + 2 < −x + 3 ⇒ 4x < 1 ⇒ x <  n + 1 − x, se n ≤ x ≤ n + 1
4
1 a) Esboce o gráfico de f para 0 ≤ x ≤ 6.
2) –x + 3 ≤ x + 4 ⇒ −2x ≤ 1 ⇒ x ≥ −
2
1
b) Encontre os valores de x, 0 ≤ x ≤ 6, tais que f(x) = .
A interseção desses dois conjuntos é: 5

Resolução:
 x, se 0 ≤ x ≤ 1
a) n=
1 ⇒ f(x) =

2 − x, se 1 ≤ x ≤ 2
x − 2, se 2 ≤ x ≤ 3
n =⇒
3 f(x) =
2 − x, se 3 ≤ x ≤ 4

x − 4, se 4 ≤ x ≤ 6
n =⇒
5 f(x) =
D O DePacordo
 6 − x, se 5 ≤ x ≤ 6
 1 1
S = x ∈  | − ≤ x < 
L R Ocom as funções acima, temos o seguinte gráfico.
 2 3
I A FE
Exercício Resolvido R

S
TE

01. A empresa Alpha dedica-se exclusivamente à digitalização

SO
de documentos. Um funcionário leva 4 horas para digitalizar um
MA

documento, a empresa opera durante 250 dias por ano e não


há estoque de documentos antigos para digitalizar. Em 2014,

R
os funcionários têm uma jornada de trabalho de 8 horas diárias,
mas têm exatamente 2 horas de ociosidade por dia. Em relação a
2014, o número de novos documentos que chegam para serem
digitalizados aumentará 10.000 por ano nos próximos três anos.
Sem novas contratações, em 2017, os funcionários precisarão
trabalhar 8 horas por dia sem qualquer tempo ocioso para R
conseguir processar toda a demanda de 2017.
b) Considerando f(x) = 1 , temos:
MA

a) Qual é o número atual de funcionários da empresa?


SO

5
b) Quantos documentos deverão ser digitalizados em 2015?
1 1 9
c) Representando o ano de 2014 como x = 0, 2015 como f(x) = ⇒ 2 − x = ⇒ x =
5 5 5
T

x = 1, 2016 como x = 2, e assim por diante, é possível expressar


S

1 1 11
R
Y (demanda da empresa, em número de documentos para f(x) = ⇒ x − 2 = ⇒ x =
E

5 5 5
E

IA
digitalização) em função de x, para o período de 2014 a 2017,
1 F 1 19

L DO PRO
como Y(x) = a + bx. Nesta expressão, a representa o número f(x) = ⇒ 4 − x = ⇒ x =
de documentos digitalizados em 2014. Determine o valor de b. 5 5 5
1 1 21
f(x) = ⇒ x − 4 = ⇒ x =
Resolução: 5 5 5
a) Seja d o número anual de documentos que serão digitados 1 1 29
f(x) = ⇒ 6 − x = ⇒ x =
anualmente e f o número de funcionários. De acordo com as 5 5 5
informações do texto, podemos escrever: 1 9 11 19 21
Portanto, x = ou x = ou x = ou x = ou x =
Em 2014: d=
( 8 − 2) ⋅ f ⋅ 250 ⇒ d= 375 ⋅ f (equação 1) 29 5 5 5 5 5
4 ou x = .
5
8 ⋅ f ⋅ 250
Em 2017: d + 30000= ⇒d
= 500 ⋅ f − 30000
(equação 2) 4
Igualando as equações 1 e 2, temos: 500 · f – 30000 = 375f
⇒ 125 · f = 30000 ⇒ f = 240. Portanto, a empresa tem 240
funcionários.
b) Em 2014 o número de documentos digitados foi de:
d = 375 · 240 = 90000
Logo em 2015, teremos 90.000 + 10.000 = 100.000
documentos digitados.
c) O valor de b é a taxa de variação da função linear, como já
foi dito que esta variação é de 10.000 documentos ao ano,
podemos considerar que b = 10.000.

PROMILITARES.COM.BR 23

PM_BOOK05_MAT.indb 23 01/11/2021 13:40:34


FUNÇÃO AFIM

Exercício Resolvido

03. A quantidade de cópias vendidas de cada edição de uma


revista jurídica é função linear do número de matérias que abordam
julgamentos de casos com ampla repercussão pública. Uma
edição com quatro matérias desse tipo vendeu 33 mil exemplares,
enquanto que outra contendo sete matérias que abordavam
aqueles julgamentos vendeu 57 mil exemplares.
a) Quantos exemplares da revista seriam vendidos, caso fosse
publicada uma edição sem matéria alguma que abordasse
julgamento de casos com ampla repercussão pública?
b) Represente graficamente, no plano cartesiano, a função
da quantidade (Y) de exemplares vendidos por edição, pelo Determine o tempo x0, em horas, indicado no gráfico.
número (X) de matérias que abordem julgamentos de casos
com ampla repercussão pública.
Resolução:
c) Suponha que cada exemplar da revista seja vendido a
R$ 20,00. Determine qual será o faturamento, por edição, em De acordo com as informações do problema, temos: yA = 720 – 10x
função do número de matérias que abordem julgamentos de e yB = 60 + 12x.
casos com ampla repercussão pública. O valor x0 indicado no gráfico é o valor de x quando yA = yB, ou seja:
720 – 10x = 60 + 12x ⇒ -22x = -660 ⇒ x = 30. Logo, x0 = 30 horas.

D O Exercício
Resolução:

L P R OResolvido
Seja f :  →  a função afim definida por f(x) = ax + b, em que

IA F E dos calçados obedece a padrões distintos,


f(x) é o número de cópias vendidas e x é o número de matérias
que abordam julgamentos de casos com ampla repercussão
R 05. A numeração
pública. Sabendo que o gráfico de f passa pelos pontos (4,33000)
57000 − 33000
conforme o país. No Brasil, essa numeração varia de um em um,

S
TE

e (7,57000),=tem-se que a = 8000. e vai de 33 a 45, para adultos. Nos Estados Unidos a numeração
7−4 varia de meio em meio, e vai de 3,5 a 14 para homens e de 5 a

SO
Logo, 33000 = 8000 · 4 + b ⇔ b = 1000. 15,5 para mulheres.
MA

a) O valor inicial da função f, definida acima, é igual a 1000. a) Considere a tabela abaixo.

R
b) O gráfico pedido é
Numeração brasileira (t) Comprimento do calçado (x)
35 23,8 cm
42 27,3 cm
Suponha que as grandezas estão relacionadas por funções
R
afins t(x) = ax + b para a numeração brasileira e x(t) = ct +
MA

d para o comprimento do calçado. Encontre os valores dos


SO

parâmetros a e b da expressão que permite obter a numeração


dos calçados brasileiros em termos do comprimento, ou os
valores dos parâmetros c e d da expressão que fornece o
T

comprimento em termos da numeração.


S

R
E

b) A numeração dos calçados femininos nos Estados Unidos


E

IA F
pode ser estabelecida de maneira aproximada pela função real

L DO PRO f definida por f(x) = 5(x – 20) / 3, em que x é o comprimento


do calçado em cm. Sabendo que a numeração dos calçados
nk forma uma progressão aritmética de razão 0,5 e primeiro
termo n1 = 5, em que nk = f (ck), com k natural, calcule o
c) Seja g:  →  a função definida por g(x) = 20 · f(x), em que
comprimento c5.
g(x) é o faturamento por adição e f(x) é o número de cópias
vendidas, conforme definido em (a).
Portanto, segue-se que g(x) = 20 · (8000x + 1000) = 160000x Resolução:
+ 20000. a) t(x) = ax + b
27,3.a + b = 42

 23,8.a + b =35
Exercício Resolvido
Resolvendo o sistema, temos: a = 2 e b = –12,6. Logo
04. O reservatório A perde água a uma taxa constante de 10 t(x) = 2x – 12,6. Agora escrevendo x em função de t, temos:
litros por hora, enquanto o reservatório B ganha água a uma taxa x(t) = 0,5t + 6,3, portanto c = 0,5 e t = 6,3.
constante de 12 litros por hora. No gráfico, estão representados, 5.(x − 20)
no eixo y, os volumes, em litros, da água contida em cada um dos b) f(x) =
3
reservatórios, em função do tempo, em horas, representado no
eixo x. n1 = 5, n2 = 5,5, n3 = 6, n4 = 6,5 e n5 = 7.
5.(c5 − 20)
Fazendo 7 = , temos: 5c5 – 100 = 21 ⇒ 5c5 = 121
3
⇒ c5 = 24,2 cm

24 PROMILITARES.COM.BR

PM_BOOK05_MAT.indb 24 01/11/2021 13:40:35


FUNÇÃO AFIM

EXERCÍCIOS DE 06. (EAM) Considere o gráfico abaixo de uma função real, definida

FIXAÇÃO por y = ax + b:
y

01. (CFN) Qual o valor de X na inequação 1/2 + 2X/3 > 3/2?


a) X > 4
2
b) X < 5/2
c) X < 3/2
d) X > 3/2 2
e) X > –3/2 0 x

02. (CFN) Coloque C (certo) ou E (Errado) na afirmação sobre as


inequações, assinalando a seguir a opção correta.
( ) Se – 2x > 4, então x < – 2.
Com base nesse gráfico, é correto afirmar que a equação que define
( ) Se 3x > – 18, então x < – 6. essa função é:
( ) Se – 6 < – x, então 6 > x. a) 4y = -4x+16
( ) Se – 5x < 35, então x > – 7 b) 4y = -4x+8

D O d) Py =R2x +2O
c) y = -2x +4
a) C, C, E, E
b) C, E, C, C L
c) E, E, C, C IA e) 2y = x-2 F
E
d) C, E, C, E R 07. (EEAR) Na função f(x) = mx – 2(m – n), m e n ∈ ℜ Sabendo que

S
TE

e) E, C, C, E f(3) = 4 e f(2) = –2 , os valores de m e n são, respectivamente

SO
a) 1 e -1
03. (CFN) O gráfico abaixo pode representar qual das expressões?
MA

b) -2 e 3
y c) 6 e -1

R
d) 6 e 3

3 08. (EEAR) Sejam f e g funções polinomiais de primeiro grau, tais que


o gráfico de f passa por (2,0) e o de g, por (–2,0). Se a intersecção dos
gráficos é o ponto (0,3), é correto afirmar que
a) y = 2x – 3. a) f e g são crescentes.
R
MA

b) y = –2x + 3. -2 b) f e g são decrescentes.


SO

c) y = 1,5x + 3. 0 x c) f é crescente e g é decrescente.


d) 3y = –2x. d) f é decrescente e g é crescente.
T

e) –2y = 3x.
S

R
E

09. (EEAR) O ponto de intersecção dos gráficos das funções f(x) = x + 2


E

IA
04. (EAM) A função f :  →  definida por f(x) = –3x + 6 é F
e g(x) = 2x – 1 pertence ao ____ quadrante.
a) crescente para todos os reais.
b) crescente para x > 2.
L D Ob) 2ºP R O
a) 1º c)
d) 4º

c) decrescente para todos os reais.


d) decrescente para x < 2. 10. (EEAR) A função definida por y = m(x – 1) + 3 – x, m ∈ , será
crescente, se
e) decrescente para x ≥ 2.
a) m ≥ 0. c) –1 < m < 1.
05. (EAM) O conjunto solução no campo dos reais da inequação b) m > 1. d) – 1 < m ≤ 0.
3x + 5 > –7x + 3 é
 2 EXERCÍCIOS DE
a) x ∈  / x ≥ + 

TREINAMENTO
 10

 2
b) x ∈  / x < − 
 10

 2  01. (CESGRANRIO) O valor de um carro novo é de R$ 9.000,00 e,


c)  − 10 , + ∞ 
com 4 anos de uso, é de R$ 4.000,00. Supondo que o preço caia com
o tempo, segundo uma linha reta, o valor de um carro com 1 ano de
2 
d) 10 , + ∞  uso é:
a) R$ 8.250,00 d) R$ 7.500,00
 2  b) R$ 8.000,00 e) R$ 7.000,00
e)  −∞, − 10 
c) R$ 7.750,00

PROMILITARES.COM.BR 25

PM_BOOK05_MAT.indb 25 01/11/2021 13:40:35


FUNÇÃO AFIM

02. Na figura a seguir tem-se o gráfico da função f, onde f(x) 07. (EEAR) Se f(x) = (k – 4)x + 2 é uma função do 1º grau decrescente,
representa o preço pago em reais por x cópias de um mesmo original, então
na Copiadora Reprodux. a) k < 4. c) k = 5.
b) k > 6. d) k = 8.

08. (ESPCEX) Utilizando o fato de a água congelar a 0° Celsius ou


32° Fahrenheit e ferver a 100º Celsius ou 212° Fahrenheit, e sabendo
que existe uma relação linear entre as duas escalas de temperaturas,
conforme o gráfico abaixo, pode-se completar adequadamente a
tabela abaixo com os seguintes valores aproximados ou exatos.
Celsius –10°
Fahrenheit 0° 68°

De acordo com o gráfico, é verdade que o preço pago nessa Copiadora por
a) 228 cópias de um mesmo original é R$ 22,50.
b) 193 cópias de um mesmo original é R$ 9,65.
c) 120 cópias de um mesmo original é R$ 7,50.
L DO PRO
d) 100 cópias de um mesmo original é R$ 5,00
e) 75 cópias de um mesmo original é R$ 8,00. I A FE
R
a) –17,7°; 14°; 20°

S
03. Uma pessoa, pesando atualmente 70 kg, deseja voltar ao peso
TE

normal de 56 kg. Suponha que uma dieta alimentar resulte em um

SO
b) –32°; 42°; 168°
emagrecimento de exatamente 200g por semana. Fazendo essa dieta,
MA

c) 32°; 90°; 100º


a pessoa alcançará seu objetivo ao fim de
d) –18,8°; 50°; 112°

R
a) 67 semanas.
b) 68 semanas. 09. (ESPCEX) Sabendo que a função y = ax + b, pode-se afirmar que:
c) 69 semanas. a) O gráfico da função passa sempre pela origem.
d) 70 semanas. b) O gráfico da função corta sempre o eixo das ordenadas.
e) 71 semanas. b
c) O zero da função é . R
a
MA

04. (EEAR) A função que corresponde ao gráfico a seguir é f(x) = ax + b, d) A função é crescente para a < 0.
SO

em que o valor de a é e) O gráfico da função nunca passa pela origem.

10. (ESPCEX) O crescimento de um vegetal, sob certas condições e a


T

partir de uma determinada altura, segue a função do gráfico abaixo.


R
E

a) 3 IA h(cm)
F
b) 2
c) –2
L DO PRO
3
d) –1
2

05. (EEAR) Hoje, o dobro da idade de Beatriz é a metade da idade de 1


Amanda. Daqui a 2 anos, a idade de Amanda será o dobro da idade
0 5 10 t(dias)
de Beatriz. A idade de Beatriz hoje é _____ ano(s).
a) 1
Mantidas tais condições, pode-se afirmar que a função que representa
b) 2 o crescimento do vegetal e sua altura no 12° dia são, respectivamente:
c) 3 1 12
a) =
h(t) t −5 e h= cm
d) 4 2 15
1 5 12
06. (EPCAR) Para que o número x satisfaça simultaneamente as b) =
h(t) t− e h= cm
3 3 5
desigualdades 3x + 2 < 7 − 2x, 48x ≤ 3x + 10 e 11− 2(x − 3) > 1− 3(x − 5)
é suficiente que 1 17
c) =
h(t) t +1 e h= cm
5 5
a) – 1 < x ≤ 2/9
1 17
b) 2/9 ≤ x < 1 d) h(t) = t +1 e h= cm
4 5
c) –1<x<1
t −5 12
d) – 1 < x < 2/9 e) h(t) = e h= cm
5 15

26 PROMILITARES.COM.BR

MAT123 - FUNÇÃO AFIM.indd 26 01/11/2021 14:21:23


FUNÇÃO AFIM

EXERCÍCIOS DE Com base nisso, pode-se dizer que, quando o número de pessoas

COMBATE
atingiu 78.000, o relógio marcava
a) 15 horas e 30 minutos
b) 15 horas e 45 minutos
c) 16 horas
01. O único valor de “x” que verifica a equação, na incógnita “x”,
(x – 2)2 + (x + 1)·(x – 1) = 2(x + 5)2 – 167, é divisor de: d) 16 horas e 30 minutos

a) 54 e) 16 horas e 45 minutos

b) 12
05. (AFA) Na figura abaixo, tem-se representado as funções f, g e
c) 97 h que indicam os valores pagos, respectivamente, às locadoras de
d) 33 automóveis x ∈ ]m, +∞[, m ∈  para x quilômetros rodados por dia.
e) 75 Uma pessoa pretende alugar um carro e analisa as três opções.
y (valor pago em reais)
g
02. (CMRJ) Considere a função afim f, representada no gráfico abaixo. h
Sabendo-se que A (3, 1); B(0, 1) e que C é o ponto de interseção do
gráfico de f com o eixo das ordenadas, a área do triângulo ABC é, em 70
unidades de área, igual a: 50 f

D O P R20 O
30

L
IA 0 FE 100 x (km rodados por dia)
R Após a análise, essa pessoa conclui que optar pela locadora α ao invés

S
TE

das outras duas locadoras, é mais vantajoso quando x ∈ ]m, +∞[, m ∈ .

SO
a) 10 O menor valor possível para m é
MA

b) 9 a) 60 c) 80
c) 8,5

R
b) 70 d) 90
d) 7,5
e) 6 06. (AFA) Na figura abaixo, tem-se o gráfico da função real f em
que f(x) representa o preço, pago em reais, de x quilogramas de um
03. (EPCAR 3°ANO) A reta do gráfico abaixo indica a quantidade de determinado produto. (Considere f(x) ∈ )
soro (em ml) que uma pessoa deve tomar, em função de seu peso R
(dado em Kgf), num tratamento de imunização. A quantidade total de
MA

soro a ser tomada será dividida em 10 injeções idênticas. Quantos ml


SO

de soro receberá um indivíduo de 65 Kgf em cada aplicação?


T

R
E

IA F
a) 20
b) 40 L DO PR O
c) 2 De acordo com o gráfico, é INCORRETO afirmar que
d) 4 a) o preço pago por 30 quilogramas do produto foi R$ 18,00.
b) com R$ 110,00, foi possível comprar no máximo 55 quilogramas
04. (EPCAR 3° ANO) Em um jogo de futebol amistoso entre Brasil do produto.
e Argentina, no Mineirão, compareceram 90.000 torcedores. Quatro c) com R$ 36,00, foi possível comprar 72 quilogramas do produto.
portões foram abertos às 12 horas, e até as 14 horas entrou um número
constante de pessoas por minuto. Entre 14 horas e 15 horas não d) com R$ 32,00, compra-se tanto 53,333... quilogramas, quanto 64
entrou ninguém. Às 15 horas, abriram mais 4 portões, aumentando o quilogramas do produto.
fluxo de pessoas e, às 17 horas, os portões foram fechados. O gráfico
abaixo indica o número de pessoas dentro do estádio em função do 07. (EFOMM) Uma churrascaria cobra, num almoço, R$ 10,00 por
horário de entrada. pessoa. Após as 15h, esse valor cai para R$ 8,00. Estima-se que o
custo total de um almoço seja de R$ 6,00 por pessoa. Em certo dia, na
churrascaria almoçaram 100 pessoas; x dos quais permaneceram até
as 15h. Assinale a alternativa que representa o intervalo de variação
de x a fim de que 300 < L(lucro) < 400.
a) maior que 100
b) menor que 50
c) entre 50 e 100
d) menor que 50 e maior que 100
e) maior que 50

PROMILITARES.COM.BR 27

MAT123 - FUNÇÃO AFIM.indd 27 01/11/2021 14:21:24


FUNÇÃO AFIM

08. (EFOMM) Todos os anos uma fábrica aumenta a produção em


uma quantidade constante. No 5º ano de funcionamento, ela produziu ANOTAÇÕES
1460 peças, e no 8º ano, 1940. Quantas peças, então, ela produziu no
1º ano de funcionamento?
a) 475 b) 520 c) 598 d) 621 e) 820

09. (EPCAR) O gráfico a seguir é de uma função polinomial do 1º grau


e descreve a velocidade v de um móvel em função do tempo t:

Assim, no instante t = 10 horas o móvel está a uma velocidade de


55 km/h, por exemplo. Sabe-se que é possível determinar a distância
que o móvel percorre calculando a área limitada entre o eixo horizontal
t e a semirreta que representa a velocidade em função do tempo.
L DO PRO
I A
Desta forma, a área hachurada no gráfico fornece a distância, em km,
percorrida pelo móvel do instante 6 a 10 horas. É correto afirmar que
FE
R
a distância percorrida pelo móvel, em km, do instante 3 a 9 horas é de

S
TE

a) 318 b) 306 c) 256 d) 212

SO
10. (EPCAR) João, ao perceber que seu carro apresentara um defeito,
MA

optou por alugar um veículo para cumprir seus compromissos de

R
trabalho. A locadora, então, lhe apresentou duas propostas:
- plano A, no qual é cobrado um valor fixo de R$ 50,00 e mais
R$ 1,60 por quilômetro rodado.
- plano B, no qual é cobrado um valor fixo de R$ 64,00 mais R$ 1,20
por quilômetro rodado.
João observou que, para certo deslocamento que totalizava k R
quilômetros, era indiferente optar pelo plano A ou pelo plano B, pois
MA

o valor final a ser pago seria o mesmo. É correto afirmar que k é um


SO

número racional entre


a) 14,5 e 20 c) 25,5 e 31
T

b) 20 e 25,5 d) 31 e 36,5
R
E

IA F
RESOLUÇÃO EM VÍDEO
Abra o ProApp, leia o QR Code, assista à resolução
L DO PRO
de cada exercício e AVANCE NOS ESTUDOS!

GABARITO
EXERCÍCIOS DE FIXAÇÃO
01. D 04. C 07. C 10. B
02. B 05. C 08. D
03. C 06. B 09. A
EXERCÍCIOS DE TREINAMENTO
01. C 04. C 07. A 10. C
02. B 05. A 08. A
03. D 06. A 09. B
EXERCÍCIOS DE COMBATE
01. E 04. D 07. C 10. D
02. B 05. A 08. E
03. D 06. B 09. A

28 PROMILITARES.COM.BR

MAT123 - FUNÇÃO AFIM.indd 28 01/11/2021 14:21:24


FUNÇÃO QUADRÁTICA

FUNÇÃO QUADRÁTICA PONTO DE INTERSEÇÃO


Definição: Uma aplicação de f de  em  recebe o nome de DA PARÁBOLA COM Oy/y

função quadrática ou função do segundo grau quando associa a cada Quando buscamos a intersecção com o eixo Oy estamos
elemento x ∈  o elemento (ax² + bx + c) ∈ , onde a ≠ 0. Isto é: buscando o ponto da forma (0,f(0)), ou seja, o valor da função f para
f :  → . x = 0. Dessa forma temos f(0) = a · 0² + 0 · x + c, sendo f(0) = c,
x → ax² + bx + c, a ≠ 0 portanto o ponto em que a parábola corta o eixo das ordenadas é o
ponto (0,c).
Exemplos:
a) f(x) = x² – 3x + 2, onde a = 1, b = –3 e c = 2
b) f(x) = 2x³ + 4x – 3, onde a = 2, b = 4 e c = -3
L DO PRO
c) f(x) = x² – 4, onde a = 1, b = 0 e c = –4
I
d) f(x) = 3x² + 5x – 1, onde a = –3, b = 5 e c = –1
A FE
R
O gráfico da função quadrática é uma parábola.

S
TE

Uma parábola é o conjunto de pontos no plano que são

SO
equidistantes de um ponto dado F (foco) e uma reta dada d (diretriz)
que não contém F.
MA

R
CONCAVIDADE
A parábola representativa da função quadrática y = ax² + bx + c
pode ter concavidade voltada para “cima” ou para “baixo”.
Se a > 0, a concavidade está voltada para cima

ZEROS OU RAÍZES DA FUNÇÃO QUADRÁTICA


R
MA

Os zeros ou raízes da função quadrática f(x) = ax² + bx + c são os


SO

valores de x reais tais que f(x) = 0 e, portanto, as soluções da equação


do segundo grau ax² + bx + c.
−b ± b2 − 4ac
T

x=
R 2a
E

IA F
Discussão – Observe que a existência da raiz da equação do

L DO PR O
segundo grau ax² + bx + c fica condicionada ao fato de ∆ ∈ R. Assim
temos 3 possibilidades distintas.
I. ∆ > 0, a equação apresentará duas raízes reais e distintas.
II. ∆ = 0, a equação apresentará duas raízes reais e iguais.
Se a < 0, a concavidade está voltada para baixo. III. ∆ < 0, a equação não apresentará raízes reais.
Cada sinal do determinante irá determinar uma maneira diferente

que a parábola pode ou não interceptar o eixo Ox .

SOMA E PRODUTO DAS RAÍZES


Podemos encontrar a soma e o produto das raízes sem antes
calcular essas raízes pela fórmula de Baskara.
−b + ∆ −b − ∆
=
Sendo x1 = e x2 teremos que
2a 2a
 −b + ∆   −b − ∆  − 2b b
S = x1 + x 2 =  + = = −
 2a   2a  2 a a
   

PROMILITARES.COM.BR 29

PM_BOOK05_MAT.indb 29 01/11/2021 13:40:39


FUNÇÃO QUADRÁTICA

Ponto máximo (a < 0):


( )
2
 −b + ∆   −b − ∆  ( −b ) − ∆
2

=P x=1.x 2   .  =  =
 2a   2a  4a2
b2 − ( ∆ ) b2 − (b2 − 4ac) 4a c c
= = = =
4a2 4a2 4a 2 a
b c
Daí S =
− eP= .
a a

GRÁFICO
Seguem os tipos de gráficos que podemos obter.

D O PVamos
R
ver um exemplo.
Esses foram os gráficos para a > 0, agora veremos paraLa < 0. SabemosOque 2m + n = 8, com m e n números reais. Devemos
I A determinar o parF(m,n) que torna o produto m · n máximo.

R E isolar n em função de m.
Veja que podemos
n = 8 – 2m

S
TE

Dessa forma vamos chamar o produto de P e assim

SO
P = mn = m(8 – 2m) = -2m² + 8m
MA

Acabamos
 de criar uma função
a por P, onde m “fará papel de

R
eixo Ox ” e P “fará papel de eixo Oy ”. Como nossa função possui
a > 0 teremos realmente um ponto máximo.

MÁXIMO E MÍNIMO
 −b −∆ 
Vértice da parábola: O ponto V  ,  é chamado vértice da
R
 2a 4a 
MA

parábola representativa da função quadrática.


SO

Máximo e mínimo: O ponto do vértice da parábola é o ponto


onde a imagem (ordenada eixo y) e o x são mínimos para a > 0 e são
máximos para a < 0.
T

Ponto mínimo (a > 0):


R
E

IA F
L DO PRO

Usando a ideia de x do vértice encontraremos o valor de m que


torna o produto P máximo.
b 8
x v ==
m − = − =
2
2a 2 ( −2)

Assim m sendo igual a 2 teremos n = 8 – 2 · 2 = 4.


Assim o produto máximo será P = mn = 2 · 4 = 8.
Note que não calculamos yV pois não era nosso interesse o produto
máximo mas sim os valores que tornavam máximo esse produto. Se
precisássemos calcular o produto máximo poderíamos ter utilizado

yv = −

=

( 82 − 4.( −2) .0) =

64
=8.
4a 4 ( −2) −8
Note que o valor máximo ou mínimo também pode ser encontrando
fazendo yV = f(xV). Assim yV = f(2) = -2(2²) + 8 · 2 = -2 · 4 + 16 = 8.

30 PROMILITARES.COM.BR

PM_BOOK05_MAT.indb 30 01/11/2021 13:40:40


FUNÇÃO QUADRÁTICA

Eixo de simetria: O gráfico da função quadrática admite um eixo Exemplo:


de simetria perpendicular ao eixo dos x e (paralelo a y) que passa pelo Resolver a inequação x² – 2x + 2 > 0.
vértice.
Considerando f(x) = x² – 2x + 2, temos a = 1 > 0 e ∆ = –4 < 0
entao f(x) > 0 para qualquer x ∈ .

Como a inequação é f(x) > 0 vem S = .


Resolver a inequação x² – 2x + 1 ≤ 0
Considerando f(x) = x² – 2x + 1, temos a = 1 > 0, ∆ = 0, e uma raiz
f ( x ) > 0 ∀x ∈
dupla igual a 1 então  .
DO PRO  f ( x ) 0=
= se x 1

A L
I FE
xv =

b
2a
e yv =


4a R

S
TE

SO
FORMA FATORADA
MA

Se f(x) = ax² + bx + c possui raízes r1 e r2, podemos fatorar

R
f(x) = a(x – r1) (x – r2).
Exemplo:
Seja f(x) = 6x² – 5x – 1. As raízes desta função quadrática Como a inequação é f(x) ≤ 0, temos S = {1}.
1
são 1 e − . Assim, podemos fatorar tal função como
6 Exercício Resolvido
 1
R
6 ( x − 1)  x +  =( x − 1)( 6x + 1) . 01. (UFF 2001) Considere a função f : + →  definida por f (x) =
MA

 6
SO

(3 – x) (x – 1). Identifique a melhor representação do gráfico de f.


INEQUAÇÕES DO SEGUNDO GRAU a) d)
Se a ≠ 0 as inequações ax² + bx + c > 0, ax² + bx + c < 0, ax² + bx + c ≥ 0,
T

ax² + bx + c ≤ 0 são denominadas inequações do segundo grau.


R
E

IA
Resolver por exemplo a inequação ax² + bx + c > 0 é responder a
F
pergunta “existe x ∈ R tal que f(x) = ax² + bx + c seja positiva?”
L R O
A resposta a essa pergunta encontra-se na análise do sinal
que pode, inclusive ser feito pelo gráfico de f(x). DO
de f(x),
P
a>0e∆=0 a>0e∆=0 a>0e∆<0
y y
y b) e)

x1 x2 x x1 = x2 x ∃x∈ x

a<0e∆>0 a<0e∆=0 a<0e∆<0


y y y c)
x1 x2 x1 = x2 ∃x∈
x x x

PROMILITARES.COM.BR 31

PM_BOOK05_MAT.indb 31 01/11/2021 13:40:40


FUNÇÃO QUADRÁTICA

Resolução: D Resolução:
f : ℝ+ → ℝ, f(x) = (3 – x)(x – 1) = –x² + 4x – 3. C = 5 + 10n ⇒ V = – 5n² + 100n – 320, 4 ≤ n ≤ 16.
- Domínio: [0,+∞[
- Concavidade para baixo a) Lucro: L = – 5n² + 100n – 320 – (5 + 10n) = -5n² + 90n – 325.
- Raízes: 1 e 3 Raízes: n = 5 ou n = 13
- Interseção com o eixo Oy: x = 0 ⇒ y = – 3

Exercício Resolvido L > 0 ⇔ 5 < n < 13 ⇒ 6 ≤ n ≤ 12.

02. (UERJ 2005) Numa operação de salvamento marítimo, foi −90


b) O máximo da função L ocorre para= n = 9; n = 9 ⇒
lançado um foguete sinalizador que permaneceu aceso durante L = – 5 × 81 + 90 × 9 – 325 = 80. 2 ( −5)
toda sua trajetória. Considere que a altura h, em metros, alcançada
por este foguete, em relação ao nível do mar, é descrita por
h = 10 + 5t – t², em que t é o tempo, em segundos, após seu Exercício Resolvido
lançamento. A luz emitida pelo foguete é útil apenas a partir de
14 m acima do nível do mar. O intervalo de tempo, em segundos, 05. (UFF 2001) A reta de equação y = –1 é tangente à parábola de
no qual o foguete emite luz útil é igual a: equação y = mx² – 4x + 1. O valor da constante m é:
a) 3
a) –2 c) 0 e) 2
b) 4
b) –1 d) 1

D O Resolução:
P R OE
c) 5
d) 6
A L
Resolução: A I FE
h = 10 + 5t – t² = 14 ⇒ t² – 5t + 14 = 0 R
⇒ t = 1 ou t = 4;
∆t = 4 – 1 = 3.

S
TE

SO
( −4 )
2
− 4 ⋅ m ⋅1
MA

m > 0 e y V =−1 ⇒ − =−1 ⇒16 – 4 m = 4 m ⇒ m = 2.


4m

R
Exercício Resolvido

03. (UNIRIO 2000) Em uma fábrica, o custo de produção de x EXERCÍCIOS DE

FIXAÇÃO
produtos é dado por c(x) = –x² + 22x + 1. Sabendo-se que cada
produto é vendido por R$ 10,00, o número de produtos que
devem ser vendidos para se ter um lucro de R$ 44,00 é:
R
a) 3 c) 12 e) 15
MA

01. (CFT) Uma função real f tem a propriedade f(x + 1) = x² + 2, para


SO

b) 10 d) 13
x ∈ . O valor de f(3) é
Resolução: E a) 3. b) 4. c) 5. d) 6.
T

Custo de x unidades: c(x) = –x² + 22x + 1


S

R 02. (CFT) Considerando-se a função f(x) = ax² + bx + 1/3 , cujos zeros


E

Venda de 1 unidade: R$ 10,00 ⇒ Receita com a venda de x


E

unidades: r(x) = 10x.


IA F
são – 1/3 e 1/3, pode-se afirmar que

L D Ob) aP= 0 R
Lucro na venda de x unidades: L(x) = r(x) – c(x) = 10x + x² – 22x – 1 Oa) a ≠ 0 e b ≠ 0.
e b ≠ 0.
c) a ≠ 0 e b = 0.
d) a = 0 e b = 0
= x² – 12x – 1.
L(x) = 44 = x² – 12x – 1 ⇒ x² – 12x – 45 = 0 ⇒ x = 15 ou x = –3.
03. (CFT) Sejam a função f(x) = 2x² – 5x + 2 e o intervalo A = ]0,2[.
Se x ∈ A, a função f
Exercício Resolvido a) é crescente para x < 1/2 e decrescente para x > 1/2.
b) é sempre crescente.
04. (UENF 2005) Considere as seguintes funções, relativa a uma
ninhada de pássaros: c) tem uma raiz real.

C = 5 + 10n d) tem duas raízes reais.

C = custo mensal, em reais, para a manutenção de n pássaros 04. (CFT) Seja a parábola que representa a função y = kx² – x + 1. Os
valores de k, para os quais essa parábola não intercepta o eixo das
V = -5n² + 100n – 320
abscissas, são tais que
V = valor mensal arrecadado, em reais, com a venda de n a) k > 1/4. c) – 4 < k < 1/4.
pássaros, para 4 ≤ n ≤ 16
b) k > – 4. d) – 1/4 < k < 4.
Sabe-se que o lucro mensal obtido é dado pela diferença entre os
valores de venda V e custo C.
05. A equação do 2º grau, (m + 1)x² – 2mx + (m – 1), de incógnita x,
a) Determine os possíveis valores de n, para que haja lucro nas vendas. 1 1
possui + = 3. Sendo assim o valor de m² é
b) Calcule o valor de n que proporciona o maior lucro possível e x1 x 2
o valor, em reais, desse lucro. a) 1 b) 4 c) 16 d) 25 e) 9

32 PROMILITARES.COM.BR

PM_BOOK05_MAT.indb 32 01/11/2021 13:40:41


FUNÇÃO QUADRÁTICA

06. (EEAR) O vértice da parábola y = –x² + 6x – 5 é o ponto cuja 06. (CFOE) A parábola de equação 2x² + y = bx + c passa pelo ponto
ordenada é (1, 0) e seu vértice é o ponto de coordenadas (3, t). Então t é igual a:
a) – 2 c) 4 a) 8 c) 6
b) – 1 d) 6 b) –4 d) –5

07. Na parábola descrita pela função de  →  f(x) = x² + bx + 1, 07. (CFOE) Uma caixa d’água tem a forma de paralelepípedo reto-
b ∈ , o vértice tem coordenadas (–0,5; –0,75). Então: retângulo, cujas medidas internas são, em m, “x”, “20 – x” e “2”. O
maior volume, em m³, que ela poderá conter é igual a
a) b < –1. c) 0 ≤ b < 1.
a) 150 c) 220
b) –1 ≤ b < 0. d) 1 ≤ b < 2.
b) 200 d) 250
08. (EEAR) As raízes da equação – x² + 7x – 6 = 0 são dois números 08. Em uma P.A. crescente de 10 termos o sexto e oitavo termos são
a) simétricos. raízes da equação x² – 4x – 5 = 0. A diferença entre o maior e o menor
b) naturais pares. termo dessa P.A. é:
c) primos entre si. a) 13 c) 27 e) 20
d) inteiros e múltiplos de 3. b) 14 d) 15

09. Sabendo que x1 = –1 e x2 = 2 são raízes da equação quadrática


09. (EEAR) As dimensões de um retângulo são numericamente iguais
ax² + bx + c = 0, com a > 0 e b² – 4ac > 0, assinale a afirmativa correta.
às coordenadas do vértice da parábola de equação y = − 4x² + 12x − 8.
A área desse retângulo, em unidades de área, é a) 9a + 3b + c < 0 d) 4a – 2b + c < 0
a) 1. b) 1,5. c) 2. d) 2,5.
L D O c) Pa +Rb +Oc < 0
b) c > 0 e) 4a + 2b + c > 0

10. (ESA – ADAPTADA) O lucro de uma empresa é dado


I A(“empormilhares
uma lei FE a² − b²
R
L(x) = –x² + 8x – 7, que que x é a quantidade vendida
de unidades”) e L é o lucro (em milhares de Reais). Qual o valor do
10. Para que a equação
devemos ter:
x² − ax +
4
=
0 tenha raízes iguais,

S
TE

lucro máximo em reais? a) a = 2b d) b = 0

SO
a) 9000 c) 6000 e) 10000 b) a² – b² = 0 e) a = b
MA

b) 7000 d) 8000 c) a=0

R
EXERCÍCIOS DE

TREINAMENTO
EXERCÍCIOS DE

COMBATE
01. (EEAR) Dada a função f:  → , definida por f(x) = −x² + 3x – 2, R
é correto afirmar que 01. (EEAR) Para que a função quadrática y = – x² + 3x + m − 2 admita
MA

SO

a) f(x) ≥ 0, para x ≤ 1 ou x ≥ 2. 3
o valor máximo igual a − , o valor de m deve ser
b) f(x) < 0, para qualquer valor de x. 4
a) –3 c) –1
c) f(x) ≤ 0, para nenhum valor de x.
T

b) –2 d) 0
S

d) f(x) > 0, para 1 < x < 2.


R
E

IA F
02. (EEAR) O conjunto solução da inequação x + 6 ≥ x² é
02. (EEAR) Para que a equação x²+ mx + m² – m – 12 = 0 tenha uma
raiz nula e outra positiva, o valor de m, deve ser
x≤3
O
{x ∈  / ________ }
L D Oa) −P2 ≤ R c) −3≤x≤2
a) – 4. c) 4.
b) − 2 ≤ x ≤ 2 d) − 3 ≤ x ≤ 3
b) – 3. d) 3.
03. (EEAR) A função ƒ(x) = ax² + bx + c, cuja soma das raízes é 2,
03. (EEAR) Para que a função f(x) = (k – 4)x² + kx – (k –2) seja
é representada graficamente por uma parábola com concavidade
quadrática, deve-se ter k ≠ __ .
voltada para cima e que passa pelo ponto (0, –1). Sobre os sinais de a,
a) –2. c) 2. b e c, é correto afirmar que
b) 0. d) 4. a) ab > 0 c) bc > 0
04. (EEAR) O ponto de maior ordenada, pertencente ao gráfico da b) ac > 0 d) abc < 0
função real definida por f(x) = (3 – x)(x + 1), é o par ordenado (m,n).
Então, “m – n” é igual a 04. (EEAR) Seja a função quadrática ƒ(x) = ax² + bx + 1. Se ƒ(1) = 0 e
ƒ(–1) = 6, então o valor de a é
a) –3. c) 5.
a) 5 c) 3
b) 3. d) -5
b) 4 d) 2
05. (EEAR) A fórmula que define a função quadrática, cuja
representação gráfica é uma parábola, cuja concavidade é voltada 05. (EEAR) Considere a inequação x² – 1 ≤ 3 . Está contido no conjunto
para baixo e que não intercepta o eixo das abscissas, é solução dessa inequação o intervalo
a) y = – x2 – 2x – 1 c) y = 3x – 2x2 – 2 a) [–3, 0] c) [1, 3]
b) y = – 5x + x + 7
2
d) y = – 6 – x2 – 5x b) [–1, 1] d) [3, 4]

PROMILITARES.COM.BR 33

PM_BOOK05_MAT.indb 33 01/11/2021 13:40:42


FUNÇÃO QUADRÁTICA

06. (EEAR) Seja a função ƒ(x) = 2x² + 8x + 5. Se P(a, b) é o vértice do


gráfico de ƒ, então |a + b| é igual a ANOTAÇÕES
a) 5 c) 3
b) 4 d) 2

07. (EEAR) Seja a função quadrática ƒ(x) = ax² + bx + c, de raízes reais


distintas x’ e x”. Se a > 0, então se pode afirmar corretamente que no
intervalo ]x’, x”[ a função f é
a) positiva. c) crescente.
b) negativa. d) decrescente.

08. (EEAR) A função ƒ(x) = x² – 2x – 2 tem um valor ________, que


é ______ .
a) mínimo; –5
b) mínimo; –3
c) máximo; 5
d) máximo; 3

09. (AFA) O retângulo, com base no eixo das abcissas, está inscrito
numa parábola, conforme figura abaixo. O valor de x que faz esse
retângulo ter perímetro máximo é
L DO PRO
I A FE
R

S
TE

SO
MA

a) 1
b) 0,5 R
c) 0,25 R
MA

d) 0,125
SO

10. Quantos são os números inteiros não negativos k para os quais a


equação x2 + 6x + k = 0 tem soluções inteiras?
T

a) 2 c) 4 e) 6
R
E

b) 3 d) 5
IA F
RESOLUÇÃO EM VÍDEO L DO PRO
Abra o ProApp, leia o QR Code, assista à resolução
de cada exercício e AVANCE NOS ESTUDOS!

GABARITO
EXERCÍCIOS DE FIXAÇÃO
01. D 04. A 07. D 10. A
02. C 05. E 08. C
03. C 06. C 09. B
EXERCÍCIOS DE TREINAMENTO
01. D 04. A 07. B 10. D
02. B 05. C 08. C
03. D 06. A 09. C
EXERCÍCIOS DE COMBATE
01. C 04. D 07. B 10. C
02. A 05. B 08. B
03. C 06. A 09. B

34 PROMILITARES.COM.BR

PM_BOOK05_MAT.indb 34 01/11/2021 13:40:42


FUNÇÃO MODULAR

DEFINIÇÃO 1) |x| < k ⇔ –k < x < k


Sendo x ∈ , define-se módulo ou valor absoluto de x que se Devemos pensar em todos os números reais que possuem o
indica através da relaçao. módulo menor que um número k, que é o mesmo que pensar em
todos os números que possuem a distância até a origem da reta real
=
| x | x, se x ≥ 0 menor que a distância do númerok. Assim vemos que se trata da parte
 interna entre k e –k.
| x |=− x, se x < 0

1) o módulo de um número real não negativo é igual ao próprio


número.

número.
D O P2)DeR
2) o módulo de um número real negativo é igual ao oposto desse
L
|x| > k ⇔ x < –k ou x > k
O análoga
maneira iremos pensar agora em todos os números
I A F
reais que possuem a distância até a origem da reta real maior do que
Exemplo:
|2| = 2, |–7| = 7, |0| = 0 R o número . E

S
TE

EQUAÇÕES MODULARES

SO
MA

1° Tipo Exemplo 1:
Resolver em , |2x +1| < 3.
x =k⇔x=k ou x =−k

R
| 2x + 1| < 3
2° Tipo − 3 < 2x + 1< 3
− 2 < x <1
x =k ⇔ x =k ou x =−k
S = {x ∈  | − 2 < x <1}

Exemplo 1:
R
MA

Resolver |2x – 1| = 3 Exemplo 2:


SO

2x − 1 = 3 ⇒ x = 2 Resolver em , |4x – 3| > 5.


2x − 1 = 3 ⇒ 
2x − 1 =−3 ⇒ x =−1 |4x – 3| > 5 ⇒ 4x – 3 < –5 ou 4x – 3 > 5
T

S = {2,1} 1
x<− ou x > 2
R
E

2
E

IA  1F 

L DO PRO
Exemplo 2: S= x ∈  / x < − ou x > 2
 2 
Resolver |3x – 1| = |2x + 3|
3x − 1= 2x + 3 ⇒ x = 4

 2 OUTROS TIPOS DE EQUAÇÕES E
3x − 1 =−2x − 3 ⇒ x =− 5 INEQUAÇÕES MODULARES
S = {4,–2/5} Existem outros tipos de equações ou inequações modulares,
geralmente quando há alguma operação entre 2 ou mais módulos
distintos, onde precisaremos decompor cada módulo individualmente
INEQUAÇÕES MODULARES e arrumar em uma tabela para melhor visualização dos intervalos. Mas
Podemos interpretar o módulo de um número como sua distância antes vamos aprender a decompor módulos corretamente.
a origem na retal real. Dessa forma sempre que tivermos um número x, se x ≥ 0
real k haverá o seu simétrico –k que terá a mesma distância a origem Por terem visto a seguinte decomposição x =
da reta real e dessa forma consequentemente o mesmo módulo. − x, se x < 0
muitos copiam e colam para os demais módulos e acabam errando,
por exemplo:
x + 1, se x ≥ 0
x +1 =  , veja que está errado pois o correto é
−x − 1, se x < 0
f(x), se f(x) ≥ 0
o seguinte f(x) =  , assim o que devemos fazer
− f(x), se f(x) < 0
realmente é estudar o sinal da função que está dentro do módulo
para depois decompô-lo.

PROMILITARES.COM.BR 35

PM_BOOK05_MAT.indb 35 01/11/2021 13:40:46


FUNÇÃO MODULAR

Veja: 1
Se −3 ≤ x <
|x + 1| ⇒ f(x) = x + 1, fazendo o zero da função: 2
x + 1 = 0 ⇒ x = –1 −x + 4 = 4 ⇒ x = 0
1
A solução x = 0 está dentro do intervalo −3 ≤ x < .
2
1
Se x ≥
2
2
3x + 2 = 4 ⇒ 3x = 2 ⇒ x =
x + 1, se x ≥ −1 3
Sendo assim o correto x + 1 =
−x − 1, se x < −1 A solução x =
2
é solução para este intervalo.
3
Dessa forma teremos duas soluções S = 0,  .
2
Depois de visto isso vamos ver nossas equações e inequações:
 3
1) |x + 3| + |1 – 2x| = 4
|x + 3| ⇒ f(x) = x + 3, fazendo o zero da função 2) |2x – 4| – |x + 1| ≥ 3
x + 3 = 0 ⇒ x = –3 2x − 4, se x ≥ 2 x + 1, se x ≥ −1
=2x − 4  = e x +1 
4 − 2x, se x < 2 − x − 1, se x < −1

L DO PRO
I A FE
R

S
TE

x + 3, se x ≥ −3
x+3 =

SO
−x − 3, se x < −3
MA

Se x < –1
|1 – 2x| ⇒ g(x) = 1 – 2x, fazendo o zero da função x–5≥3⇒x≥8

R
1− 2x = 0 ⇒ x =
1 Teremos de fazer ]−∞, −1[  [8, ∞[ .
2 ]−∞, −1[  [8, ∞[ = ∅

Se –1 ≤ x < 2
3x – 3 ≥ 3 ⇒ 3x ≥ 6 ⇒ x ≥ 2 R
Teremos de fazer [ −1, 2[  [2, ∞[ .
MA

SO

[ −1, 2[  [2, ∞[ =∅
Se x ≥ 2
T

− x + 5 ≥ 3 ⇒ − x ≥ −2 ⇒ x ≤ 2

R
1
E

1− 2x, se x <


E

1− 2x = 
2
IA F
Teremos de fazer [2, ∞[  ]−∞,2] = {2} .

L DO PRO
2x − 1, se x ≥ 1 Dessa forma dentre todos os intervalos temos apenas um conjunto
 2 unitário como solução, S = {2}.

Assim iremos montar uma tabela


GRÁFICO DA FUNÇÃO MODULAR

A função modular f :  →  possui o gráfico bem simples de
 x → x
ser traçado, veremos como podemos traçar gráficos das diversas
funções modulares, não só de f (x) = |x|.
1) f (x) = |x|
Podemos proceder da seguinte forma:
Se x < –3 1°) traçamos o gráfico da função que está dentro do módulo.
–3x – 2 = 4 ⇒ –3x = 6 ⇒ x = –2 2°) “Rebatemos” a parte negativa do gráfico (se houver) para
cima.
Mas repare que estamos no intervalo x < -3, assim –2 ∉ x < –3,
f(x) = |x| ⇒ f(x) = |g(x)| onde g(x) = x, assim faremos o gráfico de
então ∃ x ∈  .
g (x) = x.

36 PROMILITARES.COM.BR

PM_BOOK05_MAT.indb 36 01/11/2021 13:40:47


FUNÇÃO MODULAR

Por fim

Vamos ver outro exemplo.

2) f (x) = |2x + 1| + 2
Daí podemos dizer que f(x) = |g(x)| + 2 onde g(x) = 2x +1.
Assim traçamos primeiro g(x) = 2x + 1

DO PRO
Verificamos que a simetria da parte negativa do gráfico deve ser
mantida depois de rebatermos para cima.

A L
I FE
R

S
TE

SO
MA

R
R
MA

SO
T

R
E

IA F
Depois modulamos f(x) = |g (x)|
Fazendo f (x) = |g (x)|
L DO PRO

PROMILITARES.COM.BR 37

PM_BOOK05_MAT.indb 37 01/11/2021 13:40:47


FUNÇÃO MODULAR

3) f (x) = |x + 2| – | 1 – x|

x + 2, se x ≥ −2 1− x, se x < 1
x+2  e 1− x 
−x − 2, se x < −2 −1+ x, se x ≥ 1

 −3, se x < −2

Assim teremos f(x)= 2x + 1, se − 2 ≤ x < 1
 3, se x ≥ 1

E agora iremos deslocar 2 unidades para cima:


f(x) = |g(x)| + 2
L DO PRO
I A FE
R

S
TE

SO
MA

R
4) f(x) = ||2x + 1| – 1| R
MA

 1
2x + 1, se x ≥ − 2
SO

2x + 1  , daí teremos
−2x − 1, se x < − 1
 2
T

R 1
E

Se x < − ⇒ f(x) = −2x − 1− 1 = 2x − 2 e


E

IA 2 F
L DO PRO
1
se x ≥ − ⇒ f(x)
= 2x + 1− 1 = 2x
2

Também podemos traçar o gráfico de funções modulares decom-


Se quiséssemos poderíamos ter divido mais, veja:
pondo os módulos e criando assim funções de sentenças abertas.

38 PROMILITARES.COM.BR

PM_BOOK05_MAT.indb 38 01/11/2021 13:40:48


FUNÇÃO MODULAR

2x + 2, se x ≥ −1 2x, se x ≥ 0
2x + 2  e 2x 
 −2x − 2, se x < −1 −2x, se x < 0

−2x − 2, se x < −1

2x + 2, se − 1 ≤ x < − 1
 2
f(x) = 
−2x, se − 1 ≤ x < 0 Agora usamos o auxílio do quadro abaixo onde usamos a regra
 2
2x, se x ≥ 0 dos sinais para a multiplicação.

INEQUAÇÕES SIMULTÂNEAS
A dupla desigualdade f (x) < g (x) < h (x), se decompõem em 2
inequações simultâneas
 f(x) < g(x)

f(x) < g(x) < h(x) ⇔  e
g(x) < h(x)

 1
Exemplo: S= x ∈  | x < −2 ou x > 
 2
 
D O PA inequação
II

3x
 +< − x +3 ≤ x + 4 e 3x + 2 < − x + 3 ≤ x + 4
2
L R O f(x) · g(x) ≥ 0 tem como solução a união das soluções
I

I A FE
de f(x) · g(x) > 0 com as soluções de f(x) · g(x) = 0.
Temos que resolver as 2 inequações
1) 3x + 2 < – x + 3 ⇒ 4x < 1 ⇒ x < 1/4 R f(x).g(x) = 0

f(x).g(x) ≥ 0 ⇔  ou

S
TE

2) – x + 3 ≤ x + 4 ⇒ –2x ≤ 1 ⇒ x ≥ –1/2 f(x).g(x) > 0

SO

A interseção desses dois conjuntos é
MA

Exemplo:

R
Resolver a inequação (3x + 1) · (2x – 5) ≥ 0 em .
Fazemos o estudo de f(x) = (3x + 1) e g(x) = (2x – 5).
Sinal de f(x) = 3x + 1
1
Zero da função: f ( x ) = 0 ⇒ 3x + 1 = 0 ⇒ x = −
3
R
MA

SO

 1 1
S = x ∈  | − ≤ x ≤ 
 2 4
T

R
E

INEQUAÇÕES PRODUTO
E

IA F
Sinal de g(x) = 2x – 5
Sendo f(x) e g (x) duas funções na variável x, as inequações
L D O Zero O
P daRfunção: g(x) = 0 Þ 2x - 5 = 0 Þ x = 2
5
f(x) × g(x) > 0, f(x) × g(x) < 0, f(x) × g(x) ³ 0 e f(x) × g(x) £ 0 são
denominadas inequações produto.
Exemplo:
(x + 2) · (2x – 1) > 0. Um processo prático é fazer o estudo de cada
função separadamente.
Fazemos o estudo de f(x) = (x + 2) e g(x) = (2x + 1).
Sinal de f(x) = x + 2
Zero da função: f(x) = 0 ⇒ x + 2 = 0 ⇒ x = –2
Fazendo o quadro de sinais teremos

Sinal de g(x) = 2x + 1
1
Zero da função: g ( x ) = 0 ⇒ 2x + 1 = 0 ⇒ x = −  1 5
2 S= x ∈  | x ≤ − ou x ≥ 
 3 2

PROMILITARES.COM.BR 39

PM_BOOK05_MAT.indb 39 01/11/2021 13:40:49


FUNÇÃO MODULAR

Dentre as inequações produto são importantes as inequações do


tipo (f(x))n > 0 , (f(x))n < 0 , (f(x))n ≥ 0 e (f(x))n ≤ 0 . Para resolver
essas inequações vamos relembrar as regras de potências:
1) toda potência de base real e expoente ímpar conserva o sinal
da base.
a2n +1 > 0 ⇔ a > 0
a2n +1 = 0 ⇔ a = 0
Sinal de g (x) = 1 – x
a2n +1 < 0 ⇔ a < 0 (n ∈ N)
Zero da função: g(x) = 0 ⇒ 1 – x = 0 ⇒ x = 1
2) toda potência de base real e expoente positivo resulta num
número não negativo.
a2n ≥ 0, ∀a ∈ , ∀n ∈ N
Assim sendo, temos:

f(x) > 0 se n é ímpar


[(f(x))]
n
>0⇔
 f(x) ≠ 0 se n é par Fazendo o quadro de sinais

ïìf(x) < 0 se n é ímpar


[(f(x))] < 0 Û ïí
n

ïïî $
/ x Î  se n é par

f(x) ≥ 0 se n é ímpar L DO PRO


[(f(x))] A FE
n
≥0⇔
∀ x ∈ D(f) se n é par I
R
f(x) ≤ 0 se n é ímpar
[(f(x))]
n

S
TE

≤0⇔  2 
 f(x) = 0 se n é par S= x ∈  | x ≤ − ou x > 1

SO
 5 
MA

Exemplos:

R
 2 Exercício Resolvido
1º ) ( 3x − 2) > 0 ⇒ 3x − 2 > 0 ⇒ S = x ∈  | x > 
3

 3
01. (UEPB) A soma das raízes que a equação modular ||x – 2| – 7| = 6 é
 3 a) 15
2º ) ( 4x − 3) > 0 ⇒ 4x − 3 ≠ 0 ⇒ S = x ∈  | x ≠ 
6

 4 b) 30
c) 4 R
 1
3º ) ( 2x + 1) < 0 ⇒ 2x + 1 < 0 ⇒ S = x ∈  | x < − 
5
MA

2 d) 2

SO

e) 8
4º ) ( x − 2) < 0 ⇒ S =∅
4

Resolução: E
T

Temos ||x – 2| – 7| = 6 ⇔ |x – 2| – 7 = ± 6.
INEQUAÇÕES QUOCIENTER
E

IA F
Logo, |x – 2| = 13 ⇔ x – 2 = ± 13 ⇔ x = 15 ou x = –11
As regras para se resolver as inequações quociente são as mesmas
O
ou |x – 2| = 1 ⇔ x – 2 = ± 1 ⇔ x = 3 ou x = 1.
L D O Portanto,
P Ro resultado é 15 + (–11) + 3 + 1 = 8.
para as inequações produtos, apenas devemos atentar para as
restrições de não se poder deixar zerar o denominador da fração pois
nao existe divisão por zero.
Exemplo:
3x + 4 Exercício Resolvido
Resolver em  a inequação ≤ 2 , temos
1− x 02. (PUCRS) A expressão |x – a| < 16 também pode ser representada
3x + 4 3x + 4 por
≤2⇒ −2≤ 0 ⇒
1− x 1− x
a) x – a < 16
3x + 4 − 2(1− x) 5x + 2
⇒ ≤0⇒ ≤0 b) x + a > 16
1− x 1− x
c) – a – 16 < x < a + 16
 f( x )

5x + 2 d) – 16 + a < x < a + 16
−x
1 e) x – a < – 16 ou x – a >0
g( x )

Sinal de f (x) = 5x + 2 Resolução: D

2 x − a < 16 ⇒ −16 < x − a < 16 ⇒ −16 + a < x < a + 16


Zero da função: f ( x ) = 0 ⇒ 5x + 2 = 0 ⇒ x = −
5

40 PROMILITARES.COM.BR

PM_BOOK05_MAT.indb 40 01/11/2021 13:40:50


FUNÇÃO MODULAR

EXERCÍCIOS DE
Exercício Resolvido

03. (PUCRJ) Considere as funções reais f(x) = x2 + 4x e g(x) = x.


Qual é o maior inteiro para o qual vale a desigualdade f(x) < g(x)?
FIXAÇÃO
a) –3 d) 3
01. (EEAR) Se a e b são dois números reais e a razão de a para b é 0,7,
b) –1 e) 4
pode-se afirmar sempre que
c) 0
a) a >b
Resolução: B b) a>b
Calculando:
c) a <b
x 2 + 4x < x → x 2 + 3x < 0 d) a<b
−3 < x < 0
Logo, a alternativa que se encontra dentro do intervalo é a 02. (EEAR) A quantidade de números inteiros positivos que verificam
x
apresentada no item B. as inequações 3x − 8 < e x + 20 > 10x, ao mesmo tempo, é
2
a) 1.
b) 2.
Exercício Resolvido
c) 3.
04. (IFCE) O conjunto solução S ⊂  da inequação (5x2 – 6x – 8) d) 4.
(2 – 2x) < 0 é
 4  L D O 03.P(EEAR)
RO Dada a inequação 2 – x < 3x + 2 < 4x + 1, o menor valor
a) S =  − ,2 ∪ ]−∞,1[ .
 5  I A F Eé um número múltiplo de
inteiro que a satisfaz

b) =
S ]2, +∞[ ∪  −
4 
,1 .
R a) 3.
b) 2.
 5 

S
TE

c) 7.

SO
 4 
c) S =  − ,2 ∪ ]1, +∞[ . d) 5.
MA

 5 
 4 5

R
d) S =  −∞, −  ∪ ]1,2[ . 04. (EAM) O conjunto solução, nos reais, da inequação >1 é o
 5 intervalo x −1
 4  a) ]5,6[
e) S =  − ,1  ∪ ]2, +∞[ .
 5  b) ]-∞,6[
Resolução: E c)  R
Tem-se que d) ]1,+∞[
MA

e) ]1,6[
SO

 4
(5x 2 − 6x − 8)(2 − 2x) < 0 ⇔  x +  (x − 1)(x − 2) > 0
 5
05. (EEAR) A solução da inequação 2(x + 2) + 5x ≤ 4 (x + 3) é um
4
T

⇔ − < x < 1 ou x > 2. intervalo real. Pode-se afirmar que pertence a esse intervalo o número
S

5
R a) 2.
E

IA b) 3. F
Exercício Resolvido
L D Od) 5.P R O c) 4.

05. (UEPB) Com relação ao número de soluções inteiras da equação


(5 − x 2 )(x 2 − 2)
> 0, podemos garantir que existem: 3x + 1 ≥ 4x − 6
x 2 − 2x + 5 06. (EEAR) A solução do sistema  é
 x+3>0
a) ]−3,7].
a) infinita
b) [−3,7].
b) quatro
c) [−7,3[.
c) três
d) ]−7,3].
d) seis
e) duas 07. (EEAR) Seja f(x) = |x – 3| uma função. A soma dos valores de x para
os quais a função assume o valor 2 é
Resolução: E a) 3 c) 6
(5 − x 2 )(x 2 − 2) b) 4 d) 7
Em primeiro lugar, > 0 é uma inequação.
x 2 − 2x + 5
Como x² – 2x + 5 = (x – 1)² + 4 > 0 para todo x real, a inequação 08. (EEAR) A função modular f(x) = |x – 2| é decrescente para todo x
dada é equivalente a real tal que
a) 0 < x < 4. c) x > 4.
(x − 5)(x + 5)(x − 2)(x + 2) < 0 ⇔ − 5 < x < − 2 ou 2 < x < 5.
b) x > 0. d) x ≤ 2.
Portanto, as únicas soluções inteiras são x = -2 e x = 2.

PROMILITARES.COM.BR 41

PM_BOOK05_MAT.indb 41 01/11/2021 13:40:51


FUNÇÃO MODULAR

09. Observe a inequação modular |3x – 2| = 8 + 2x e identifique a 06. (EEAR) A equação |x|² + |x| – 6 = 0
alternativa que apresenta uma das possíveis raízes. a) só tem uma solução.
a) 6 d) –6 b) tem duas soluções, tais que seu produto é = – 6.
b) –10 e) 8/3 c) tem duas soluções, tais que seu produto é = – 4.
c) 10 d) tem duas soluções, tais que seu produto é igual a 0.

10. (EEAR) Em , o conjunto solução da equação |x – 2| = 2x + 1 é 07. (EEAR) A inequação (x² – 5x + 6)(x – 3) ≥ 0 tem para conjunto
formado por solução
a) dois elementos, sendo um negativo e um nulo.
a) {x ∈  / x ≤ 3} . c) {x ∈  / 2 ≤ x ≤ 3} .
b) dois elementos, sendo um positivo e um nulo.
b) {x ∈  / x ≥ 2}. d) {x ∈  / x ≤ 2 ou x ≥ 3}.
c) somente um elemento, que é positivo.
d) apenas um elemento, que é negativo. 08. (EEAR) A solução da inequação |x – 2| + |x + 4| ≥ 6, em U = , é
o conjunto:

EXERCÍCIOS DE a) { x  x ≥ 6}
S =∈

TREINAMENTO b)
c)
S=
S=
{ x ∈ x ≤ 0 }
{ x ∈ x ≤ 0 e x ≥ 6 }
d) S={ x ∈ x ≤ 0 ou x ≥ 6 }
D O 09.
P(EEAR)
01. (EEAR) A soma das raízes da equação |2x – 3| = x – 1 é
a) 1.
L R Resolvendo a inequação (2x – 6)(4x + 8) ≤ 0, para x ∈ ,
obtemos O
b)
5
. IA a) –2 < x < 3 F
E c) –6 < x < 1
3
10
R b) –2 ≤ x ≤ 3 d) –6 ≤ x ≤ 1
c) .

S
TE

3 10. (EEAR) O menor valor inteiro positivo que pertence ao conjunto-

SO
d) 5. solução da inequação (–3x² + 12)(x² – 6x + 8) < 0 é o
MA

a) 2 c) 4
 2x + 3 ≥ 0

R
02. (EEAR) Resolvendo, em , o sistema de inequações  , b) 3 d) 5
tem-se como solução o conjunto x − 8 < 3x − 5
 3
a) S =∈
x  | 0 ≤ x ou x ≥ 
 2 EXERCÍCIOS DE

b)


3
S = x ∈  | 0 ≤ x ≤ 
2
COMBATE R
MA

SO

 3
c) S= x ∈  | x > − 
 2 01. (EEAR) Sendo S o conjunto-solução da equação em  |3x – 1| =
–3x + 1, pode-se afirmar que
T

 3
S

d) S= x ∈  | x ≥ −  1
 2
R a) ∈S c)
3 1
 , ⊂ S
E

IA
2
F 5 3

O
x 2
03. (EEAR) No conjunto solução da inequação 1 − < 5 , a quantidade 1 2
de números inteiros pares é 3
L DO PR b)
3
∈S d)  ,  ⊂ S
5 7
a) 14.
02. (EEAR) A expressão que completa o conjunto S= {x ∈ / ...........} ,
b) 12. solução das inequações x 2 +1< 2x 2 – 3 ≤ –5x , é
c) 10. 1
a) −2 < x ≤ .
d) 8. 2
1
04. (EEAR) Considere a equação |3x – 6| = x + 2. Com respeito às raízes b) ≤ x <2.
2
dessa equação, podemos afirmar que elas pertencem ao intervalo
c) –3 ≤ x < –2.
a) [1,2].
1
b) ]2,5[. d) x < –2 ou x ≥ .
2
c) ]0,4].
03. (EEAR) O conjunto dos valores reais de x para os quais a expressão
d) ]1,4].
x −1
é estritamente positiva é
x 2 − 10x + 21
05. (EEAR) Seja a inequação |x – 1| ≤ 3. A soma dos números inteiros
que satisfazem essa inequação é a) {x ∈  / x >1}.
a) 8.
b) {x ∈  / x > 3 e x ≠ 7} .
b) 7.
c) {x ∈  / x <1 ou 3 < x < 7}.
c) 5.
d) {x ∈  / x >1, x ≠ 3 e x ≠ 7}.
d) 4.

42 PROMILITARES.COM.BR

PM_BOOK05_MAT.indb 42 01/11/2021 13:40:53


FUNÇÃO MODULAR

x2 − x − 6
04. (ESPCEX) O domínio da função f ( x ) = é
3x − 6
RESOLUÇÃO EM VÍDEO
a) [ −2, 2[ ∪ [3, +∞[ Abra o ProApp, leia o QR Code, assista à resolução
b) [ −2, 0] ∪ ]2, 3] de cada exercício e AVANCE NOS ESTUDOS!

c) [0, 2[ ∪ [3, +∞[


d) ]−∞, −2] ∪ ]2, 3] GABARITO
e) ]−∞, 0] ∪ ]2, 3] EXERCÍCIOS DE FIXAÇÃO
01. D 04. E 07. C 10. C
05. (ESPCEX) O número de elementos do conjunto
 20  02. B 05. A 08. D
A = x ∈  | x − 5 ≤
*
− 4 , é
 x  03. B 06. A 09. C
EXERCÍCIOS DE TREINAMENTO
a) 4 c) 6 e) 10
01. C 04. C 07. B 10. D
b) 5 d) 8
02. C 05. B 08. D
06. (AFA) Se x é variável real, então o campo de definição da função 03. A 06. C 09. B
x +1 EXERCÍCIOS DE COMBATE
f ( x ) = log 2 é o conjunto
x +1 01. D 04. A 07. C 10. B
a) {x ∈  | −1 < x < 1} 02. C 05. B 08. B
b) {x ∈  | −1 < x ≤ 1}
L D O 03.PD R O 06. D 09. B

c) {x ∈  | 0 < x < 1}
I A ANOTAÇÕES FE
d) {x ∈  | 0 ≤ x ≤ 1} R

S
TE

3x 3 + x 2 − 5x + 10

SO
07. (AFA) A solução da inequação 2x 2 – 3x + 8 > ,
x+2
MA

no conjunto dos números reais, é dada pelo intervalo


a) –2 < x < 5 c) –1 < x < 3

R
b) –2 < x < 3 d) –1 < x < 5

08. (ESPCEX) O conjunto solução da inequação x 2 + x + 1 ≤ x 2 + 2x − 3 é


 1 
a) x ∈  | − ≤ x ≤ 2 ou x ≥ 4
 2  R
MA

 1 
SO

b) x ∈  | −2 ≤ x ≤ ou x ≥ 4
 2 
 1 
c) x ∈  | x < − ou 2 < x ≤ 4
T

 2 
R
E

d)
 1 
x ∈  | x ≤ −2 ou ≤ x ≤ 4 IA F
L DO PRO
 2 

09. (ESPCEX) O domínio e a imagem da função f(x) = |2x² – 2x| + 4


são, respectivamente
a)  e [4,5;+∞[ d)  e ]–∞;4,5]
b)  e [4;+∞[ e) + e [4;+∞[
c) + e ]–∞;4]

10. (AFA) O conjunto-solução da inequação |1 + 2x – 3x²| < 5 é


 19 1 + 19 
a) x ∈  | <x< 
 3 3 

 1 − 19 1 + 19 
b) x ∈  | <x< 
 3 3 

 −1 − 19 −1 + 19 
c) x ∈  | <x< 
 3 3 

 1 − 19 1 + 19 
d) x ∈  | x ou x 
 3 3 

PROMILITARES.COM.BR 43

PM_BOOK05_MAT.indb 43 01/11/2021 13:40:54


FUNÇÃO MODULAR

ANOTAÇÕES

L DO PRO
I A FE
R

S
TE

SO
MA

R
R
MA

SO
T

R
E

IA F
L DO PRO

44 PROMILITARES.COM.BR

PM_BOOK05_MAT.indb 44 01/11/2021 13:40:54


FUNÇÃO EXPONENCIAL

POTÊNCIA COM EXPOENTE NATURAL Exemplo:


3
Dado um número real a e um número natural n diferente de zero,
a) 25  5 2³  5 8
chama-se potência de base a e expoente n no número an que é igual
1
ao produto de n fatores iguais a a.  1 1 1
b) 4 2
 1
 
a = a ⋅ a ⋅ a ... a, onde:
n 4 2
4 2

a = base
Dado um número real a (a > 0 e a 1), denomina-se função
n = expoente exponencial de base a uma função f de por f(x) = ax ou y = ax.

Exemplo:
L D O P R FUNÇÃO
O
CRESCENTE

IA FE
4 = 4 · 4 · 4 º 4 = 256
4
y
(–4)3 = (–4) · (–4) · (–4) = –64
R

S
TE

Para n = 1, temos: a1 = a

SO
Exemplo: 61 = 6 y = ax
MA

(a > 1)

R
Propriedades:
Dados a e b reais e m e n naturais, as seguintes propriedades são
válidas:
(0,1)
a) am .an = am + n
am x
b) n am −n para a ≠ 0
=
R
a
MA

c) (ab)m = am .bm
SO

FUNÇÃO DECRESCENTE
m
a  am 
=
d)    m  para b ≠ 0 y
b b 
T

e) (a=) (a=
R
m n n m
) amn
E

IA F
Observação
Para expoentes iguais a zero, convencionou-se que a a =L1, com R O y = ax

a diferente de zero.
0
DO P (0 < a < 1)

POTÊNCIA COM EXPOENTE INTEIRO


(0,1)
NEGATIVO
1 x
=a−m com a ≠ 0
am
Exemplo:
1 EQUAÇÃO EXPONENCIAL
a) 71 
7 A regra básica é tentar igualar as bases pois daí teremos que 2
1 1 potências de mesma base serão iguais quando os expoentes também
b) 32  2 
3 9 forem.
Exemplo 1:
POTÊNCIA COM EXPOENTE RACIONAL 1 7 x −1
= 49
FRACIONÁRIO 7
m
a n = n am

PROMILITARES.COM.BR 45

PM_BOOK05_MAT.indb 45 01/11/2021 13:40:56


FUNÇÃO EXPONENCIAL

 x −1  INEQUAÇÕES EXPONENCIAIS
7−1 =  49 7 
  se a> 1, (fun çã o crescente), ent ão
x −1
1º caso:  ,
ax  ak  x  k
( )
7−1 = ( 7)
2 7 
(mantém a desigualdade)
2x − 2
7 = ( 7)
−1
7

2x − 2 5 se 0  a  1, (fun çã o de crescente), ent ão


=−1 → 2x − 2 =−7 → x =− 2º caso:  ,
7 2  ax  ak  x  k
(inverte a desigualdade)

Exemplo 2: Exemplo 1:
3x −1 − 3x + 3x +1 + 3x + 2 =
306 2x > 128
m+n Como a base (2) é maior que devemos manter a desigualdade
Usando as propriedades de potências a = am .an teremos:
para os expoentes 2x > 27 ⇔ x > 7
3x.3−1 − 3x + 3x.31 + 3x.32 =
306
3x Exemplo 2:
− 3x + 3 ⋅ 3x + 32 ⋅ 3x =306
3 x
 3 27
  ≥
Multiplicando toda a equação por 3, para reduzirmos os 5
  125

D O PComo
denominadores. Podemos colocar 3x em evidência ou fazer uma  3
substituição, como 3x = a
L RO a base   é um número maior que 0 e menor que 1,
5

IA F
a devemos inverter a desigualdade para os expoentes
9a 306 × ( 3)
 3  E 3 
− a + 3a + =
R
3 x 3 x 3
 3  3
a − 3a + 9a + 27a =
306.3 ≥ →
  3
≥    
5 5  5 5

S
TE

= 306 ⋅ 3
34a x≤3

SO
306.3 306 .3
=a = = 9.3
= 27
MA

34 34
Exercício Resolvido

R
Voltando a variável x 01. Conforme dados obtidos pelo IBGE, relativos às taxas de
3x = 27 analfabetismo da população brasileira de 15 anos ou mais, a partir
de 1960, foi possível ajustar uma curva de equação y = 30kx + 10,
3x = 33 ⇒ x = 3
onde k > 0, representada a seguir:
S={x ∈ R / x =3}
R
MA

Exemplo 3:
SO

4 x ℵ2x 2 0
Também usando as propriedades de potência
T

(=a ) (=
m n
a ) n m

R
m.n
a
E

( 2 ) − 2 − 2 =0
2 x x
IA F
( 2 ) − 2 − 2 =0
x 2 x
L DO PRO
Novamente podemos fazer uma substituição, 2x = a
a² ℵa 2 0

Resolvendo a equação do 2º grau


a) Determine o valor de k.
−( −1) ± ( −1)2 − 4.(1).( −2)
a= b) Obtenha as taxas relativas aos anos de 1960 e 2020 (valor
2.1
estimado), usando o gráfico e a equação anterior.
1± 1+ 8 1± 9 1± 3
=a = =
2 2 2 Resolução:
a = 2 ou a = −1 a) (30, 20) ⇒ 20 = 30·k30 + 10 ⇒ 30 · k30 = 10 ⇒
1 1
⇒ k 30 = ⇒k = 30 .
Voltando a variável x 3 3
2x = 2 ⇒ x =1 ou 2x = −1 ⇒ ∃ x ∈ R b) 1960: x = 0 ⇒ y = 30 + 10 = 40%.
S={x ∈ R / x =
1}  1
60
40
2020: x = 60 ⇒ y = 30  30  + 10 = ≡ 13,3% .
 3 3

46 PROMILITARES.COM.BR

PM_BOOK05_MAT.indb 46 01/11/2021 13:40:58


FUNÇÃO EXPONENCIAL

Exercício Resolvido Exercício Resolvido

02. Numa população de bactérias, há P(t) = 109 · 43t bactérias 05. Considere as funções
no instante t medido em horas (ou fração da hora). Sabendo-
se que inicialmente existem 109 bactérias, quantos minutos são f:ℝ→ℝ g:ℝ→ℝ h:ℝ→ℝ
necessários para que se tenha o dobro da população inicial? x→y=x–3 x → y = 2x x→y=|x|
a) 20 d) 15
Determine o conjunto-imagem da função f o g o h.
b) 12 e) 10
c) 30 Resolução:

Resolução: E (
= f g (h ( x )=
fogoh(x) ) |) ) f ( 2| x=
) f ( g (|x = |
) 2| x | − 3 .
P(t) = 10 ·4
9 3t A partir do gráfico de y = 2x, obtém-se o gráfico da função fogoh:
1
⇒ t=
P(t) =2 ⋅ 109 =109 ⋅ 43t ⇒ 2 =26t ⇒ 6t =1 ⇒ h= 10 min .
6

Exercício Resolvido

03. Em um meio de cultura especial, a quantidade de bactérias,

DO PRO
em bilhões, é dada pela função Q definida, para t ≥ 0, por

L
Q(t) = k · 5kt, sendo t o tempo, em minuto, e k uma constante. A

A
quantidade de bactérias, cuja contagem inicia-se com o cálculo
I
de Q(0), torna-se, no quarto minuto, igual a 25 · Q(0). Assinale FE
R
a opção que indica quantos bilhões de bactérias estão presentes
nesse meio de cultura no oitavo minuto.

S
TE

a) 12,5

SO
Im fogoh = [–2, ∞[
b) 25
MA

c) 312,5

R
d) 625
e) 1000

Resolução: C
Q(t) = k · 5kt R
1 EXERCÍCIOS DE
Q(4) = 25 ⋅ Q(0) ⇒ k ⋅ 54k = 25 ⋅ k ⋅ 50 ⇒ 54k = 52 ⇒ k = .

FIXAÇÃO
MA

2
SO

1
1 ⋅8
Q(8) = ⋅ 52 =312,5 bilhões.
2
T

R 01. (EEAR) Na equação 2x+1 + 2-x – x = 3, é verdadeira a afirmativa:


E

Exercício Resolvido
IA F
a) Uma das raízes é 1.

04. A automedicação é considerada um risco, pois, a utilização


L D Oc) OPproduto O
b) A soma das raízes é um número inteiro positivo.
R das raízes é um número inteiro negativo.
desnecessária ou equivocada de um medicamento pode
comprometer a saúde do usuário: substâncias ingeridas difundem- d) O quociente das raízes pode ser zero (0).
se pelos líquidos e tecidos do corpo, exercendo efeito benéfico ou
maléfico. Depois de se administrar determinado medicamento a 02. (EEAR) A soma dos valores de x que verificam a equação
um grupo de indivíduos, verificou-se que a concentração (y) de 52x – 7 ⋅ 5x + 10 = 0 é
certa substância em seus organismos alterava-se em função do
tempo decorrido (t), de acordo com a expressão y = y0 · 2-0,5t, em a) log10.
que y0 é a concentração inicial e t é o tempo em hora. Nessas b) log5 10.
circunstâncias, pode-se afirmar que a concentração da substância c) log2 5 + log5 2.
tornou-se a quarta parte da concentração inicial após:
d) log2 2 + log2 5.
a) 1/4 de hora
b) meia hora 03. (EEAR) Sejam as funções f , g , h e t definidas, respectivamente,
x
c) 1 hora  2
−x
 10 
por f(x) =  , g(x) = ( 2)− x e t(x) =
πx , h(x) =   . Dessas
d) 2 horas  3  3 
e) 4 horas quatro funções, é(são) decrescente(s)
a) todas.
Resolução: E
b) somente três.
y = y0 · 2-0,5t
c) somente duas.
y0
y= = y 0 ⋅ 2−0,5t ⇒ 2−0,5t = 2−2 ⇒ t = 4 h d) somente uma.
4

PROMILITARES.COM.BR 47

PM_BOOK05_MAT.indb 47 01/11/2021 13:40:59


FUNÇÃO EXPONENCIAL

04. (EEAR) A raiz real da equação 25 x


− 24 ⋅ 5 x
= 25 é um número EXERCÍCIOS DE
múltiplo de
a) 7. c) 3. TREINAMENTO
b) 5. d) 2.

− x2
05. (EEAR) Considere que o número de células de um embrião,  1
01. (EEAR) O conjunto solução da inequação   ≥ 2, sendo U = , é
contadas diariamente desde o dia da fecundação do óvulo até o 30º  2
dia de gestação, forma a sequência: 1, 2, 4, 8, 16, ...
a) {x ∈  / x ≤ –1 ou x ≥ 1}.
A função que mostra o número de células, conforme o número de dias
b) [–1, 1].
x, é f : {x ∈ ; 1 ≤ x ≤ 30} → ; f(x) =
c) {0}
a) 2x-1
d) ∅.
b) 2x – 1
e) .
c) 2x – 1
d) x² – 1 02. (EEAR) A população de uma determinada bactéria cresce segundo
a expressão P(x) = 30 ⋅ 2x, em que x representa o tempo em horas.
06. (ESA) A função n(t) = 1000 ⋅ 20 indica o número de bactérias
0,2t
Para que a população atinja 480 bactérias, será necessário um tempo
existentes em um recipiente, em que t é o número de horas decorridas. igual a _____ minutos.
Em quantas horas, após o início do experimento, haverá 16000
a) 120
bactérias?
b) 240
a) 10
b) 50 L D O c) P360R O
IA FE
d) 400
c) 15
d) 30
e) 20
R  2
x

03. (EEAR) Sabe-se que   = 4 x. Dessa forma, x + 2 é igual a

S
TE

 3

SO
a) 5
07. (EEAR) O valor da raiz da equação 2x +1 + 2x −1 =
40 é um número
MA

b) 4
a) inteiro positivo.
c) 3

R
b) irracional.
d) 2
c) inteiro negativo.
d) imaginário puro. x +2
04. (EEAR) Na função f ( x ) = 27 x
, tal que x ≠ 0, o valor de x para que
08. (EEAR) O conjunto-solução da inequação (0,5)x(x-2) < (0,25)x-1,5 é f ( x ) = 36 , é um número
a) {x ∈  | x < 1}. a) divisível por 2
R
MA

{x ∈  | x > 3}. b) divisível por 3


SO

b)
c) divisível por 5
c) {x ∈  |1 < x < 3}.
d) divisível por 7
T

d) {x ∈  | x < 1 ou x > 3}.


S

R
E

e) {x ∈ | x < −1 ou x > −3}.


IA F
05. O valor real que satisfaz a equação 4 x – 2x – 2 = 0 é um número

> (0,8) L são b) entre R


O
a) entre –2 e 2
09. (EEAR) Os valores de x para os quais (0,8) ˘ 2− 3( x +1)
P
D Oc) maior que 4
2e4

3 1
a) − <x< d) menor que –2
2 2
1 3 x
b) − <x<  1
2 2 06. (EEAR) A raiz da equação 2x + 2 =   é um número
 2
3 1 a) inteiro positivo.
c) x< − ou x >
2 2 b) inteiro negativo.
1 3 c) irracional.
d) x < − ou x >
2 2 d) nulo.

07. (EEAR) Seja a função g(x) = 3 ⋅ 2x. Se P(0, a) é um ponto do gráfico


10. (EEAR) Se ( 0,0625)x + 2 = 0,25 , então ( x + 1) vale
6
de g, então o valor de a é ___.
3 a) 2
a) − c) 64
2 b) 3
1 1 c) 5
b) d)
32 64 d) 6

48 PROMILITARES.COM.BR

PM_BOOK05_MAT.indb 48 01/11/2021 13:41:01


FUNÇÃO EXPONENCIAL

( )
x
08. (EEAR) No conjunto dos números reais, a equação 3x = 98 tem 05. (ESA) O conjunto solução da equação exponencial 4 x − 2x =
56 é
por raízes
a) {–7,8}
a) um número positivo e um negativo.
b) {3,8}
b) um número negativo e o zero.
c) {3}
c) dois números negativos.
d) {2,3}
d) dois números positivos.
e) {8}

09. (EEAR) Seja uma função real definida por f ( x ) = ( x +1) ⋅ m x −1


. Se 06. (AFA) A cada ano que passa, o valor de uma máquina diminui
f(2) = 6, então m é igual a 10% em relação ao do valor do ano anterior. Se V for o valor da
a) 4. máquina no ano da compra, após 10 anos será
b) 3. a) (0,9)10 V
c) 2. b) (0,5)9 V
d) 1. c) (0,1)9 V
d) (0,1)10 V
2x +1 5
10. (EEAR) O conjunto solução da inequação 2 < ⋅ 2x + 2 − 2 é
4 07. (AFA) A equação 2x + 1 + 4x = 80 para x real:
a) S = {x ∈  | -1/2 < x < 2}
b) S = {x ∈  | -1 < x < 1} a) tem duas soluções

c) S = {x ∈  | 0 < x < 1} b) tem uma única solução

d) S = {x ∈  | x > 1}
L D O c)d) Padmite
RO
não admite solução

I A FE
x = 4 como solução

EXERCÍCIOS DE R 08. (ESPCEX) Se f ( x ) = 5 , com x ∈ , o valor de f ( x + 2) – f ( x + 1) é:


x

COMBATE
a) 30 ⋅ f(x)

S
TE

SO
b) 24 ⋅ f(x)
MA

c) 20 ⋅ f(x)
d) 9 ⋅ f(x)

R
3x − 5 x
 1  1 e) 5 ⋅ f(x)
01. (EEAR) A desigualdade   >  tem como conjunto
solução  2 4
09. (ESPCEX) A soma e o produto das raízes da equação
a) S = {x ∈  | x > 1}
x2 − x − 9
b) S = {x ∈  | x < 5}  3 243
9  = são, respectivamente
c) S = {x ∈  | x > 5} 5 125 R
a) 1 e – 12
MA

d) S = {x ∈  | 1 < x < 5}
SO

b) 7 e 12
4 c) –2e–8
02. (EEAR) Se f ( x=) ax + b é uma função tal que f ( 0 ) = e f ( −1) =
1
então o valor de “a” é 3 d) – 1 e 12
T

a) 1
R e) 7 e 10
E

b) 2
IA F
L DO PRO
1
c) 1/2 10. (ESPCEX) O domínio da função f(x) = é
−x −2 1
d) 3/2 3 −
9
a) *−
03. (ESA) Se 5x + 2 = 100, então 52x é igual a:
b) −
a) 4
c) +
b) 8
d) *+
c) 10
e) 
d) 16
e) 100
RESOLUÇÃO EM VÍDEO
04 (ESA) Quantos algarismos são necessários para escrever o produto Abra o ProApp, leia o QR Code, assista à resolução
(16)13 ⋅ (25)25 ? de cada exercício e AVANCE NOS ESTUDOS!
a) 50
b) 51
c) 54
d) 52
e) 53

PROMILITARES.COM.BR 49

PM_BOOK05_MAT.indb 49 01/11/2021 13:41:02


FUNÇÃO EXPONENCIAL

GABARITO
EXERCÍCIOS DE FIXAÇÃO
01. D 04. D 07. A 10. D
02. B 05. A 08. D
03. D 06. E 09. B
EXERCÍCIOS DE TREINAMENTO
01. A 04. A 07. B 10. B
02. B 05. A 08. A
03. D 06. B 09. C
EXERCÍCIOS DE COMBATE
01. B 04. B 07. B 10. A
02. D 05. C 08. C
03. D 06. A 09. A

ANOTAÇÕES

L DO PRO
I A FE
R

S
TE

SO
MA

R
R
MA

SO
T

R
E

IA F
L DO PRO

50 PROMILITARES.COM.BR

PM_BOOK05_MAT.indb 50 01/11/2021 13:41:02


FUNÇÃO LOGARÍTMICA

DEFINIÇÃO b) Definimos o logaritmo de um quociente através da diferença


entre o logaritmo do numerador e do denominador.
Em uma base a, onde a > 0 e a ≠ 1, o logaritmo de N é α.
O logaritmo de um número é definido como o expoente que é  M
loga=
  log a M − lo g a N
elevado para que potência obtida seja igual a N. N
Veja, como podemos simbolizar:
logaN = α ⇔ aα = N c) Definimos o logaritmo de uma potência através do expoente
Logaritmo é um número real localizado na base que é elevado multiplicado pelo logaritmo da base da potência.
para obter o logaritmando ou o antilogaritmo (N = antiloga α = aα),
( )
D O PR
que nesse caso é representado por N. a é a base e α é o logaritmo.
loga N m = m ⋅ loga N, ∀m ∈ 

O logaritmo α existe se:


A L O
N>0
I Fnirmos
d) Para defi
Eraizomultiplicado
logaritmo de uma raiz basta saber o inverso
a>0
a≠1 R do índice da pelo logaritmo do radicando.

( N) =
1

S
TE

O cologaritmo de N na base a é o oposto do logaritmo de N na loga m


⋅ log N, m ∈  *
a

SO
base a. m
MA

cologaN = –logaN
e) Quando a base de um logaritmo estiver elevada a um
Antilogaritmo de N na base α é definido por

R
expoente faremos
antilogαN = x ⇔ αN = x 1
=
log am N ⋅ logaN
( ) m
Exemplos:
a) log216 = 4, pois 24 = 16
b) log5 25 = 2, pois 52 = 25 MUDANÇA DE BASE
R
−4
MA

 1 Para definirmos o logaritmo de um número N (N > 0), numa base


SO

c) log1/3 81 = −4, pois   =


81
 3 a (a > 0 e a ≠ 1) basta saber o quociente entre o logaritmo de N e o
d) log4 1 = 0, pois 40 = 1 logaritmo de a, com os dois na base b, onde com b > 0 e b ≠ 1.
Para trocar as bases, devemos seguir a regra abaixo:
T

e) antilog7 2 = 72 = 49
S

R
E

logbN
E

f) antilog2 6 = 26 = 64
IA F loga N =
logba
g) colog24 = –log24 = –2
h) colog3 27 = – log3 27 = –3
L DO PRO
FUNÇÃO LOGARÍTMICA
A função logarítmica de base a, com a > 0 e a ≠ 1, é a função
CONSEQUÊNCIA DA DEFINIÇÃO f : ∗+ → :
a) loga1 = 0, pois a0 = 1 Determinada por:
f(x) = logax
b) logaa = 1, pois a = a1

c) loga (aa) = a, pois aa = aa 1° caso: α > 1 2° caso: 0 < α < 1


(função crescente) (função decrescente)
d) a logaN
= N pois loga N = k, então a = N e daí a
k logaN
=a =N
k

PROPRIEDADES DOS LOGARITMOS


Para: M > 0, N > 0, a > 0 e a ≠ 1.
a) Definimos o logaritmo de um produto através da soma dos
logaritmos dos fatores.
Simbolicamente:
loga (M · N) = logaM + logaN

PROMILITARES.COM.BR 51

PM_BOOK05_MAT.indb 51 01/11/2021 13:41:03


FUNÇÃO LOGARÍTMICA

FUNÇÃO LOGARÍTMICA VERSUS INEQUAÇÕES LOGARÍTMICAS


FUNÇÃO EXPONENCIAL
1º CASO:
 Se a > 1, então

(x ) loga g (x ) ⇒ f (x ) > g (x ) > 0
loga f =

(mantém a desigualdade)

2º CASO:
 Se 0 < a < 1, então

log
 a =f ( x ) loga g ( x ) ⇒ 0 < f ( x ) < g ( x )

(inverte a desigualdade)
Exemplo:
LOGARITMO NEPERIANO (lnx) 1) log2(2x – 1) < log26
O logaritmo neperiano é o logaritmo de base e, um número
Como a base do logaritmo é maior que 1, devemos manter a
irracional aproximadamente igual a 2,71828. É definido para todos os
desigualdade
números reais estritamente positivos.
log2(2x – 1) < log26 ⇔ 2x – 1< 6 e 2x – 1 > 0

D O P01 <<R2x2x O
g(x) = lnx é a função inversa da função exponencial f(x) = ex. Fazer a
–2<6
L
opção lnx é equivalente a logex, só que só veremos escrito da forma lnx.

EQUAÇÕES LOGARÍTMICAS I
A 1
<7
7
FE
R 2
<x<
2

S
TE

1º TIPO:  1 7
S = x ∈R / < x < 

SO
Se a > 0; a ≠ 0 e α ∈   2 2
MA

logax = α → x = aα
Exemplo: Exercício Resolvido

R
1) log2(3x + 1) = 4
01. (UFF 2000) A figura representa o gráfico da função f definida
Usando a definição de logaritmo: por f(x) = log2 x. A medida do segmento PQ é igual a:
24 = 3x + 1
y
3x + 1 = 16
3x = 15 ⇒ x = 5
R Q
MA

SO

2º TIPO: P
logax = logaN ⇒ x = N
T

Exemplo:
S

R
E

2) log2(3x – 5) = log2 7
E

Igualando os logaritmandos: IA 0 F 2 4 x
3x – 5 = 7 ⇒ 3x = 12 ⇒ x = 4
L DO PRO
3º TIPO:
incógnita auxiliar
Exemplo:
3) (log2x)² – log2x = 2 a) 6
Fazendo log2x = a teremos: b) 5
a² – a = 2 c) log2 5
a² – a – 2 = 0 d) 2
Resolvendo a equação do 2º grau: e) log 2
2
−( −1) ± ( −1) − 4(1)( −2) 1± 1+ 8 1± 9 1± 3
=a = = =
2 ⋅1 2 2 2 Resolução: B
f(2) = log22 = 1 ⇒ P(2,1)
a = 2 ou a = –1
f(4) = log24 = 2 ⇒ Q(4,2)
Voltando a variável x:
log2x = 2 ⇔ 2² = x ⇒ x = 4 PQ= ( 4 − 2) 2 + ( 2 − 1) 2 = 5
ou
1
log2 x =−1 ⇔ 2−1 =x ⇔ x =
2

52 PROMILITARES.COM.BR

PM_BOOK05_MAT.indb 52 01/11/2021 13:41:04


FUNÇÃO LOGARÍTMICA

Exercício Resolvido EXERCÍCIOS DE

02. (UERJ 2004) O número, em centenas de indivíduos, de um


determinado grupo de animais, x dias após a liberação de um
FIXAÇÃO
predador no seu ambiente, é expresso pela seguinte função:

f(x) = log (x 4 ) 01. (EEAR) A equação log2 (9x −1 + 7) =2 + log2 (3x −1 + 1) possui
53 5
a) duas raízes positivas.
Após cinco dias da liberação do predador, o número de indivíduos b) duas raízes negativas.
desse grupo presentes no ambiente será igual a:
c) duas raízes simétricas.
a) 3
d) uma única raiz.
b) 4
c) 300 02. (EEAR) Se log 2,36 = 0,3729, então antilog 3,3729 é
d) 400 a) 236.
b) 23,6.
Resolução: C c) 2360.
4y d) 23600.
(
f(5) =log53 5 (54 ) =y ⇒ 5 3 5 )
y
=54 ⇒ 5 3
=54 ⇒
4y 03. (EEAR) Se log3 2 = a e log7 3 = b, então log3 14 =
= 4 ⇒ y = 3 centenas.
3
D O a) Pb a+R1
A L O
Exercício Resolvido I b)
a +1 FE
R
03. (UFF 2000) São dados os números reais positivos a, b e x tais
b

c)
ab + 1

S
TE

que a ≠ 1 e b ≠ 1. Sabe-se que logax = 2 e logbx = 4. b

SO
ab + 1
d)
MA

Calcule logab a x . a

R
04. (EEAR) O menor número inteiro que satisfaz a inequação
Resolução:
log2 (3x – 5) > 3 é um número
logax = 2 e logbx = 4.
a) par negativo.
1
loga a x loga a + 2 loga x 1+ 1 b) par positivo.
=
logab a x = = ; c) ímpar negativo.
loga ab loga a + loga b 1+ loga b R
d) ímpar positivo.
MA

loga x loga x 1
logb x = ⇒ loga b = = .
SO

loga b logb x 2 3
05. (EEAR) O logaritmo de 8 é , se a base do logaritmo for igual a
2 4 4
Então, logab =
a x = . a) 4.
T

1 3
S

1+
2
R b) 8.
E

IA c) 16. F
Exercício Resolvido L DO PR d) 64. O
04. (UERJ 2003) O logaritmo decimal do número positivo 06. (EEAR) Sendo a > 0 e a ≠ 1, o conjunto solução da equação
x é representado por log x. Então, a soma das raízes de 10loga ( x² − 3x + 2) = 6loga 10 está contido no conjunto
log² x – logx² = 0 é igual a: a) {1, 2, 3, 4}.
a) 1 b) {–4, –3, –2, –1, 0, 1}.
b) 101 c) {–1, 0, 1, 2, 3, 4}.
c) 1000 d) {0, 1, 2, 3, 4}.
d) 1001
07. (EEAR) Se log 8 = a, então log 3 2 vale
Resolução: B a) a/2.
log²x – logx² = 0 b) a/4.
log² x – 2 · log x = 0 c) a/9.
logx(logx – 2) = 0 d) a/6.
logx = 0 ou logx = 2.
logx = 0 ⇒ x = 100 = 1
logx = 2 ⇒ x = 10² = 100
Soma das raízes = 101.

PROMILITARES.COM.BR 53

PM_BOOK05_MAT.indb 53 01/11/2021 13:41:04


FUNÇÃO LOGARÍTMICA

08. (EEAR) Estudando um grupo de crianças de uma determinada Se a = log2 1024 e x0 = a – 6, então o valor da função no ponto x0 é
cidade, um pediatra concluiu que suas estaturas variavam segundo dado por
=
a fórmula ( )
h log 100,7 ⋅ i , onde h é a estatura (em metros), e i é a a) 2/3
idade (em anos). Assim, segundo a fórmula, a estatura de uma criança b) 3/2
de 10 anos dessa cidade é, em m,
c) 2
a) 1,20.
d) 3
b) 1,18.
e) 1
c) 1,17.
d) 1,15. 04. (EEAR) Se o logaritmo de um número na base “n” é 4 e na base
“n/2” é 8, então esse número está no intervalo
09. (EEAR) Sejam m, n e b números reais positivos, com b ≠ 1. Se logb a) [1,50]
m = x e se logb n = y, então logb (m ⋅ n) + logb   é igual a
n
b) [51,100]
m
a) x c) [101,200]
b) 2y d) [201,500]
c) x+y
d) 2x – y 05. Das sentenças abaixo, quantas são verdadeiras de modo que são
satisfeitas por qualquer número real “x”?
(x − 4)
2
10. (EEAR) Se log 2 ≅ 0,3 e log 36 ≅ 1,6, então log 3 ≅ _____. I. =x 2 − 16

D O III. P 1R >O


a) 0,4 II. 8 = 2 ⋅ 4x
x

L
x x
b) 0,5  1

I A  2   3 F
 

1) E
c) 0,6
IV. log 3 ( x + = log 3 + log ( x + 1)
R
2 2
d) 0,7 2 2 2

S
TE

a) 1

SO
EXERCÍCIOS DE b) 2

TREINAMENTO
MA

c) 3
d) 4

R
e) nenhuma

01. (CFOE) O domínio da função f(x) = ln (x² + x + 1) é o conjunto


06. (EEAR) A curva da figura representa o gráfico da função y = loga x,
dos números:
(a > 1). Dos pontos B(3,0) e C(9,0) saem perpendiculares ao eixo das
a) reais positivos abscissas, as quais interceptam a curva em D e E, respectivamente. Se
a área do trapézio retângulo BCED vale 9, a área do triângulo ABD,
b) negativos
R
MA

c) reais entre -1 e 1 onde A(1,0) vale


SO

d) reais
1
a) .
02. (EEAR) Na figura abaixo, a curva representa o gráfico da função 2
T

b) 2.
R
y = log x, para x > 0. Assim, a soma das áreas das regiões hachuradas
E

é igual a
IA c)
3
. F
L DO PRO
2
d) 1.
e) 0.

07. (EEAR) Todo número real positivo pode ser escrito na forma 10x.
Tendo em vista que 8 ≅ 100,90, então o expoente x, tal que 125 = 10x,
vale aproximadamente,
a) log 2 a) 1,90.
b) log 3 b) 2,10.
c) log 4 c) 2,30.
d) log 6 d) 2,50.
e) 2,90.
03. (EEAR) Considere a função f :  →  definida por
 08. (EEAR) Sejam a, b e c números reais positivos, com b ≠ 1. Se
2x − 1, se x ≤1
 a2b
=
f(x) 0, se 1< x ≤ 3 logb a = 1,42 e logb c = -0,16 o valor de logb é
c
 x−2 a) 3
 , se x > 3
 2x − 5 b) 4
c) 5
d) 6

54 PROMILITARES.COM.BR

PM_BOOK05_MAT.indb 54 01/11/2021 13:41:05


FUNÇÃO LOGARÍTMICA

 64  06. (EEAR) Para que exista a função f(x) = log(x – m), é necessário
09. (EEAR) O valor de log3 1 + log 3    é
   27  que x seja
4
a) 3/4
b) 9/4 a) maior que m.

c) 0 b) menor que m.

d) –3 c) maior ou igual a m.
d) menor ou igual a m.
10. (ESA) Mudando para base 3 o log5 7, obtemos:
log5 3 07. (EEAR) Sejam as funções logarítmicas f(x) = loga x e g(x) = logb x.
a) Se f(x) é crescente e g(x) é decrescente, então
log7 3
a) a > 1 e b > 1.
b) log3 7
b) a > 1 e 0 < b < 1.
log7 3 c) 0 < a < 1 e b > 1.
c)
log5 3
d) 0 < a < 1 e 0 < b < 1.
d) log3 5
log3 7 08. (EEAR) A razão entre o logaritmo de 16 e o de 4, numa mesma
e) base b, sendo 0 < b ≠ 1, é
log3 5
a) 1/4.
b) 1/2.

D O d) P2. R O
c) 4.
EXERCÍCIOS DE
L
COMBATE I A F
09. (EEAR) Se log xE+ log y = k, então logx + logy é 5 5

R a) 10k

S
TE

01. (EEAR) As funções logarítmicas f(x) = log0,4 x e g(x) = log4 x são, b) k10

SO
respectivamente, c) 5k
MA

a) crescente e crescente d) k5
b) crescente e decrescente

R
c) decrescente e crescente 10. (EEAR) Considerando n > 1, se loga n = n, então o valor de a é
d) decrescente e decrescente a) n.
b) nn.
02. (ESA) Utilizando os valores aproximados log2 = 0,30 e log3 = 0,48, 1
c) .
encontramos para log 3 12 o valor de: n R
a) 0,33
MA

1
d) nn .
SO

b) 0,36
c) 0,35
d) 0,31
T

RESOLUÇÃO EM VÍDEO
e) 0,32
R
E

IA F
Abra o ProApp, leia o QR Code, assista à resolução

L DO PRO
de cada exercício e AVANCE NOS ESTUDOS!
03. (EEAR) A soma dos logaritmos de dois números positivos, na base
1
9, é . O produto desses números é
2
a) 3. c) 8. GABARITO
b) 4. d) 9.
EXERCÍCIOS DE FIXAÇÃO
01. A 04. D 07. C 10. B
04. (EEAR) Seja a função g :  ⇒  , definida por g(x) = log2 x. O valor
*
+
de x para o qual g(x) = 3 é 02. C 05. C 08. A
a) 6. 03. C 06. C 09. B
b) 7. EXERCÍCIOS DE TREINAMENTO
c) 8. 01. D 04. D 07. B 10. E
d) 9. 02. A 05. B 08. B
03. A 06. D 09. D
05. (EEAR) Se log2 = a e log3 = b, então a solução da equação EXERCÍCIOS DE COMBATE
10x = 60 é
01. C 04. C 07. B 10. D
a) 2a + b
02. B 05. B 08. D
b) a + b + 1
03. A 06. A 09. C
c) a + 2b
d) 2a + 2b + 1

PROMILITARES.COM.BR 55

PM_BOOK05_MAT.indb 55 01/11/2021 13:41:06


FUNÇÃO LOGARÍTMICA

ANOTAÇÕES

L DO PRO
I A FE
R

S
TE

SO
MA

R
R
MA

SO
T

R
E

IA F
L DO PRO

56 PROMILITARES.COM.BR

PM_BOOK05_MAT.indb 56 01/11/2021 13:41:06


ANÁLISE COMBINATÓRIA: CONTAGEM,
PERMUTAÇÃO E ARRANJO

PRINCÍPIO ADITIVO
Se um evento E1 possui α resultados distintos e um segundo
evento E2 possui β resultados distintos, sendo E1 e E2 conjuntos
disjuntos, então o número de resultados para que ocorra o evento E1
ou o evento E2 é dado pela soma α + β.
Podemos também dizer que há α maneiras de ocorrer o evento
E1 ou β maneiras de ocorrer o evento E2. Assim a maneira de ocorrer
E1 ou E2 é α + β.

DO PRO
Perceba da noção de conjuntos o uso da conjunção “ou” que nos

L
remete a operação de união.

I A FE
Em uma cantina de uma escola há 4 salgados R
Exemplo:
diferentes disponíveis FATORIAL

S
TE

e 3 sabores de sucos distintos. De quantas maneiras uma pessoa pode Dado um número natural n ≥ 2, representa-se e define-se o
se servir de um salgado ou um suco?

SO
fatorial de n por n! = 1 ⋅ 2 ⋅ 3 ⋅ … ⋅ (n – 1) ⋅ n .
MA

Perceba que por mais que se tenha vontade de multiplicar 4 ⋅ 3 só Casos particulares: 1! = 1 e 0! = 1.
temos um “espaço” (queremos salgado ou suco). Perceba também que

R
salgado e suco são coisas totalmente disjuntas (distintas), precisamos
de um dentre os 7 disponíveis (4 + 3). Isso caracteriza o princípio
aditivo. PERMUTAÇÕES SIMPLES
Uma permutação simples de n objetos distintos é uma das
PRINCÍPIO MULTIPLICATIVO sequências ordenadas de n elementos em que os objetos podem ser
Se um evento E1 possui α resultados distintos e um segundo colocados. R
evento E2, independente de E1, possui β resultados distintos, então Notação: Pn (número de permutações simples de n elementos
MA

SO

o número de resultados para o par de eventos (E1, E2) é dado pelo distintos).
produto α ⋅ β. Propriedade: Pn = n!
Podemos também dizer que há α maneiras de ocorrer o evento E1
T

e β maneiras de ocorrer o evento E2. Assim a maneira de ocorrer E1


S

e E2 é α ⋅ β.
R Exemplo
E

I AL
Perceba da noção de conjuntos o uso da conjunção “e” que nos Fque cada pessoa é um objeto então também
De quantas maneiras distintas 5 pessoas podem formar uma fila?
O
Se pensarmos
PR
remete a operação de intersecção.
D Oteremos
5 espaços para distribuir esses objetos. Daí usando o princípio
Exemplo:
multiplicativo teremos que a 1ª posição pode ser ocupada por
Uma pessoa tem no guarda-roupas: 3 camisetas, 2 bermudas e qualquer um dos 5 objetos. Ocupada a 1ª posição a 2ª posição poderá
dois pares de tênis. De quantas maneiras diferentes essa pessoa pode ser ocupada com qualquer um dos 4 objetos restantes ... E assim por
se vestir? diante até que sobra um objeto para o ultimo espaço.
A pessoa pode usar qualquer uma das três camisetas E dois tipos
de bermudas diferentes E dois dos pares de tênis. Como pode usar
5⋅4⋅3⋅2⋅1
uma coisa E a outra devemos usar o princípio multiplicativo, daí
3 ⋅ 2 ⋅ 2 = 12
Como essa operação de se multiplicar sequencialmente de um
Perceba que uma escolha influencia no caso geral, bermuda azul
valor n até se chegar ao valor 1 (uma possibilidade), essa operação foi
com camiseta branca é diferente de bermuda vermelha com camiseta
definida como n!. Então a permutação de n objetos é definida como
branca, por exemplo. Veja que também cabe a montagem da chamada
n!. Daí
árvore de possibilidades.
5 ⋅ 4 ⋅ 3 ⋅ 2 ⋅ 1 = P5 = 5! = 120

PROMILITARES.COM.BR 57

PM_BOOK05_MAT.indb 57 01/11/2021 13:41:07


ANÁLISE COMBINATÓRIA: CONTAGEM, PERMUTAÇÃO E ARRANJO

PERMUTAÇÃO COM REPETIÇÃO Utilizando o princípio multiplicativo teremos o seguinte: a 1ª


posição pode ser ocupada por qualquer um dos 10 atletas, escolhido
Quando precisamos permutar n objetos em n espaços porém, o campeão teremos 9 opções para o vice campeão, por fim, escolhido
existem objetos repetidos, a permutação se dará de maneira diferente. o vice nos sobram 8 possibilidades para a 3ª posição. Como acabaram
Exemplo: Quantos anagramas possui a palavra BANANA? os espaços acabou também a multiplicação de possibilidades.
(Anagrama é a permutação das letras formando-se outra palavra,
10 ⋅ 9 ⋅ 8 = 720
com sentido ou não).
Como BANANA possui 6 letras (objetos) basta distribuirmos essas O princípio multiplicativo se torna uma maneira mais simples de
letras dentro dos 6 espaços disponíveis. Daí teríamos uma permutação se resolver questões de arranjo sem a necessidade de se recorrer a
de 6 objetos. fórmulas e haver confusão com a combinação. Arranjo é o tipo de
P6 = 6! = 6 ⋅ 5 ⋅ 4 ⋅ 3 ⋅ 2 ⋅ 1 = 720 distribuição em que a ordem dos objetos importa.
Porém vejamos uma das possíveis permutações. Exercício Resolvido

01. Apenas com os algarismos 2, 4, 5, 6 ou 9, foram escritos


N1 B A1 N2 A2 A3
todos os números possíveis com cinco algarismos. Cada um
desses números foi registrado em um único cartão, como está
Perceba que numerando as letras que se repetem podemos exemplificado a seguir.
ver mais facilmente que se trocarmos de posições as letras N1 com
N2 ou as letras A1 com A2 ou com A3, não mudaríamos a palavra,
continuaria sendo NBABAA. Então da forma que está teremos
contado palavras mais de uma vez, daí precisamos descontar. As 2
letras N podem permutar entre si de 2! Maneiras, enquanto as 3 letras
A podem permutar entre si de 3! Maneiras. Então para fazer o acerto L D O Alguns
P Rdesses
O
cartões podem ser lidos de duas maneiras, como é o

A
caso dos cartões C, D e E. Observe:
I
iremos dividir as 720 maneiras pela quantidade de vezes que as letras F E
repetidas podem permutar entre si.
6! 720 720
R

S
TE

   60
2! 3! 2  1 3  2  1 12 O total de cartões que admitem duas leituras é:

SO
a) 32
MA

Daí criamos uma fórmula para permutação quando há objetos


b) 64
repetidos.

R
c) 81
“Querendo permutar n objetos em n espaços sendo destes n
objetos, a iguais entre si, b iguais entre si, ...., até c iguais entre si d) 120
teremos.
n! Resolução: A
Pna, b, ..., c 
a! b! ...  c! Os cartões que admitem duas leituras são os que apresentam
R
apenas os algarismos 6 ou 9. Logo, como existem duas escolhas
MA

(Tendo objetos que não se repetem, só aparecem uma única vez, para cada dígito, pelo Princípio Multiplicativo, segue que a resposta
SO

não há a necessidade de se dividir por 1!). é 2 · 2 · 2 · 2 · 2 = 32.


T

Exercício Resolvido
ARRANJOS SIMPLES R
E

IA F
02. Quantos são os números inteiros positivos com três dígitos

L D O a) P136.R O
Um arranjo simples de n objetos distintos tomados p a p (sendo distintos nos quais o algarismo 5 aparece?
p ≤ n), é uma das sequências ordenadas de p elementos em que
podem ser colocados p objetos, selecionados entre os n objetos dados.
b) 200.
Notações: Anp ou An,p (número de arranjos simples de n elementos
distintos tomados p a p). c) 176.
n! d) 194.
Propriedade: An 
p

(n  p)!
Resolução: B
Nota: Todo arranjo simples pode ser resolvido utilizando o
Primeiramente vamos calcular quantos são os números inteiros
princípio multiplicativo. É de costume se dizer que estamos fazendo
positivos com três dígitos distintos.
uma permutação de n objetos só que somente para p espaços (n > p).
Existem 9 possibilidades para o algarismo das centenas, pois o zero
Exemplo: De quantas maneiras podemos formar um podium (1º,
deve ser descartado; 9 escolhas para o algarismo das dezenas e 8
2º e 3º lugares) de uma competição de judô que possui 10 atletas?
possibilidades para o algarismo das unidades. Logo, pelo Princípio
O arranjo simples resolve esse problema. Arranjar 10 objetos em Multiplicativo, temos 9 · 9 · 8 = 648 números.
3 espaços. Para o arranjo costumamos dizer que a ordem dos objetos
Agora, vamos determinar quantos são os números inteiros positivos
é importante. Isso fica bem claro, como é um podium, a ordem faz
com três dígitos distintos em que o algarismo 5 não figura.
diferença. ABC é bem diferente de BAC, na 1ª o atleta A é o campeão
e o atleta B o vice, na 2ª acontece o inverso. Temos 8 escolhas para o algarismo das centenas, 8 possibilidades
para o algarismo das dezenas e 7 escolhas para o algarismo das
10! 10  9  8  7! unidades. Em consequência, pelo Princípio Multiplicativo, existem
3
A10    720
(10  3)! 7! 8 · 8 · 7 = 448 números em que o 5 não figura.
A resposta é 648 – 448 = 200.

58 PROMILITARES.COM.BR

PM_BOOK05_MAT.indb 58 01/11/2021 13:41:09


ANÁLISE COMBINATÓRIA: CONTAGEM, PERMUTAÇÃO E ARRANJO

02. (CFT) Deseja-se colorir os seis triângulos da figura com cores


Exercício Resolvido diferentes.
03. Os alunos do curso de Computação Gráfica do campus Olinda
estão desenvolvendo um vídeo com todos os anagramas da palavra
CARNAVAL. Se cada anagrama é mostrado durante 0,5 s na tela, a
animação completa dura
a) menos de 1 minuto. d) menos de 10 minutos. Dispondo-se de sete cores, o número de maneiras diferentes de
b) menos de 1 hora. e) mais de 1 hora. conseguir o que se deseja é
c) menos de meia hora. a) 3200 b) 4700 c) 5040 d) 6090

Resolução: B 03. (EEAR) O número de anagramas da palavra SARGENTO, que


O número de anagramas da palavra CARNAVAL será dado por: começam por consoante e terminam por vogal é
8! a) 1.080 b) 1.800 c) 10.800 d) 18.000
P=
8
3
= 6720 anagramas. Como são 0,5 s para cada anagrama, o
3!
tempo total será: 6720 x 0,5 = 3360 s (menos que 1 hora = 3600 s). 04. (EEAR) Com os algarismos 2, 3, 4, 5, 6 e 7 posso escrever ____
Ou seja, a resposta correta é a opção [B], menos de 1 hora. números pares de quatro algarismos distintos.
a) 120 b) 180 c) 240 d) 360

Exercício Resolvido 05. (EEAR) Um professor montará uma prova com as 4 questões que

DO PRO
ele dispõe. O número de maneiras diferentes que o professor pode
04. O número de anagramas da palavra COLEGA em que as letras
L, E e G aparecem juntas em qualquer ordem é igual a:
A L montar essa prova, levando em conta apenas a ordem das questões, é

I
a) 20
Fb)E 22 c) 24 d) 26
a) 72
b) 144
c)
d) 60
120
R
e) 24
06. (ESA) Sendo n um número natural, n! equivale a
n ⋅ (n – 1) ⋅ (n – 2) ⋅ ... ⋅ 2 ⋅ 1 e ainda 0! = 1 e 1! = 1, então identifique

S
TE

Resolução: B a afirmativa verdadeira.

SO
Calculando: a) 5! = 120. c) 3! = 7. e) 6! = 600.
MA

LEG ⇒ 4 ⋅ ( 3 ⋅ 2 ⋅ 1) =24  b) 4! = 10. d) 2! = 3.

R

LGE ⇒ 4 ⋅ ( 3 ⋅ 2 ⋅ 1) =24  07. (EEAR) Considere todos os anagramas que podem ser formados
ELG ⇒ 4 ⋅ ( 3 ⋅ 2 ⋅ 1) =24  com as letras da palavra COLHER. O número dos que começam com
 ⇒ 6 ⋅ 24 =
144 a letra C é
EGL ⇒ 4 ⋅ ( 3 ⋅ 2 ⋅ 1) =24 
24 
a) 2. b) 6. c) 24. d) 120.
GLE ⇒ 4 ⋅ ( 3 ⋅ 2 ⋅ 1) =
24 
GEL ⇒ 4 ⋅ ( 3 ⋅ 2 ⋅ 1) =
R
08. (EEAR) Uma urna contém uma bola vermelha (V), uma preta (P) e
MA

uma amarela (A). Extrai-se uma bola, observa-se sua cor e repõe-se a
SO

bola na urna. Em seguida, outra bola é extraída e sua cor é observada.


O número das possíveis sequências de cores observadas nestas duas
Exercício Resolvido etapas consecutivas é
T

R a) 9. b) 10. c) 11. d) 12.


E

F
A 7, 4
05. (EEAR) Em análise combinatória a razão é igual a:
IA
L DO PRO
P5 09. (EEAR) Os taifeiros da EPCAR prepararam para o almoço dos
a) 7 c) 3 alunos: 2 tipos de salada, 3 tipos de massa, 2 tipos de carne e 3 tipos
de sobremesa. Com as opções acima, o número de possibilidades para
b) 5 d) 1 servir o almoço com 1 tipo de salada, 1 tipo de massa, 1 tipo de carne
e 1 tipo de sobremesa foi ____.
Resolução: A
a) 10 b) 12 c) 24 d) 36
n!
Como An,p  e Pn  n! teremos
(n  p)!
10. (EEAR) Para elaborar uma prova de Inglês, um professor utilizará 6
7! 7  6  5  4  3! questões de vocabulário e 4 de gramática. O número de maneiras que
A 7, 4 (7  4 )! 3! 765 4 7 6  54 ele pode ordenar aleatoriamente essas questões é dado por ______ .
    7
P5 5! 5! 5! 5  4  3  2 1
1 a) (6 + 4)! b) (6 – 4)! c) 6!.4! d) 6!/4!

EXERCÍCIOS DE

EXERCÍCIOS DE
TREINAMENTO
FIXAÇÃO
01. (EEAR) Formato, tamanho e cor são as características que diferem
as etiquetas indicadoras de preço dos produtos de uma loja. Se elas
01. (CFT) A quantidade de números de quatro algarismos distintos podem ter 2 formatos, 3 tamanhos e 5 cores, o número máximo de
que podem ser formados com os algarismos 3, 4, 5, 7 e 9 é preços distintos dos produtos da loja é
a) 120 b) 140 c) 160 d) 210 a) 24. b) 30. c) 32. d) 40.

PROMILITARES.COM.BR 59

PM_BOOK05_MAT.indb 59 01/11/2021 13:41:11


ANÁLISE COMBINATÓRIA: CONTAGEM, PERMUTAÇÃO E ARRANJO

02. (EEAR) O número de anagramas da palavra SOLEIRA que a) 1315 e 1330. c) 1345 e 1360.
começam com vogal é b) 1330 e 1345. d) 1360 e 1375.
a) 2720. b) 2780. c) 2860. d) 2880.
03. (EEAR) O número de anagramas formados com as letras da
03. (EEAR) Sobre uma mesa tem-se 2 livros de Física, 1 de Matemática, palavra ROMA de modo que não apareçam vogais ou consoantes
2 de Inglês e 1 de História. De quantas formas podemos colocá-los em juntas é igual a
uma prateleira, de modo que os livros de Exatas fiquem juntos? a) 4! b) 4 c) 8 d) 2 e) 1
a) 36 b) 72 c) 144 d) 288
04. (EEAR) Com os dígitos 1, 2, 3, 6 e 0, podemos formar x números
04. (EEAR) Dos 10 judocas que participam de uma competição, de 4 algarismos distintos. Então, x é igual a:
os 3 melhores subirão em um pódio para receber uma premiação. a) 160 b) 96 c) 180 d) 108 e) 120
Lembrando que cada atleta pode ocupar o 1º, 2º ou 3º lugar no pódio,
o número das possíveis formas de os atletas comporem o pódio é
05. (EEAR) Sejam: A = {1,2,3}, B = {a,e,i,o,u}. e a função ƒ: A → B. O
a) 720. b) 680. c) 260. d) 120. número de funções injetoras definidas em f é igual a
a) 10 b) 15 c) 60 d) 75
05. (EEAR) Um determinado brinquedo possui uma haste onde devem
ser colocadas 4 peças de formatos diferentes. O número de maneiras
06. (EEAR) As atuais placas de automóveis possuem três letras do
diferentes de se montar esse brinquedo é
alfabeto latino (incluindo K, W, Y) e quatro algarismos. O número de
placas que não repetem nem letras e nem algarismos é
a) 4. 26!10!
DO PRO
a) c) 26! 10!
b) 12. 23! 6!
c) 24.
A L d)
26!10!
d) 36. I b) 26 ⋅ 10 FE 3 4
4! 3!

R 07. (EEAR) No emplacamento de automóveis da cidade paulista X,


06. (EEAR) A metade do número de anagramas da palavra PRISMA são usadas duas letras do alfabeto seguidas de quatro algarismos.

S
TE

que começam por S é O número de placas, começadas pela letra “A”, seguida de vogal,

SO
a) 10. b) 20. c) 30. d) 60. inclusive “A”, e de quatro algarismos distintos, sendo dois (2) o último
MA

algarismo, é
07. (ESA) Em um guarda-roupa há quatro camisas, cinco calças e três a) 2.520. c) 160. e) 1800.

R
sapatos, então identifique a alternativa que apresenta a quantidade b) 720. d) 3.600.
de formas diferentes que se pode utilizá-las.
a) ∞ c) 1 e) 60 08. (EEAR) O número de anagramas da palavra ALAMEDA que não
b) 453 d) 12 apresenta as 4 vogais juntas é
a) 96 b) 744 R c) 816 d) 840
08. (ESA) Uma corrida é disputada por 8 atletas. O número de
MA

resultados possíveis para os 4 primeiros lugares é: 09. (ESA) Com as letras da palavra SARGENTO foram escritos todos
SO

a) 336 c) 1530 e) 4096 os anagramas iniciados por vogais e com as consoantes todas juntas.
Quantos são esses anagramas?
b) 512 d) 1680
a) 120960 b) 40320 c) 2160 d) 720 e) 120
T

R
09. (EEAR) O número de anagramas da palavra ESCOLA, que
E

distintos, formados F
10. (AFA) A quantidade de números naturais de 4 algarismos
começam por S e terminam por L, é
a) 720. b) 120. c) 24.
I d)A 12. ou o algarismoO
por 1, 2, 3, 4, 5 e 6, que contém o algarismo 3
L D a) 196 P R b) 286

10. (EEAR) Na equação (y + 3)! + (y + 2)! = 15 (y + 1)!, o conjunto O


c) 336 d) 446
solução é
a) {−7, 1}. b) {−7}. c) {1}. d) {2}.
RESOLUÇÃO EM VÍDEO
Abra o ProApp, leia o QR Code, assista à resolução
de cada exercício e AVANCE NOS ESTUDOS!

EXERCÍCIOS DE

COMBATE GABARITO
EXERCÍCIOS DE FIXAÇÃO
01. A 03. C 05. C 07. D 09. D
01. (EEAR) Considere todos os números de 4 algarismos distintos
02. C 04. B 06. A 08. A 10. A
formados com os algarismos 2, 3, 4, 5 e 6. Se colocarmos esses
números em ordem decrescente, a posição ocupada pelo número EXERCÍCIOS DE TREINAMENTO
4652 será a 01. B 03. C 05. C 07. E 09. C
a) 49ª b) 50ª c) 59ª d) 60ª 02. D 04. A 06. D 08. D 10. C
EXERCÍCIOS DE COMBATE
02. (EEAR) Se existem k maneiras possíveis de pintar uma parede com
01. B 03. C 05. C 07. A 09. C
3 listras verticais, de mesma largura e de cores distintas, dispondo de
12 cores diferentes, então o valor de k está compreendido entre 02. A 04. B 06. A 08. B 10. C

60 PROMILITARES.COM.BR

PM_BOOK05_MAT.indb 60 01/11/2021 13:41:12


ANÁLISE COMBINATÓRIA:
COMBINAÇÃO E BINÔMIO DE NEWTON

PERMUTAÇÃO CIRCULAR No caso de n objetos podemos girar o círculo n vezes que não
alteramos a ordem dos objetos. Então:
Imagine que devemos distribuir n objetos em volta de uma mesa
circular por exemplo (em forma de círculo). Teríamos uma distribuição n! n  (n  1)!
  (n  1)!
dessa maneira. n n
PCn  Pn 1  (n  1)!

Exemplo: De quantos modos distintos podemos distribuir 5


crianças numa roda para se brincar de ciranda?

D O usarPComo
R
vamos permutar objetos ao longo de um círculo devemos

A L uma
O
permutação circular.

I F E permutar as 5 crianças.
Vamos inicialmente

R P5 = 5!

S
TE

SO
Na figura abaixo temos uma das 5! Permutações.
MA

R
Teríamos Pn = n!, mas repare que podemos “girar” o círculo que a
ordem dos objetos não muda.

R
MA

SO
T

R
E

IA F
L D O Porém O
P Rveja que podemos ter 5 “giros” que ainda sim temos a
mesma configuração.

PROMILITARES.COM.BR 61

PM_BOOK05_MAT.indb 61 01/11/2021 13:41:14


ANÁLISE COMBINATÓRIA: COMBINAÇÃO E BINÔMIO DE NEWTON

p
Notações: Cn ou Cn,p (número de combinações de n elementos
distintos tomados p a p).
n!
Propriedade: Cpn =
p!(n − p)!

Vamos imaginar que temos os objetos A, B, C, D, E, F e G e que


destes queremos escolher 4 objetos. Dentre as vária possibilidades
vamos imaginar o seguinte cenário.

ESCOLHIDOS: {A, B, E, F}
NÃO ESCOLHIDOS: {C, D, G}

Podemos começar permutando todos os 7 objetos, ou seja 7!.


Mas como queremos 2 grupos, onde não há distinção alguma entre a
ordem de seus elementos, tanto que {A, B, E, F} = {B, A, F, E} = ... tanto
para o conjunto de escolhidos como o de não escolhidos devemos
dividir 7! Pela quantidade de cada conjunto.
7!
4!⋅ 3!

L D O PDaíRtemos
O a noção de combinação simples, de onde sempre que
I A F (n – p) objetos, assim
queremos formar um grupo de p objetos dentre n também formamos
um outro grupo deE
R n!
Cn,p 

S
TE

p! n  p !

SO
MA

E dessa ideia podemos perceber que escolher 4 dentre 7 é o


mesmo que deixar de escolher 3 dentre 7. Isso se chama combinação

R
complementar.
Escolher p dentre n é o mesmo que escolher (n – p) dentre n.
Assim
Cn,p = Cn,n – p
Também podemos representar uma combinação simples Cn,p por
R
 n  n  n 
MA

 p , assim  p =  n − p .


SO

Exemplo: Num restaurante há 8 tipos de saladas, de quantas


T

maneiras distintas uma pessoa pode se servir de 4 saladas diferentes?


S

R Nesse caso é como formarmos um grupo de 4 saladas com as 8


E

IA F
que temos disponíveis. Por exemplo se escolhermos salada de alface

L DO PRO
(A), rúcula (R), ovos (O) e tomate (T), formamos o grupo de saladas (A,
R, O, T). Repare que se trocarmos a ordem das saladas (R, A, T, O), por
exemplo, as saladas no prato continuam sendo: alface, rúcula, ovos e
tomate. Então ao se formar grupos, onde não há hierarquia ou cargos,
a ordem não importa. Essa é a característica da combinação, a ordem
4
não importa. Daí teremos C8 .

8! 8  7  6  5  4! 8  7  6  5
C84     2  7  5  70
4! (8  4 )! 4! 4! 4  3 .2  1
Por isso devemos dividir 5! Por 5, como é a ideia da permutação
circular.
Nota: Ao somarmos as combinações de objetos para 0 espaço com
5! 5  4! a combinação de n objetos para 1 espaço e assim até a combinação de
  4!
5 5 n objetos para n espaços teremos:
PC5  P5 1  (5  1)!  4!  4  3  2  1  24
Cn,0 + Cn,1 + Cn,2 + ... + Cn,n −1 + Cn,n

Isso é o somatório completo de uma linha do triângulo de Pascal,


COMBINAÇÃO SIMPLES que é igual a 2n. O somatório das combinações de 0 a n é o mesmo
que se fazer o total de subconjuntos. Essa informação pode poupar
Uma combinação simples de n objetos distintos tomados p a p muitos cálculos numa questão.
(sendo p ≤ n), é um dos subconjuntos com p elementos em que podem
ser colocados p objetos, selecionados entre os n objetos dados. Cn,0 + Cn,1 + Cn,2 + ... + Cn,n −1 + Cn,n = 2n

62 PROMILITARES.COM.BR

PM_BOOK05_MAT.indb 62 01/11/2021 13:41:21


ANÁLISE COMBINATÓRIA: COMBINAÇÃO E BINÔMIO DE NEWTON

Exemplo – Uma pastelaria oferece uma promoção: Monte seu Logo podemos escolher nossos 8 refrigerantes de 45 maneiras
pastel com até 5 recheios, carne, queijo, frango, calabresa e pizza. diferentes.
Quantos pastéis diferentes podem ser feitos sabendo-se que um Assim quando vamos escolher p objetos dentre n disponíveis mas
mesmo recheio não pode ser colocado mais de uma vez? desses p objetos podemos ter objetos repetidos teremos.
A resposta seria dentre 5 recheios escolhermos somente 1 ou de
5 recheios escolhermos 2 até que de 5 recheios escolhêssemos os 5.  n + p − 1
Cn + p −1,p = 
 p 
C5,1 + C5,2 + C5,3 + C5,4 + C5,5

Porém a ideia das bolinhas e barrinhas é sempre mais interessante.


Onde podemos usar a teoria do somatório do triangulo de pascal
para poupar cálculos, mas repare que falta a combinação C5,0. Exercício Resolvido
A combinação Cn,0 fazendo analogia a teoria de conjuntos essa
combinação é equivalente ao conjunto vazio, o que para essa questão 01. (EEAR) uma classe tem 10 meninos e 9 meninas. Seu professor
não nos interessa pois não queremos pastel vazio. Como Cn,0 é igual necessita formar comissões de 7 crianças, sendo 4 meninos e 3
a 1, temos: meninas, que incluam obrigatoriamente o melhor aluno dentre os
meninos e a melhor aluna dentre as meninas. O número possível
C5,1 + C5,2 + C5,3 + C5,4 + C5,5 = 25 − 1 = 32 − 1 = 31 de comissões é
a) igual a 2300
b) maior que 2400
COMBINAÇÃO COM REPETIÇÃO c) menor que 2300
Imagine que você vai a um supermercado e deseja comprar 8
D O Resolução:
d) igual a 2352
refrigerantes tendo 3 marcas diferentes disponíveis. É fácil perceber
L P R OD
A
que não há possibilidade de levar somente marcas distintas, logo
I
alguma marca terá que se repetir, essa é a ideia da combinação com FE
Então temos uma comissão conjuntas, 4 vagas para os meninos,
repetição.
R
Podemos fazer uma assimilação da combinação com repetição
dentre 10 disponíveis e mais 3 vagas para as meninas dentre as
9 disponíveis. Porém os melhores menino e menina devem estar

S
TE

com a quantidade de soluções inteiras e não negativas da equação. na comissão, daí teremos somente 3 vagas para os meninos e 2

SO
x+y+z=8 vagas para as meninas, e 9 meninos que poderão ser distribuídos,
MA

o melhor ja está na comissão e 8 meninas para distribuir, pois a


Onde x, y e z são as marcas de refrigerante que temos disponíveis melhor também já está na comissão.
e 8 é a quantidade total que queremos levar. Assim veja que se

R
imaginarmos as soluções inteiras e não negativas dessa equação é
algo interessante.
Solução {0, 3, 5} – não compramos nenhum refrigerante da marca
1, 3 refrigerantes da marca 2 e 5 refrigerantes da marca 3.
Solução {2, 2, 4} – compramos 2 refrigerantes da marca 1, 2 R
refrigerantes da marca 2 e 4 refrigerantes da marca 3. Como se trata de uma escolha, formar grupos, então a ordem não
MA

Solução {0, 8, 0} – compramos todos os 8 refrigerantes da marca 2. importa, assim devemos combinar as 8 meninas restantes para as 2
SO

Então a quantidade soluções inteiras e não negativas dessa vagas e os 9 meninos restantes para as 3 vagas restantes.
equação nos dá a quantidade de escolhas. Para criar um modo de 8! 9! 8! 9!
C8,2  C9,3     
calcular vamos imaginar cada sinal de + como uma “barra” e a 2!(8  2)! 3!(9  3)! 2! 6! 3! 6!
T

quantidade total que se deseja a quantidade de “bolas” a se distribuir.


R 8  7  6! 9  8  7  6!
E

IA

F .
2  1 6! 3  2  1 6!
 4  7  3  4  7  2352

L DO PR O
A quantidade de “bolas” que ficarem a esquerda da 1º barra será Exercício Resolvido
a quantidade x. 02. (EEAR) Um sargento da FAB tem 8 soldados sob seu
A quantidade de “bolas” que ficarem entre a 1º e a 2ª barra será comando. Tendo que viajar a serviço, deixa a seus comandados
a quantidade y. uma determinação: “Ao chegar, quero encontrar no mínimo
A quantidade de “bolas” que ficarem a direita da 2º barra será a um de vocês no pátio, fazendo Educação Física.”. Dessa forma,
quantidade z. o sargento tem maneiras de encontrar seus soldados fazendo
Educação Física.
Logo no exemplo teríamos a solução {2, 3, 3}.
a) 256
b) 255
c) 64
d) 16
Já essa solução seria {0, 0, 8}.
Então podemos pensar que temos 10 objetos no total (8 bolas e Resolução: B
2 barras), onde 8 são iguais entre si (8 bolas) e 2 são iguais entre si
O sargento quer formar grupos de uma pessoa, ou de 2 pessoas,
(2 bolas). Logo para se obter uma solução diferente basta permutar a
ou de 3 pessoas, ..., ou até mesmo que todos os 8 estejam no
posição dos objetos, assim
pátio. Assim queremos formar todos esses grupos. Quando é feita
10! 10  9  8! uma escolha é feita uma combinação, daí
P108,2    5  9  45 8
8! 2! 8!  2  1 C8,1 + C8,3 + ... + C8,7 + C8,8 = 2 − 1 = 256 − 1 = 255

PROMILITARES.COM.BR 63

PM_BOOK05_MAT.indb 63 01/11/2021 13:41:24


ANÁLISE COMBINATÓRIA: COMBINAÇÃO E BINÔMIO DE NEWTON

Exercício Resolvido COEFICIENTES BINOMIAIS


03. (EEAR) Dentre 8 candidatos, 5 devem ser selecionados para
comporem uma comissão de formatura. O número de formas DEFINIÇÃO
distintas de se compor essa comissão é Dados os naturais n e k, sendo n ≥ k chama-se coeficiente
a) 56  n
binomial n sobre k e se indica   ao número definido por:
b) 48  k
c) 46  n  n n!
=1 e   = se 1 ≤ k ≤ n.
d) 38  0  k k!(n − k )!

Resolução: A Da definição que para 1 ≤ k ≤ n tem-se:


Toda escolha é uma combinação, não importa a ordem dos objetos
 n n! n(n − 1)(n − 2)...(n − k + 1)
= = onde o numerador da
C8,5 
8!

8  7  6  5! 8  7  6
  56  k k!(n − k )! k(k − 1)(k − 2)...1
5!(8  5)! 5!  3! 3  2 1
fração contém k fatoriais.
Exemplo:
Exercício Resolvido (n + 1)!
Calcule n, sabendo-se que =7.
n!
04. (CFOE) Uma lanchonete dispõe-se de 6 variedades de Solução: Temos que:
coberturas para sorvetes. De quantas maneiras é possível escolher (n  1)!  (n  1)  n  (n  1)  ...  3  2  1  (n  1)  n!
pelo menos 3 coberturas?
L D O PLogo,
R nO!(n + 1) = 7 ⇒ n + 1 = 7 ⇒ n = 6 .
a) 32
I A n!F
b) 35
c) 38 R E
TRIÂNGULO DE PASCAL

S
TE

d) 42 O triângulo de Pascal é um triângulo aritmético formado por

SO
números que têm muitas relações entre si. Algumas dessas relações
Resolução: D foram descobertas por Pascal.
MA

Toda vez que fazemos uma escolha simples estamos formando Onde Ckn representa combinação de n escolhe k, n é o número da

R
um grupo, onde não importa a ordem dos objetos. Temos uma linha e k representa o número da coluna.
combinação simples e como queremos pelo menos 3 coberturas,
Os quadros abaixo representam o triângulo de Pascal:
então podemos escolher 3, 4, 5 ou até mesmo 6 dentre as 6
disponíveis, temos C00
C6,3  C6,4  C6,5  C6,6 C10 C11
6! 6  5  4  3! C02 C12 C22
R
C6,3    20
MA

3! 3! 3!  3  2  1 C03 C13 C23 C33


SO

6! 6  5  4! C04 C14 C24 C34 C44


C6,4    15
4! 2! 4!  2 C50 C15 C52 C53 C54 C55
0 1 2 3 4
6! C C C C C C56 C66
T

C6,5   6 e C6,6  1 6 6 6 6 6
5! 1!
R ...
E

C6,3  C6,4  C6,5  C6,6  20  15  6  1  42


IA F1
L DO PRO 1 1
Exercício Resolvido 1 2 1
05. (CFOE) Num batalhão estão de plantão 3 sargentos e 7 1 3 3 1
soldados. O número de equipes de 5 pessoas que podemos formar
1 4 6 4 1
contendo exatamente 1 sargento é igual a:
a) 105 1 5 10 10 5 1
b) 125 1 6 15 20 15 6 1
c) 150 ...
d) 250

Resolução: A
RELAÇÃO DE STIFEL
Vamos determinar as casas e quem pode ocupa-las
Trata-se de uma propriedade que permite construir o Triângulo de
Pascal dada pela seguinte fórmula: Cpn + Cpn +1 = Cpn ++11.
Somando 2 elementos consecutivos de uma mesma linha obtemos
o elemento localizado abaixo da segunda parcela.

7! 7  6  5  4! Prova: Considere o número de comissões de p + 1 pessoas que


3  C7,4  3   3  3  7  5  105 podemos formar a partir de um grupo de n + 1 pessoas, sabendo que
4! 3! 4!  3  2  1 uma destas n + 1 pessoas é Pedro.

64 PROMILITARES.COM.BR

PM_BOOK05_MAT.indb 64 01/11/2021 13:41:32


ANÁLISE COMBINATÓRIA: COMBINAÇÃO E BINÔMIO DE NEWTON

p +1
O total dessas comissões é igual a Cn +1 , por outro lado podemos O número de termo da forma (2) é, então é igual ao número de
contar este total separando em dois casos; primeiro caso comissões modos de escolhermos p letras a em p dos n binômios x + a, isto é, Cpn .
que Pedro participa que são Cpn e o segundo caso, as comissões na Portanto, reduzindo todos os termos da forma apxn–p, encontramos
p−1
qual Pedro não participa que são Cn . um único termo, Cpnap xn − p .
Finalmente, fazendo em p variar de 0 até n, encontramos todos os
TEOREMA DAS LINHAS termos do desenvolvimento de (x + a)n.
n
A soma dos termos da linha n é igual a 2n.
Cn0 + C1n + Cn2 + ... + Cnn −1 + Cnn = 2n.
Então, ( x + a) =
n
∑ Cax
p=0
p p n−p
n .

Expandindo o somatório acima, temos:


Prova: Seja o conjunto A={1,2,3,...,n} o total de subconjuntos é  n  n  n  n −1
( x + a)n = xn +   a xn −1 +   a2xn − 2 + ... +  a x + an (I)
igual a Cn0 + C1n + Cn2 + ... + Cnn −1 + Cnn , por outro lado para formar um  1  2  n − 1
subconjunto de A cada elemento de A tem duas opções , participar
ou não do subconjunto.
Concluímos que o total de subconjuntos é igual a 2n. TERMO GERAL DO
DESENVOLVIMENTO DE (X + A)N
TEOREMA DAS COLUNAS Todos os termos do desenvolvimento de (x + a)n são obtidos de
p p n−p
A soma dos elementos de uma coluna do triângulo, começando Cax n quando fazemos neste termo, p variar de 0 a n.

O PChamando
no primeiro termo da coluna, é igual ao elemento que está na linha e
D
p p n−p
Por este motivo, C a x é chamado de termo geral.
na coluna posteriores ao último elemento da soma.
L R O
n

o 1º, 2º, 3º, ... termos do desenvolvimento de (x + a) n

Cpp + Cpp +1 + Cpp + 2 + ... + Cpp + n =C


I
p +1
A respectivamenteFpor T , T , T , ..., podemos observar que:
E T =Ca x
p + n +1 1 2 3

Prova: Considere o conjunto A = {1,2,...,p,...,n+p,n+p+1} com


R Para p = 0 obtemos 1

Para p = 1 obtemos T2 = C1na1xn −1


0 0 n
n

S
TE

p < n, o total de subconjuntos de p + 1 elementos de A é igual a Cpn ++1p +1 .

SO
Para p = 2 obtemos T3 = Cn2a2xn − 2
Por outro lado cada um destes subconjuntos tem um elemento
MA

3 3 n−3
que é o máximo (maior elemento do conjunto). Para p = 3 obtemos T4 = Cna x
...

R
• Primeiro caso: p + 1 é o máximo, os outros p elementos são Isto é, a ordem de cada termo é igual à taxa da combinação
escolhidos à partir de {1, 2, ..., p}. correspondente mais 1. Como a taxa da combinação do termo geral é
p p n−p
Total de subconjuntos = Cpp . p, segue-se que este termo é de ordem p + 1. Isto é, Tp +1 = Cna x .

• Segundo caso: p + 2 é o máximo; agora devemos escolher EXERCÍCIOS DE


R
FIXAÇÃO
MA

os outros p elementos de p + 1 elementos {1, 2, ..., p + 1}.


SO

p
Total de subconjuntos = Cp +1 .
Caso N: n + p + 1 é o máximo; Agora devemos escolher os outros p
T

elementos à partir do conjunto {1, 2, 3, ..., n + p}.


S

R
01. (EEAR) Uma lanchonete tem em sua dispensa 5 espécies de frutas.
E

Total de subconjuntos = Cpn + p .


E

IA F
Misturando 3 espécies diferentes, pode-se preparar _____ tipos

L DO PRO
Igualando as duas formas, o teorema está provado. de suco.
a) 24.
OUTRAS PROPRIEDADES b) 15.
Cn0 + C1n +1 + Cn2+ 2 + ... + Cpn + p + Cpn ++1p +1 ; teorema das diagonais. c) 10.
d) 8.
Cpn = Cnn − p relação das combinações complementares.
n −1 02. (EEAR) Em uma turma de 20 alunos, será formado um grupo de
Cpn < Cpn +1 se p< .
2 6 alunos para representá-la em uma feira de Ciências. Se 8 alunos
não querem participar do grupo, o número de maneiras de se formar
esse grupo é
BINÔMIO DE NEWTON a) 732.
Consideremos a igualdade: b) 846.
( x + a)n = ( x + a)( x + a)...( x + a) (n fatores) (1) c) 924.
Para se formar um termo do produto ( x + a)( x + a)...( x + a) d) 1038.
devemos escolher uma das duas parcelas em cada um dos n fatores
x + a e efetuar o produto das mesmas. 03. (EEAR) Um maestro escolherá 5 músicas distintas, das 10 que
Por exemplo, se escolhermos p letras a em p dos n binômios, e dispõe, e montará uma apresentação. Para a escolha das músicas e
n – p letras x dos n – p binômios restantes, então um termo genérico da ordem que elas serão tocadas, o maestro possui um número de
 n   possibilidades cujo algarismo das unidades é
do desenvolvimento de (x + a)n é da forma: a a ... ax x ... x = ap xn − p a) 0 c) 4
     
com p = 0, 1, 2, ..., n (2) p n.p
b) 2 d) 6

PROMILITARES.COM.BR 65

PM_BOOK05_MAT.indb 65 01/11/2021 13:41:48


ANÁLISE COMBINATÓRIA: COMBINAÇÃO E BINÔMIO DE NEWTON

04. (EEAR) De um grupo de 10 (dez) pessoas, 5 (cinco) serão escolhidas EXERCÍCIOS DE


para compor uma comissão. Ana e Beatriz fazem parte dessas 10 (dez)
pessoas. Assim, o total de comissões que podem ser formadas, que
tenham a participação de Ana e Beatriz, é
TREINAMENTO
a) 24
b) 36 01. (CFOE) Se n é um número natural tal que 4 ⋅ Cn,3 =
Cn +1, 3 , então
c) 48 n3 − 2 é igual a:
d) 56 a) 3 b) 5 c) 7 d) 8

05. (EEAR) Suponha que na EEAR existam 13 Arrumadores. O 02. (CFOE) Em uma cidade existem 30 farmácias. Se a cada noite
número de maneiras diferentes de se formar um grupo com 4 desses 6 ficam de plantão, quantos grupos diferentes de farmácias pode-se
Arrumadores formar para um plantão em uma determinada noite?
a) está entre 300 e 500. a) 593.775
b) está entre 100 e 300. b) 545.125
c) é menor que 100. c) 485.238
d) é maior que 500. d) 490.276

06. (ESA) O número de anagramas diferentes com as letras da palavra 03. (CFOE) Num batalhão estão de plantão 3 sargentos e 7 soldados.
MILITAR que não possuem consoantes consecutivas que se pode obter é: O número de equipes de 5 pessoas que podemos formar contendo

D O a) P105R O
exatamente 1 sargento é igual a:
a) 60
b) 72
A L
c) 120
I b) 125 FE
d) 186 R c)
d) 250
150

e) 224

S
TE

SO
10
04. (EEAR) No desenvolvimento de  m3 −  , o coeficiente de m6 é
3
A10 1
MA

07. (EEAR) Ao calcular , obtém-se m


3
C10 
a) 3! a) 45

R
b) 4! b) 120
c) 5! c) 210
d) 6! d) 245

08. (EEAR) Dentre 8 candidatos, 5 devem ser selecionados para 05. (EEAR) Marcelo deseja comprar 4 camisas e 3 calças todas de
R
comporem uma comissão de formatura. O número de formas distintas modelos diferentes. Ao entrar em uma loja verificou que havia
MA

de se compor essa comissão é 6 modelos de camisas e 4 modelos de calça. De quantas maneiras


SO

a) 56 Marcelo poderá efetuar a compra?


b) 48 a) 48
T

b) 60
S

c) 46
R
E

c) 72
E

d) 38
IA F
O
d) 56
L D O06. (CFOE)
09. (EEAR) Em um campeonato de tênis estão inscritos 10 militares.
Para disputar o campeonato, esses militares podem formar_______ P RUma lanchonete dispõe-se de 6 variedades coberturas
duplas diferentes. para sorvete. De quantas maneiras é possível de se escolher até 3
a) 34 coberturas?
b) 35 a) 32
c) 44 b) 35
d) 45 c) 38
d) 42
10. (EEAR) Um sargento da FAB tem 8 soldados sob seu comando.
Tendo que viajar a serviço, deixa a seus comandados uma  Cn,p = 78
determinação: “Ao chegar, quero encontrar no mínimo um de vocês 07. (CFOE) No sistema  tem-se
An,p = 156
no pátio, fazendo Educação Física.” Dessa forma, o sargento tem
______ maneiras de encontrar seus soldados fazendo Educação Física. a) n = 2 e p = 12
a) 256 b) n = 3 e p = 10
b) 255 c) n = 2 e p = 13
c) 64 d) n = 13 e p = 2
d) 16

66 PROMILITARES.COM.BR

PM_BOOK05_MAT.indb 66 01/11/2021 13:41:49


ANÁLISE COMBINATÓRIA: COMBINAÇÃO E BINÔMIO DE NEWTON

08. (EEAR) uma classe tem 10 meninos e 9 meninas. seu professor 05. O número de valores de x, para os quais os coeficientes binomiais
necessita formar comissões de 7 crianças, sendo 4 meninos e 3 6 6
meninas, que incluam obrigatoriamente o melhor aluno dentre os   e  2  sejam iguais, é
2x
  x 
meninos e a melhor aluna dentre as meninas. o número possível de
comissões é a) 1 c) 3 e) 5
a) igual a 2300 b) 2 d) 4
b) maior que 2400
06. No desenvolvimento de x(2x + 1)10 o coeficiente de x³ é
c) menor que 2300
a) 480. c) 260.
d) igual a 2352
b) 320. d) 180.
09. (ESA) Um colégio promoveu numa semana esportiva um
campeonato interclasses de futebol. Na primeira fase, entraram na 07. (UERN) A soma dos algarismos do termo independente de x no
8
disputa 8 times, cada um deles jogando uma vez contra cada um dos 2 
outros times. O número de jogos realizados na 1ª fase foi desenvolvimento do binômio de Newton  + x  é
x 
a) 8 jogos a) 3 c) 6
b) 13 jogos b) 4 d) 7
c) 23 jogos
n

∑  p  = 256, então o valor de n vale


d) 28 jogos n
08. (MACKENZIE) Sabendo que
e) 35 jogos p=0

L D O b) P7 R O
10. (ESA) Para o time de futebol da EsSA, foram convocados 3
a) 8 d) 5
e) 4

IA FE
goleiros, 8 zagueiros, 7 meios de campo e 4 atacantes. O número
c) 6
de times diferentes que a EsSA pode montar com esses jogadores

de campo e 1 atacante é igual a


R
convocados de forma que o time tenha 1 goleiro, 4 zagueiros, 5 meios
09. (IFAL) O termo independente no desenvolvimento do binômio

S
TE

5
a) 84 d) 17640  2 3 é
 2x − 3 

SO
 x 
b) 451 e) 18560
MA

c) 981 a) -720. d) 360.

R
b) -360. e) 720.
c) 0.
EXERCÍCIOS DE

COMBATE
10. (ESPCEX) O coeficiente de x5 no desenvolvimento de (x + 2)9 é:
a) 64 d) 1024
b) 126
R e) 2016
MA

c) 524
SO

01. (AFA) Quatro pontos não-coplanares determinam, exatamente,


quantos planos?
a) 1 b) 2 c) 3 d) 4 RESOLUÇÃO EM VÍDEO
T

R Abra o ProApp, leia o QR Code, assista à resolução


E

02. (ESPCEX) Num determinado setor de um hospital, trabalham 4


F
de cada exercício e AVANCE NOS ESTUDOS!

IA
L DO PRO
médicos e 8 enfermeiras. O número de equipes distintas, constituídas
cada uma de 1 médico e 3 enfermeiras, que podem ser formadas
nesse setor é de
a) 60 c) 495 e) 11880 GABARITO
b) 224 d) 1344 EXERCÍCIOS DE FIXAÇÃO
01. C 04. D 07. A 10. B
03. (AFA) Numa demonstração de paraquedismo, durante a queda
02. C 05. D 08. A
livre, participam 10 paraquedistas. Em um certo momento, 7 deles
devem dar as mãos e formar um círculo. De quantas formas distintas 03. A 06. B 09. D
eles poderão ser escolhidos e dispostos nesse círculo? EXERCÍCIOS DE TREINAMENTO
a) 120 c) 86400 01. B 04. C 07. D 10. D
b) 720 d) 151200 02. A 05. B 08. D
03. A 06. D 09. D
04. Desenvolvendo-se o binômio P(x) = (x + 1)5, podemos dizer que a
soma de seus coeficientes é EXERCÍCIOS DE COMBATE
a) 16 d) 40 01. D 04. C 07. B 10. E
b) 24 e) 48 02. B 05. B 08. A
c) 32 03. C 06. D 09. E

PROMILITARES.COM.BR 67

PM_BOOK05_MAT.indb 67 01/11/2021 13:41:49


ANÁLISE COMBINATÓRIA: COMBINAÇÃO E BINÔMIO DE NEWTON

ANOTAÇÕES

L DO PRO
I A FE
R

S
TE

SO
MA

R
R
MA

SO
T

R
E

IA F
L DO PRO

68 PROMILITARES.COM.BR

PM_BOOK05_MAT.indb 68 01/11/2021 13:41:50


PROBABILIDADE

CONCEITO DE PROBABILIDADE ESPAÇO AMOSTRAL


É conveniente dispormos de uma medida que exprima a incerteza O espaço amostral de um experimento, denotado por Ω, é o
em afirmações tais como “É possível que chova amanhã” ou “Não há conjunto de todos os resultados experimentais.
chance de vitória”, em termos de uma escala numérica que varie do Exemplos:
impossível ao certo.
I. Jogar uma moeda.
O conceito de probabilidade é fundamental para o estudo de
Ω = {Cara, Coroa}
situações em que os resultados são variáveis, mesmo quando mantidas
inalteradas as condições de sua realização. Por exemplo: II. Selecionar uma peça para inspeção
Ω = {Defeituosa, Não defeituosa}
D O PIII. RLançar
• As declarações de despesas por funcionário de uma empresa

L Ω = {1,O
podem assumir uma variedade de valores; um dado

IA FE
• A audiência estimada de um comercial de TV com 30 2, 3, 4, 5, 6}
segundos de duração não é a mesma para cada exibição.

Observação
R A probabilidade atribuída a cada um dos resultados experimentais

S
TE

deve situar entre 0 e 1, inclusive. Se admitimos que Ei denota o í-ésimo


Probabilidade pode ser definida como teoria matemática utilizada

SO
resultado experimental e que P(Ei) é a sua probabilidade, então esse
para se estudar a incerteza oriunda de fenômeno de caráter aleatório. requisito pode ser escrito na seguinte forma:
MA

0 ≤ P(Ei ) ≤ 1 para todo i

R
FENÔMENO ALEATÓRIO A soma das probabilidades de todos os resultados experimentais
Fenômeno: qualquer acontecimento natural. deve ser igual a 1,0. Para n resultados experimentais, esse requisito
pode ser escrito na seguinte forma:
Os fenômenos podem ser classificados, quanto aos seus possíveis
resultados, de dois tipos: P(E1)  P(E2 )    P(En )  1
• Determinísticos; R
• Aleatórios; MÉTODOS DE ATRIBUIÇÃO DE PROBABILIDADE
MA

SO

– Fenômenos Determinísticos: são aqueles que repetidos Através das frequências de ocorrências (método de frequência
sob mesmas condições iniciais conduzem sempre a um relativa)
só resultado. Observamos o experimento aleatório n vezes e determinamos a
T

– Fenômenos Aleatórios: são aqueles que repetidos sob frequência relativa com que cada resultado ocorre.
R
E

mesmas condições iniciais podem conduzir a mais de um


E

resultado. IA F
Números de vezes em que o evento ocorreu

L D OObservação O Número total de repetições do experimento

EXPERIMENTO ALEATÓRIO PR
Experimento: processo que gera resultados bem definidos. Este método é apropriado quando se tem dados disponíveis para
São fenômenos aleatórios que possuem as seguintes características: estimar a proporção do tempo em que o resultado experimental
• Repetitividade: pode ser repetido quantas vezes quisermos; ocorrerá se o experimento for repetidos inúmeras vezes.
• Regularidade: diz respeito à possibilidade da ocorrência dos
resultados do fenômeno. Através de suposições teóricas (método clássico).

Exemplo de experimentos aleatório e seus respectivos resultados Número de casos favoráveis ao evento
experimentais: Número de casos possíveis
RESULTADOS Observação
EXPERIMENTO
EXPERIMENTAIS
Apropriado quando todos os resultados experimentais são
Jogar uma moeda Cara, coroa igualmente prováveis.
Selecionar uma peça para
Defeituoso, não defeituoso
inspeção EVENTO
Fazer um contato de vendas Comprar, não comprar Subconjunto do espaço amostral do experimento.
Notação: A, B, C, ...
Lançar um dado 1, 2, 3, 4, 5, 6
φ (conjunto vazio): evento impossível.
Jogar uma partida de futebol Ganhar, perder, empatar Ω: espaço amostral

PROMILITARES.COM.BR 69

PM_BOOK05_MAT.indb 69 01/11/2021 13:41:51


PROBABILIDADE

Exemplo: COMPLEMENTAR
Lançamento de um dado. A negação do evento A, denotado por A, é chamado de evento
Espaço amostral: Ω = {1, 2, 3, 4, 5, 6} complementar de A.
Alguns eventos: A e A são complementares se sua intersecção é vazia e sua união
é o espaço amostral, isto é,
A: sair face par A = {2, 4, 6} ⇒ ⊂ Ω
B: Sair face maior que 3 B = {4, 5, 6} ⇒ ⊂ Ω A  A  O e A  A  
C: Sair face 1 C = {1} ⇒ ⊂ Ω

OPERAÇÕES COM EVENTOS


INTERSEÇÃO
O evento interseção de dois eventos A e B equivale à ocorrência
de ambos.
Notação: A ∩ B

L DO PRO
I A F
PROBABILIDADE DE UM EVENTO
R E
É uma função que atribui um número aos eventos que pertence
ao espaço amostral (se A é um evento de Ω, P(A) é a probabilidade de

S
TE

A), que satisfaz as seguintes condições:

SO
0  P( A )  1,  A  
MA

EVENTOS MUTUAMENTE EXCLUSIVOS P( )  1;

R
Dois eventos A e B dizem-se mutuamente exclusivos, ou
Se A e B são eventos mutuamente excludentes, então
mutuamente excludentes, quando a ocorrência de um deles
P( A  B)  P( A )  P(B) .
impossibilita a ocorrência do outro.
Exprime-se este fato escrevendo-se A ∩ B = ∅.
TEOREMAS FUNDAMENTAIS
P(O )  0
R
MA

P( A )  1  P( A )
SO

Se A  B, P( A )  P(B) ;
T

REGRA DA SOMA
S

R
E

Se A e B são eventos quaisquer de Ω, então:


E

IA F
P( A  B)  P( A )  P(B)  P( A  B) .
O
L D OPROBABILIDADE
PR CONDICIONAL
A probabilidade do evento A, quando se sabe que o evento B
UNIÃO ocorreu, é chamado probabilidade condicional de A dado B; denota-
O evento união de A e B equivale à ocorrência de A, ou de B, ou se por P(A|B) e é calculada por:
ambas.
P( A  B)
Notação: A ∪ B P( A | B) 
P(B)

desde que P(B) > 0.


Da definição de probabilidade condicional podemos obter a regra
de produtos de probabilidades

P( A  B)  P(B)P( A | B)

Analogamente, se P(A) > 0,

P( A  B)  P( A )P(B | A )

70 PROMILITARES.COM.BR

PM_BOOK05_MAT.indb 70 01/11/2021 13:41:56


PROBABILIDADE

Exercício Resolvido Exercício Resolvido


01. Uma caixa contém 4 pistolas e 4 fuzis, sendo uma pistola e 2 03. O Centro Paula Souza administra Escolas Técnicas (Etecs) e
fuzis defeituosos. Duas armas são retiradas da caixa sem reposição. Faculdades de Tecnologia (Fatecs) estaduais em 149 municípios,
A probabilidade de pelo menos uma arma ser defeituosa ou ser no Estado de São Paulo. Para participar de um simpósio sobre
pistola é igual a educação a distância, a Fatec São Paulo enviou cinco alunos, sendo
dois homens; a Fatec Sorocaba enviou três alunos, sendo uma
27 mulher; e a Fatec da Baixada Santista enviou quatro alunos, sendo
a)
28 dois homens. Para a abertura desse simpósio, será selecionada, ao
13 acaso, uma dessas Fatecs e dela se escolherá, também ao acaso,
b) um aluno para representar o Centro Paula Souza. A probabilidade
14
de que o aluno escolhido seja uma mulher é
6
c)
7 16 43
a) . d) .
11 45 90
d)
14 37 28
b) . e) .
5 90 45
e)
7 19
c) .
45
Resolução: A

D O Mulher
Resolução: D
A probabilidade de se retirar dois fuzis sem defeito:
C2,2 1 1 L P RdeOSão Paulo: 1 ⋅ 3 =1
IA F E 13 15 15
P1   
C8,2 8! 28
2! (8  2)!
R Mulher de Sorocaba: ⋅ =
3 3 9

S
TE

Logo, a probabilidade de se retirar de pelo menos uma arma ser 1 2 1


defeituosa ou ser pistola é igual a: Mulher da Baixada Santista: ⋅ =

SO
3 4 6
1 27
MA

P  1 P’  1  . 1 1 1 43
Somando: + + = .
28 28 5 9 6 90

R
Exercício Resolvido
Exercício Resolvido
04. (UERJ 2005)
02. O Centro Paula Souza administra Escolas Técnicas (Etecs) e
Faculdades de Tecnologia (Fatecs) estaduais em 149 municípios, R
no Estado de São Paulo. Para participar de um simpósio sobre
MA

educação a distância, a Fatec São Paulo enviou cinco alunos, sendo


SO

dois homens; a Fatec Sorocaba enviou três alunos, sendo uma


mulher; e a Fatec da Baixada Santista enviou quatro alunos, sendo
dois homens. Para a abertura desse simpósio, será selecionada, ao
T

acaso, uma dessas Fatecs e dela se escolherá, também ao acaso, O poliedro acima, com exatamente trinta faces quadrangulares
R
E

F
um aluno para representar o Centro Paula Souza. A probabilidade numeradas de 1 a 30, é usado como um dado, em um jogo.
de que o aluno escolhido seja uma mulher é IA O
Admita que esse dado seja perfeitamente equilibrado e que, ao ser

a)
16
45
d)
43
90
L D O Calcule:
PR
lançado, cada face tenha a mesma probabilidade de ser sorteada.

37 28 a) a probabilidade de obter um número primo ou múltiplo de 5,


b) e) ao lançar esse dado uma única vez;
90 45
19 b) o número de vértices do poliedro.
c)
45 Resolução:
7
Resolução: D a) ;
15
1 3 1 Experiência: sortear um número do conjunto {1, 2, 3, ..., 30} →
Mulher de São Paulo: . =
3 5 5 30 resultados;
Evento: do conjunto anterior, sortear um número primo ou
1 1 1 múltiplo de 5: {2, 3, 5, 7, 10, 11, 13, 15, 17, 19, 20, 23, 25, 29}
Mulher de Sorocaba: . =
3 3 9 → 14 resultados.
14 7
Probabilidade: =
p = .
1 2 1 30 15
Mulher da Baixada Santista: . =
3 4 6 b) 32.
1 1 1 43
Somando:    . 30 faces quadrangulares → 30 × 4 = 120 lados → 60 arestas.
5 9 6 90
F = 30, A = 60;
V + F = A + 2 → V = 32.

PROMILITARES.COM.BR 71

PM_BOOK05_MAT.indb 71 01/11/2021 13:42:06


PROBABILIDADE

Exercício Resolvido 07. (ESA) Um aluno da ESA tem uma habilidade muito boa nas provas
de tiro com pistola, possuindo um índice de acerto no alvo de quatro
05. (UFF 2004) Considere o conjunto S = {1, 2, 3, 8, 9}. em cada cinco tiros. Se ele atirou duas vezes, a probabilidade de que
Seja M o conjunto de todos os números de três algarismos distintos ele tenha errado os dois tiros é:
que podem ser formados com os elementos de S. a) 16/25 b) 8/25 c) 1/5 d) 2/5 e) 1/25
a) Determine o número de elementos de M.
08. (EEAR) Com os dígitos 1, 2, 3, 4 e 5 são formados números de 4
b) Escolhendo-se, ao acaso, um elemento de M, qual a
algarismos distintos. Um deles é escolhido ao acaso. A probabilidade
probabilidade de o elemento escolhido ser um múltiplo de 3?
desse número ser par é:
a) 1/3 b) 2/5 c) 3/5 d) 2/3
Resolução:
S = {1, 2, 3, 8, 9} 09. (EEAR) Uma urna contém 3 bolas verdes e 4 amarelas. Ao retirar,
a) Elementos de M: _ _ _ (3 algarismos distintos) 5 × 4 × 3 = 60 sem reposição, duas bolas, a probabilidade delas serem amarelas é
números. a) 2/7 b) 3/7 c) 4/7 d) 6/7
b) Experiência: escolher um elemento de M ⇒ 60 resultados.
Evento: escolher um múltiplo de 3: 10. (EEAR) No lançamento de 2 dados, qual a probabilidade de que a
soma dos dois seja 8, supondo que no 1º resultado tenha sido menor
Algarismos {1, 2, 3} ⇒ 3! = 6 casos que 5?
Algarismos {1, 2, 9} ⇒ 3! = 6 casos a) 10% b) 12,5% c) 14% d) 16,5%
Algarismos {1, 3, 9} ⇒ 3! = 6 casos
Algarismos {1, 8, 9} ⇒ 3! = 6 casos
Então, 6 × 4 = 24 resultados. L DO PRO EXERCÍCIOS DE

Probabilidade:=
p
24 2
=
60 5
.
R
I A TREINAMENTO
FE

S
TE

01. (EEAR) Dois dados são lançados conjuntamente. A probabilidade

SO
EXERCÍCIOS DE
da soma dos números das faces superiores ser 10 ou maior que 10 é
MA

FIXAÇÃO
a) 5/36 b) 1/12 c) 1/6 d) 1/3

R
02. (EEAR) Dentre as 7 notas musicais, dois músicos escolherão,
individualmente, uma nota. A probabilidade de que eles escolham
notas iguais é
01. (EEAR) Em um grupo de jovens, 25 praticam futebol, 20 praticam
vôlei, 5 praticam futebol e vôlei e 10 não praticam nenhum esporte. a) 1/7 b) 2/7 c) 1/49 d) 2/49
Ao selecionar, aleatoriamente, um jovem desse grupo, a probabilidade R
dele praticar apenas futebol é 03. (EEAR) Em um lote com 250 peças, foi constatado que existem
MA

a) 0,6 b) 0,5 c) 0,4 d) 0,3 exatamente seis defeituosas. Retirando-se, ao acaso, uma peça desse
SO

lote, a probabilidade de que ela seja perfeita é de %.


02. Um número inteiro é escolhido ao acaso entre 1 e 20 inclusive. a) 82,3 b) 85,5 c) 97,6 d) 98,2
Qual a probabilidade de o número escolhido ser um quadrado
T

perfeito?
R 04. (EEAR) Uma bomba está prestes a explodir e um militar tentará
E

a)
1
b)
1
c)
3
I d)A 1
F
desativá-la cortando um de seus fios de cada vez. Ela possui 10 (dez)
20 10 20
L
5
O fios, dos quais 1 (um) a desativa, 7 (sete) causam a explosão e os

PR
outros 2 (dois) não causam efeito algum. A probabilidade do militar
03. (EEAR) Uma urna contém bolas verdes e azuis. Sabe-se queD
a O
ter uma segunda chance para desativar a bomba é de %.
a) 5
probabilidade de se retirar uma bola azul é de 6/11 A probabilidade de b) 10 c) 15 d) 20
ser retirada, em uma única tentativa, uma bola verde é de
05. (EEAR) Em um lançamento simultâneo de dois dados, sabe-se que
a) 1/11 b) 2/11 c) 4/11 d) 5/11
ocorreram somente números diferentes de 1 e 4. A probabilidade de
o produto formado por esses dois números ser par é
04. Jogando-se um dado comum de seis faces e não viciado, a
probabilidade de ocorrer um número primo e maior que 4 é de a) 1/2 b) 3/4 c) 3/5 d) 7/12
a) 1/3 b) 1/2 c) 1/6 d) 2/3 e) 5/6
06. (EEAR) Para participar de um sorteio, um grupo de 152 pessoas
respondeu à pergunta: “Você é fumante?”. Se 40 pessoas responderam
05. (EEAR) Dentre os números inteiros de 1 a 50, um número é “SIM”, a probabilidade da pessoa sorteada não ser fumante é
escolhido aleatoriamente. Qual a probabilidade de ele divisível por 5?
a) 11/16. b) 17/18. c) 15/17. d) 14/19.
a) 1/50 b) 1/5 c) 1/2 d) 3/4
07. (EFOMM) Um atirador, em um único tiro, tem probabilidade de
06. (ESA) A probabilidade de um jogador de futebol marcar o gol ao 80% de acertar um específico tipo de alvo. Num exercício ele dá seis
cobrar um pênalti, é de 80%. Se esse jogador cobrar dois pênaltis tiros seguidos nesse mesmo tipo de alvo. Considerando-se que os tiros
consecutivos, a probabilidade dele fazer o gol, em ambas as cobranças, são independentes, em cálculo aproximado, qual é a probabilidade de
é igual a: o atirador errar o alvo exatamente duas vezes?
a) 16% c) 32% e) 80% a) 4,12% c) 24,58% e) 40,25%
b) 20% d) 64% b) 18,67% d) 27,29%

72 PROMILITARES.COM.BR

PM_BOOK05_MAT.indb 72 01/11/2021 13:42:07


PROBABILIDADE

08. (ESPCEX) Dispondo-se de duas urnas, com 4 fichas cada uma, 06. (AFA) Seja S o espaço amostral de um experimento aleatório e
numeradas de 1 a 4, realiza-se o experimento de retirar aleatoriamente A um evento de S. A probabilidade de ocorrer evento A é dada por
uma ficha de cada urna e somar os números indicados nas duas fichas n − 10
sorteadas. Nessas condições, a probabilidade de, em uma retirada, P(A) = 4 . O número máximo de elementos de A é
obter-se para a soma dos números das fichas um número primo é de: a) 10 b) 11 c) 14 d) 15
a) 1/4 b) 5/16 c) 9/16 d) 3/8 e) 3/4
07. (AFA) Em um balcão de supermercado, foram esquecidas 2
09. (AFA) Dois dados são lançados simultaneamente. Qual a sacolas. Uma continha 3 latas de atum, 2 latas de ervilha e 5 de
probabilidade da soma ser menor do que 4? sardinha; a outra, x latas de atum, 3 latas de ervilha e 3 de sardinha.
a) 1/6 b) 1/8 c) 1/12 d) 1/16 Escolhe-se ao acaso uma sacola e retira-se uma lata. Qual é o menor
valor de x para que a probabilidade de tratar-se de uma lata de atum
seja, no mínimo, 50%?
10. (ESA) Em uma urna existem 5 bolinhas numeradas de 1 a 5.
Quatro dessas bolinhas são retiradas, uma a uma, sem reposição. Qual a) 13 b) 14 c) 15 d) 16
a probabilidade de que a sequência de números observados, nessas
retiradas, seja crescente? 08. (AFA) Em uma urna contendo 12 bolas amarelas, 15 bolas
1 1 1 2 1 brancas e 18 bolas pretas, a probabilidade de retirar três bolas de
a) b) c) d) e) cores diferentes é:
12 24 36 5 5
a) 38% b) 22.8% c) 11.4% d) 1/376

09. (ESPCEX) Enrico guardou moedas em um cofrinho por um certo


EXERCÍCIOS DE período de tempo e, ao abri-lo, constatou que:

COMBATE L D O I. Po cofrinho
R O contém apenas moedas de R$ 0,25, R$ 0,50 e R$ 1,00.
IA F E retirar uma moeda de R$0,50.
II. a probabilidade de retirar uma moeda de R$0,25 é o triplo da
probabilidade de
R III. se forem retiradas 21 moedas de R$ 0,25 desse cofrinho, a
01. (AFA) Com os dígitos 1, 2, 3, 4 e 5 são formados números de 4

S
TE

9
algarismos distintos. Um deles é escolhido ao acaso. A probabilidade probabilidade de retirar uma moeda de R$ 0,50 passa a ser .
40

SO
desse número ser par é:
IV. se forem retiradas 9 moedas de R$ 0,50 desse cofrinho, a
MA

a) 1/3 b) 2/5 c) 3/5 d) 2/3 1


probabilidade de retirar uma moeda de R$ 1,00 passa a ser .
4

R
02. (AFA) Uma urna A contém x bolas vermelhas e y bolas brancas. Diante dessas constatações, podemos afirmar que a quantidade de
Uma urna B contém z bolas vermelhas e w bolas brancas. Uma bola é moedas de R$ 0,25 nesse cofrinho era
retirada da urna A e colocada na urna B e, então, uma bola é retirada
da urna B. A probabilidade dessa última bola ser vermelha é: a) 27. b) 32. c) 33. d) 81. e) 108.
z +1
a) 10. (ESPCEX) Em uma população de homens e mulheres, 60% são
z + 1+ w R
mulheres, sendo 10% delas vegetarianas. Sabe-se, ainda, que 5%
MA

x+z dos homens dessa população também são vegetarianos. Dessa forma,
SO

b) selecionando-se uma pessoa dessa população ao acaso e verificando-


x+y+z+w
se que ela é vegetariana, qual é a probabilidade de que seja mulher?
1  x + xz + zy  a) 50% c) 75% e) 85%
c) ⋅
T


S

x + y  z + w +1 
R b) 70% d) 80%
E

d) 1  x − xz − zy 
IA F
⋅ 
O
L D O RESOLUÇÃO
PR
x − y  z − w −1 
EM VÍDEO
03. (AFA) Numa urna são colocados números maiores que 2500 Abra o ProApp, leia o QR Code, assista à resolução
de cada exercício e AVANCE NOS ESTUDOS!
formados com os algarismos 1, 2 ,3, 4 e 5 sem repetição. A
probabilidade de se retirar dessa urna um número com apenas quatro
algarismos é:
a) 0,333... c) 0,373737... GABARITO
b) 0,343434... d) 0,393939...
EXERCÍCIOS DE FIXAÇÃO
04. (AFA) A probabilidade de observarmos um número na face superior 01. C 04. C 07. E 10. B
de um dado viciado é diretamente proporcional a esse número. Ao 02. D 05. B 08. B
lançarmos esse dado, a probabilidade de ocorrer um número par é:
03. D 06. D 09. A
a) 1/2 b) 11/21 c) 4/7 d) 13/21
EXERCÍCIOS DE TREINAMENTO
01. C 04. D 07. C 10. B
05. (AFA) Na Academia da Força Aérea, existem 8 professores de
matemática e 6 de física. Para participar de um congresso no Rio 02. A 05. B 08. C
de Janeiro, deverá ser formada uma comissão de 4 professores. 03. C 06. D 09. C
A probabilidade de participarem dessa comissão 3 professores de
EXERCÍCIOS DE COMBATE
matemática e 1 de física é de
01. B 04. C 07. B 10. C
a) 3/1001 c) 21/286
02. C 05. B 08. B
b) 48/143 d) 4/13
03. D 06. C 09. D

PROMILITARES.COM.BR 73

PM_BOOK05_MAT.indb 73 01/11/2021 13:42:08


PROBABILIDADE

ANOTAÇÕES

L DO PRO
I A FE
R

S
TE

SO
MA

R
R
MA

SO
T

R
E

IA F
L DO PRO

74 PROMILITARES.COM.BR

PM_BOOK05_MAT.indb 74 01/11/2021 13:42:08


NOÇÕES DE ESTATÍSTICA

CONCEITOS BÁSICOS Exemplo:


Um pesquisador, contratado pela empresa de cervejas, deseja
estudar quantas cervejas por semana seus clientes bebem. A amostra
ESTATÍSTICA
com 10 clientes resultou nos seguintes números: 2, 3, 7, 1, 10, 11,
É a ciência que utiliza a coleta de dados, sua classificação, sua 5, 2, 8, 9.
apresentação, sua análise e sua interpretação para se tomar algum
A amplitude desta amostra é igual a 11 – 1 = 10.
tipo de decisão.

ESTATÍSTICA DESCRITIVA ROL

D O emPtabelas
Os dados coletados em uma amostra podem ser organizados
É o ramo da Estatística que se ocupa em organizar e descrever os
dados que podem ser expressos por tabelas e gráficos.
L R crescentes
sequências O
ou gráficos. Para isso, antes devemos organizá-los em

I A F ou decrescentes denominadas Rol.

ESTATÍSTICA INFERENCIAL
R
Exemplo:
E
No exemplo anterior organizando em ordem crescente temos: 1,
É o ramo da Estatística que utiliza técnicas de análise e interpreta-

S
TE

2, 2, 3, 5, 7, 8, 9, 10, 11.
ção de dados, a partir de uma amostra de uma população, e fornece

SO
conclusões sobre este conjunto.
DADOS BRUTOS
MA

POPULAÇÃO Podemos considerar como dados brutos aqueles que não estão

R
numericamente organizados.
Na coleta de dados sobre determinado assunto, chama-se
população estatística, o conjunto formado por todos os elementos Exemplo:
que possam oferecer dados relativos ao assunto em questão. 2, 3, 7, 1, 10, 11, 5, 2, 8, 9.
Podemos dizer que população é qualquer conjunto que reúna
todos os elementos que tenham pelo menos uma característica VARIÁVEIS R
comum, objeto de estudo.
Uma outra definição que aparece na análise de dados estatísticos
MA

é o conceito de variável.
SO

AMOSTRA Uma variável é quantitativa quando seus valores podem ser


É um subconjunto de uma população. A seleção da amostra pode representados por contagem (variáveis quantitativas discretas) ou
ser feita de várias maneiras, dependendo, entre outros fatores, do grau mensuração (variáveis quantitativas contínuas).
T

R
de conhecimento que temos da população, da quantidade de recursos Uma variável é qualitativa quando apresentam como resultado um
E

disponíveis e outros fatores. A seleção da amostra deve fornecer um


IA F
atributo, qualidade ou preferência de um entrevistado.
subconjunto de valores mais parecido possível com a população original.
L D OSÉRIES O
Exemplo:
Uma pesquisa típica de audiência na televisão utiliza uma amostra
P RESTATÍSTICAS
Uma série estatística é um conjunto de dados estatísticos que
de 5000 lares e, com base nestes dados, formula conclusões acerca de
fazem referência aos seguintes fatores: tempo, local e fenômeno.
uma população de todos os milhões de lares no país.
Os exemplos mais comuns de séries estatísticas são:

CENSO • a série temporal, cronológica ou histórica;

É o método utilizado para um trabalho com uma população. • varia o tempo;


• a série geográfica, territorial ou espacial;
DADOS ESTATÍSTICOS • varia o local;
Os dados são denominados quantitativos quando são representa- • a série específica ou especificativa;
dos por números ou medidas, como por exemplo as alturas de uma • varia o fenômeno.
população, o número de filhos e o salário bruto. Quando os dados
representam contagens são discretos e quando representam mensu-
rações são contínuos. Elementos essenciais em uma tabela:
Os dados são chamados de qualitativos ou nominais quando • Título: é a indicação contida na parte superior da tabela,
são definidos por categorias tais como: cor dos olhos, sexo, nível de onde deve estar definido o fato observado;
escolaridade, naturalidade. • Corpo: é constituído por linhas e colunas, que fornecem o
conteúdo das informações prestadas;
AMPLITUDE DE UMA AMOSTRA • Cabeçalho: é a parte da tabela que apresenta a natureza do
A amplitude total dos dados é a diferença entre o valor máximo e que contém cada coluna.
o valor mínimo da amostra.

PROMILITARES.COM.BR 75

PM_BOOK05_MAT.indb 75 01/11/2021 13:42:08


NOÇÕES DE ESTATÍSTICA

Há ainda os elementos que complementam a tabela como, por a) calcular a amplitude da amostra; 60 – 48 = 12 cm
exemplo: b) dividir o intervalo em subintervalos de mesmo comprimento.
• Fonte: designação da instituição que forneceu os dados Como exemplo, tomemos 4 subintervalos de comprimento igual
estatísticos. a 3: [48, 51[; [51,54[; [54,57[; [57,60]. Esses subintervalos são
Exemplo: chamados de classes e o comprimento de cada um é chamado de
amplitude da classe;
Datafolha, IBOPE, IBGE, INPE e etc.
c) conte quantas observações se situam em cada classe, respeitando
• Notas: esclarecimentos de natureza geral. os intervalos fechados à esquerda e abertos à direita, e coloque as
observações numa tabela do tipo abaixo.
TABELAS DE FREQUÊNCIAS
Um processo que possibilita uma leitura mais sucinta dos dados é COMPRIMENTO fi fri fac frac
a construção de uma tabela de frequências.
Exemplo 1: 5
[48, 51[ 5 = 0=
, 25 25% 5 25%
Uma entrevista com 20 pessoas é realizada no estado do Rio de 20
Janeiro. O objetivo da pesquisa era saber qual o time do entrevistado.
Dos 20 entrevistados foram encontrados os seguintes resultados 6
[51, 54[ 6 = 0=
, 3 30% 11 55%
para a frequência absoluta dos entrevistados: 20
Flamengo (f1 = 10)
Vasco (f2 = 6) 4
[54, 57[ 4 = 0=
, 20 20% 15 75%

DO PRO
Fluminense (f3 = 2) 20
Botafogo (f4 = 1)
A L
Note que f1 + f2 + f3 + f4 = 20.
Ipor uma variável
[57, 60] FE 5
5
= 0=
20
, 25 25% 20 100%

R
Definimos frequência relativa absoluta assumido
como a razão entre a frequência absoluta e o número total de dados. Total 20

S
TE

fi

SO
fri = Onde,
n
MA

fi (frequência absoluta): frequência que o intervalo aparece na


distribuição;

R
FREQUÊNCIA FREQUÊNCIA fri (frequência relativa): é a percentagem do valor dos dados em
TIME ABSOLUTA RELATIVA PORCENTAGEM relação ao total da amostra;
(fi) (fri)
fac (frequência acumulada): é a soma das frequências absolutas
começando pelo menor valor;
10
Flamengo 10 50% frac (frequência relativa acumulada): é a porcentagem do valor
20 R
das frequências acumuladas em relação ao total da amostra.
MA

SO

6
Vasco 6
20
30% GRÁFICOS
T

GRÁFICO EM LINHA
S

3
R
E

Fluminense 3 15%
E

F
Esse tipo de gráfico é usado sobretudo quando temos observações
20
IA temporais de uma variável em estudo e desejamos representá-la

Botafogo 1
1 L DO PRO
5%
no tempo (abscissa) a fim de reconhecer possíveis tendências e/ou
sazonalidade (comportamento periódicos repetidos). O exemplo a
20 seguir ilustra bem a utilidade do gráfico em linha para a evolução do
preço da ação da empresa PETRO S.A.
TOTAL 20 1 100%

Exemplo 2:
Para avaliar o tempo de permanência em um supermercado,
o gerente mensurou em minutos o tempo de permanência de 20
clientes na loja. Os tempos estão representados na tabela abaixo.

49 52 56 52 50

54 57 60 48 59

48 49 57 53 55

51 53 52 55 57

GRÁFICO EM BARRAS HORIZONTAIS


Para representar esses dados em uma tabela de frequência Os dados que estejam organizados em colunas ou linhas em uma
devemos: tabela podem ser representados em um gráfico de barras horizontais.

76 PROMILITARES.COM.BR

PM_BOOK05_MAT.indb 76 01/11/2021 13:42:13


NOÇÕES DE ESTATÍSTICA

Gráficos de barras ilustram comparações entre itens individuais. Voltando ao exemplo 2:


Considere a utilização de um gráfico de barras horizontais quando:
TEMPO fi
• os rótulos dos eixos forem longos;
• os valores mostrados forem durações. [48, 51[ 5

[51, 54[ 6

[54, 57[ 4

[57, 60] 5

TOTAL 20

O Histograma referente à tabela é

GRÁFICO DE COLUNAS
L DO PRO
A ideia é expressar informações individualizadas, e representadas

I A
por barras cuja altura representa a frequência nas categorias. Vejamos FE
R
o exemplo a seguir, representando em barras o número de cópias de
jornais (em milhares de exemplares) em alguns países.

S
TE

SO
MA

R
POLÍGONO DE FREQUÊNCIA R
MA

O polígono de frequência é obtido unindo-se os pontos médios


SO

da parte superior de cada retângulo do histograma com segmentos de


reta. É importante notar que tanto o histograma quanto o polígono de
frequência indicam a frequência absoluta de cada classe.
T

Voltando ao exemplo 2 temos:


R
E

GRÁFICO EM SETORES
IA F
L DO PRO
É utilizado quando se deseja mostrar partes do total, conforme
ocorre em produções, vendas e orçamentos de países e etc.

O gráfico em linhas representativo de uma distribuição de


frequências acumuladas é chamado polígono de frequências
acumuladas ou ogiva de Galton. No caso de dados agrupados em
intervalos de classe, os pontos do gráfico são os pontos correspondentes
aos limites superiores das classes das bases superiores dos retângulos.

HISTOGRAMA CARTOGRAMA
Quando as classes são intervalos reais, a interpretação da Um cartograma é um mapa que mostra informação quantitativa
distribuição de frequências em um sistema de eixos é feita por um mantendo um certo grau de precisão geográfica das unidades
tipo de gráfico chamado Histograma. espaciais mapeadas.

PROMILITARES.COM.BR 77

PM_BOOK05_MAT.indb 77 01/11/2021 13:42:14


NOÇÕES DE ESTATÍSTICA

% de Prefeitos Eleitos em 2012: PT Exemplo:


12  13  14  14  14  15  15  16  16  17
x  14, 6
10

PROPRIEDADES DA MÉDIA
1. Ao adicionarmos um mesmo valor a cada um dos valores
assumidos pela variável, a média aritmética fica adicionada
desse valor.
2. Ao multiplicarmos cada um dos valores assumidos pela variável
por um mesmo valor, a média aritmética fica multiplicada por
esse valor.
Seja x uma variável quantitativa e x1, x2, x3, ..., xk os valores assumidos
por essa variável, com frequências absolutas respectivamente iguais
a n1, n2, n3, ..., nk. A média aritmética (x) desses valores é igual à
soma de cada um os valores assumidos pela variável multiplicado
pela sua frequência dividida pela soma das frequências, ou seja,
k

 x n
i1
i i
x1  n1  x 2  n2    xk  nk .
x k

n1  n2    nk
n
D O PA média
i

R Oaritmética é muito influenciada por valores discrepantes


i1

A L
PICTOGRAMA I (“outliers”). FE
R tipos de gráficos
É comum em jornais e revistas ilustrar os vários
MEDIANA
com figuras relacionadas ao assunto, tornando-os mais atraentes.

S
TE

Esses são os pictogramas. Seja x1 ≤ x2 ≤ x3 ≤ ... ≤ xn os valores ordenados assumidos pela

SO
variável quantitativa x. A mediana (Me) é dada por:
MA

x  n+1 , se n é ímpar
  2 

R

Me =  x n + x n
   
  2   +1
2 
 , se n é par
 2

Dessa forma, a mediana é tal que a quantidade de valores


R
menores ou iguais à mediana é igual à quantidade de valores maiores
MA

ou iguais à mediana.
SO

A mediana é uma medida de centralidade menos sensível a


valores discrepantes.
Exemplo:
T

x5  x 6 14  15
R No nosso exemplo a mediana é   14, 5.
E

IA F 2 2

O
L D O APmodaR de um conjunto é o valor que ocorre mais vezes ou de
MODA

maior frequência simples (absoluta ou relativa) numa distribuição de


MEDIDAS DE CENTRALIDADE: MÉDIA frequências.
ARITMÉTICA, MÉDIA ARITMÉTICA No nosso exemplo a moda é igual a 14.

PONDERADA, MEDIANA, MODA A moda pode também não existir ou não ser única.

Para apresentar os conceitos a seguir, vamos considerar um grupo Exemplo:


de 10 estudantes com as seguintes idades: 0, 1, 1, 2, 2, 3 tem modas 1 e 2 (bimodal).
12, 13, 14, 14, 14, 15, 15, 16, 16, 17.
MEDIDAS DE DISPERSÃO: DESVIO
MÉDIA ARITMÉTICA E MÉDIO, VARIÂNCIA, DESVIO PADRÃO
MÉDIA ARITMÉTICA PONDERADA Considere uma turma de 5 alunos em que todos tiraram nota 5 e
Seja x uma variável quantitativa e x1, x2, x3, ..., xn os valores outra com a mesma quantidade de alunos com as seguintes notas: 1, 3,
assumidos por essa variável. A média aritmética (x) de x é igual a soma 5, 7 e 9. Essas duas turmas têm a mesma média aritmética e a mesma
de todos os valores assumidos pela variável dividida pelo número de mediana que é 5. Mas a dispersão dos valores é completamente
valores, ou seja, diferente e pode ser calculado.
n

x
i1
i
x1  x 2    xn
DESVIO MÉDIO ABSOLUTO (DMA)
x  O desvio em relação à média aritmética é a diferença entre cada
n n
valor e a média aritmética.

78 PROMILITARES.COM.BR

PM_BOOK05_MAT.indb 78 01/11/2021 13:42:17


NOÇÕES DE ESTATÍSTICA

δi = xi – x PROPRIEDADES DO DESVIO PADRÃO


Se adicionarmos um mesmo valor a cada um dos valores assumidos
A soma de todos os desvios em relação à média aritmética é
pela variável, o desvio padrão não se altera.
sempre nula.
Se multiplicarmos cada um dos valores assumidos pela variável
O desvio médio absoluto (DMA) é a média aritmética dos desvios
por um mesmo valor, o desvio padrão fica multiplicado por esse valor.
em módulo.
n n
Observação
   x x
i1
i
i1
i
Se o desvio padrão for calculado sobre uma amostra em vez
DMA  
n n de sobre toda a população, teremos então o chamado desvio
n n
O desvio médio absoluto da segunda turma do nosso exemplo é
4  1 0  1 4  2
i  x  x 
i
2

DMA   2. padrão amostral que é dado por S  i1


 i1
5 n 1 n 1
VARIÂNCIA POPULACIONAL k k

A variância populacional (σ²) é a média aritmética da soma



i1
2
i  fi  x  x 
i1
i
2
 fi
ou para dados agrupados por S   .
dos quadrados dos desvios em relação à média de um conjunto de n 1 n 1
números.
n
n n A razão entre o desvio padrão amostral e o populacional é
  x  x  n −1
2
i2 i chamado fator de correção de Bessel.
2  i1
 i1
n n
L D O normal
PSe Ros(unimodal,
O
valores se distribuem de acordo com uma distribuição
gaussiana, simétrica, de afunilamento médio)
Outra fórmula para o cálculo da variância populacional é
I A F valores encontram-se a uma distância da média
podemos dizer que:
• 68% dos E
  n  
2

1

n

  x 
i

 R inferior a um desvio padrão;
2

   xi  2  i1 
.

S
TE

n  i1 n  • 95% dos valores encontram-se a uma distância da média

SO
inferior a duas vezes o desvio padrão;
MA

PROPRIEDADES DA VARIÂNCIA • 99,7% dos valores encontram-se a uma distância da média


inferior a três vezes o desvio padrão.

R
Se adicionarmos um mesmo valor a cada um dos valores assumidos
pela variável, a variância não se altera. Esta informação é conhecida como a regra dos “68 - 95 - 99,7”.
Se multiplicarmos cada um dos valores assumidos pela variável
por um mesmo valor, a variância fica multiplicada pelo quadrado desse
valor.
Se a variância for calculada sobre uma amostra em vez de sobre R
toda a população, teremos então a chamada variância amostral que
MA

SO
n

2
 x  x 
i1
i
2

é dada por S  .
n 1
T

DESVIO PADRÃO POPULACIONAL R


E

O desvio padrão populacional (σ) é a média quadrática dos IA F


L D O Exemplo: O Fonte: Wikipédia

PR
desvios em relação à média de um conjunto de números ou a raiz
quadrada da variância.
n n


i1
2
i  x  x 
i1
i
2 Sejam 10 estudantes com as seguintes idades: 13, 13, 14, 14, 14,
14, 15, 15, 15, 16.
 
n n
IDADE Ni δi δ²i
Outra fórmula para o cálculo do desvio padrão populacional é
 13 2 −1,3 1,69
 
2
 n

1

 n
  x  

i
14 4 −0,3 0,09
 
 xi 
n  i1
2  i1
n
.
 15 3 0,7 0,49
Se os dados forem agrupados em intervalos de classe, então o 16 1 1,7 2,89
quadrado de cada desvio deve ser multiplicado pela sua frequência
absoluta (ni) ou relativa (fi).
k O desvio padrão populacional é
 x  x 
2
 fi
i k 2  1, 69  4  0, 09  3  0, 49  1 2, 89
 i1
n
  x  x 
i1
i
2
 fri 
2  4  3 1
 0, 81  0, 9.

PROMILITARES.COM.BR 79

PM_BOOK05_MAT.indb 79 01/11/2021 13:42:24


NOÇÕES DE ESTATÍSTICA

Exercício Resolvido Exercício Resolvido

01. As idades dos atletas que participaram da Seleção Brasileira 03. A tabela seguinte informa a quantidade de pessoas que
Masculina de Basquete, convocados para a preparação dos Jogos compraram ingressos antecipados de um determinado show, cujos
Olímpicos 2016, variaram de 24 a 36 anos, como se pode observar preços eram modificados semanalmente.
na tabela a seguir:
VALOR DO INGRESSO (R$) NÚMERO DE PESSOAS
Idade (anos) 24 26 28 30 32 33 35 36
50 |– 75 300
Número de atletas 3 1 1 1 1 4 1 2
75 |– 100 640
De acordo com a tabela, a média, a mediana e a moda dessas 100 |– 125 500
idades são, respectivamente:
125 |– 150 1.310
a) 30,5; 32,5 e 33
b) 31; 32 e 33 150 |– 175 850
c) 31,5; 31 e 33
d) 30,5; 31 e 24
∑ = 3.600
e) 31; 24 e 33 O percentual de pessoas que adquiriram o ingresso por menos de
R$ 125,00 foi
Resolução: A a) 40% c) 50%

D O Resolução:
b) 45% d) 55%
Calculando:

L
3 ⋅ 24 + 26 + 28 + 30 + 32 + 4 ⋅ 33 + 35 + 2 ⋅ 36
P R OA
Média = 30,5
I
3 + 1+ 1+ 1+ 1+ 4 + 1+ 2 A FE
Tem-se que a resposta é dada por
300 + 640 + 500
⋅ 100% =
40%.
Já a mediana será a média entre o sétimo e R
o oitavo termo, ou 3.600
32 + 33

S
TE

=
seja: Mediana = 32,5 . E a moda será o termo que mais
2

SO
aparece, ou seja, 33 anos. Exercício Resolvido
MA

04. O Centro de Estudos, Resposta e Tratamento de Incidentes de


Exercício Resolvido

R
Segurança no Brasil (CERT.br) é responsável por tratar incidentes de
segurança em computadores e redes conectadas à Internet no Brasil.
02. Uma agência de viagem entrevistou 50 idosos perguntando- A tabela abaixo apresenta o número de mensagens não solicitadas
lhes quantas viagens eles tinham feito para o exterior. O gráfico a (spams) notificadas ao CERT.br no ano de 2015, por trimestre. Qual
seguir apresenta os resultados dessas entrevistas. dos gráficos abaixo representa os dados dessa tabela?
TRIMESTRE R NOTIFICAÇÕES
MA

4° T 135.335
SO

3° T 171.523
2° T 154.866
T

1° T 249.743
R
E

IA a) F d)

L DO PRO

Baseando-se na informação do gráfico, a mediana do número de b) e)


vezes que esses idosos viajaram para o exterior é de
a) 0,5
b) 0,0
c) 2,0
d) 1,0
e) 1,5 c)

Resolução: A
Como o número de observações é par, segue que a mediana
corresponde à média aritmética simples das observações de ordem
0 +1
25 e 26, ou seja, = 0,5.
2

80 PROMILITARES.COM.BR

PM_BOOK05_MAT.indb 80 01/11/2021 13:42:25


NOÇÕES DE ESTATÍSTICA

02. (EEAR) Os resultados da prova de Ciências aplicada a uma turma


Resolução: D de um certo colégio estão apresentados no gráfico. Baseado neste
Desde que o total de notificações é igual a 135335 + 171523 gráfico, podemos afirmar que a porcentagem de alunos dessa turma
+ 154866 + 249743 = 711467, tem-se que o percentual com nota inferior a 5,0, nessa prova de Ciências, foi de
135335
correspondente ao quarto trimestre é ⋅ 100% ≅ 19%.
711467
Portanto, o gráfico que representa os dados da tabela é o da
alternativa D.

Exercício Resolvido

05. O número de atletas nas Olimpíadas vem aumentando nos a) 37,5%


últimos anos, como mostra o gráfico. Mais de 10.000 atletas b) 42,5%
participaram dos Jogos Olímpicos de Sydney, em 2000.
c) 47,5%
d) 52,5%

03. (EEAR) A altura de 80 homens de uma comunidade está distribuída


de acordo com a tabela. A porcentagem de homens com altura maior
ou igual a 1,80 m é

L D O P R OALTURA (m) NÚMERO DE HOMENS

IA F E 1,70
1,60 |–– 1,65 04

Nas últimas cinco Olimpíadas, esse aumento R


1,65 |–– 12
ocorreu devido ao
crescimento da participação de 1,70 |–– 1,75 18

S
TE

a) homens e mulheres, na mesma proporção. 1,75 |–– 1,80 26

SO
b) homens, pois a de mulheres vem diminuindo a cada Olimpíada. 1,80 |–– 1,85 10
MA

c) homens, pois a de mulheres praticamente não se alterou. 1,85 |–– 1,90 08

R
d) mulheres, pois a de homens vem diminuindo a cada Olimpíada. 1,90 |–– 1,95 02
e) mulheres, pois a de homens praticamente não se alterou. Total 80

Resolução: E a) 25%
Na olimpíada de 1984, realizada em Los Angeles, 6.434 homens b) 30% R
e 1498 mulheres participaram, totalizando 7932 atletas. Desse c) 60%
MA

modo, como nos jogos de Sydney tivemos 6416 homens e 3905


SO

d) 75%
mulheres, num total de 10.321 atletas, segue que o aumento no
número de atletas ocorreu devido ao crescimento da participação
04. (ESA) Numa sala de aula, a média das idades dos 50 alunos era
de mulheres, pois a de homens praticamente não se alterou.
T

de 22,5 anos. No cálculo da média, foram consideradas idades com


S

R anos completos. Transcorridas algumas semanas, houve a desistência


E

IA F
de um aluno e a média das idades caiu para 22 anos. Considerando-se

L DO PRO
EXERCÍCIOS DE que nesse período nenhum dos alunos da turma fez aniversário, então

FIXAÇÃO
a idade do aluno que desistiu é igual a:
a) 47 anos.
b) 45 anos.
c) 37 anos.
01. (EEAR) Numa prova de matemática, três classes obtiveram as d) 35 anos.
seguintes médias e desvios:
e) 27 anos.
classe A: x = 4,5 e δ =2,5
classe B: x = 4,5 e δ =3,1 05. (ESA) A média aritmética das notas de Matemática em uma turma
classe C: x = 4,5 e δ =2,8 de 25 alunos em um dos doze Colégios Militares existentes no Brasil
diminui em 0,1, se alterarmos uma das notas para 6,8. A referida nota
Se for sorteado um aluno em cada classe, em qual delas é mais
sem ser alterada é:
provável que a nota desse aluno esteja entre 3,0 e 6,0?
a) 4,3
a) Classe A
b) 8,8
b) Classe B
c) 4,8
c) Classe C
d) 9,3
d) Classes B e C
e) 9,8

PROMILITARES.COM.BR 81

PM_BOOK05_MAT.indb 81 01/11/2021 13:42:26


NOÇÕES DE ESTATÍSTICA

06. (ESA) Em uma turma a média aritmética das notas é 7,5. Sabe-se 02. (EEAR) Um teste de inteligência, aplicado aos alunos das 4as
que a média aritmética das notas das mulheres é 8 e das notas dos séries do Ensino Fundamental da Escola A, apresentou os seguintes
homens é 6. Se o número de mulheres excede o de homens em 8, resultados:
pode-se afirmar que o número total de alunos da turma é
Pontos nº de alunos Pontos nº de alunos
a) 4. d) 16.
b) 8. e) 20. 90 |–– 95 40 115 |–– 120 140
c) 12. 95 |–– 100 60 120 |–– 125 120
100 |–– 105 140 125 |–– 130 30
07. (ESA) A média aritmética de n números é 29. Retirando-se o
105 |–– 110 160 130 |–– 135 20
número 12 a média aumenta para 30. Podemos afirmar que o valor
de n será 110 |–– 115 180 135 |–– 140 10
a) 17.
A frequência relativa da classe modal é
b) 11.
a) 0,2
c) 42.
b) 0,22
d) 41.
c) 0,25
e) 18.
d) 0,5
08. (ESA) A média aritmética de todos os candidatos de um concurso
foi 9,0, dos candidatos selecionados foi 9,8 e dos eliminados foi 7,8. 03. (EEAR) Na distribuição a seguir, as 6 classes foram justapostas,
isto é, o limite superior de uma classe é o inferior da classe seguinte.
DO PRO
Qual o percentual de candidatos selecionados?
Se fi é frequência absoluta, então a frequência acumulada absoluta
a) 20%
A L da 4ª classe é
b) 25%
I FE
c) 30%
d) 50%
R Classes

fi
5 |−− 10

8
10 |−− 15

12
15 |−− 20

21
20|−− 30

16
25|−− 30

6
30 |−−

S
TE

e) 60% a) 57

SO
MA

b) 20
09. (ESA) Qual é a média de idade de um grupo em que há 6 pessoas
de 14 anos, 9 pessoas de 20 anos e 5 pessoas de 16 anos? c) 41

R
a) 17,2 anos d) 16
b) 18,1 anos
04. (EEAR) A média de um conjunto de quatro valores é 4,25. Se
c) 17,0 anos aumentarmos de 5 unidades o menor desses valores, e diminuirmos
d) 17,5 anos de 3 unidades o maior deles, a nova média será
e) 19,4 anos a) 4,75
R
MA

b) 5,25
SO

10. (ESA) Identifique a alternativa que apresenta a frequência absoluta c) 5


(fi) de um elemento (xi) cuja frequência relativa (fr) é igual a 25 % e
cujo total de elementos (N) da amostra é igual a 72. d) 5,5
T

a) 18.
R 05. (EEAR) A tabela traz as idades, em anos, dos filhos de 5 mães.
E

b) 36.
IA F
L DO PRO
c) 9. Nome da
Ana Márcia Cláudia Lúcia Eloisa
Mãe
d) 54.
Idade dos
e) 45. 7, 10, 12 11, 15 8, 10, 12 12, 14 9, 12, 15, 16, 18
filhos

A idade modal desses 15 filhos é inferior à idade média dos filhos de


EXERCÍCIOS DE Eloísa em _______ anos(s).

TREINAMENTO a) 4
b) 3
c) 2
01. (ESA) O exército realizou um concurso de seleção para contratar d) 1
sargentos e cabos. A prova geral foi igual para ambos. Compareceram
500 candidatos para sargento e 100 para cabo. Na prova, a média de 06. (EEAR) Na distribuição dos salários de 800 empregados de uma
todos os candidatos foi 4, porém, a média apenas entre os candidatos empresa, o ponto médio da 4ª classe é R$ 1400,00. Se as 8 classes
a sargento foi 3,8. Desse modo, qual foi a média entre os candidatos dessa distribuição têm a mesma amplitude de R$ 200,00 e são do tipo
a cabo? [a,b[, então a 6ª classe não inclui, com certeza, o salário de R$
a) 3,9 a) 1900,00
b) 1,0 b) 1850,00
c) 6,0 c) 1800,00
d) 4,8 d) 1750,00
e) 5

82 PROMILITARES.COM.BR

PM_BOOK05_MAT.indb 82 01/11/2021 13:42:26


NOÇÕES DE ESTATÍSTICA

07. (EEAR) Sejam x1, x2, x3, ..., x81 os valores ordenados de uma variável xi 1,5 2,0 2,5 3,0 3,5 4,0 4,5 5,0 5,5 6,0 6,5 7,0 7,5 8,0 8,5
X. A mediana desse conjunto de valores é igual a
fi 1 2 2 3 5 6 7 8 9 7 6 5 4 3 2
a) x41
b) x40 a) 14 c) 24
x 40 + x 41 b) 19 d) 29
c)
2
x 41 + x 42 02. (EEAR) Os dados de uma pesquisa, cujo objetivo era saber o
d) número de filhos, por família, realizada em uma certa comunidade,
2
estão na tabela:
08. (EEAR) Os resultados de uma pesquisa realizada com 20 alunos
de uma escola, a respeito da área da carreira pretendida, estão Nº DE FILHOS 0 1 2 3 4 5
apresentados na tabela:
Nº DE FAMÍLIAS 2 8 10 14 18 15
FREQUÊNCIA FREQUÊNCIA
ÁREA É correto afirmar que o número
ABSOLUTA RELATIVA
a) modal de filhos é maior que o número médio.
Humanas 8 M
b) médio de filhos coincide com o número modal.
Biológicas P 0,35
c) mediano e o número modal de filhos são iguais.
Exatas R S
d) modal, o mediano e o número médio de filhos são iguais.
Total 20 1,00

Os valores de M, P, R e S são, respectivamente:


L D O 03.P(EEAR)
R OQuando
comportamento
o objetivo de uma pesquisa é comparar o
de uma mesma variável em populações com números
a) 0,35; 5; 7 e 0,35.
IA F E a frequência mais conveniente é a
diferentes de elementos,
b) 0,4; 7; 5 e 0,4.
c) 0,4; 7; 5 e 0,25.
R a) total.
b) relativa.

S
TE

d) 0,25; 5; 7 e 0,25. c) absoluta.

SO
d) acumulada.
MA

09. (EEAR) Sendo fi as frequências absolutas, a classe mediana da


distribuição é a: 04. (EEAR) Feito um levantamento sobre a altura dos 50 alunos da 5ª

R
série A de um colégio, chegou-se aos seguintes resultados:
classe [10,20[ [20,30[ [30,40[ [40,50[ [50,60[ [60,70[ [70,80[
fi 25 18 10 05 09 12 15 Altura (cm) nº de alunos Altura nº de alunos

a) 2ª c) 4ª 150  154 6 162  166 8

154  158 12 166  170 6


b) 3ª d) 5ª
R
MA

158  162 14 170  174 4


SO

10. (EEAR) A tabela mostra as idades dos alunos matriculados no


Centro de Educação Infantil “X”, em 2005. Nessas condições, o número de alunos da 5ª A que não atingem
1,58 m de altura, e a porcentagem de alunos cuja altura é maior ou
T

Idade (anos) Número de alunos igual a 1,62 m são, respectivamente,


R
E

2
3
3
3 IA
a) 12 e 12%
F
L D Oc) 18Pe 36R%O
b) 12 e 20%
4 5
5 14
6 25 d) 18 e 20%
Total 50
05. (EEAR) A revista Época publicou, em janeiro de 2000, os resultados
A média das idades dos alunos dessa escola, em anos, é, de uma pesquisa por ela realizada em setembro de 1999. Cada
aproximadamente: participante indicava o nome de uma personalidade mundialmente
conhecida, do século XX, da qual ele mais se lembrava. O gráfico a
a) 4,1 c) 5,1
seguir traz o percentual de pessoas que indicaram cada uma dessas
b) 4,5 d) 5,6 personalidades.

EXERCÍCIOS DE

COMBATE
01. (EEAR) A tabela a seguir traz o resultado de uma prova de Ciências.
Nela, xi são as notas e fi são as frequências absolutas. Agrupando os
dados em 5 classes do tipo [a,b[, de amplitude 1,5, sendo o limite
inferior da 1ª classe a nota 1,5, a frequência absoluta da 3ª classe da
nova tabela será igual a

PROMILITARES.COM.BR 83

PM_BOOK05_MAT.indb 83 01/11/2021 13:42:27


NOÇÕES DE ESTATÍSTICA

Sabendo que participaram dessa pesquisa 60 mil pessoas, Ayrton 10. A Moda da distribuição representada pelo Polígono de Frequência é
Senna foi indicado por ______ pessoas.
a) 12 800 c) 16 900
b) 15 300 d) 18 600

06. (EEAR) Os alunos da 6ª série A de um colégio foram pesquisados


em cinco diferentes objetos de estudo: sexo, idade, cor dos olhos,
disciplina favorita e estatura. Desses cinco objetos, são variáveis
qualitativas
a) todas. c) apenas três.
b) apenas quatro. d) apenas duas.

07. (EEAR) Os salários mensais, em reais, dos 24 funcionários de uma


empresa são
a) 6
800 840 880 880 1000 1050 1060 1060 b) 8
1100 1150 1200 1210 1230 1250 1280 1300 c) 10
1340 1380 1450 1480 1500 1500 1520 1550 d) 12

O salário mensal mediano dessa empresa, em reais, é


a) 1200 c) 1220
L D O RESOLUÇÃO
PRO EM VÍDEO
IA FE
b) 1210 d) 1230 Abra o ProApp, leia o QR Code, assista à resolução
de cada exercício e AVANCE NOS ESTUDOS!

R dos sapatos dos


08. (EEAR) A tabela abaixo mostra os números
candidatos ao Curso de Formação de Sargentos 1/2018 da Força

S
TE

Aérea Brasileira.

SO
GABARITO
N° do sapato fi
MA

33 182 EXERCÍCIOS DE FIXAÇÃO

R
34 262 01. A 04. A 07. E 10. A
35 389 02. A 05. D 08. E
36 825
03. A 06. D 09. A
37 1441
38 2827 EXERCÍCIOS DE TREINAMENTO
39 3943 01. E 04. A R 07. A 10. C
40 2126 02. A 05. C 08. C
MA

41 1844
SO

42 1540 03. A 06. A 09. C


43 989 EXERCÍCIOS DE COMBATE
44 421 01. C 04. C 07. C 10. B
T

Total 16789
R 02. A 05. B 08. C
E

A Moda dessa Distribuição é IA 03. B


F
06. C 09. B
O
L D OANOTAÇÕES
PR
a) 33
b) 36
c) 39
d) 44

09. (EEAR) O gráfico abaixo refere-se aos índices de desistência em


um curso de Informática, verificados nos anos de 2010 a 2014. Com
base no gráfico, pode-se afirmar que os índices mediano e médio
(aproximado) de desistência do curso nesses anos são, respectivamente

a) 10% e 10%
b) 9% e 10%
c) 10% e 9%
d) 9% e 9%

84 PROMILITARES.COM.BR

PM_BOOK05_MAT.indb 84 01/11/2021 13:42:27


PROGRESSÃO ARITMÉTICA

SEQUÊNCIAS EXPRESSANDO CADA TERMO EM FUNÇÃO DE SUA


POSIÇÃO
REPRESENTAÇÃO DE UMA SEQUÊNCIA É dada uma fórmula expressando o termo geral an em função de
sua posição n.
NUMÉRICA
Exemplo:
=
an 2n2 + 3
n =1 ⇒ a1 = 2 ⋅ 12 + 3 =5

D O Pn =R3 ⇒Oa = 2 ⋅ 3 + 3 = 21
n = 2 ⇒ a2 = 2 ⋅ 2² + 3 = 11

L 2

IA FE
3
...

R
Sequência: {5,11,21,...,2n² + 3}

S
TE

POR PROPRIEDADE DOS TERMOS

SO
É dada uma propriedade em que todos os termos devem obedecer
MA

Exemplo:
Escrever os 4 primeiros termos da sequência formada pelos

R
números primos positivos em ordem crescente.
Sequência: {2,3,5,7}
Termo geral da sequência: an, sendo n ∈ N*.
PROGRESSÃO ARITMÉTICA (P. A.)
LEI DE FORMAÇÃO A progressão aritmética é uma sequência de recorrência onde
R
cada termo, a partir do segundo, é igual ao antecessor acrescido de
MA

Nos interessam as sequências que obedecem a uma lei de


SO

formação, e esta pode se dar de 3 maneiras. um valor fixo (razão r).

POR RECORRÊNCIA DEFINIÇÃO


T

Temos 2 regras, uma que identifica o 1° termo (a1) e outra que


R Dados os números a ∈ R e r ∈ R, a sequência (a1, a2 , a3 ,) é uma
E

F
permite calcular cada termo (an) a partir do antecessor (an-1).
Exemplos: IA P.A. se e somente se

O
=
a1 3 e =an an −1 + 4
n = 2 ⇒ a2 = a1 + 4 = 3 + 4 = 7
L D O Pa = R
a 1
∀ n ∈ N, n ≥ 2
=a a + r, n n−1
n = 3 ⇒ a3 = a2 + 4 = 7 + 4 = 11
Sequência:{3,7,11,...} Termo geral da P.A.: an = a1 + (n – 1)r

1 Exemplos:
a1 = 6 e an = an −1
1
2
1
S1 ={−2,3,8,13,...} ⇒ a1 = −2 e r =5
n = 2 ⇒ a2 = a1 = ⋅ 6 = 3 S= { 4,4,4,4,...} ⇒ a= 4 e =
r 0
2 2 2 1

1 1 3 S3 ={5,2, −1, −4,...} ⇒ a1 =5 e r =−3


n = 3 ⇒ a3 = a2 = ⋅ 3 =
2 2 2
...
 3  CLASSIFICAÇÃO DA P.A.
Sequência:6,3, ,...
 2  Crescente ⇔ r > 0
Constante ⇔ r = 0
* As progressões aritméticas e as progressões geométricas são Decrescente ⇔ r < 0
exemplos de sequências de recorrência.

PROMILITARES.COM.BR 85

PM_BOOK05_MAT.indb 85 01/11/2021 13:42:28


PROGRESSÃO ARITMÉTICA

REPRESENTAÇÕES DE UMA P.A. a1 + an = 2a1 + nr − r 


+ ⇒ 2(a1 + a2 + ... + an )= ( 2a1 + nr − r ) ⋅ n ⇒
1º) P.A. de 3 termos: (x − r,x,x + r) a2 + an −1 = 2a1 + nr − r 
 (a + a ) ⋅ n
2º) P.A. de 5 termos: (x − 2r,x − r,x,x + r,x + 2r)
... =
 2Sn ( a1 + a1 + (n − 1)r ) ⋅ n ⇒ Sn = 1 n
an + a1 = 2a1 + nr − r  2
3º) P.A. de 4 termos:
(x − 3y,x − y,x + y,x + 3y) ⇒ ATENÇÃO : r =
2y
ProBizu
1º) A soma de 2 termos equidistantes numa P.A. é sempre constante
CONDIÇÃO PARA QUE 3 TERMOS EM e para verificar quais são os pares de termos equidistantes em
SEQUÊNCIA FORMEM UMA P.A. qualquer P.A. basta lembrar que a1 e an são sempre equidistantes
e seus índices somam “n + 1” assim qualquer outro par de termos
(...,an −1,an ,an +1,...) ⇒ an +1 − an = an − an −1 ⇒
que os índices somarem “n + 1” também serão equidistantes.
a +a
2an = an +1 + an −1 ⇒ an = n +1 n −1
2
Exemplo: ProBizu
Encontre o valor de x para que os termos (x + 1,2x − 3,4x − 2)
1°) A soma de 2 termos equidistantes numa P.A. é sempre constante
formem, nessa ordem, uma progressão aritmética.
e para verificar quais são os pares de termos equidistantes em
4x − 2 − 2x + 3 =2x − 3 − x − 1 ⇒ 2x + 1 =x − 4 ⇒ x =−5 qualquer P.A. basta lembrar que a1 e an são sempre equidistantes
4x − 2 − 2x + 3 =2x − 3 − x − 1 ⇒ 2x + 1 =x − 4 ⇒ x =−5 e seus índices somam “n + 1” assim qualquer outro par de termos

D O P.A.
que os índices somarem “n + 1” também serão equidistantes.
( −4, −13, −22) ⇒ r =−9
L P deR28 termos: {a ,a ,...,a ,a } ⇒ n + 1 =29, assim por
exemplosO
IA
1 2 27 28

TERMO GERAL DA P.A. FE


todos os termos que os índices somarem 29 serão
equidistantes, como a e a , a e a como tantos outros.
a=
2 a1 + r R
 (n − 1) termos ⇒ a + a + ... + a + a =
5 24 11

2°) Outro fato importante quanto a equidistância de termos se


18

S
TE

 2 3 n −1 n dá na fórmula da soma dos termos de uma P.A. A soma (a1 + an)


a3 = a2 + r  pode ser substituída por qualquer outra soma de pares de termos
= + + + + − ⇒

SO
 a1 a2 ... an −1 (n 1)r
. ..  equidistantes, veja o exemplo.
MA

a = a + (n − 1)r
an = an −1 + r  n 1
Exemplo:

R
Numa P.A. de 15 termos o 8° termo é igual a 12, encontre a soma
dos 15 termos dessa P.A.
INTERPOLAÇÃO ARITMÉTICA ( a1 + a15 ) .15 , onde a soma
Numa sequência (a1,a2 ,...,an −1,an ) teremos as “pontas” {a1,an } Pela fórmula da soma teremos S15 =
2
chamadas de extremos e o restante dos termos {a2 ,a3 ,..., an − 2 ,an −1} a1 + a15 pode ser substituída pela soma de quaisquer 2 outros
chamados de meios. pares de termos equidistantes, pela nossa regra todos os termos
R
Dessa forma interpolar y meios aritméticos é colocar y termos que os índices somarem 16 (n + 1) serão equidistantes, dessa
MA

entre os extremos {a1,an } . forma são equidistantes: a1 e a16 , a e a15 , a3 e a15 ,..., a8 e a8
SO

(numa P.A. de quantidade de termos ímpar o termo central é


Exemplo: equidistante de si mesmo) assim podemos usar na fórmula da
Interpole 5 meios aritméticos entre 5 e 41. ( a8 +=
a8 ) .15 ( 2a8 ) .15 2.12.15
T

=
soma dos termos S15 = = 180 ,
S

 
R
2 2 2
E

a1 = 5 e an =41 ⇒ 5,__,__,__,__,__,41

 a1


a7 
⇒
IA F
assim por exemplos todos os termos que os índices somarem 29
serão equidistantes, como a5 e a24, a11 e a18, como tantos outros.

L DO PRO

teremos 7 termos (2 extremos e 5 meios), assim utilizaremos a fórmula 2°) Outro fato importante quanto a equidistância de termos se
do termo geral. dá na fórmula da soma dos termos de uma P.A. A soma (a1 + an)
pode ser substituída por qualquer outra soma de pares de termos
a7 =a1 + 6r ⇒ 41 =5 + 6r ⇒ 6r =36 ⇒ r =6
equidistantes, veja o exemplo.
  Exemplo:
5,11,17,23,29,35,41
   Numa P.A. de 15 termos o 8° termo é igual a 12, encontre a soma
 a1 a7 
 dos 15 termos dessa P.A.
( a1 + a15 ) .15
Pela fórmula da soma teremos S15 = , onde a soma
SOMA DOS TERMOS DE UMA P.A. FINITA 2
a1 + a15 pode ser substituída pela soma de quaisquer 2 outros pares
{a1,a1 + r,a1 + 2r,...,a1 + (n − 2)r,a1 + (n − 1)r=
} {a1,a1 + r,a1 + 2r,...,a1 + nr − 2r,a + nr − r}equidistantes, pela nossa regra todos os termos que os
de1termos
índices somarem 16 (n + 1) serão equidistantes, dessa forma são
} {1 1
.,a1 + (n − 2)r,a1 + (n − 1)r= a ,a + r,a 1 + 2r,...,a1 + nr − 2r,a1 + nr − r}
a1 e a16 , a e a15 , a3 e a15 ,..., a8 e a8
equidistantes: (numa P.A.
{a1 + a1 + r, a1 + 2r, ..., a1 + nr – 2r, a1 + nr – r} de quantidade de termos ímpar o termo central é equidistante de
si mesmo) assim podemos usar na fórmula da soma dos termos
{a1 + nr − r,a1 + nr − 2r,...,a1 + 2r,a1 + r,a1} ( a8 +=
a8 ) .15 ( 2a8 ) .15 2.12.15
=S15 = = 180 .
Somando os termos equidistantes 2 2 2

(a1 + an ,a2 + an −1,...,an + a1) teremos sempre

86 PROMILITARES.COM.BR

PM_BOOK05_MAT.indb 86 01/11/2021 13:42:29


PROGRESSÃO ARITMÉTICA

Exercício Resolvido Exercício Resolvido

01. (UNI-RIO 2000) As idades inteiras de três irmãos formam uma 03. (UNI-RIO 2004) Passando em uma sala de aula, um aluno
P.A., e a soma delas é igual a 15 anos. A idade máxima, em anos, verificou que, no quadro-negro, o professor havia escrito os
que o irmão mais velho pode ter é: números naturais ímpares da seguinte maneira:
a) 10 1
b) 9
3 5
c) 8
7 9 11
d) 7
13 15 17 19
e) 6
21 23 25 27 29
Resolução: A
Idades: x − r, x, x + r . O aluno achou interessante e continuou a escrever, até a décima
linha.
x − r + x + x + r = 15 ⇒ x = 5 .
Somando os números dessa linha, ele encontrou
O irmão mais novo pode ter menos de 1 ano ⇒ x − r = 0 ⇒ r = 5.
a) 800
Então, x + r = 5 + 5 = 10 .
b) 900
c) 1000
d) 1100
D O e)P 1200
Exercício Resolvido

L RO
A
02. (UERJ 2002) Leia com atenção a história em quadrinhos.
I Resolução: C FE
R 1

S
TE

3 5

SO
7 9 11
MA

13 15 17 19

R
21 23 25 27 29
............................
(O Globo, 16/03/2001)
A quantidade de algarismos anteriores à 10ª linha é
Considere que o leão da história acima tenha repetido o 9
convite por várias semanas. Na primeira, convidou a Lana para sair 1 + 2 + 3 +  + 9 = 10 ⋅ = 45 .
2
19 vezes; na segunda semana, convidou 23 vezes; na terceira, 27 Então, a 10ª linha contém os termos a46 , a47 ,  , a55 da P.A.
R
vezes e assim sucessivamente, sempre aumentando em 4 unidades
MA

1, 3, 5,  , a46 ,  , a55
 .
SO

o número de convites feitos na semana anterior. 10ª linha

Imediatamente após ter sido feito o último dos 492 convites, o a46 =1 + 45 × 2 = 91
número de semanas já decorridas desde o primeiro convite era
a55 =1 + 54 × 2 =109
T

igual a:
R
E

a) 10 10
E

IA F ( 91 109 )
A soma dos termos da 10ª linha é S =+
2
=
1.000 .

O
b) 12
c) 14 L DO PR
d) 16
Exercício Resolvido
Resolução: B
04. (UFRJ 2004) Felipe começa a escrever números naturais em
P.A.: 19, 23, 27,  , an uma folha de papel muito grande, uma linha após a outra, como
n n mostrado a seguir:
Sn= (19 + an ) = 492 ⇒ (19 + 19 + (n − 1) ⋅ 4 ) = 492
2 2
1
41
⇒ (17 + 2n) n= 492 ⇒ n= 12 ou n = − . 2 3 4
2
3 4 5 6 7
4 5 6 7 8 9 10
5 6 7 8 9 10 11 12 13
6 7 8 9 10 11 12 13 14 15 16
...............
...............

Considerando que Felipe mantenha o padrão adotado em todas


as linhas:

PROMILITARES.COM.BR 87

PM_BOOK05_MAT.indb 87 01/11/2021 13:42:31


PROGRESSÃO ARITMÉTICA

03. (EEAR) Do conjunto dos números naturais menores ou iguais a


a) determine quantos números naturais ele escreverá na 50ª 100 retiram-se os múltiplos de 5 e, em seguida, os múltiplos de 6. O
linha; número de elementos que permanecem no conjunto é
b) determine a soma de todos os números escritos na 50ª linha. a) 66. c) 68.
b) 67. d) 69.
Resolução:
1 04. (EEAR) A soma dos 10 primeiros termos de uma P.A., cujo termo
2 3 4 geral é dado pela expressão ak = 3k – 16, é
a) 5. c) 18.
3 4 5 6 7
b) 14. d) – 6.
4 5 6 7 8 9 10
5 6 7 8 9 10 11 12 13 05. (EEAR) O perímetro de um triângulo retângulo é 36 cm, e os
números que expressam as medidas de seus lados formam uma P.A. O
6 7 8 9 10 11 12 13 14 15 16 cateto maior desse triângulo, em cm, mede
............... a) 15. c) 8.
b) 12. d) 6.
...............
a) A quantidade de elementos em cada linha é 06. (EEAR) Considere esses quatro valores x, y, 3x, 2y em P.A. crescente.
1, 3, 5, 7,  ,  50 ,  ,  n ,  ⇒  50 =1 + 49 ⋅ 2 = 99 Se a soma dos extremos é 20, então o terceiro termo é
.

D O b) P12R O
a) 9 c) 15
b) Os elementos da 50ª linha são
50, 51, 52,  , a ⇒ a = 50 + 98 = 148
A L d) 18
99 99

I F E aritmética, cuja fórmula do termo geral é


07. (EEAR) A progressão
(50 148 ) =
A soma dos elementos é S =+
99
R.
99
2
2
dada por a = 5n – 18 tem razão igual a
n

S
TE

a) –5 c) 5

SO
b) –8 d) 8
MA

Exercício Resolvido 08. (CFOE) O 78º termo de uma progressão aritmética de razão 3 é

R
igual a 32. O maior termo negativo dessa sequência é
05. Numa sala de aula, cada um dos 100 alunos recebe um a) a65 c) a66
número que faz parte de uma sequência que está em progressão
aritmética. Sabendo-se que a soma de todos os números é 15.050 b) a67 d) a68
e que a diferença entre o 46º e o 1º é 135, determine o 100º
número. 09. As medidas dos lados de um triângulo são x + 1, 2x, x² – 5 e estão
em progressão aritmética, nesta ordem. O perímetro do triângulo
R
mede:
MA

Resolução: 299
SO

P.A.: a1, a2 ,  , a100 a) 4 d) 33


b) 12 e) 24
100
= 15.050
S100 = ( a1 + a100 ) c) 8
T

2
S

⇒ 15.050 = [a1 + a1 + 99r ] 50 ⇒ 2a1 + 99r = 301 . R


E

IA F
10. (CFOE) A soma dos 9 primeiros termos de uma P.A. de razão 2 é

L. D O P R O
a46 − a1= 135 ⇒ a1 + 45r − a1= 135 ⇒ r= 3. nula. Assim, pode-se afirmar que seu sexto termo é igual a
Então, 2a1 + 99 × 3= 301 ⇒ a1= 2 ⇒ a100 a) 0
= 2 + 99 × 3 = 299 c) 6
b) 2 d) 7

EXERCÍCIOS DE
EXERCÍCIOS DE

FIXAÇÃO TREINAMENTO
01. (EEAR) O termo geral de uma P.A. é an = 3n – 16. A soma de seus
01. (EEAR) Os números que expressam as medidas, em cm ou em cm², 10 primeiros termos é
do lado, da superfície e do perímetro de um quadrado, dados nessa a) 18. c) 5.
ordem, formam uma P.A. O lado desse quadrado, em cm, mede
b) 14. d) – 6.
a) 5/2. c) 3/4.
b) 5/3. d) 3/2. 02. (EEAR) Um número, seu logaritmo 2 e a base do logaritmo
formam, nessa ordem, uma P.A. Esse número é
02. (EEAR) A soma dos números múltiplos de 7, compreendidos entre 9 − 17 −1 + 17
20 e 300, é a) c)
2 2
a) 6250. c) 6350.
b) 6300. d) 6400. 9 + 17 −1 − 17
b) d)
2 2

88 PROMILITARES.COM.BR

PM_BOOK05_MAT.indb 88 01/11/2021 13:42:32


PROGRESSÃO ARITMÉTICA

03. Em um triângulo, os três ângulos estão em progressão aritmética 02. Numa P.A., cujo 2º termo é igual a 5 e o 6º termo é igual a 13 o
e o maior ângulo é o dobro do menor. Então o menor ângulo mede: 20º termo é igual a:
a) 10° d) 15° a) 13 c) 41
b) 20° e) 40° b) 40 d) 42
c) 30°
10
03. Os números , x – 3 e x + 3 são os 3 primeiros termos de uma
04. (EEAR) As casas de uma rua foram numeradas em ordem crescente x
segundo as regras: os números formam uma P.A. de razão 5; cujo P.A., de termos positivos, sendo x ≠ 0. O décimo termo desta P.A. é
primeiro termo é 1; as casas à direita são ímpares e as à esquerda, igual a:
pares. Assim, se Tiago mora na 3ª casa do lado esquerdo, o nº da a) 50 d) 57
casa dele é b) 53 e) 55
a) 26 c) 36 c) 54
b) 31 d) 41
04. A soma dos termos de uma P.A. é dada por Sn = n² – n,
05. (EEAR) As medidas, em cm, dos lados de um pentágono estão em n = 1, 2, 3,... Então o 10° termo da P.A. vale:
Progressão Aritmética (P.A.). Se o perímetro desse polígono é 125 cm, a) 18 d) 100
o terceiro elemento da P.A é
b) 90 e) 9
a) 25 c) 35
c) 8
b) 30 d) 40

D O 05.
(a P
A soma dos 11 primeiros termos da progressão aritmética
, a ,R
06. (ESA) Em uma progressão aritmética cujo primeiro termo é 1,87 e

A L O
..., a , ...) é 176. Se a = a + 30 então, para qualquer n ∈ 
1 2 n 11 1

FE
a razão é 0,004, temos que a soma dos seus dez primeiros é igual a:
temos:
a) 18,88 d) 18,9 I a) a = 3n - 2
b) 9,5644 e) 18,99 R n

b) an = 2n - 3

S
TE

c) 9,5674 c) an = n + 3

SO
d) an = 2n + 3
07. (ESA) Em uma Progressão Aritmética com 6 termos, temos que a
MA

soma de seus termos é igual a 102 e seu último termo é 27. Com base e) an = 3n + 2

R
nessas informações, a razão dessa progressão é:
06. Os ângulos internos de um triângulo estão em progressão
a) 3 d) 4
aritmética e o menor deles é a metade do maior. O maior ângulo do
b) 5 e) 7 triângulo mede:
c) 11 a) 60º d) 90º
b) 75º e) 120º
08. (EEAR) Numa P.A., o 2° termo é 1 e o 5° termo é 16. O termo R
igual a 31 é o c) 80º
MA

SO

a) 7°. c) 10°.
07. Em uma progressão aritmética, o primeiro termo é 5 e o décimo
b) 8°. d) 11°. primeiro termo é 45. Pode-se afirmar que o sexto termo é igual a
T

a) 15. d) 29.
S

09. (EEAR) O 15º termo da sequência (–2, 3, 8, ...) é


R
E

b) 21. e) 35.
E

a) 68. c) 72.
IA c) 25. F
b) 70. d) 81.
L D O08. Numa O
Rsequência
10. (ESA) Em um treinamento de condicionamento físico, um soldado
P aritmética de 17 termos, sabe-se que a …= 3 e
a …= 7. Então a soma de todos os termos é:
5

13
inicia seu primeiro dia correndo 800 m. No dia seguinte corre 850 m.
No terceiro 900 m e assim sucessivamente até atingir a meta diária de a) 102 d) 78
2.200 m. Ao final de quantos dias, ele terá alcançado a meta? b) 85 e) 90
a) 31 d) 25 c) 68
b) 29 e) 23
09. (ESA) Quantos múltiplos de 9 ou 15 há entre 100 e 1000?
c) 27
a) 100 d) 160
b) 120 e) 180
EXERCÍCIOS DE c) 140

COMBATE 10. (ESA) O número mínimo de termos que deve ter a


P.A. (73, 69, 65, …) para que a soma de seus termos seja negativa é
a) 18
01. (EEAR) Numa P.A., o 10º termo e a soma dos 30 primeiros termos b) 19
valem, respectivamente, 26 e 1440. A razão dessa progressão é c) 20
a) 2. c) 4. d) 37
b) 3. d) 6. e) 38

PROMILITARES.COM.BR 89

PM_BOOK05_MAT.indb 89 01/11/2021 13:42:32


PROGRESSÃO ARITMÉTICA

RESOLUÇÃO EM VÍDEO
Abra o ProApp, leia o QR Code, assista à resolução
de cada exercício e AVANCE NOS ESTUDOS!

GABARITO
EXERCÍCIOS DE FIXAÇÃO
01. A 04. A 07. C 10. B
02. B 05. B 08. B
03. B 06. B 09. E
EXERCÍCIOS DE TREINAMENTO
01. C 04. A 07. D 10. B
02. A 05. A 08. B
03. E 06. A 09. A
EXERCÍCIOS DE COMBATE
01. C 04. A 07. C 10. E
02. C 05. A 08. B
L DO PRO
03. E 06. C 09. C
I A FE
ANOTAÇÕES R

S
TE

SO
MA

R
R
MA

SO
T

R
E

IA F
L DO PRO

90 PROMILITARES.COM.BR

PM_BOOK05_MAT.indb 90 01/11/2021 13:42:33


PROGRESSÃO GEOMÉTRICA

PROGRESSÃO GEOMÉTRICA TERMO GERAL DA P.G.


A progressão geométrica é uma sequência de recorrência onde a2 = a1 ⋅ q 
(n − 1) termos ⇒ a2 ⋅ a3 ⋅ ... ⋅ an −1 ⋅ an =
cada termo, a partir do segundo, é igual ao antecessor multiplicado a3 = a2 ⋅ q 
 = a1 a2 .....an −1 ⋅ q
⋅ q
⋅
q ⋅ ...
⋅q ⇒
por um valor fixo (razão q). ...  n −1
an = an −1 ⋅ q an = a1 ⋅ qn −1
DEFINIÇÃO
Dados os números a ∈ R e q ∈ R, a sequência (a1, a2, a3, ...) é uma
P.G. se e somente se INTERPOLAÇÃO GEOMÉTRICA

D O chamadas
Numa sequência (a1, a2, ..., an-1, an) teremos as “pontas” {a1, an}
a1 = a

=
an an−1 ⋅ q ,
∀ n ∈ N, n ≥ 2
L P R deOextremos e o restante dos termos {a , a , ..., a , a } 2 3 n-2 n-1

I A chamados de
Finterpolar
meios.

Termo Geral da P.G.: an= a1 ⋅ qn−1


R
Dessa forma
E y meios geométricos é colocar y termos
entre os extremos {a , a }. 1 n
Exemplos:
Exemplo:

S
TE

S=
1 {2,6,18,54,...} ⇒ a= 1 2eq = 3

SO
Interpole 4 meios geométricos (reais) entre 4 e 972.
S=
2 {4, 4, 4, 4,...} ⇒ a=
1 4 e =
q 1
MA

 
1  
S3= {8, 4,2,1,...} ⇒ a=
1 =
8eq a1 =4 e an =972 ⇒  4 , __, __, __, __,97 2 ⇒ teremos 6 termos

R
2 
 a6 
{3, −6,12, −24,...} ⇒ a1 =
S3 = 3eq=−2  a1 
(2 extremos e 4 meios), assim utilizaremos a fórmula do termo geral.
972
CLASSIFICAÇÃO DA P.G. a6= a1.q5 ⇒ 972= 4.q5 ⇒ q5= = 243 ⇒ q= 5 243= 3
4
Crescente: q > 1 R
Constante: q = 1  
MA

 
SO

Decrescente: 0 < q < 1  4 ,12,36,108,324,972


 
Alternante: q < 0  a6 
 a1 
T

REPRESENTAÇÕES DE UMA P.G.


R SOMA DOS TERMOS DE UMA P.G. FINITA
E

x 
1°) P.G. de 3 termos:  , x, xq  IA F
L DO PRO
q  
Sn = a1 + a2 + ... + an −1 + an xq

 x x 2 q ⋅ Sn = a1 ⋅ q + a2 ⋅ q + ... + an −1 ⋅ q + an ⋅ q = a2 + a3 + ... + an + an +1  –
2°) P.G. de 5 termos:  2 , , x, xq, xq 
q q  Sn − q ⋅ Sn =a1 + a2 − a2 + a3 − a3 + ... + an − an − an +1
 
 x x 3 a1 − a1 ⋅ qn a1 (1 − q )
n
2 a (1 − qn )
3°) P.G. de 4 termos:  3 , , xy, xy  ⇒ ATENÇÃO : q = y Sn (1 − q) = a1 − an +1 ⇒ Sn = = ⇒ Sn = 1
y y  1− q 1− q 1− q

CONDIÇÃO PARA QUE 3 TERMOS EM


SEQUÊNCIA FORMEM UMA P.G. SOMA DOS TERMOS DE UMA P.G. INFINITA
an+1 an (0 < q < 1)
(..,an−1,an ,an+1,...) ⇒ =⇒ Para uma P.G. decrescente (0 < q < 1) é possível calcular a soma
an an−1
an 2 an+1 ⋅ an−1 =
= ⇒ an an+1 ⋅ an−1 dos termos de uma P.G. infinita, pois na fórmula Sn =
a1 qn − 1 ( )
q −1
Exemplo: podemos observar que qn para uma razão q onde, 0 < q < 1, tenderá
Encontre o valor de x para que os termos (x + 1, x + 9, x + 15) a um valor muito próximo de 0 quando o expoente n tender a um
formem, nessa ordem, uma progressão aritmética. a1(q∞ − 1) −a1 a1
valor muito grande. Assim=S∞ = = .
 x +1 x + 9  2 0 q −1 q − 1 1− q
 =  ⇒ (x + 9) = (x + 1)(x + 15) ⇒
 x + 9 x + 15 
(x 2 + 18x + 81) =
(x 2 + 16x + 15) ⇒ 2x =
−66 ⇒ x =
−33

PROMILITARES.COM.BR 91

PM_BOOK05_MAT.indb 91 01/11/2021 13:42:34


PROGRESSÃO GEOMÉTRICA

Exercício Resolvido Resolução:


01. (CEFET 2000) A soma do sexto ao décimo primeiro termo de Sendo S1, S2, S3, ... as áreas obtidas nas etapas sucessivas, tem-se:
uma progressão geométrica é 5. A soma do décimo ao décimo S1 = 1
quinto vale 1280. A razão dessa PG é:
1 1
a) 2 d) 8 S2 = S1 + 3 ⋅ = 1+
9 3
b) 4 e) 12
1 1 1
c) 5 S3 = S2 + 9 ⋅ =1+ + , etc. No limite:
81 3 9
Resolução: B
1 1 1 1 1 3
a, aq,  , aq5 ,  , aq9 , aq10 ,  , aq14
SF =1+ + + + + = =
3 9 27 81 1 2
↑ ↑ ↑ ↑
1−
a6 a10 a11 a15 3

aq5 + aq6 +  + aq10 =


5;
Exercício Resolvido
aq9 + aq10 +  + aq14
= 1280 ⇒
04. (UNI-RIO 2004) Há exatamente um ano, José iniciou uma
⇒ q4 ( aq5 + aq6 +  + aq10=
) 1280 ⇒ criação de coelhos e, durante este período, o número de coelhos
⇒ q4 ⋅ 5 = 1280 ⇒ q4 = 256 = 28 ⇒ q = 22 = 4 duplicou a cada 3 meses. Hoje, preocupado com a falta de espaço
para os coelhos, José vai vender parte dessa criação, de modo

DO PRO
que apenas a quantidade inicial fique com ele. Se N0 denota a
Exercício Resolvido
A L quantidade inicial de coelhos, então a quantidade a ser vendida é

I
a) 15 N 0
FE
R
02. (UERJ 2005) Um veículo com velocidade constante de V b) 13 N0
km/h percorre S km em um intervalo de tempo de T horas, sendo
c) 12 N0
T diferente de 1. Considere que T, V e S estejam em progressão

S
TE

geométrica, nessa ordem. d) 8 N0

SO
A alternativa que indica a relação entre o espaço percorrido S e a e) 7 N0
MA

velocidade V é:
Resolução: B

R
a) S = V²
P.G.: N0, 2 N0, 4 N0, 8 N0, 16 N0.
b) S = V2
População hoje: 16 N0 .
c) S=V
A vender: 16 N0 – N0 = 15 N0.
d) 3
S= V

Exercício Resolvido
R
Resolução: D
MA

SO

V2 05. (UFF 2000) Numa progressão geométrica (P.G.) decrescente


P.G.: T, V, S ⇒ V 2 = T ⋅ S ⇒ T = ; o primeiro termo é um número real positivo e cada termo, a
S
S S partir do terceiro, é igual à sexta parte da soma dos dois termos
T

Velocidade ⇒ V = ⇒T= .
S

imediatamente anteriores.
T V
R
E

Determine a razão dessa P.G.


Então,
V2 S
= ⇒ V 3 =S2 ⇒ 6 V 3 = 6 S2 ⇒ V = 3 S . IA F
S V
O
Resolução:
L D O P.G.:Pa, aq,R aq², ...
a + aq 1 1
Exercício Resolvido aq2 = ⇒ 6q2 − q − 1 =0 ⇒ q =− ou q = .
6 3 2
1
03. (UFRJ 2003) A região fractal F, construída a partir de um a > 0 e P.G. decrescente ⇒ q = .
2
quadrado de lado 1 cm, é constituída por uma infinidade de
quadrados e construída em uma infinidade de etapas. A cada
nova etapa consideram-se os quadrados de menor lado ()
acrescentados na etapa anterior e acrescentam-se, para cada um EXERCÍCIOS DE
destes, três novos quadrados de lado /3. As três primeiras etapas
de construção de F são apresentadas a seguir. FIXAÇÃO
01. (EEAR) Na P.G. (y, 2y + 2, 3y + 3, ...), o 4º termo, que é diferente
de zero, vale
a) 2.
3
b) .
2
c) – 4.
27
Calcule a área de F. d) − .
2

92 PROMILITARES.COM.BR

PM_BOOK05_MAT.indb 92 01/11/2021 13:42:36


PROGRESSÃO GEOMÉTRICA

 3 3  10. (EEAR) Sabe-se que a sequência (x; y; 10) é uma P.A. e a sequência
02. (EEAR) A soma dos infinitos termos da P.G.  , ,…  é
 2 3  1 
3 2 3  ; 2 ; 3x + 4  é uma P.G. Nessas condições, é correto afirmar que
a) . c) .  y 
2 3
2 a) a razão da P.A. é 2.
b) . 3 3
3 d) . b) a razão da P.G. é 26.
2
c) x + y = 0.
03. (EEAR) Numa P.G., onde o 1.º termo é 3, a soma dos três primeiros d) x · y = –16.
termos é 21. Se a soma dos quatro primeiros termos é 45, o quinto
termo é
a) 51. EXERCÍCIOS DE
b) 50.
c) 49. TREINAMENTO
d) 48.

04. (EEAR) Os números que expressam as medidas das arestas que 01. (EEAR) Numa P.G. crescente temos a1 + a3 = 100 e a2 = 14. A soma
concorrem em um mesmo vértice de um paralelepípedo retângulo dos algarismos do quarto termo dessa sequência é igual a:
estão em progressão geométrica. Se a maior dessas arestas mede 6m, a) 12
e o volume desse sólido é 27 m3, então a sua área total, em m2, é b) 16

O c)d) P2018R
a) 63.
b) 57.
L D O
c) 53.
I A F E adicionar a cada um dos números k + 3,
d) 47.
R 02. (AFA) Quanto devemos
k, k – 2 para que, nesta ordem, formem uma Progressão Geométrica?

S
TE

05. (EEAR) A soma dos n primeiros termos da PG (1, – 2, 4, – 8, ...) a) 6 – k

SO
é – 85. Logo, n é b) 6 + k
MA

a) 8. c) 1 – 6k
b) 10. d) 1 + 6k

R
c) 12.
d) 14. π π π
03. (EEAR) O valor de sen  + 2 + ... + n  , n ∈ , é
2 2 2 
06. (EEAR) Quatro números estão dispostos de forma tal que a) –1
constituem uma PG finita. O terceiro termo é igual a 50 e a razão é b) 0
igual a 5. Desta maneira, o produto de a1 · a4 vale 1
R
c)
MA

a) 10 2
SO

b) 250 d) 1
c) 500
T

04. (EEAR) Dada a equação 20x + 10x + 5x + ... = 5, em que o


S

d) 1.250
R primeiro membro representa a soma dos termos de uma progressão
E

IA
07. (EEAR) Na progressão geométrica onde o primeiro termo é m3, o F
geométrica infinita, o valor de 1/x é

L D Ob) 10P R O
último é (–m²¹) e a razão é (–m²), o número de termos é a) 12
a) 8.
b) 9. c) 8
c) 11. d) 5
d) 10.
05. (ESA) Se (40, x, y, 5, ...) é uma progressão geométrica de razão q e
(q, 8 – a, 7/2, ...) é uma progressão aritmética, determine o valor de a.
08. (EEAR) A soma 1 + 2 + 22 + 23 + ... + 2999 + 21000 é igual a
a) 8 d) 6
a) 21000 – 1
b) 25/4 e) 7
b) 21001 – 1
c) 23/4
c) 21000 + 1
d) 21001 + 1 06. (EEAR) O 6º termo da sequência 2, 8, 32, 128, ... é um número
cuja soma dos algarismos é
09. (CFOE) Em uma sequência de circunferências, em que a
a) 10 c) 14
primeira tem raio r = 1, os diâmetros formam uma PA de razão 6.
Podemos afirmar que os valores dos raios e dos comprimentos dessas b) 12 d) 16
circunferências, respectivamente, formam uma
a) PA de razão 3 e uma PA de razão 6π 07. (EEAR) Seja a PG (a1, a2, a3, a4, ...) de razão q = 2. Se a1 + a5 = 272,
o valor de a1 é
b) PA de razão 3 e uma PG de razão 3π
a) 8 c) 18
c) PA de razão 6 e uma PA de razão 6π
b) 6 d) 16
d) PA de razão 3 e uma PG de razão 6π

PROMILITARES.COM.BR 93

PM_BOOK05_MAT.indb 93 01/11/2021 13:42:36


PROGRESSÃO GEOMÉTRICA

08. (EEAR) Seja (a1, a2, a3, a4, a5, ...) uma PG de termos não nulos. Se 07. A soma dos quatro primeiros termos de uma progressão
2(a2 + a4) = a3 + a5, pode-se afirmar corretamente que a razão dessa geométrica (PG) de razão 3 é igual a 60, e a soma dos quatro primeiros
PG é termos de uma progressão aritmética (PA) também vale 60. Sabe-se
a) 4 que o primeiro termo da PA é igual ao primeiro termo da PG. A razão
da PA é:
b) 2
a) -3
c) 1/2
b) 3/2
d) 2
c) 3
d) 2/3
09. (EEAR) Os valores que expressam as idades, em anos, dos 5 filhos
1 e) 9
de Joana formam uma PG de razão . Se o filho mais novo tem 1
2
ano, então a idade do filho mais velho de Joana, em anos, é 08. Uma P.G é formada por 6 termos, onde a1 = 4 e a6 = 972. A soma
a) 10. dos elementos que formam os meios geométricos existentes entre a1
e a6 vale:
b) 12.
a) 320
c) 14.
b) 324
d) 16.
c) 40
10. (EEAR) Se a sequência (4, x, 2y, 108) é uma PG de razão 3, então d) 450
x + y é igual a ___ . e) 480
a) 24
L D O 09.PAsRmedidas
O do lado, do perímetro e da área de um quadrado
b) 30
I A F
estão em progressão geométrica, nessa ordem. A área do quadrado
c)
d) 48
36
R será: E
a) 256

S
TE

b) 64

SO
c) 16
MA

EXERCÍCIOS DE
d) 243

COMBATE
R
e) 729

10. Sejam as sequências de números reais (–3, x, y, ...) que é uma


progressão aritmética de razão r, e (x, y, 24, ...) que é uma progressão
  r
01. (EEAR) Se an  log1,log0,001,log 1 729,… e bn =  − , , −1,… ,
1 1
= geométrica de razão q. O valor de pertence ao intervalo
 3   9 3  q
a) [0,1/2[ R
são PA e PG, respectivamente, então a diferença entre o décimo termo
MA

de an e o nono termo de bn é b) [1/2,1[


SO

a) –756 c) [1,2[
b) –270 d) [2,3[
T

c) 702
R
E

F EM VÍDEO
d) 756
IA RESOLUÇÃO
3 9
1+ +
4 16
+… L DO PRO Abra o ProApp, leia o QR Code, assista à resolução
de cada exercício e AVANCE NOS ESTUDOS!
02. Simplifique a expressão
15 75
3+ + +…
8 64

03. Numa PG de quatro termos, a razão é 5 e o último termo é 375. GABARITO


Calcular o primeiro termo dessa PG.
EXERCÍCIOS DE FIXAÇÃO
04. Numa PG a1 = ¼ e a7 = 16. Calcule a razão dessa PG. 01. D 04. A 07. D 10. B
02. D 05. A 08. B
05. Em uma P.G., o primeiro termo é 4 e o quinto termo é 324. A 03. D 06. C 09. A
razão dessa P. G. é: EXERCÍCIOS DE TREINAMENTO
a) 3 d) 2 01. D 04. C 07. D 10. B
b) 4 e) ½ 02. A 05. D 08. B
c) 5 03. B 06. C 09. D
EXERCÍCIOS DE COMBATE
06. A soma da série infinita 1 + 1/5 + 1/25 + 1/125 + ... é:
01. C 04. 2 07. E 10. C
a) 6/5 d) 2
1 05. A 08. E
b) 7/5 e) 7/4 02.
2 06. C 09. A
c) 5/4 03. 3

94 PROMILITARES.COM.BR

PM_BOOK05_MAT.indb 94 01/11/2021 13:42:37


MATRIZES

MATRIZES a11 = 2(1) + 1 = 3


a21 = 2(2) + 1 = 5
DEFINIÇÃO a12 = 2(1) + 2 = 4
Matriz m × n é uma tabela de m × n números reais dispostos em m a22 = 2(2) + 2 = 6
linhas (filas horizontais) e n colunas (filas verticais). Exemplos:
 3 4
1 −2 3  Assim, A = 
1. A =   5 6
 é uma matriz 2 × 3;
0 4 2
MATRIZES ESPECIAIS
 4 0
2. B    é uma matriz 2 × 2;
 1 1  L DO PRO
IA F E tipo 1 × n, isto é, com uma única linha.
MATRIZ LINHA
3 −2
1 0
5
2
R É toda matriz do
Exemplo:

S
TE

3. C = 0 4 3 é uma matriz 4 × 3. A = (4 7 –3 1)1 x 4.

SO
1
MA

−1 −6 MATRIZ COLUNA
2
É toda matriz do tipo n × 1, isto é, com uma única coluna.

R
Como podemos notar nos exemplos 1, 2 e 3 respectivamente,
uma matriz pode ser representada por colchetes, parênteses ou duas Exemplo:
barras verticais.
4
B =  −1
REPRESENTAÇÃO DE UMA MATRIZ  0  3x1
As matrizes costumam ser representadas por letras maiúsculas e R
seus elementos por letras minúsculas, acompanhadas de dois índices
MA

que indicam, respectivamente, a linha e a coluna ocupadas pelo MATRIZ QUADRADA


SO

elemento. É toda matriz do tipo n × n, isto é, com o mesmo número de linhas


Exemplo: e colunas. Neste caso, dizemos que a matriz é de ordem n.
T

Exemplo:
S

Uma matriz A do tipo m × n é representada por:


R  4 −1 0
E

a11
a
a12 a13  a1n 
 IA F  4 7
C= 

D = 0 π 3

 21
A  a31
a22 a23
a32 a33
 a2n
 a3n

 L DO PR O  2 −1 2x 2  2 7 3 
3x 3
 
      Matriz de ordem 2 Matriz de ordem 3
a am2 am3  amn 
 m1  Seja A uma matriz quadrada de ordem n.
ou, abreviadamente, A = aij m x n , onde i e j representam,
1 ≤ i ≤ m Diagonal principal de uma matriz quadrada é o conjunto de
respectivamente, a linha e a coluna que o elemento ocupa,  . elementos dessa matriz, tais que i = j.
1 ≤ j ≤ n
Por exemplo, na matriz anterior, a23 é o elemento da segunda
linha com o da terceira coluna. Diagonal secundária de uma matriz quadrada é o conjunto de
elementos dessa matriz, tais que i + j = n + 1.
Exemplo 1: Exemplo:
 −1 2 5
Seja a matriz A = aij  2 x 2 , onde aij = 2i + j:
A 3 =  3 0 −3
a a   
Genericamente, temos: A =  11 12  . Utilizando a regra de  5 7 −6
 a21 a22  2 x 2
formação dos elementos dessa matriz, temos:
aij = 2i + j Descrição da matriz:
• O subscrito 3 indica a ordem da matriz;
• A diagonal principal é a diagonal formada pelos elementos
–1, 0 e –6;

PROMILITARES.COM.BR 95

PM_BOOK05_MAT.indb 95 01/11/2021 13:42:49


MATRIZES

• A diagonal secundária é a diagonal formada pelos elementos MATRIZ OPOSTA


5, 0 e 5;
Chamamos de matriz oposta de uma matriz A a matriz que é
• a11 = –1 é elemento da diagonal principal, pois i = j = 1; obtida a partir de A, trocando-se o sinal de todas os seus elementos.
• a31 = 5 é elemento da diagonal secundária, pois i + j = n + 1
= 3 + 1.
Notação: – A
3 0  −3 0
MATRIZ NULA Exemplo: Se A =  então − A =  
4 -1   −4 1
É toda matriz em que todos os elementos são nulos.

Notação: Om × n OPERAÇÕES
0 0 0
Exemplo: O2 x 3 = 
0 0 0 IGUALDADE DE MATRIZES
Duas matrizes, A e B, do mesmo tipo m × n, são iguais se, todos os
elementos que ocupam a mesma posição são idênticos.
MATRIZ DIAGONAL
Notação: A = B.
É toda matriz quadrada onde só os elementos da diagonal
principal são diferentes de zero.
Exemplo:
 4 0 0
 2 0  2 c
Exemplo: A 2 =  B3 =  0 3 0 .  2 0
DO PRO
   Se A =   B=   e A = B, então c = 0 e b = 3
0 1  −1 b  −1 3
L
 0 0 7

IA FE
Simbolicamente: A = B ⇔ a = b ij ij para todo 1 < i < m e todo
MATRIZ IDENTIDADE 1 < i < n.
R
É toda matriz quadrada onde todos os elementos que não estão
ADIÇÃO DE MATRIZES

S
TE

na diagonal principal são nulos e os da diagonal principal são iguais

SO
a 1. Dadas as matrizes A = [aij]m x n e B = [bij]m x n, chamamos de soma
MA

das matrizes A e B a matriz C = [cij]m x n, tal que cij = aij + bij, para todo
Notação: In onde n indica a ordem da matriz identidade. 1 < i < m e todo 1 < i < n.

R
 1 0 0
 1 0 Notação: A + B = C
Exemplo: I2 =   I3 =  0 1 0
0 1  
 0 0 1
OBS: A + B existe se, e somente se, A e B são do mesmo tipo (m × n).
1, se i = j
ou In = aij  , aij = 
0, se i ≠ j
R
Propriedades: A, B e C são matrizes do mesmo tipo (m × n),
MA

valem as seguintes propriedades:


SO

MATRIZ TRANSPOSTA
Chamamos de matriz transposta de uma matriz A a matriz que Associativa:
T

é obtida a partir de A, trocando-se ordenadamente suas linhas por


R (A + B) + C = A + (B + C)
E

colunas ou suas colunas por linhas.


E

IA F
L R O
Comutativa

P
Notação: At.

 2 3 0
2 −1  DO A + B = B +A
Exemplo: Se A =   então A t = 3 −2  
 −1 −2 1  0 1
Elemento Neutro
Desse modo, se a matriz A é do tipo m × n, A é do tipo n × m.
t

Note que a primeira linha de A corresponde à primeira coluna de At e A+O=O+A=A


a segunda linha de A corresponde à segunda coluna de At.
onde O é a matriz nula m × n.
MATRIZ SIMÉTRICA
Elemento Oposto
Uma matriz quadrada de ordem n é simétrica quando A = At.
 2 3 1  2 3 1 A + (–A) = (–A) + A = O
Exemplo: Se A =  3 2 4 A t =  3 2 4
   
 1 4 5 3x 3  1 4 5 3x 3 Exemplos:
 1 4  2 −1  1 + 2 4 + ( −1) 3 3
MATRIZ ANTISIMÉTRICA 0 7  + 0 = =
   2 0 + 0 7 + 2 0 9
Uma matriz quadrada de ordem n é simétrica A = –A t

 0 4 1  0 4 1 2 3 0 3 1 1 2 + 3 3+1 0 + 1 5 4 1
0 + = =
Exemplo: Se A =  −4 0 3 − A t =  −4 0 3  1 −1  1 -1 2  0 + 1 1 + ( −1) −1 + 2  1 0 1
   
 −1 −3 0  −1 −3 0

96 PROMILITARES.COM.BR

PM_BOOK05_MAT.indb 96 01/11/2021 13:43:01


MATRIZES

SUBTRAÇÃO DE MATRIZES 1 6 4  5 0 2 
Exemplo: Sejam A=  e B=  . Os
Dadas as matrizes A = [aij]m × n e B = [bij]m × n, chamamos de diferença 3 0 2 7 3 4 
entre as matrizes A e B a soma de A com a matriz oposta de B
elementos a 13 = 4 e b 13 = 2 são elementos correspondentes.
Notação: A - B = A + (–B)
Decorrência da definição: A matriz produto A ⋅ B existe apenas
se o número de colunas da primeira matriz (A) é igual ao número de
OBS: A + B existe se, e somente se, A e B são do mesmo tipo
linhas da segunda matriz (B).
(m × n).

Assim: Am x p e Bp x n   A  Bm x n
Exemplo:
Note que a matriz produto terá o número de linhas (m) do primeiro
3 0  1 2  3 0  −1 -2   3 − 1 0 − 2  2 −2
4 − = + = = fator e o número de colunas (n) do segundo fator.
 −7  0 -2   4 −7   0 2  4 + 0 −7 + 2  4 −5

Exemplos:
MULTIPLICAÇÃO DE UM NÚMERO REAL POR 1) Se A 3 x 2 e B2 x 5 ⇒ ( A.B)3 x 5
UMA MATRIZ 2) Se A 4 x 1 e B2 x 3 ⇒ não existe o produto
Dados um número real x e uma matriz A do tipo m × n , o produto
3) A 4 x 2 e B2 x 1 ⇒ ( A.B)4 x 1
de x por A é uma matriz do tipo m × n, obtida pela multiplicação de
cada elemento de A por x. Propriedades: Verificadas as condições de existência, para a

L DO PRO multiplicação de matrizes são válidas as seguintes propriedades:

Notação: B = x · A
OBS.: Cada elemento bij de B é tal que b = xa I
A FE
1) Associativa:

R ij ij
(A ⋅ B) ⋅ C = A⋅(B ⋅ C)

S
TE

Propriedades - Sendo A e B matrizes do mesmo tipo (m × n) e x e


2) Distributiva em relação à adição:

SO
y números reais quaisquer, valem as seguintes propriedades:
MA

a) A⋅(B + C) = A ⋅ B + A ⋅ C
Associativa:

R
b) (A + B)⋅ C = A ⋅ C + B ⋅ C
x·(y·A) = (x·y)·A

Distributiva de um número real em relação a adição de matrizes: 3) Elemento Neutro:


x·(A + B) = x·A + x·B A ⋅ In = In ⋅ A = A R
MA

Distributiva de uma matriz em relação a soma de dois números onde In é a matriz identidade de ordem n.
SO

reais:
Atenção: Não valem as seguintes propriedades:
(x + y) · A = x·A + y·A
T

Elemento Neutro: x·A = A, para x = 1, ou seja:


R 1) Comutativa, pois, em geral, A ⋅ B ≠ B ⋅ A
E

IA F
2) Sendo Om × n uma matriz nula, A ⋅ B = Om × n não implica,

L DO PRO
1·A=A necessariamente, que A = Om x n ou B = Om × n.

Exemplo:
 2 7  3.2 3.7  6 21 MATRIZ INVERSA
3.  = = 
 −1 0 3. ( −1) 3.0  −3 0  Seja A uma matriz quadrada de ordem n. A matriz inversa de
A, simbolizada por (A–1), é o elemento inverso da multiplicação de
matrizes. A matriz inversa A–1 é a matriz que multiplicada por A resulta
MULTIPLICAÇÃO DE MATRIZES na matriz identidade de ordem n (In).
O produto de uma matriz por outra não pode ser determinado
A · A–1 = In
através do produto dos seus respectivos elementos. A multiplicação de
matrizes não é análoga à multiplicação de números reais. Por enquanto iremos mostrar como encontrar uma matriz inversa
Assim, o produto das matrizes A = [aij]m x p e B = [bij]p x n é a matriz de ordem 2 utilizando um sistema 2 x 2.
C = [cij]m x n, onde cada elemento cij é obtido através da soma dos
produtos dos elementos correspondentes da i-ésima linha de A pelos  2 −1
Exemplo: Seja a matriz A =  encontre A–1.
elementos da j-ésima coluna de B.  3 0 
Observação Vamos utilizar a definição A ⋅ A–1 = In,
Elementos correspondentes de matrizes do mesmo tipo m x n, são  2 1 1  1 0 
os elementos que ocupam a mesma posição nas duas matrizes.
ou seja  A   .
3 0   0 1

PROMILITARES.COM.BR 97

PM_BOOK05_MAT.indb 97 01/11/2021 13:43:07


MATRIZES

 a b  2 1  a b   1 0 
Vamos fazer A −1 =  e assim    .
 c d  3 0   c d   0 1
Resolução: B
Utilizando a multiplicação e igualdade de matrizes teremos a12 = 2; a22 = 5; a32 = 1.
 2a − c 2b − d  1 0 Unidades do composto 2: 3 × 2 + 2 × 5 + 5 × 1 = 21.
= .
 3a 3b   0 1

2a − c = 1 2b − d = 0 1
 e  ⇒ a = 0 e –c = 1 ⇒ c = –1; b = e Exercício Resolvido
 3a = 0  3b = 1 3
02. (UENF 2005) A temperatura corporal de um paciente foi
2 medida, em graus Celsius, três vezes ao dia durante cinco dias.
2b = d ⇒ d = .
3 Cada elemento aij da matriz abaixo corresponde à temperatura
 1 observada no instante i do dia j.
0
−1  a b  3
Assim A = =   .  35, 6 36, 4 38, 6 38, 0 36, 0 
 c d  2
−1  36,1 37, 0 37, 2 40,5 40, 4 
 3  
 35,5 35, 7 36,1 37, 0 39, 2 

ProBizu Determine:
I) A multiplicação, por não ser necessariamente comutativa, se a) o instante e o dia em que o paciente apresentou a maior
numa igualdade precisarmos multiplicar os 2 lados da igualdade temperatura;

DO PRO
por uma mesma matriz o lado que se multiplica terá importância. b) a temperatura média do paciente no terceiro dia de observação.

A L
Exemplo:
I
Resolução:
F E 3 do dia 5.
a = 39, 2 ⇒ instante
R
A ⋅ B = C se quisermos “sumir” com a matriz A da equação utilizando
A–1 precisaremos nos atentar para o lado da multiplicação.
35

38, 6 + 37, 2 + 36,1


= 37, 3 .

S
TE

A  B  C  A 1  A  B  A 1  C  In  B  A 1  C  B  A 1  C 3

SO
Está errado A–1 ⋅ A . B = C ⋅ A–1.
MA

R
II) Propriedades da transposição de matrizes
Exercício Resolvido
 A  B   A t  Bt
t

03. (UERJ 2000) Considere as matrizes A e B:


a  A   a  At
t

 1,se i é par
A = (aij) é quadrada de ordem n em que aij = 
A  t t
A
−1,se i é impar
B = (bij) é de ordem n×p em que bij = ji
R
A  B
t
 Bt  A t
MA

SO

a) Calcule a soma dos elementos da diagonal principal da matriz A.


III) Propriedades das matrizes inversas b) O elemento da quarta linha e da segunda coluna da matriz
A   A
1 1 produto AB é igual a 4094.
T

R Calcule o número de linhas da matriz B.


E

A   A 
t 1 1 t
E

 A t
IA F
R −O
A  B
1 Resolução:
 B 1  A 1
L DO P  1 −1 −1 … −1 
 1 1 1 … 1 
 
Exercício Resolvido a) A =  −1 −1 −1 … −1  .
      

01. (UNIRIO 2005) Um laboratório farmacêutico fabrica 3 tipos
 ( −1) ( −1)n ( −1)n … ( −1) 
n n
de remédios utilizando diferentes compostos. Considere a matriz
A = (aij) dada a seguir, onde aij representa quantas unidades do A soma S dos elementos da diagonal principal é tal que S = 0 se n
composto j serão utilizadas para fabricar uma unidade do remédio é par e S = –1 se n é ímpar.
do tipo i.
 −1 −1 −1 … −1   11 21 … p1 
1 2 4  1  2
1 1 … 1
 1 2
2
… p2 
A =  2 5 3  
b)  −1 −1 −1 … −1  1 23
3
… p3 
 0 1 4     
        
 ( −1)n n  n n
Quantas unidades do composto 2 serão necessárias para fabricar 3 ( −1)n ( −1)n … ( −1)   1 2n … p
remédios do tipo 1; 2 remédios do tipo 2 e 5 remédios do tipo 3?
A ⋅ B = C ⇒ c 42 = 21 + 22 + 23 +  + 2n = 4094 ⇒
a) 18 d) 27
b) 21 e) 30

(
2 2n − 1 ) = 4094 ⇒ 2 n
= 2048 = 211 ⇒ n = 11
c) 24 2 −1

98 PROMILITARES.COM.BR

PM_BOOK05_MAT.indb 98 01/11/2021 13:43:19


MATRIZES

EXERCÍCIOS DE

FIXAÇÃO
Exercício Resolvido

 a 0
04. (UNI-RIO 2000) Seja B =   , a, b ≠ 0 uma matriz que
0 b 
0 9 0 −3  1 −1  2 1
satisfaz a equação B−1 ⋅ A + 3A = 
5 0  , onde A = 2 0  . A 01. (EEAR) Considere as matrizes= A = ,B   e
     2 0  0 1
1 1
soma dos elementos da diagonal principal de B é: C=  . Então AB + C é igual a
1 1
1
a)
3 3 0  3 5
a)   c)  
b) –1  1 1  1 3

c) −
11  3 1  −1 1
b)   d)  
6  5 3  2 1
13
d) −
6 1 2  1 0
02. (EEAR) Sabendo-se que M + N =   e M – N = 0 0  , a
matriz N é igual a  3 4   
19
e) −
6  1 1  0 1
a) 3 . c) 3 .
 2  2
Resolução: D
m n L DO PRO 2  2 

Sendo B−1 =   a matriz inversa da B, tem-se:


 p q I A  1 0
b) 3 . FE d)
 3
1 2  .
 a 0 m n  1 0 am an  1 0 1R 
2
2

 
0 2 
0 b  p q = 0 1 ⇒  bp bq = 0 1 ⇒ m = a ; n = 0;

S
TE

        

SO
1 03. (EEAR) Sendo A uma matriz 3 × 4 e B uma matriz N × M, coloque
p = 0; q = .
MA

V (Verdadeira) ou F (Falsa) nas afirmações a seguir:


b
1  ( ) Existe A + B se, e somente se, N = 4 e M = 3.

R
−1 0 9  a 0  0 −3 0 −9 0 9 ( ) Existe A · B se, e somente se, N = 4 e M = 3.
B ⋅ A + 3A =   ⇒  + = ⇒
5 0   0 1   2 0   6 0  5 0  ( ) Existem A · B e B · A se, e somente se, N = 4 e M = 3.
 b 
 3 ( ) A + B = B + A se, e somente se, A = B.
 0 − a  0 −9 0 9 ( ) A · B = B · A se, e somente se, A = B.
 + = ⇒
 2 0  6 0  5 0  Assinale a alternativa que contém a sequência correta:
R
 b 
MA

a) V - V - V - V - V
SO

 3 1 b) F - V - F - V - F
 − a − 9 = 9 ⇒ a = − 6 1 13 c) F-F-V-F-F
 ⇒ a+b = − −2= − .
 2 + 6 = 5 ⇒ b = −2 6 6
T

d) V - V - V - F – V
S

 b
R
E

IA F
04. (EEAR) A = (aij) é a matriz quadrada de ordem 2 em que

Exercício Resolvido
L DO PRO  2, se i < j

aij =
i + j, se i = j , então o determinante da matriz A é
i − j, se i > j

2 7 a) –10.
05. (UNI-RIO 2003) Sendo A =   , obtenha a matriz B tal que
 1 3 b) 10.
B × A = I2, onde I2 é a matriz identidade de ordem 2. c) –6.
d) 6.
Resolução:
 −3 7  2 −1 1 2 
B=   05. (EEAR) Se B =   é a matriz inversa de A = 
 1 −2 x y  , então
x–yé   1 4 
2a + b = 1 a = −3 a) 2.
7a + 3b = 0 b = 7
a b 2 7  1 0   b) 1.
B × A = I2 ⇒     =  ⇒ ⇒
c d  1 3 0 1 2c + d = 0 c = 1 c) –1.
7c + 3d = 1 d = −2
d) 0.

PROMILITARES.COM.BR 99

PM_BOOK05_MAT.indb 99 01/11/2021 13:43:30


MATRIZES

 1 −1  −1 1  2 3 4 5
06. (EEAR) Sejam as matrizes A =   e B =  0 −3 . Se A e 02. (EEAR) Considere as tabelas das lojas A e B, A = 
t

 2 2    5 4 4 3 4 5 5 4 
Bt são as matrizes transpostas de A e de B, respectivamente, então e B=  , em que cada elemento aij ou bij representa o
At + Bt é igual a 3 3 4 2
número de unidades vendidas do produto i no dia j. Considerando as
0 2  0 2 quantidades vendidas nas duas lojas juntas, por dia, o melhor dia de
a) 0 −1 c)  −2 −2
    vendas foi o dia ____.
2 1 0 −1 a) 4
b)  −2 −3 d) 0 5  b) 3
   
c) 2
4 a  b d) 1
07. (EEAR) Sejam as matrizes A =   e B =   . Se A ⋅ B é uma
 2 −1  2
matriz nula 2 × 1, então a + b é  x2 1  9 z 
a) –1.
03. (EEAR) Considere as matrizes reais A =   eB= .
Se A = B , então y + z é igual a
t 2 y + z  y −x 
b) 0.
a) 3
c) 1.
b) 2
d) 2.
c) 1
d) –1
 1 3 0 1
08. (EEAR) Dadas as matrizes A =   e B=  , o produto
D O 04.P(CFT)R SeO 3 4 −2 =  −3a4c
A · B é a matriz  2 0   1 2
b − 1
L  , então a + b + c é igual a

IA
3 7  3 7
a) 2 2
 
c) 0 2
 
a) –1/4.  0
F E1   1 

b)
 4 7
d)
4 4  R b) –3/4.
c) 3.
 2 2 0 2

S
TE

    d) 4.

SO
MA

 1 a   b −1 05. (EEAR) Seja a matriz A = ( aij )2 x 2 tal que a= i2 − j2 . A soma dos
09. (EEAR) Se   e   são matrizes opostas, os valores de
ij

R
 −1 2   x 2k  elementos de A é igual a
a, b, x e k são respectivamente a) 3
a) 1, –1, 1, 1 b) 6
b) 1, 1, –1, –1 c) 9
c) 1, –1, 1, –1 d) 12
d) –1, –1, –2, –2
R
MA

 1 0  −1 1 
SO

06. (CFT) Se A =   e B=  , então A · B =


 1 3 1  1 0 2  0 2  0 2
10. (EEAR) Sendo as matrizes A =  2 0 4  , B =  4 1 0  e a  −1 1   1 −1
  a)  c) 
 −1 2 3   2 3 4   
T

 1 2 0 2 
S

R
E

matriz X – 2A + B = 0, a soma dos elementos da 1ª linha da matriz x é:


E

a) 7 IA b)
 −1 0 
  F d)
 −1 1 
 

L DO PRO
 0 4  0 4
b) 5
c) 4
d) 3  4 2 1
07. (EEAR) Seja a matriz A =   . A matriz X = A tem como
 −6 2  2
soma de seus elementos o valor
EXERCÍCIOS DE a) 7.

TREINAMENTO b) 5.
c) 4.
d) 1.
3 0   2 1
01. Dadas as matrizes A =   e B=  , então A · B – B · A  1 0 −1
é igual a:  1 −4   −1 0
08. (EEAR) Na matriz A =  2 1  faltam 2 elementos. Se nessa
 
a)
0 0 
d)
 −3 1  5 … 3 
0 0   2 7 matriz aij = 2i – j, a soma dos elementos que faltam é
   
a) 4.
2 −3  −3 −1
b) 5 0  e) 7 2 b) 5.
   
c) 6.
 −1 7
c)  9 1 d) 7.
 

100 PROMILITARES.COM.BR

PM_BOOK05_MAT.indb 100 01/11/2021 13:43:31


MATRIZES

0 1 2 6 7 8   2 −1 1 1 
09. (CFT) Sejam as matrizes A =   , B = 9 10 11 e 06 Seja A −1 =   a matriz inversa de A =   . Sabendo que
 3 4 5     −1 x  1 2 
C = ( cij )2 x 3 . Se C = A + B, então c12 + c21 – c23 é igual a A · A-1 = I2, o valor de x é
a) 3. c) 1.
a) –5. c) 1.
b) 2. d) 0.
b) –2. d) 4.
07. A soma dos elementos da diagonal principal da matriz A = (aij)3×3,
 1 1
10. (EEAR) Seja P =   e P a matriz transposta de P. A matriz
t  i2 se i ≠ j
Q=P·P é t 0 1 tal que aij =  , é um número
i + j se i =j
1 1 1 1
a) 1 2 c) 1 0 a) múltiplo de 3. c) divisor de 16.
   
b) múltiplo de 5. d) divisor de 121.
2 1  1 1
b)  1 1 d) 2 0 
    5 −3
08. Sendo a matriz A =   , então a transposta de A é
2

6 −4 
6 7 36 25
a)   c)  
 −2 −3  16 9 
EXERCÍCIOS DE

COMBATE
25 36 7 6
b)  9 16  d)  −3 −2
   

L D O 09.P(EEAR)
R OO elemento X da matriz solução da equação matricial
 0, seA  1 1 F
3,2

tal que a =  I
i= j 10 4 
01. (EEAR) Seja a matriz A = ( aij ) . A soma
3 ⋅ X + 2 4  =  2E 16
R
ij
dos elementos de A é
2 x2
i + j, se i ≠ j   é
a) 4. 6 8   0 8 

S
TE

b) 5.

SO
a) 0 c) 3
MA

c) 6. b) –2 d) 1
d) 7.

R
10. (EEAR) O par (x,y), solução da equação matricial
02. (EEAR) Sejam as matrizes Am x 3 , Bp x q e C5 x 3 . Se A · B = C, então x −4   x 2   13 2x − 4 
m + p + q é igual a  2 ⋅ =  é
x y   y 1  x3 + y2 8 
a) 10.
b) 11. a) (6, ± 3 ) R
c) 12.
( ± 5, −2)
MA

b)
d) 13.
SO

 1 
 3 4 5 −2
c)  ± , −5 
03. Sendo A =   2 
 e B=  , a soma dos elementos da 2ª
T

 −2 1 0 3 
S

 7 4
R − , 
E

linha de (A – B)t é igual a d)


E

IA F  3 5

O
a) –4
b) –2 L DO PR
c) 2 RESOLUÇÃO EM VÍDEO
d) 4 Abra o ProApp, leia o QR Code, assista à resolução
de cada exercício e AVANCE NOS ESTUDOS!

 2 −1  4 5 3
04. Sendo A =   e B=  , a soma dos elementos da
4 5   −1 0 3 GABARITO
1ª linha “A · B” é EXERCÍCIOS DE FIXAÇÃO
a) 22 01. B 04. D 07. A 10. A
b) 30 02. C 05. C 08. C
c) 46 03. C 06. A 09. C
d) 58 EXERCÍCIOS DE TREINAMENTO
01. C 04. B 07. D 10. B
 2 1   x  6  02. B 05. B 08. D
05. Se  ⋅  =   , então o valor de x + y é
 1 −1  y  0 03. A 06. D 09. D
a) 4. EXERCÍCIOS DE COMBATE
b) 5. 01. C 04. A 07. A 10. B
c) 6. 02. B 05. A 08. D
d) 7. 03. D 06. C 09. A

PROMILITARES.COM.BR 101

PM_BOOK05_MAT.indb 101 01/11/2021 13:43:33


MATRIZES

ANOTAÇÕES

L DO PRO
I A FE
R

S
TE

SO
MA

R
R
MA

SO
T

R
E

IA F
L DO PRO

102 PROMILITARES.COM.BR

PM_BOOK05_MAT.indb 102 01/11/2021 13:43:33


DETERMINANTES

DEFINIÇÃO Solução:
Determinante é um número associado a uma matriz quadrada. 1ºPasso: Repetir a duas primeiras colunas ao lado da 3ª:

Aplicações dos determinantes na matemática:


a11 a12 a13 a11 a12
• Cálculo da matriz inversa;
a21 a22 a23 a21 a22
• Resolução de alguns tipos de sistemas de equações lineares;
a31 a32 a33 a31 a32
• Cálculo da área de um triângulo, quando são conhecidas as
coordenadas dos vértices.

D O principal
P2º Passo: Encontrar a soma do produto dos elementos da diagonal
RcomOos dois produtos obtidos com os elementos das paralelas
DETERMINANTE DE PRIMEIRA ORDEM
Dada uma matriz quadrada de 1ª ordem M = [a ],A
L
determinante associado à matriz M o número real aI .
chamamos de
11
a essa diagonal.
F E ser precedida do sinal positivo, ou seja:
R 11 OBS.: A soma deve

S
TE

Notação: det M ou |a11| = a11    a11a22a33  a12a23a31  a13a21a32 

SO
Exemplos:
MA

1. M1  5  det M1  5 ou 5  5
3º Passo: Encontrar a soma do produto dos elementos da diagonal

R
2. M2   3  det M1  3 ou -3  3 secundária com os dois produtos obtidos com os elementos das
paralelas a essa diagonal.
OBS.: A soma deve ser precedida do sinal negativo, ou seja:
DETERMINANTE DE SEGUNDA ORDEM
a a    a13a22a31  a11a23a32  a12a21a33 
Dada a matriz M =  11 12  , de ordem 2, por definição, temos R
a a
 21 22 
MA

SO

que o determinante associado a essa matriz, ou seja, o determinante


Assim D    a13a22a31  a11a23a32  a12a21a33 
a a 
de 2ª ordem é dado por detM   11 12   a11a22   a12a21    a11a22a33  a12a23a31  a13a21a32 
a21 a22 
T

Assim detM  a11a22   a12a21 


R
E

OBS.: Se desenvolvêssemos esse mesmo determinante de 3ªordem


E

Exemplo: IA F
com o auxílio do teorema de Laplace, veríamos que as expressões são
 2 3
Sendo M =  L DO PR O
idênticas, pois representam o mesmo número real.
 , então
 4 5
MENOR COMPLEMENTAR
2 3 Chamamos de menor complementar relativo ao elemento aij de
det M=  2  5  3  4  10  12  2 .
4 5 uma matriz M, quadrada e de ordem n > 1, o determinante MCij, de
ordem n – 1, associado à matriz obtida de M quando suprimos a linha
Logo det M = –2 e a coluna que passam por aij.
Exemplo:
Conclusão: O determinante de uma matriz de ordem 2 é dado a a 
pela diferença entre o produto dos elementos da diagonal principal e 1. Dada a matriz M=  11 12  , de ordem 2, para determinarmos
o produto dos elementos da diagonal secundária. a21 a22 
o menor complementar relativo ao elemento a11 (MC11),
retiramos a linha 1 e a coluna 1;
REGRA DE SARRUS
Dispositivo prático para calcular o determinante de 3ª ordem.
MC = menor complementar
Exemplos:
1. Calcular o seguinte determinante através da Regra de Sarrus.
 a11 a12 
a11 a12 a13 a  , logo, MC11 = |a22| = a22
D= a21 a22 a23  21 a22 
a31 a32 a33

PROMILITARES.COM.BR 103

PM_BOOK05_MAT.indb 103 01/11/2021 13:43:43


DETERMINANTES

Da mesma forma temos que o MC relativo ao elemento a12  a11 a12 a13 
 a11 a12  2. Sendo M = a 
é dado por:   , logo, MC12 = |a21| = a21 e assim por  21 a22 a23  , vamos calcular os cofatores A22,
diante.  a21 a22  A23 e A31. a31 a32 a33 
 a11 a13 
A 22  ( 1)2  2   ( 1)4  a11a33   a13a31    ( 1)  a11a33   a13a31  
Exemplo: a31 a33 
 a11 a12 a13   a11 a12 
A 23  ( 1)2  3   ( 1)5  a11a32   a12a31    ( 1)  a11a32   a12a31  
2. Dada a matriz M= a21 a22 a23  , de ordem 3, vamos a31 a32 
determinar: a31 a32 a33   a12 a13 
A 31  ( 1)3 1    ( 1)  a12a23   a13a22    ( 1)  a12a23   a13a22   .
4
a) MC11 a22 a23 
b) MC12
c) MC13
d) MC21
MATRIZ ADJUNTA
A matriz transposta da matriz dos cofatores de uma matriz A é
chamada adjunta de A.
Solução:
Assim: adjA = (A)t
 a11 a12   A11 A 21 
OBS.: Vamos denotar “menor complementar” por MC = A =  então adjA  
a a
 21 22   A12 A 22 
a) retirando a linha 1 e a coluna 1 da matriz dada acima
 a11 a12 a13   A11 A12 A13 
DO PRO
 a11 a12 a13 
a  a22 a23     
= A =a21 a22 a23  então adjA  A 21 A 22 A 23 
 21 22 a a23  , temos que MC = 
L
  a22a33   a23a32    A A 33 
IA FE
 a31 a32 a33 
11
a a
 32 33   a31 a32 a33   31 A 32

a21 a23 
R
b) retirando a linha 1 e a coluna 2 da matriz dada acima, temos E assim para qualquer ordem.

S
TE

que MC12 =   = a21a33 – (a23a32)


a31 a33  TEOREMA DE LAPLACE

SO
O determinante de uma matriz quadrada M  aij m x m m  2
MA

c) retirando a linha 1 e a coluna 3 da matriz dada acima, temos


a21 a22  pode ser obtido pela soma dos produtos dos elementos de uma fila

R
que MC13 =   = a21a32 – (a22a31)
a31 a32  qualquer (linha ou coluna) da matriz M pelos respectivos cofatores.
m
d) retirando a linha 2 e a coluna 1 da matriz dada acima, temos
 a12 a13 
Assim, fixando j  N, tal que 1  j  m , temos detM 
m
a A ij ij


i 1
que MC21 =   = a12a33 – (a13a32) onde, é o somatório de todos os termos de índice i, variando
a32 a33  R
i 1
MA

de 1 até m, m ∈ N e Aij é o cofator ij.


SO

COFATOR Exemplo:
Chamamos de cofator (ou complemento algébrico) relativo ao Calcular com o auxílio do Teorema de Laplace, os seguintes
elemento aij de uma matriz quadrada de ordem n o número Aij, tal
T

determinantes:
S

que Aij  ( 1)i  j  MCij .


R 1
E

2 3 4
E

IA 2
F
3 4
0 0 2 0

L DO PRO
Exemplos: a) D1  2 1 2 b) D2 
3 1 1 1
0 5 6
a a  1 0 2 3
1. Dada M =  11 12  , os cofatores relativos a todos os
a a
 21 22  Solução
elementos da matriz M são 2 3 4
A11  ( 1)11  a
22  ( 1)  a22  a22
2 a) D1  2 1 2
MC11 0 5 6
1 2
A12  ( 1)  a21  ( 1)3  a21  a21

MC12
Aplicando Laplace na coluna 1, temos
A 21  ( 1)2 1  a12  ( 1)3  a12  a12
 11
1 2 3 4 3 4
MC21 D1  2 (-1) 2) (-1)2 1
 ( 0 3 1
 (-1) 1 2 
5 6 5 6
A 22  ( 1)2  2  a11  ( 1)4  a11  a11 a11
 a21    a31   
 A11( cofator11) CofatorA 21 CofatorA 31
MC22
1 2 3 4
Assim, podemos também determinar a matriz dos cofatores (que  D1  2 2 0
5 6 5 6
 A11 A12   a22 a21 
será denotada por A) como sendo A    .  D1  2(6-10)  2(18  20)  2(-4)  2(38) 
 A 21 A 22   a12 a11 
 D1  8  76  68

b) Como três dos quatro elementos da 2ª linha são nulos,


convém aplicar Laplace nessa linha.

104 PROMILITARES.COM.BR

PM_BOOK05_MAT.indb 104 01/11/2021 13:43:54


DETERMINANTES

2 3 4 1 Exemplos:
0 0 2 0 1 3 4
D2  1) 2 4 6 = 0 pois C1 + C2 = C3
3 1 1 1
1 0 2 3 3 2 5

2 3 1
3 4 1
D2  0  0  2( 1)2  3 3 1 1 +0 
2) 1 2 3 = 0 pois 2L1 + L2 = L3
1 0 3
 7 10 5
D
MC23

OBS.: Então podemos rescrever D2 como D2 = –2D (I) OBS.: Definição de combinação linear:
Um vetor v é uma combinação linear dos vetores v1, v2, ... ,vk, se
existem escalares a1, a2, ... ,ak tal que:
Agora precisamos calcular o valor de D para substituirmos em
(I) Para isso aplicamos Laplace na 3ª linha (mais conveniente, v= a1. v1+...+ ak. vk
pois um dos elementos é nulo), e obtemos
3 -1 2 3 V. Teorema de Jacobi: O determinante de uma matriz não
D  1( 1)3 1  3( 1)3  3  se altera quando somamos aos elementos de uma fila uma
-1 1 3 -1

  combinação linear dos elementos correspondentes de filas
MC31 MC33
paralelas.
D  1(3  1)  3( 2  9) 
 1(2)  3( 11)  2  33  D  35 L D O PExemplo:
R3
1 2 O
I A 2 1 2 =9 FE
Finalmente, substituindo esse valorRem (I), obtemos 2 4 3
D2  2D  D2  -2(-35)  D2  70

S
TE

Substituindo a 1ª coluna pela soma dessa mesma coluna com o

SO
dobro da 2ª, temos:
MA


 

C1  2C2
PROPRIEDADES DOS 1 2  2 2 3 5 2 3
DETERMINANTES

R
2  1 2 1 2  4 1 2  9
As propriedades a seguir são relativas a determinantes associados 2  4  2 4 3 10 4 3
a matrizes quadradas de ordem n. Estas propriedades, muitas vezes
nos permite simplificar os cálculos.
I. Quando todos os elementos de uma fila (linha ou coluna) são
nulos, o determinante dessa matriz é nulo. VI. O determinante de uma matriz e o de sua transposta são
R
iguais.
MA

Exemplos:
Exemplo:
SO

4 9 8 7
3 0 15 1 2 3 1 2 2
0 0 0 0
1) 0 2) 2 0 3  0 Det A = 2 1 2 = 9 Det At = 2 1 4 = 9
T

3 2 1 3
S

1 0 7 2 4 3 3 2 3
R
E

18 12 9 3
E

IA F
L DO PRO
II. Se duas filas paralelas de uma matriz são iguais, então seu VII. Multiplicando por um número real todos os elementos de
determinante é nulo. uma fila em uma matriz, o determinante dessa matriz fica
Exemplo: multiplicado por esse número.

2 5 3 5 Exemplos:
4 2 9 8 1 2 3
=0 pois, L1 = L3
2 1 3 5 2 1 1  4 Multiplicando C1 por 2, temos:
9 7 4 3 3 2 1
2 2 3
III. Se duas filas paralelas de uma matriz são proporcionais, então
o seu determinante é nulo. 4 1 1  2   4   8
Exemplo: 6 2 1

1 4 2
2 1 4 =0 pois C3 = 2C1 5 10 0
1
3 2 6 3 7 4  145 Multiplicando L1 por , temos:
5
2 0 1
IV. Se os elementos de uma fila de uma matriz são combinações
lineares dos elementos correspondentes de filas paralelas, 1 2 0
1
então o seu determinante é nulo. 3 7 4    145  29
5
2 0 1

PROMILITARES.COM.BR 105

PM_BOOK05_MAT.indb 105 01/11/2021 13:44:06


DETERMINANTES

VIII. Quando trocamos as posições de duas filas paralelas, o MATRIZ INVERSA


determinante de uma matriz muda de sinal.
A matriz inversa (A–1) pode ser encontrada através do determinante
Exemplo: da matriz A e da matriz adjunta de .
1 2 3
1
 A
t
2 1 1  4 A 1 
detA
3 2 1

Trocando as posições de L1 e L2, por exemplo, temos: Este método é muito útil quando é preciso encontrar um elemento
de uma matriz inversa.
2 1 1
Exemplo:
1 2 3  4
 1 2 3 
3 2 1
Sendo A   2 0 4  encontre o elemento que ocupa a 3ª
 1 2 2 
 
IX. Quando, em uma matriz, os elementos acima ou abaixo da linha e 1ª coluna de A–1.
diagonal principal são todos nulos, o determinante é igual ao
produto dos elementos dessa diagonal.
Exemplos: Solução
1
  A  primeiro encontramos detA
1 t
a 0 0 x g h Como A 
detA
1) d b 0  a  b  c 2) 0 y i  x  y  z
e f c 0 0 z
L D O P 1R 2O3 1 2
X. Quando, em uma matriz, os elementos acima I Aou abaixo da 1 2 2 E1 2
F
2 0 4 2 0  1 0  ( 2)  2  4  ( 1) 

diagonal secundária são todos nulos, o R


determinante é igual
( 3)  2  2  2  2  ( 2)  1 4  2  ( 3)  0  1)
ao produto dos elementos dessa diagonal, multiplicado por

S
TE

n n 1
 1 . detA  8  12  8  8  20

SO
2
MA

Exemplos: Como precisamos de (A)t faremos o cofator do elemento a13.


0 0 a 2 0

R
A13   1  1  4   4
1 3
0 a 
1)  a  b 2) 0 b x  a  b  c 1 2
b x
c y z

Assim para (A)t o cofator A13 irá ocupar a 3ª linha e 1ª coluna, por
XI. Para A e B matrizes quadradas de mesma ordem n, temos: 1 1
fim chamando B = A–1 teremos b31  4   .
det (AB) = det A ⋅ det B R 20  5
MA

SO

Obs.: Como A ⋅ A-1 = I, na propriedade acima, temos: Observação


1 Utilizando a teoria anterior podemos afirmar para uma matriz
det (A-1) =
 a22 a12 
T

det A
S

 detA  detA 
R  a11 a12 
E

1
A 2x 2    que A   a .
E

IA  a21 a22  F 
21 a11 


L DO PRO
A condição para que uma matriz possua inversa é que seu  detA detA 
determinante seja não nulo, ou seja A 1 existe  detA  0 .

Exercício Resolvido
Exemplo:
1 0 4 2 01. (CEFET 2000) Pode-se afirmar que o determinante
Se A = 2 1 , B = e A⋅B= , então 1 log 2 log 4
3 4 2 2 11 8
−1 log 4 log 8 é:
det  AB  det A  det
B 1 log 8 log16
   5 2
10
a) 0
b) 1
XII. Se k ∈ R, então det (k ⋅ A) = kn ⋅ detA. c) –4log 2
Exemplo: d) –8log 2
2 1 6 3 e) –4log² 2
Sendo k = 3, A = ek⋅A= , temos
4 5 12 15
Resolução: E
det k  A   kn  det
 A log 4  log16  log 2  log 8  log 4  log 8  log 4  log 4 
 2 6
3
54
 log 8  log 8  log 2  log16 
 8 log2 2  3log2 2  6 log2 2  4 log2 2  9 log2 2  4 log2 2  4 log2 2

106 PROMILITARES.COM.BR

PM_BOOK05_MAT.indb 106 01/11/2021 13:44:20


DETERMINANTES

Exercício Resolvido Exercício Resolvido

cos a − sen a 0 05. (UERJ 2001) Os números 204, 782 e 255 são divisíveis por 17.
02. (UNI-RIO 2000) O valor de sen a cos a 0 é: 2 0 4
0 0 2 Considere o determinante de ordem 3: 7 8 2 . Demonstre que
a) 4(cos a + sen a) 2 5 5
b) 4 esse determinante é divisível por 17.
c) 2(cos² a – sen a)
d) 2 Resolução:
e) 0 Adicionando-se à terceira coluna a 1ª coluna multiplicada
por 100 mais a 2ª coluna multiplicada por 10, tem-se
Resolução: D 2 0 4 2 0 204
2(cos² a + sen² a) = 2. 7 8 2 = 7 8 782 . A terceira coluna é divisível por 17,
2 5 5 2 5 255
então o determinante é divisível por 17.
Exercício Resolvido

03. (UFF 2000) Numa progressão aritmética, de termo geral an


e razão r, tem-se a = r = 1 . Calcule o determinante da matriz
1 2 L DO PRO EXERCÍCIOS DE
a
 5
a
a  4
a12  . I A FE
FIXAÇÃO
 4
R

S
TE

Resolução:  3 2 2

SO
1 3 5 01. Sendo a matriz A =  −3 0 1  , calcule o det A e os cofatores
MA

P.A.: 2 , 1, 2 , 2, 2 ,  A11 e A32.  1 1 4


 

R
1 1
a12   11  6
2 2 02. (ESA) Seja A uma matriz de ordem 3 tal que Det (A) = 4. Então
Det (2A) vale
a4   
5
 a5 2 a) 8 c) 32 e) 128
a   2
 4 a12    b) 16 d) 64
 2 6 R
det  15  4  11.
MA

1 0 2
SO

03. (EEAR) O valor do determinante −1 0 −2 é


2 3 4
T

a) – 2 b) 0 c) 1 d) 2
S

Exercício Resolvido
R
E

04. (UERJ 2003) Observe a matriz abaixo:


IA F x −1 x + 2
O
04. (EEAR) O número real x, tal que = 5, é
sen x cos x 1
L D Oa) –P2 R b) – 1 −3 x
2

  c) 0 d) 1
sen x cos x 0
sen x
 1 1
05. (EEAR) Seja uma matriz M do tipo 2 × 2. Se det M = 2, então
Resolvendo seu determinante, será obtido o seguinte resultado: det (10M) é
a) 1 a) 20. b) 80. c) 100. d) 200.
b) sen x
06. (ESA) Sabendo-se que uma matriz quadrada é invertível se, e
c) sen² x somente se, seu determinante é não-nulo e que, se A e B são duas
d) sen³ x matrizes quadradas de mesma ordem, então det (A ⋅ B) = (det A) ⋅ (det B),
pode-se concluir que, sob essas condições
Resolução: D a) se A é invertível, então A ⋅ B é invertível.
sen x cos x  sen x  sen x cos x  sen x cos2 x  b) se B não é invertível, então A é invertível.
 sen x 1  cos2 x   sen x sen2 x  sen3 x c) se A ⋅ B é invertível, então A é invertível e B não é invertível.
d) se A ⋅ B não é invertível, então A ou B não é invertível.
e) se A ⋅ B é invertível, então B é invertível e A não é invertível.

PROMILITARES.COM.BR 107

PM_BOOK05_MAT.indb 107 01/11/2021 13:44:26


DETERMINANTES

0 2 −1 −3
07. (EEAR) Considere a soma S: 04. (EEAR) Sendo m =
en= , pode-se afirmar que
4 6 −5 −7
cos1º cos 2º sen1º sen2º cos 3º cos 4º sen3º sen4º cos 9º cos10º sen9º sen10º
=S + + + + ... + a) m = n. + b) m = –n. c) m = 2n. d) n = 2m.
cos 2º cos1º sen2º sen1º cos 4º cos 3º sen4º sen3º cos10º cos 9º sen10º sen9º
os 4º sen3º sen4º cos 9º cos10º sen9º sen10º  1, se i = j
+ + ... + + . O valor de log S é 05. (EEAR) Considere a matriz A = (aij)3×3 tal que aij =  .O
os 3º sen4º sen3º cos10º cos 9º sen10º sen9º 0, se i ≠ j
valor do determinante de A é
a) zero. c) negativo.
a) a unidade.
b) positivo. d) inexistente.
b) um número primo.
08. (EEAR) Seja A uma matriz de ordem 2, cujo determinante é – 6. Se c) um número par positivo.
det (2A) = x – 87, então o valor de x é múltiplo de d) um número ímpar negativo.
a) 13. b) 11. c) 7. d) 5.
4 x
06. (EEAR) Na equação = x o valor de x é ___.
1 −1 1  5 x+2
  a) 5 b) 4 c) 3 d) 2
09. (EEAR) Para que o determinante da matriz 1 0 b  seja 3, o
valor de b deve ser igual a 1 2 1 
 
a) 2 b) 0 c) –1 d) -2 x 1 1
07. Calcule o valor de x, na equação: 3 1 1 = 24 e assinale a
2 3 6 opção correta. 2 −3 1
10. (EEAR) Seja 4 x 0 = 64. O valor de x que torna verdadeira
a igualdade é −2 0 −2 L D O a) P11R O c) 9 e) 7

a) 4. b) 5. c) –4. A
I d) –5.
b) 10
FE d) 8

R 2 1 3
 2 3
matrizes A = 

S
TE

08 (EEAR) Sejam as = . O valor de


EXERCÍCIOS DE 0 5 1 e B 0 9
(det A) : (detB) é  

TREINAMENTO
SO
3 2 1
MA

a) 4 b) 3 c) – 1 d) – 2

R
a b   −2a 2c 
01. (EEAR) Os valores de x que tornam verdadeira a igualdade
09. (EEAR) Se as matrizes   e   tem determinantes
c d  −3b 3d
x 0 2 x
respectivamente iguais x e y, e ad≠bc, então o valor de é
−1 −1 1 = −2 são tais que seu produto p é elemento do conjunto y
a) 2 c) –6
3 1 x
b) 3 d) –4
R
a) {p ∈  | p > –3}
MA

b) {p ∈  | –3 < p ≤ 2}
SO

1 1 1 
{p ∈  | p < –6}
M  2 −3 x  . Se det M = ax² + bx + c,
c)
10. (EEAR) Seja a matriz=
d) {p ∈  | –6 ≤ p < 2} então o valor de a é
 4 9 x² 
T

R a) 12 c) –5
E

02. Informe se é verdadeiro (V) ou falso (F) o que se afirma abaixo e


E

IA
depois assinale a alternativa que apresenta a sequência correta. b) 10 F d) – 7
(
L DO PRO
) O determinante de uma matriz quadrada de ordem 3 e que
apresenta todos seus elementos iguais a 2 é igual a 0.
( ) Os valores dos determinantes de uma matriz de ordem 2 e de sua EXERCÍCIOS DE

(
transposta são sempre iguais.
) Uma matriz quadrada A e uma matriz quadrada B obtida pela troca
de duas linhas ou duas colunas de A apresentam determinantes
COMBATE
iguais.
( ) O determinante de uma matriz identidade de ordem 3 é igual a 0. 1 0 0 3
2 3 5 1 
a) V – F – F – V
01. (EEAR) O determinante da matriz  é
b) F – F – V – V 1 2 3 −1
 
c) F–V–V–F 3 0 1 4
d) V – V – F – F a) 0 c) 7
e) V – V – V – F b) 8 d) 6

0 x y  2x y 0
03. (EEAR) Se A =  x 0 2  e detA = 4 3 , então x²y² é igual a 02. (EEAR) Se z 0 2y = 16 3, então (xyz)² é igual a
 y 2 0 0 2z 0
 
a) 24 c) 6 a) 8 c) 24
b) 12 d) 3 b) 12 d) 36

108 PROMILITARES.COM.BR

PM_BOOK05_MAT.indb 108 01/11/2021 13:44:27


DETERMINANTES

03. (EEAR) Sejam A = (aij) uma matriz real quadrada de ordem 2 e I2 a


matriz identidade também de ordem 2. Se “r1” e “r2” são as raízes da
equação det (A – r ⋅ I2 ) = n ⋅ r, onde n é um número inteiro positivo,
RESOLUÇÃO EM VÍDEO
podemos afirmar que Abra o ProApp, leia o QR Code, assista à resolução
de cada exercício e AVANCE NOS ESTUDOS!
a) r1 + r2 = a11 + a22
b) r1 + r2 = n (a11 + a22 )
c) r1 ⋅ r2 = det A
d) r1 ⋅ r2 = – n ⋅ det A GABARITO

04. (EEAR) O elemento X3,2 da matriz solução da equação matricial EXERCÍCIOS DE FIXAÇÃO
 1 1 10 4  01. det A = 17, A11 = –1 e A32 = –9
3 ⋅ X + 2 4  =  2 16 é
  02. C 05. D 08. C
6 8   0 8  03. B 06. D 09. B
a) 0 c) 3 04. B 07. D 10. B
b) – 2 d) 1 EXERCÍCIOS DE TREINAMENTO
01. D 04. A 07. C 10. C
−1 −1 0 0
2 3 0 −1 02. D 05. A 08. D
05. (EEAR) Calculando o valor do determinante ,
−2 −1 0 0 03. D 06. B 09. C
obtém-se
a) – 3. c) 1.
0
L D O 01.PC R O
0 −1 1 EXERCÍCIOS DE COMBATE
04. A 07. D 10. C
b) – 1. d) 3.
I A 02. B FE05. B 08. A

06. (AFA) Sendo A = (aij) uma matriz R 2ª


de ordem, com
03. C 06. A 09. D
aij = (–i)j + l – 3j2 e B a matriz dos cofatores dos elementos de A, o valor

S
TE

ANOTAÇÕES
do determinante de A + B é:

SO
a) 680 b) 288 c) –288 d) –680
MA

R
1 2,4 9
07. (AFA) Se −0,5 0 k =
10, então k é:
−2 0,4 −1
a) menor que –4
53
b) igual a − R
22
MA

SO
83
c) igual a −
26
5
d) igual a −
T

2
R
E

IA
08. (AFA) Sejam A, B e C matrizes reais 3 × 3 satisfazendo as seguintes F
L DO PRO
relações A ⋅ B = C–1, B = 2A. Se o determinante de C é 32, qual o valor
do módulo do determinante de A?
1 1 1 1
a) b) c) d)
16 8 4 2

1 0 −1
09. (ESPCEX) O conjunto solução da inequação k 1 3 ≤ 0 é
a) {k ∈  / −4 ≤ k ≤ 1} 1 k 3
b) {k ∈  / −1 ≤ k ≤ 4}
c) {k ∈  / k ≤ −1 ou k ≥ 4}
d) {k ∈  / k ≤ −4 ou k ≥ 1}
e) ∅

 
10. (ESPCEX) Considere a matriz quadrada  sen18° cos72° .
 
O valor do determinante de A é: sen36° cos54° 
a) –2 c) 0
b) –1 d) 1
e) 2

PROMILITARES.COM.BR 109

PM_BOOK05_MAT.indb 109 01/11/2021 13:44:28


DETERMINANTES

ANOTAÇÕES

L DO PRO
I A FE
R

S
TE

SO
MA

R
R
MA

SO
T

R
E

IA F
L DO PRO

110 PROMILITARES.COM.BR

PM_BOOK05_MAT.indb 110 01/11/2021 13:44:28


SISTEMA LINEAR

EQUAÇÃO LINEAR MATRIZ COMPLETA


É Toda equação da forma a1x1 + a2x 2 +  + an xn = b onde É a matriz B, que obtemos ao acrescentarmos à matriz incompleta
a1, a2 , , an são números reais que recebem o nome de coeficientes uma última coluna formada pelos termos independentes das equações
das incógnitas x1, x 2 , xn e b é um número real chamado termo do sistema. Assim a matriz completa referente ao sistema anterior é
independente.  2 3 -1 0 
OBS.: Quando b = 0, a equação recebe o nome de linear B =  4 1 1 7  .
homogênea.  -2 1 1 4 
Exemplos:

D O SISTEMAS
P UmR
HOMOGÊNEOS
EQUAÇÕES LINEARES EQUAÇÕES NÃO-LINEARES
A L O
sistema é homogêneo quando os termos independentes de

1) 3x – 2y + 4z = 7 I
1) xy + 3z + t = 8 FE
todas as equações são nulos.

R Exemplo:
3x − 2y + z = 0

S
TE

2) x + y –3z – 7 t = 0 2) x² – 4y = 3t – 4 
(homogênea)  − x + 4 y − 3z = 0

SO

 2 x + 3y = 4
MA

3) –2x + 4z = 3t – y + 4 3) x –y+z=7

R
SOLUÇÕES DE UM SISTEMA HOMOGÊNEO
A n-upla (0, 0, 0, ..., 0) é sempre solução de um sistema linear
SISTEMA LINEAR homogêneo com n incógnitas e recebe o nome de solução trivial.
Definição: Um conjunto de equações lineares da forma Quando existem, as demais soluções são chamadas não-triviais.
a11x1 + a12x 2 + a13x 3 +  + a1n xn = b1 R
a x + a x + a x +  + a x = b
 21 1 22 2 CLASSIFICAÇÃO DE UM SISTEMA
MA

23 3 2n n 2
 é um sistema linear de m
SO

  LINEAR QUANTO AO NÚMERO DE


am1x1 + am2x 2 + am3x 3 +  + amn xn = bm
SOLUÇÕES
equações e n incógnitas.
T

determinado (solução única)


S

R • possível 
E

indeterminado (infinitas soluções)


E

SOLUÇÃO DO SISTEMA LINEAR


IA F
O
L D O Exemplos:
• impossível (não tem solução)

PR
Chamamos de solução do sistema a n-upla de números reais
ordenados (r1,r2 , ,rn ) que é, simplesmente, solução de todas
equações do sistema. x + y = 8

2x − y = 1
MATRIZES ASSOCIADAS A UM SISTEMA
LINEAR Tem solução única: o par ordenado (3,5). Portanto o sistema é
possível e determinado.
MATRIZ INCOMPLETA x + y = 8

É a matriz A, formada pelos coeficientes das incógnitas do sistema. 2x + 2y = 16
Exemplos: Tem infinitas soluções: algumas são dadas pelos pares ordenados:
(0,8), (1,7), (2,6), (3,5), (4,4), (5,3),.... Portanto o sistema é possível e
Seja o sistema: Matriz incompleta: indeterminado.
2x + 3y − z = 0  2 3 −1 x + y = 10
 
4 x + y + z = 7 A =  4 1 1  − x − y = 10
 −2x + y + z = 4  −2 1 1
 Não tem um par ordenado que satisfaz simultaneamente as
equações. Portanto o sistema é impossível.

PROMILITARES.COM.BR 111

PM_BOOK05_MAT.indb 111 01/11/2021 13:44:36


SISTEMA LINEAR

INTERPRETAÇÃO GEOMÉTRICA DE UM SISTEMA LINEAR

2×2

3×3

L DO PRO
I A FE
R

S
TE

SO
MA

R
R
MA

SO
T

R
E

SISTEMA NORMAL IA F
Um sistema é normal quando tem o mesmo número deL
O
R (n) e o determinante da matriz incompleta associada ao
D O P
equações (m) e de incógnitas
sistema é diferente de zero, ou seja, se m = n e det A ≠ 0, o sistema é normal.
OBS.: Todo sistema normal é possível e determinado e, portanto, tem solução única.
kx + y = 3
Exemplo: Determinar k ∈ R, de modo que o sistema  seja normal.
x + ky = 5
Solução: Para o sistema ser normal temos que observar duas condições: m = n e detA ≠ 0
1ª condição: m = 2 e n = 2 ⇒ m = n
No sistema, o número de equações (m = 2) é igual ao número de incógnitas (n = 2)
2ª condição: det A ≠ 0
k 1
det A = = k 2 − 1 ≠ 0 ⇒ k ≠ ±1
1 k
Logo, o sistema é normal para qualquer k real diferente de 1 e de –1.

REGRA DE CRAMER
Di
Todo sistema normal tem uma única solução dada por xi = , onde i ∈ {1, 2, 3,  ,n} , D = detA é o determinante da matriz
D
incompleta associada ao sistema e D i é o determinante obtido através da substituição, na matriz incompleta, da coluna i pela coluna
formada pelos termos independentes.

112 PROMILITARES.COM.BR

PM_BOOK05_MAT.indb 112 01/11/2021 13:44:39


SISTEMA LINEAR

Exemplo: Resolver com o auxílio da Regra de Cramer, os seguintes não tenham o mesmo número de equações e incógnitas (o que não é
sistemas: permitido na Regra de Cramer). Além disso, quando queremos resolver
sistemas lineares cujo número de equações (e de incógnitas) excede
2x + y = 7
a)  três, não é conveniente utilizar a Regra de Cramer, por se tornar muito
2x − 3y = 3 trabalhosa. Por exemplo, um sistema com quatro equações e quatro
incógnitas requer o cálculo de cinco determinantes de 4ª ordem. Neste
Solução: caso, usamos a técnica de escalonamento, que facilita a resolução e a
2 1 discussão de um sistema.
Temos: m = n = 2 (1ª condição) e D = = −6 − 2 = −8 ≠ 0
2 −3 Dado um sistema linear:
(2ª condição)
a11x1 + a12x 2 + a13x 3 +  + a1n x n = b1
Portanto, como o sistema é normal, podemos utilizar a Regra de a x + a x + a x +  + a x = b
Cramer para resolvê-lo.  21 1 22 2 23 3 2n n 2
S= , onde existe pelo

am1x1 + am2x 2 + am3x 3 +  + amn xn = bm
1º Passo: Calcular Dx e Dy
2 1 menos um coeficiente não-nulo em cada equação, dizemos que S
• Substituindo, na matriz incompleta   , a coluna C1 pela está escalonado se o número de coeficientes nulos antes do primeiro
 2 −3  coeficiente não-nulo aumenta de equação para equação.
coluna formada pelos termos independentes, encontramos

Dx =
7 1
= −21 − 3 = −24
PROCEDIMENTOS PARA ESCALONAR UM
3 −3 SISTEMA

L D O P1) RFixamos
da O
Substituindo, agora, C2 pela coluna dos termos independentes, como 1ª equação uma das que possuam o coeficiente
1ª incógnita diferente de zero.

IA F Eas propriedades de sistemas equivalentes,


2 7
encontramos Dy = = 6 − 14 = −8 2) Utilizando
2 3

2º Passo: Encontrar x e y:
R anulamos todos os coeficientes da 1ª incógnita das demais
equações.

S
TE

Dx −24 3) Anulamos todos os coeficientes da 2ª incógnita a partir da 3ª

SO
Assim x = = =3 equação.
D −8
MA

Dy −8 4) Repetimos o processo com as demais incógnitas, até que o


y= = =1 sistema se torne escalonado.

R
D −8

Logo, (x,y) = (3,1) é a solução do sistema dado. Exemplo:


Para discutir um sistema linear de n equações e n incógnitas, 2x − y + z = 5
calculamos o determinante D da matriz incompleta. Assim, se 
1. Vamos escalonar o sistema 3x + 2y − 4z = 0 . Para facilitar
D ≠ 0 ⇒ Sistema é possível e determinado (SPD), ou seja tem x − 2y + z = 2
R
solução única. 
MA

trocamos a 1ª e 3ª equação, pois esta possui coeficiente de x


D = 0 ⇒ Sistema pode ser possível e indeterminado (SPI) (ter
SO

igual a 1.
infinitas soluções) ou impossível (SI) (não ter solução). x 2y z 2 x 2y z 2
Observações: 3x 2y 4z 0 L2 3 L1 L2 8y 7z 6
T

1) Se o D ≠ 0, o sistema será SPD e portanto teremos uma única 2x y z 5 L3 2 L1 L3 3y z 1 3

R identificarmos
L3 L2 L3
E

solução para o problema. 8


E

2) I A os D ’s para
Se o D = 0, sistema poderá ser SPI ou SI. Para F
de ele é SPI ou SI teremos que encontrar todos
L DO PR
i
saber se o sistema é possível e indeterminado ou impossível.
O x -2y
-8y
z
7z 6
2
2ª Equação: -8y 7(2) 6 y
6 14
8
8
8
1

13z 26 3ª Equação: x 2(1) (2) 2 2


De que forma? z 2
8 8
Se todos os Di forem iguais a 0, teremos um SPI
Portanto, o sistema é possível e determinado, admitindo uma
Se pelo menos um Di diferente de zero, teremos um SI. única solução que é dada por: (x,y,z) = (2,1,2).

PROPRIEDADES DOS SISTEMAS  x − 2y + z = 3



2. Vamos escalonar o sistema 2x + y + z = 1
EQUIVALENTES 3x − y + 2z = 2

I. Trocando de posição as equações de um sistema, obtemos
um outro sistema equivalente.  x − 2y + z = 3 x − 2y + z = 3
 
II. Multiplicando uma ou mais equações de um sistema por 2x + y + z = 1 ⇒ L 2 = 2 .L1 − L 2  − 5y + z = 5 ⇒
um número k, k ∈ R*, obtemos um sistema equivalente ao 3x - y + 2z = 2 L = 3.L − L  − 5y + z = 7
 3 1 3
anterior.
x − 2y + z = 3
III. Adicionando a uma das equações de um sistema o produto 
⇒  − 5y + z = 5
de outra equação desse mesmo sistema por um número k, k
∈ R*, obtemos um sistema equivalente ao anterior. L3 = L2 − L3  0z = −2

Dessa forma fica escalonado. Como não existe valor real de z,


SISTEMAS ESCALONADOS tal que 0 ⋅ z = –2, o sistema é impossível e, portanto não tem
A técnica de escalonar um sistema linear é muito mais utilizada, solução.
pois com essa técnica podemos encontrar soluções para sistemas que

PROMILITARES.COM.BR 113

PM_BOOK05_MAT.indb 113 01/11/2021 13:44:48


SISTEMA LINEAR

 x+ y+ z=6

3. Vamos escalonar o sistema 2x + y − 2z = −1 Resolução: B
3x + 2y − z = 5
 Formando o sistema relacionando “x” à TV, “y” à DVD e “z” ao
 x+ y+ z=6 x + y + z = 6 Som utilizamos as informações dos preços e escalonamos:
 
2x + y − 2z = −1 ⇒ L2 = 2.L1 − L2  y + 4z = 13 ⇒
3x + 2y − z = 5 x + y = 1200 x + y = 1200 x + y = 1200
 L3 = 3.LL1 − L3  y + 4z = 13   
y + z = 1100 ⇒ L1 − L3 y + z = 1100 ⇒ L2 − L3 y + z = 1100
x + y + z = 6 x + z = 1500 y − z = −300 2z = 1400
   
⇒  y + 4z = 13 ⇒
L3 = L2 − L3  0z = 0

O sistema está escalonado. Entretanto, o número de equações (m)


é menor que o número de incógnitas (n). Assim, o sistema é possível Exercício Resolvido
e indeterminado, admitindo infinitas soluções.
03. Dois casais foram a um barzinho. O primeiro pagou R$ 5,40
Fazendo Z = α e substituindo esse valor na 2ª equação, obtemos: por 2 latas de refrigerantes e uma porção de batatas fritas. O
y + 4α = 13 ⇒ y = 13 − 4α segundo pagou R$ 9,60 por 3 latas de refrigerantes e 2 porções de
batatas fritas. A diferença, em reais, entre o preço de uma porção
Substituímos esses valores na 1ª equação
de fritas e de uma lata de refrigerante nesse bar é
x = 6 − 13 + 4α − α ⇒ x = −7 + 3α :
a) 1,50
Assim, a solução do sistema é dada por:
b) 1,60
S = {( −7 + 3α, 13 − 4α, α )} , sendo α ∈ R.
Para cada valor que seja atribuído a α, encontraremos D O c)d)P 1,80
RO
1,70
uma quádrupla que é solução para o sistema. Por exemplo
A L
α = 1 ⇒ S = {( −7 + 3(1), 13 − 4(1), 1)} = {( −4, 9, 1)} .I F
Resolução: D E
R Formando o sistema relacionando “x” à lata de refrigerante e “y”

S
TE

Exercício Resolvido à porção de batatas fritas utilizamos as informações dos preços e

SO
escalonamos:
01. Um jogador de basquete fez o seguinte acordo com o seu
MA

2x + y = 5, 40 2x + y = 5, 40
clube: cada vez que ele convertesse um arremesso, receberia  ⇒ 3L1 − 2L2 
R$ 10,00 do clube e, caso errasse, pagaria R$ 5,00 ao clube. Ao 3x + 2y = 9 , 60  − y = −3, 00 ⇒ y = 3, 00

R
final de uma partida em que arremessou 20 vezes, recebeu a
Logo, 2x = 5,40 – 3,00 = 2,40. Então x = 1,20.
quantia de R$ 50,00. A quantidade de arremessos que ele acertou
foi A diferença entre a porção de fritas e a lata de refrigerante é:
R$ 3,00 – R$ 1,20 = R$ 1,80.
a) 6
b) 7 R
c) 8
MA

SO

d) 9 Exercício Resolvido
e) 10
04. Os valores de x, y, e z que satisfazem ao sistema:
T

 x y z
S

Resolução: E
3 .3 .3 = 1
R
E

Será considerado que receber é um valor positivo para o jogador


IA
e pagar, negativo. Utilizando a variável “x” para o número de
 x
 2
 y z =4 F
são, respectivamente
acertos e “y” para o número de erros, o sistema que resolve a
x + y = 20
questão é: 
L DO PRO  2 .2
 −x y z 1
4 .16 .4 = 4
10x − 5y = 50
x + y = 20 x + y = 20 a) (1, 1, - 2)
 ⇒ b) (1, 1, 1)
10 x − 5y = 50 15y = 150 ⇒ y = 10
c) (- 1, 1, - 2)
d) (- 2, 1, 1)
e) (- 1, - 1, - 1)
Exercício Resolvido
Resolução: A
02. Uma loja de departamentos, para vender um televisor, um
DVD e um aparelho de som, propôs a seguinte oferta: o televisor Utilizando as propriedades das potências reescrevemos o sistema
e o DVD juntos custam R$ 1200,00; o DVD e o som juntos custam como potências de mesma base:
R$ 1100,00 e o televisor com o som custam juntos R$ 1500,00.  x y z
Um cliente que comprar os três produtos pagará, em reais 3 .3 .3 = 1 3x + y + z = 30
 x 
a) 1800,00  2
 y z =4 ⇒ 2x.2− y.2− z = 22
b) 1900,00  2 . 2  −2x 4 y 2z −2

c) 2000,00  −x y z 1 2 .2 .2 = 2
 4 . 16 . 4 =
4
d) 2100,00
e) 2200,00

114 PROMILITARES.COM.BR

PM_BOOK05_MAT.indb 114 01/11/2021 13:44:56


SISTEMA LINEAR

03. (EEAR) A tabela mostra os pedidos de 4 clientes em uma


Igualando os expoentes e escalonando, vem: lanchonete.
x + y + z = 0 x + y + z = 0
 L1 − L2 
 x − y − z = 2 ⇒ 2y + 2z = −2 ⇒ CLIENTE PEDIDOS
 −2x + 4 y + 2z = −2 2L1 + L3 6y + 4z = −2
  1 suco de laranja, 2 hambúrgueres e
1
x + y + z = 0 3 porções de batata frita.

⇒ 3L2 − L3 2y + 2z = −2 3 sucos de laranja, 1 hambúrguer e
2z = −4 ⇒ z = −2 2
 2 porções de batata frita.
2 sucos de laranja, 3 hambúrgueres e
Substituindo “z” e calculando os outros valores, temos: 3
1 porção de batata frita.
z = −2
2 1 suco de laranja, 1 hambúrguer e
2y + 2( −2) = −2 ⇒ y = = 1 4
2 1 porção de batata frita.
x + 1 + ( −2) = 0 ⇒ x = 1
Se os clientes 1, 2 e 3 pagaram, respectivamente, R$ 11,10, R$ 10,00
e R$ 11,90 por seus pedidos, então o cliente 4 pagou R$
a) 5,00. c) 5,40.
Exercício Resolvido b) 5,10. d) 5,50.

 x + 3y = m
D O 04.P(EEAR)
05. O sistema linear abaixo, nas incógnitas x e y:  .  x + my = 1
Será impossível quando: 2x − py = 2
L RO Seja 
4 x + 5y = 2
um sistema de equações do 1º grau nas

a) Nunca
IA incógnitas x e y.F
Ele será impossível se o valor de m for
b)E 3/2.
b) p ≠ –6 e m = 1
c) p ≠ –6 e m ≠ 1
R a) 5/4. c) 5/3. d) 2.

S
TE

d) p = –6 e m = 1 ax + 2y = −1  2x + y = 1
05. (EEAR) Se  e  são sistemas equivalentes,

SO
e) p = –6 e m ≠ 1  3x + by = 3 x − y = −4
MA

então o valor de a + b é
a) 11. b) 9. c) –5. d) –7.

R
Resolução: E

Se D = 0 ⇔ SPI ou SI
 x+y=3
1 3 06. (EEAR) O sistema  é possível e indeterminado para
= 0 ⇔ −p − 6 = 0 ⇔ p = −6 . Fazendo p = –6, temos: 2x − my = 6
2 −p
a) m = 2. b) m ≠2. c) m = –2. d) m ≠ –2.
 x + 3y = m R

3x + my = 0
MA

2x + 6y = 2
07. (EEAR) Para que o sistema  tenha solução diferente
SO

 x + 3y = 0
Resolvendo temos 0 = –2m + 2
da imprópria, o valor de m deve ser
Logo, o sistema será SI quando – 2m + 2 for diferente de zero, ou
a) 9. b) 0. c) 10. d) 15.
T

seja, quando m ≠ 1
S

R
E

IA F
08. (CFOE) Dado o sistema linear abaixo com solução {(x, y, z)}, o valor

L DO PRO
de (x,y,z)² é igual a
EXERCÍCIOS DE  3x − 5y − 2z = 9

FIXAÇÃO

 −2x − 3y + z = 0
 x+y+z= 2

a) 27 b) – 27 c) 8 d) – 8
 ax + by = c
01. (EEAR) Sendo abcd ≠ 0, para que o sistema  seja
px + qy = d x − y = 1
indeterminado, é necessário que p e q sejam respectivamente iguais a 
09. (EEAR) Para que valor de “K” o sistema y + 3z = 1 não possui
da bd c) ab e d . solução? 2x + Kz = 2
a) e . 
c c c c
a) – 3 c) 6
bd da . d ab
b) e d) e . b) – 6 d) 3
c c c c

3x − 2y = −4
 x+y−z=0 
10. (EEAR) O sistema x + 4 y = −6 , nas incógnitas x e y, admite uma
 2x − 3y = m
02. (EEAR) Se a solução do sistema  x − y − 2z = 1 é {(a, b, c)}, então 
o valor de “a ⋅ b ⋅ c” é x + 2y + z = 4
 única solução se, e somente se,
a) – 12. c) – 24. a) m ≠ –1 c) m = –1
b) – 18. d) – 30. b) m = 0 d) m = 2

PROMILITARES.COM.BR 115

PM_BOOK05_MAT.indb 115 01/11/2021 13:45:07


SISTEMA LINEAR

3x + Ky + z =0
EXERCÍCIOS DE 
06. O sistema de equações lineares 5x + 4y + 5z =

TREINAMENTO
0 admite mais de
uma solução se: x + y + Kz =0

a) K = 7/6
b) K= 7/5 ou K = 2
 kx − y + z =0 c) K = 7/3 ou K = 2
 d) K = 7/2 ou K = 2
01. Para que o sistema  2x − 4y − z =1 seja possível e determinado,
deve-se ter −3x + 4y − z =−1

a) k ≠ 9/8 kx + 4ky =0
07. Relativas ao sistema  ,k ∈  , considere as afirmações
b) k ≠ 2/5 I, II e III abaixo. 3x + ky =8
c) k = 7/6 I. Apresenta solução única para, exatamente, dois valores distintos
d) k = 1/3 de k.
II. Apresenta mais de 1 solução para um único valor de k.
02. (ESA) Três amigos, Abel, Bruno e Carlos, juntos possuem um III. É impossível para um único valor de k.
total de 555 figurinhas. Sabe-se que Abel possui o triplo de Bruno
menos 25 figurinhas, e que Bruno possui o dobro de Carlos mais 10 Dessa forma,
figurinhas. Desses amigos, o que possui mais tem a) somente I está correta.
a) 250 figurinhas. b) somente II e III estão corretas.
b) 365 figurinhas. c) somente I e III estão corretas.

D O e) PI, IIRe IIIO


c) 275 figurinhas. d) somente III está correta.
d) 325 figurinhas.
A L estão corretas.
e) 300 figurinhas.
I F − 2y − 2z − 2 =0
2x E
R
03. (ESA) Uma pessoa deseja totalizar a quantia de R$ 600,00

08. O sistema  2x + y + 3z =6 é possível e determinado, quando

S
TE

utilizando cédulas de um, dez e vinte reais, num total de 49 cédulas,  kx + y + 5z =9


SO
de modo que a diferença entre as quantidades de cédulas de dez e o valor de k for:
MA

de um real seja igual a nove unidades. Nesse caso, a quantidade de


cédulas de vinte reais de que a pessoa precisará será igual a: a) k ≠ 3.

R
a) 10 b) k = 5.
b) 19 c) k = 3.
c) 20 d) k ≠ 5.
d) 21 e) k = 0.
e) 29
5x + 3y + 4z =
R 3

MA

09. Considerando o sistema 15x + 9y + 8z =6 analise as


 kx + 2y − z =2
SO

 afirmativas abaixo e conclua. 20x + 12y + 16z =12


04. (ESA) O valor de k real, para que o sistema 2x − 8y + 2z = 0 seja 
possível e determinado, é:  2x + z = a) O sistema é impossível.
 4
b) O sistema é possível e indeterminado.
T

1
S

a) k≠−
R c) O sistema é possível e determinado.
E

2
E

1 IA F
d) O sistema admite como solução única x = 4, y = 8, z = –11.

L DO PRO
b) k=
2 e) O sistema admite como solução, para qualquer valor de x a terna
1 (x,x,5x).
c) k≠−
6
10. Uma empresa deve enlatar uma mistura de amendoim, castanha
3 de caju e castanha-do-pará. Sabe-se que o quilo de amendoim custa
d) k≠−
2 R$ 5,00, o quilo de castanha de caju, R$ 20,00 e o quilo de castanha-
7 do-pará, R$ 16,00. Cada lata deve conter meio quilo da mistura e o
e) k≠−
2 custo total dos ingredientes de cada lata deve ser de R$ 5,75. Além
disso, a quantidade de castanha de caju em cada lata deve ser igual
2x + ay − 4z =2 a um terço da soma das outras duas. Nesse caso, as quantidades de

05. Para que o sistema  x − 2ay − 4z =
1 seja possível e indeterminado, cada ingrediente por lata são
o valor de a deve ser  x + 2y − 4z = 3 a) 270 g de amendoim, 125 g de castanha de caju e 105 de

a) diferente de –1 castanha-do-pará.
b) igual a 1 b) 270 g de amendoim, 172,5 g de castanha de caju e 57,5 g de
c) diferente de 1 castanha-do-pará.

d) igual a –1 c) 250 g de amendoim, 125 g de castanha de caju e 125 g de


castanha-do-pará.
d) 228 g de amendoim, 100 g de castanha de caju e 72 g de
castanha-do-pará.

116 PROMILITARES.COM.BR

PM_BOOK05_MAT.indb 116 01/11/2021 13:45:08


SISTEMA LINEAR

x − 3y + kz =0
EXERCÍCIOS DE 
05. (AFA) Considere o sistema linear homogêneo 3x + ky + z =0,

COMBATE kx + y =
 0
onde k é um número real. O único valor que torna o sistema, acima,
possível e indeterminado, pertence ao intervalo
01. (EFOMM) Na Escola de Marinha Mercante, há alunos de ambos os a) (–4,–2]
sexos (130 mulheres e 370 homens), divididos entre os Cursos Básico, b) (–2,1]
de Máquinas e de Náutica. Sabe-se que do total de 130 alunos do c) (1,2]
Curso de Máquinas, 20 são mulheres. O Curso de Náutica tem 270
d) (2,4]
alunos no total e o Curso Básico tem o mesmo número de homens e
mulheres. Quantas mulheres há no Curso de Náutica? e) (4,6]
a) 50 c) 60 e) 70
b) 55 d) 65  −x − 2y + 3z =0

06. (ESPCEX) O valor de m, para que o sistema  2x + y − 4z = 0
admita soluções além da solução trivial, é 4x + my − 10z = 0
02. (AFA) O conjunto de soluções de uma única equação linear a1x + 
a2y + a3z = b é representado por um plano no sistema de coordenadas a) 1
retangulares xyz (quando a1, a2, a3 não são todos iguais a zero). Analise b) 3
as figuras a seguir.
c) 5
d) 7
e) 9

L D O 07.P(ESPCEX)
R OA soma dos valores de x, y e z que tornam o sistema
I A 5F
R
 2x + y − z =
 E
3x − 2y + z =−2 verdadeiro é:
 x+z=0

S
TE

SO
a) 1 d) 5
MA

b) 3 e) 4
Assinale a opção verdadeira. c) 2

R
a) A figura I representa um sistema de três equações com uma única
solução.
a2x + y = 1
b) A figura III representa um sistema de três equações cujo conjunto 08. (ESPCEX) Dado o sistema linear  , onde a é uma
constante real, pode-se afirmar que:  x + y =a
solução é vazio.
c) A figura II representa um sistema de três equações com uma a) o sistema é possível e determinado para a = –1.
infinidade de soluções.
R
b) existe um único valor de a que torna o sistema possível e
MA

d) As figuras I e III representam um sistema de três equações com indeterminado.


SO

soluções iguais. c) o sistema é possível e determinado somente se a ≠ –1.


d) o sistema é possível e determinado ∀x ∈ .
 x+y+z= 1 e) o sistema é impossível ∀x ∈ .
T


R
03. (ESPCEX) Sabendo que (x,y,z) é solução do sistema  x − y + 2z =3,
E

o valor de x² + y² + z² é: 2x + 3y − z =
IA
 1
F  x−y+z= 8

L DO PRO
a) 5 
09. (ESPCEX) A soma das soluções do sistema  2x + y + z =5 é:
b) 6 a) 4 x + 2y − z =−8

c) 7 b) 5
d) 9 c) 6
e) 10 d) 7
e) 8
04. (AFA) Sr. Osvaldo possui certa quantia com a qual deseja adquirir
um eletrodoméstico. Caso a loja ofereça um desconto de 40%, ainda
lhe faltarão 1000 reais. kx + 2y + 2z =5

Se o Sr. Osvaldo aplicar sua quantia a juros (simples) de 50% ao 10. (ESPCEX) Os valores de K para que o sistema linear  2x + ky + z =3
mês, ajunta, em três meses, o montante correspondente ao valor do seja possível e tenha uma única solução são  2x + 3y + z =8

eletrodoméstico sem o desconto. a) k = –{–1,2}
Assim, o valor do eletrodoméstico e da quantia que o Sr. Osvaldo b) k = –{–2,2}
possui somam, em reais, c) k = –{1,2}
a) 4000 d) k = –{3,4}
b) 5000 e) k = –{1,–2}
c) 7000
d) 8000

PROMILITARES.COM.BR 117

PM_BOOK05_MAT.indb 117 01/11/2021 13:45:12


SISTEMA LINEAR

RESOLUÇÃO EM VÍDEO
Abra o ProApp, leia o QR Code, assista à resolução
de cada exercício e AVANCE NOS ESTUDOS!

GABARITO
EXERCÍCIOS DE FIXAÇÃO
01. A 04. A 07. A 10. C
02. D 05. B 08. D
03. D 06. C 09. C
EXERCÍCIOS DE TREINAMENTO
01. A 04. D 07. B 10. C
02. B 05. D 08. D
03. C 06. B 09. B
EXERCÍCIOS DE COMBATE
01. C 04. C 07. C 10. D
02. B
03. A
05. B
06. C
08. B
09. A L DO PRO
I A FE
ANOTAÇÕES
R

S
TE

SO
MA

R
R
MA

SO
T

R
E

IA F
L DO PRO

118 PROMILITARES.COM.BR

PM_BOOK05_MAT.indb 118 01/11/2021 13:45:12


GEOMETRIA PLANA: PRINCIPAIS
TEOREMAS ANGULARES E MÉTRICOS

ÂNGULOS NA CIRCUNFERÊNCIA ÂNGULO DE SEGMENTO


Ângulo de segmento ou semi-inscrito é um ângulo com
ÂNGULO CENTRAL vértice sobre a circunferência, um lado secante e outro tangente à
circunferência. O ângulo de segmento é igual à metade do arco por
Ângulo central é um ângulo cujo vértice é o centro da ele determinado.
circunferência e seus lados são raios. O ângulo central é igual ao arco
por ele determinado.

L DO PRO
I A FE
R

S
TE

SO
MA


θ = AB

R
ÂNGULO INSCRITO
Ângulo inscrito é um ângulo com vértice sobre a circunferência e
cujos lados são secantes à circunferência. O ângulo inscrito é igual à R
metade do arco por ele determinado. 
AB
MA

θ=
SO

2
T

ÂNGULO EXCÊNTRICO INTERNO


S

R
E

Ângulo excêntrico interno é o ângulo formado por duas cordas


E

IA F
que se interceptam em um ponto interior da circunferência, distinto

L DO PRO do centro. O ângulo excêntrico interno é igual à semissoma dos arcos


por ele determinados.


AB
θ=
2

Todo ângulo reto é inscritível em uma semicircunferência e,


reciprocamente, todo ângulo inscrito em uma semicircunferência e
com lados passando pelas extremidades da mesma, é reto.

  CD
AB 

2

PROMILITARES.COM.BR 119

PM_BOOK05_MAT.indb 119 01/11/2021 13:45:13


GEOMETRIA PLANA: PRINCIPAIS TEOREMAS ANGULARES E MÉTRICOS

ÂNGULO EXCÊNTRICO EXTERNO Exercício Resolvido


Ângulo excêntrico externo é o ângulo formado por duas secantes
ou tangentes que se interceptam no exterior da circunferência.   60 e
01. Na figura, calcule , , , ,  , sabendo que AB
O ângulo excêntrico externo é igual à semidiferença dos arcos por   30 .
DE
ele determinados. No caso do ângulo formado por duas tangentes,
o ângulo excêntrico externo também pode ser calculado como o
suplemento do menor arco determinado.

L DO PRO
  CD
AB 
I A Resolução: FE

2 R   60 .
ˆ  AB
AÔB é um ângulo central, então   AOB

S
TE


ˆ
ACB ˆ  AB  60  30 .

SO
é um ângulo inscrito, então   ACB
2 2
MA

 60
AB
ˆ 
BÂG é um ângulo de segmento, então   BAG   30

R
2 2
ˆ
AFB é um ângulo excêntrico interno, então ...
ˆ é um ângulo excêntrico externo, então
DPE
  DE
AB  60  30
ˆ 
  DPE   15 .
2 2
R
MA

SO

ARCO CAPAZ
Um par de arcos capazes de θ sobre um segmento AB é o lugar
T

R geométrico dos pontos do plano que são vértices de ângulos de


E

IA F
medida θ e extremidades em A e B.

L DO PRO
  BT
AT 

2

 maior  AB
AB  menor
  menor
 180  AB
2

120 PROMILITARES.COM.BR

PM_BOOK05_MAT.indb 120 01/11/2021 13:45:21


GEOMETRIA PLANA: PRINCIPAIS TEOREMAS ANGULARES E MÉTRICOS

Arcos capazes de ângulos suplementares, relativos a um segmento QUADRILÁTERO INSCRITÍVEL


AB, e em semiplanos opostos em relação à reta suporte do segmento
são partições de uma mesma circunferência. Um quadrilátero está inscrito em uma circunferência se os seus
quatro vértices pertencem a essa circunferência.
Um quadrilátero convexo é inscritível em uma circunferência se, e
somente se, seus ângulos opostos são suplementares

L DO PRO #ABDC é Inscritível

PROPRIEDADES DA CIRCUNFERÊNCIA 
I Adeterminam
Duas retas paralelas, secantes a uma circunferência,
FE Aˆ  Cˆ  Bˆ  Dˆ  180
arcos de igual medida. R
Um quadrilátero é inscritível se, e somente se, as diagonais e dois

S
TE

lados opostos determinam ângulos congruentes.

SO
MA

R
R
MA

SO
T

R
E

  BD

E

r  s  AC
IA F #ABDC é Inscritível

L R O
Duas cordas de mesmo comprimento determinam sobre
mesma circunferência arcos congruentes e vice-versa.
uma
DO P ˆ

ˆ
BAD = BDC

Exercício Resolvido

02. Na figura abaixo, encontre o valor do ângulo BÂD = q.

  CD
AB  CD  AB 

PROMILITARES.COM.BR 121

PM_BOOK05_MAT.indb 121 01/11/2021 13:45:25


GEOMETRIA PLANA: PRINCIPAIS TEOREMAS ANGULARES E MÉTRICOS

Resolução: Exercício Resolvido


ˆ  30  80  BCE
No triângulo BCE, temos: BCE ˆ  50 (ângulo 04. Determine os valores de x e y, sendo r, s e t retas paralelas.
externo).
Como BCA ˆ  BCEˆ  50  ADB
ˆ , então o quadrilátero ABCD é
inscritível, o que implica
ˆ  180   36  50   94
ˆ  180  BCD
  BAD .

TEOREMA DE TALES
Um feixe de retas paralelas determina sobre duas secantes
quaisquer segmentos correspondentes proporcionais.
Sejam as retas Resolução:
A1A 2 A 2A 3 A A Como as retas r, s e t são paralelas, os segmentos de reta
r1  r2  r3    rn 1  rn , entª o   …  n 1 n determinados sobre as duas transversais são proporcionais. Atente,
B1B2 B2B3 Bn 1Bn
. entretanto, para a correspondência entre os segmentos em cada
uma das transversais.
Em uma das transversais, são determinados segmentos de medida
3, 2 e y e, na outra transversal, segmentos de medida 5, x e 6,

L D O 3P 2R yO 2  5 10
respectivamente. Assim, temos:

IA    xF
3  6 18
  y  .

R
5 x 6
E3 3 5 5

S
TE

SO
Uma outra maneira de identificar os segmentos proporcionais e
MA

traçar uma paralela a uma das transversais, de forma que o ponto de


interseção dessas retas seja exterior às retas r e t.

R
Exercício Resolvido R
MA

03. Determine o valor de x, sendo r, s e t retas paralelas.


SO
T

R
E

IA TEOREMA F
DAS BISSETRIZES
O
L D OTEOREMA
P R DA BISSETRIZ INTERNA
A bissetriz interna de um dos ângulos de um triângulo divide o
lado oposto internamente em segmentos proporcionais aos lados
adjacentes.
Seja AD a bissetriz interna do ângulo  de um triângulo ABC,
Resolução: BD DC
então = .
Como as retas r, s e t são paralelas, os segmentos de reta AB AC
determinados sobre as duas transversais são proporcionais. Assim,
x 6 4 6
temos:  x 3
4 8 8

122 PROMILITARES.COM.BR

PM_BOOK05_MAT.indb 122 01/11/2021 13:45:29


GEOMETRIA PLANA: PRINCIPAIS TEOREMAS ANGULARES E MÉTRICOS

Exercício Resolvido SEMELHANÇA DE TRIÂNGULOS


Se dois triângulos possuem lados respectivamente proporcionais,
05. Em um triângulo ABC de lados AB = 12, AC = 8 e .., determine
então são semelhantes.
o maior segmento que a bissetriz interna do ângulo  determina
sobre o lado BC.

Resolução:

O maior segmento determinado pelo pé da bissetriz, D, sobre BC a b c


é o correspondente ao maior dos lados adjacentes ao vértice A, ABC  A ’B’C’   
a’ b’ c ’
ou seja, BD.
Sendo BD = x, pelo teorema das bissetrizes internas, temos:
D O sãoPrespectivamente
Dois triângulos são semelhantes se, e somente se, seus ângulos
R O congruentes.
12

8
 120  12x  8x  20x  120  x  6 L
IA F E são semelhantes, então a razão entre duas
x 10  x Dois triângulos de lados respectivamente paralelos são semelhantes.

R Se dois triângulos
linhas homólogas é igual à razão de semelhança.

S
TE

TEOREMA DA BISSETRIZ EXTERNA

SO
CASOS DE SEMELHANÇA DE TRIÂNGULOS
A bissetriz externa de um dos ângulos de um triângulo divide o
MA

lado oposto externamente em segmentos proporcionais aos lados 1° caso: (A.A.) Se dois triângulos possuem dois ângulos
adjacentes. respectivamente congruentes, então são semelhantes.

R
Seja AE a bissetriz interna do ângulo  de um triângulo ABC,
BE CE
então = .
AB AC

R
MA

SO
T

R
E

IA F
L DO PRO Bˆ  Bˆ ’ 
Cˆ  Cˆ ’
  ABC  A ’B’C’
Exercício Resolvido

06. Em um triângulo ABC de lados AB = 12, AC = 8 e BC = 10,


2° caso: (LpALp) Se dois triângulos possuem dois lados proporcionais
determine a distância entre o pé da bissetriz externa do ângulo Â
adjacentes a ângulos congruentes, então são semelhantes.
e o vértice mais próximo do lado BC.

Resolução:
O pé da bissetriz externa está do junto ao menor lado. Assim,
devemos calcular a medida de CE = x.
Pelo teorema das bissetrizes externas, temos: b c
 
12 8 b’ c ’   ABC ∼ A ’B’C’
  12x  80  8x  4 x  80  x  20 .
10  x x A
A  ’


PROMILITARES.COM.BR 123

PM_BOOK05_MAT.indb 123 01/11/2021 13:45:34


GEOMETRIA PLANA: PRINCIPAIS TEOREMAS ANGULARES E MÉTRICOS

3° caso: (LpLpLp) Se dois triângulos possuem os três lados Exercício Resolvido


respectivamente proporcionais, então são semelhantes.
08. (AFA 2005) Considere o triângulo ABC, de lados AB = 15,
AC = 10, BC = 12 e seu baricentro G. Traçam-se GE e GF paralelos a
AB e AC, respectivamente, conforme a figura abaixo. O perímetro
do triângulo GEF é um número que, escrito na forma de fração
irredutível, tem a soma do numerador com o denominador igual a:

a) 43
b) 40

a b c c) 38
   ABC  A ’B’C’ d) 35
a’ b’ c ’

Resolução: B

D O Como
P RG éOo baricentro do triângulo ABC, então GM
Exercício Resolvido
1
L AM 3
= .

IA
07. Considere os quadrados da figura de lados a e b (a > b). Então
Como GE||AB Fe GF||AC, então ∆GEF ∼ ∆ABC (A.A.) e a razão de
x é igual a
R semelhança é
GME= 1 , obtida a partir das medianas homólogas.
AM 3

S
TE

Como a razão entre os perímetros dos triângulos também é igual à

SO
razão de semelhança, temos:
MA

2pGEF 2pGEF 1 37
   2pGEF 
2pABC 15  10  12 3 3

R
.
Como a fração obtida acima já se encontra em sua forma
irredutível, então a soma de seu numerador e seu denominador é
igual à 37 + 3 = 40.

R
RELAÇÕES MÉTRICAS NO
MA

SO

b2
c)
ab TRIÂNGULO RETÂNGULO
a) a+b
a−b Seja o triângulo ABC retângulo em A, conforme a figura a seguir:
T

ab
a2
R
E

b) d)
E

a−b a−b
IA F
Resolução: A L DO PRO

BC AC AB a b c a  h  b  c
ABC ~ ABH  A.A.A.        2
AB AH BH c h n c  a  n
BC AC AB a b c
ABC ~ ACH  A.A.A.        b2  a  m
AC CH AH b m h
Os triângulos retângulos “1” e “2” possuem lados paralelos, logo AB AH BH c h n
ABH ~ ACH  A.A.A.        h2  m  n
são semelhantes. AC CH AH b m h
ab b b2
Assim, temos:  x .  b2  c2  a  m  a  n  a  m  n  a  a  a2  a2  b2  c2
b x ab
(Teorema de Pitágoras)

124 PROMILITARES.COM.BR

PM_BOOK05_MAT.indb 124 01/11/2021 13:45:40


GEOMETRIA PLANA: PRINCIPAIS TEOREMAS ANGULARES E MÉTRICOS


1

1

1

a2 b2  c2 1 1 1 1 1
 2 2  2 2  2  2 2
RELAÇÕES MÉTRICAS NOS
h2 m  n b2 c2 b2  c2

bc b c h b c POLÍGONOS REGULARES
a a
TRIÂNGULO EQUILÁTERO INSCRITO
O lado de um triângulo equilátero (regular) inscrito em uma
a·h=b·c b² = a · m c² = a · n R
circunferência de raio R é l3 = R 3 e o seu apótema é a3 = .
2

1 1 1 R
h² = m · n   a² = b² + c² l3 = R 3 a3 =
h2 b2 c2 2

As relações b² = a · m e c² = a·n mostram que cada cateto é a


média geométrica da hipotenusa e da sua projeção sobre a hipotenusa.
A relação h² = m · n mostra que a altura é a média geométrica das
projeções dos catetos sobre a hipotenusa.
Considerando a lei dos cossenos, conclui-se que o recíproco do
teorema de Pitágoras também é verdadeiro, assim um triângulo de
lados a, b e c, onde a é o maior lado, é retângulo se, e somente se,
a² = b² + c².
L DO PRO
Exercício Resolvido I A FE
R
09. (ITA 2014) Considere o triângulo ABC retângulo em A.

S
TE

Sejam AE e AD a altura e a mediana relativa à hipotenusa BC,

SO
respectivamente. Se a medida de BE é  2  1 cm e a medida de
MA

AD é 1 cm, então AC mede, em cm,

R
a) 4 2 – 5.
Demonstração:
b) 3 – 2. No triângulo retângulo AOM da figura temos:
c) 6−2 2 . l3
3 l3
d) 3( 2 – 1). sen60  2 

  l3  R 3
R 2 2R R
e) 3 4 2 −5 .
a3 1 a R
MA

cos60    3  a3 
SO

Resolução: C R 2 R 2
T

QUADRADO INSCRITO
S

R
E

IA F
O lado de um quadrado (quadrilátero regular convexo) inscrito em

L DO PRO
R 2
uma circunferência de raio R é l4 = R 2 e o seu apótema é a4 = .
2

R 2
l4 = R 2 a4 = .
2

Observemos inicialmente que, em um triângulo retângulo, a


mediana relativa à hipotenusa é igual à metade da hipotenusa.
Portanto, AD = BD = DC = 1
EC  BC  BE  2   2  1  3  2

Sabemos também que, em um triângulo retângulo, o quadrado


de um cateto é igual ao produto da hipotenusa pela sua projeção.
Assim, no triângulo retângulo ABC, temos:

AC  BC  EC  2   3  2   6  2 2  AC  6  2 2 u.c.
2

PROMILITARES.COM.BR 125

PM_BOOK05_MAT.indb 125 01/11/2021 13:45:48


GEOMETRIA PLANA: PRINCIPAIS TEOREMAS ANGULARES E MÉTRICOS

Demonstração: QUADRADO CIRCUNSCRITO


No triângulo retângulo AOM da figura temos:
l4
2 l4
sen 45  2    l4  R 2
R 2 2R
a 2 a4 R 2
cos 45  4    a4 
R 2 R 2
Poderíamos observar também que o triângulo da figura é um
l4
triângulo retângulo isósceles e, portanto, a4 = .
2

HEXÁGONO REGULAR CONVEXO INSCRITO


O lado de um hexágono regular convexo inscrito em uma
R 3
circunferência de raio R é I6 = R e o seu apótema é a6 = .
2 É fácil ver pela figura que L4 = 2R e A4 = R.

R 3 HEXÁGONO REGULAR CIRCUNSCRITO


I6 = R a6 =
2

L DO PRO
I A FE
R

S
TE

SO
MA

R
R
MA

Podemos ver que o triângulo OEF é equilátero de lado L6 e altura


SO

Demonstração:
O hexágono regular pode ser dividido em seis triângulos L6 3 2R 3 2 3R
R, assim teremos que R   L6  .  .
equiláteros. Dessa forma, o lado de cada triângulo equilátero é I6 = R e 2 3 3 3
T

R 3
o apótema é igual à altura desse triângulo a6 =
R . 2 3R
E

= =
E

F
2 L6 e A6 R

TRIÂNGULO EQUILÁTERO CIRCUNSCRITO IA O


3

L DO PR
QUADRO RESUMO
POLÍGONOS INSCRITOS
TRIÂNGULO HEXÁGONO
QUADRADO
EQUILÁTERO REGULAR

LADO R 3 R 2 R

R R 2 R 3
APÓTEMA
2 2 2

POLÍGONOS CIRCUNSCRITOS

Pela razão de 2 : 1 do baricentro teremos que TRIÂNGULO HEXÁGONO


QUADRADO
EQUILÁTERO REGULAR
AO  2OR  AO  2R  AH  3R , assim
L 3 L 3 6R 3 2 3R
AH   3R   L 3  6R  L  .  2 3R . LADO 2 3R 2R
2 2 3 3 3
L3  2 3R e A 3  R APÓTEMA R R R

126 PROMILITARES.COM.BR

PM_BOOK05_MAT.indb 126 01/11/2021 13:45:58


GEOMETRIA PLANA: PRINCIPAIS TEOREMAS ANGULARES E MÉTRICOS

EXERCÍCIOS DE  mede 70º, a medida do


05. (EEAR) Na figura, AB é diâmetro. Se AC

FIXAÇÃO ângulo CÂB é

01. (EEAR) Na figura, o lado BC do triângulo ABC mede 12 cm, e a


altura relativa ao lado BC mede 8 cm. Se FG = 3EF, então o perímetro
do retângulo DEFG, em cm, é

a) 50°. c) 60°.
b) 55°. d) 65°.

06. (EEAR) A razão entre as medidas dos apótemas do quadrado


inscrito e do quadrado circunscrito numa circunferência de raio R é
2 c) 2.
a) .
a) 20. 2
b) 28. 3
b) . d) 2 3 .
c) 85/3. 2
d) 64/3.
L D O 07.P(EEAR)
RO Num trapézio isósceles ABCD as bases AB e CD medem,
I A
02. (EEAR) Na figura, O é o centro da circunferência, med (MÔN) = 62° respectivamente,F16 cm e 4 cm. Traçando-se EF paralelo às bases,
e med  PRQ
ˆ  = 65°. O ângulo MÂN mede
R sendo E ∈ AD e F ∈E BC, obtém-se os segmentos AE e DE, de modo
AE 1

S
TE

que = . O comprimento de , em cm, é


DE 5

SO
a) 8. c) 12.
MA

b) 10. d) 14.

R
08. (EEAR) Sejam as relações métricas no triângulo ABC:
A
a) 34°.
b) 36°.
cos  R b
c h
MA

c) 38°.
SO

d) 40°.
y x
03. (EEAR) Em um triângulo equilátero de 12 3 m de perímetro, a B C
a
T

soma das medidas dos raios das circunferências inscrita e circunscrita


R
E

a esse triângulo, em m, é
E

IA
I- b2 =ax
F
L D OIII - hP xyR O
a) 5. 
II - a2  b2  c 2  2bc.cosA
b) 6.
c) 7.
1 1 1
d) 8. IV -  
h2 b2 c2
04. (EEAR) Se os dados no triângulo ABC, retângulo em C, estão em
Se o triângulo ABC é retângulo em A, então o número de relações
cm, então o triângulo BCD é
verdadeiras acima é
a) 1.
b) 2.
c) 3.
d) 4.

09. (EEAR) Dois quadrados são tais que um deles tem como lado a
diagonal do outro, que por sua vez tem o lado medindo 10 cm. O
módulo da diferença entre as medidas de suas diagonais, em cm, é
a) 10(2 – 2 ).
a) obtusângulo.
b) 10( 2 – 1).
b) retângulo.
c) 5(2 – 2 ).
c) isósceles.
d) equilátero. d) 5( 2 – 1).

PROMILITARES.COM.BR 127

PM_BOOK05_MAT.indb 127 01/11/2021 13:46:02


GEOMETRIA PLANA: PRINCIPAIS TEOREMAS ANGULARES E MÉTRICOS

10. (EEAR) Um hexágono regular ABCDEF, de 30 3 cm de perímetro, 07. (EEAR) Sabe-se que a hipotenusa de um triângulo retângulo tem
está inscrito em um círculo de raio R. A medida de sua diagonal AC, 5 5 cm de comprimento e a soma dos catetos é igual a 15 cm. As
em cm, é medidas, em cm, dos catetos são
a) 5 3 . c) 15 3 . a) 6 e 9
b) 5. d) 15. b) 2 e 13
c) 3 e 12
d) 5 e 10
EXERCÍCIOS DE

TREINAMENTO 08. (EEAR) Duas cordas se cruzam num ponto distinto do centro da
circunferência, conforme esboço.

01. (EEAR) Um triângulo isósceles tem perímetro igual a 36 cm e altura


relativa à base medindo 12 cm. A área desse triângulo, em cm2, é,
a) 60. c) 48.
b) 56. d) 40.

02. (EEAR) Dois triângulos são semelhantes, e uma altura do primeiro


é igual aos 2/5 de sua homóloga no segundo. Se o perímetro do
primeiro triângulo é 140 cm, então o perímetro do segundo, em cm, é
a) 250. c) 300.
L DO PRO
b) 280. d) 350.
I A FE
R
03. (EEAR) Na figura, AD é o diâmetro da circunferência, CÂD mede
A partir do conceito de ângulo excêntrico interior, a medida do arco

S
TE

a) 40° c) 110°
35° e BD̂C mede 25°. A medida de AĈB é

SO
b) 70° d) 120°
MA

09. (EEAR) O perímetro de um triângulo retângulo é 30 cm. Se a

R
soma das medidas dos catetos é 17 cm, e a soma das medidas da
a) 30°. hipotenusa e do cateto menor é 18 cm, então a medida, em cm, do
b) 35°. cateto maior é
c) 40°. a) 8. c) 12.
d) 45°. b) 9. d) 15.
R
MA

04. (EEAR) Se ABC é um triângulo retângulo em A o valor de n é 10. (EEAR) Seja um triângulo equilátero de apótema medindo 2 3
SO

cm. O lado desse triângulo mede ______ cm.


22 a) 6
a)
3 b) 8
T

16
R c) 9
E

b)
3
IA d) 12 F
L DO PRO
c) 22
d) 16
EXERCÍCIOS DE

COMBATE
05. (EEAR) A área de um hexágono regular inscrito em um círculo de
6 cm de raio é _____ 3 cm2.
a) 6 c) 12
b) 9 3 cm2. d) 15
01. (EEAR) Um triângulo isósceles, de perímetro 24 cm, possui altura
06. (EEAR) Seja um triângulo ABC, conforme a figura. Se D e E são relativa à base medindo 6 cm. Assim, a metade da medida de sua
base, em cm, é
pontos, respectivamente, de AB e AC, de forma que AD = 4, DB = 8,
DE = x, BC = y, e se DE  BC, então a) 7/2
b) 9/2
c) 11/2
d) 13/2

a) y = x + 8 02. (EEAR) Dado um hexágono regular de 6 cm de lado, considere o


seu apótema medindo α cm e o raio da circunferência a ele circunscrita
b) y = x + 4
medindo R cm. O valor de (R + a 3 ) é
c) y = 3x
a) 12 c) 18
d) y = 2x
b) 15 d) 25

128 PROMILITARES.COM.BR

PM_BOOK05_MAT.indb 128 01/11/2021 13:46:04


GEOMETRIA PLANA: PRINCIPAIS TEOREMAS ANGULARES E MÉTRICOS

03. (EEAR) Considere o quadrilátero ABCO, de vértices A, B e C na 07. (EEAR) O polígono regular cujo ângulo externo mede 24° tem
circunferência e vértice O no centro dela. Nessas condições x mede _____ lados.
a) 20
b) 15
c) 10
d) 5

08. (EEAR) Conforme a figura, os triângulos ABC e CDE são


retângulos. Se AB = 8 cm, BC = 15 cm e CD = 5 cm, então a medida
de DE , em cm, é

a) 30°
b) 45°
c) 55°
d) 60°

04. (EEAR) Seja BDEF um losango de lado medindo 24 cm, inscrito no D


triângulo ABC. Se BC = 60 cm, então AB = _____ cm.
a) 2/5

L D O b)c) P3/2
RO
I A 8/3
d) 1/4 FE
R 09. Calcule o valor de m na figura:

S
TE

a) 36

SO
b) 40
MA

c) 42

R
d) 48

05. (EEAR) Na figura, se BC = 60 cm, a medida de DE , em cm, é

R
MA

SO
T

R
E

IA F
Onde C é o centro do círculo de raio 10.

L D Ob) 2.P R O
a) 20 a) 1. c) 3. e) 5.
b) 24 d) 4.
c) 30
d) 32 10. (AFA) Considere, no triângulo ABC abaixo, os pontos P ∈ AB,
Q ∈ BC, R ∈ AC e os segmentos PQ e QR paralelos, respectivamente,
a AC e AB.
06. (EEAR) Os pontos A, B, C e D estão alinhados entre si, assim
como os pontos A, E e F também estão. Considerando G o ponto de
interseção de FC e ED , o valor de tg α é

Sabendo que BQ = 3 cm, QC = 1 cm e que a área do triângulo ABC é


8 cm², então a área do paralelogramo hachurado, em cm², é igual a
a) 0,2 a) 2
b) 0,5 b) 3
c) 2 c) 4
d) 4 d) 5

PROMILITARES.COM.BR 129

PM_BOOK05_MAT.indb 129 01/11/2021 13:46:05


GEOMETRIA PLANA: PRINCIPAIS TEOREMAS ANGULARES E MÉTRICOS

RESOLUÇÃO EM VÍDEO
Abra o ProApp, leia o QR Code, assista à resolução
de cada exercício e AVANCE NOS ESTUDOS!

GABARITO
EXERCÍCIOS DE FIXAÇÃO
01. D 04. B 07. D 10. D
02. A 05. B 08. C
03. B 06. A 09. A
EXERCÍCIOS DE TREINAMENTO
01. A 04. B 07. D 10. D
02. D 05. B 08. B
03. A 06. C 09. C
EXERCÍCIOS DE COMBATE
01. B 04. B 07. B 10. B
02. B
03. D
05. B
06. B
08. C
09. B L DO PRO
I A FE
ANOTAÇÕES
R

S
TE

SO
MA

R
R
MA

SO
T

R
E

IA F
L DO PRO

130 PROMILITARES.COM.BR

PM_BOOK05_MAT.indb 130 01/11/2021 13:46:05


GEOMETRIA PLANA: POTÊNCIA DE PONTO,
TRIGONOMETRIA NO TRIÂNGULO RETÂNGULO,
LEIS DOS SENOS E DOS COSSENOS E ÁREAS

POTÊNCIA DE PONTO EXTERIOR Exercício Resolvido


Se por um ponto P exterior a uma circunferência são traçadas duas
02. Calcule x na figura a seguir.
secantes PAB e PCD a essa circunferência, então PA ⋅ PB = PC ⋅ PD.

L DO PRO
I A FE
R Resolução:

S
TE

PA · PB = PC · PD
Sabe-se que PC2  PA  PB  x 2  2  5  x  10

SO
MA

Exercício Resolvido

R
01. Calcule x na figura a seguir:
POTÊNCIA DE PONTO INTERIOR
Se por um ponto P interior a uma circunferência são traçadas duas
cordas APB e CPD nessa circunferência, então PA ⋅ PB = PC ⋅ PD.

R
MA

SO
T

Resolução:
R
E

PA  PB  PC  PD  2  5  3   3  x   x 
10
IA
1 F
3
3
3
L R O
Sejam dois segmentos de reta PB e PD de origem comum e os DO P
pontos A ∈ PB e C ∈ PD tais que PA ⋅ PB = PC ⋅ PD, então os pontos
A, B, C e D são concíclicos. PA · PB = PC · PD

Exercício Resolvido
POTÊNCIA DO PONTO TANGENTE 03. Calcule x na figura.

PT² = PA · PB

PROMILITARES.COM.BR 131

PM_BOOK05_MAT.indb 131 01/11/2021 13:46:07


GEOMETRIA PLANA: POTÊNCIA DE PONTO, TRIGONOMETRIA NO TRIÂNGULO RETÂNGULO, LEIS DOS SENOS E DOS COSSENOS E ÁREAS

Se conhecermos a medida dos lados do ∆ABC e de um dos lados


Resolução: do ∆A’B’C’, então é possível calcular seus outros dois lados.
AP  BP  CP  DP   4 x  2  2x   x  1  4 x  Observe que todos esses dois triângulos possuem a mesma
 4 x 2  8x  0  4 x  x  2   0  x  2 “forma”, diferindo apenas pelo “tamanho”.
Para qualquer triângulo retângulo semelhante ao ∆ABC, é sempre
Note que como 2x e 4x são medidas de segmentos de reta, então
possível, conhecendo-se um dos lados, calcular os outros dois.
x ≠ 0.
A ideia da definição das linhas trigonométricas no triângulo
retângulo é identificar características do ∆ABC que permitam calcular
os lados dos triângulos retângulos semelhantes a ele sem precisar
de um “triângulo matriz”. Para isso vamos lançar mão de uma
POTÊNCIA DE PONTO característica que todos esses triângulos semelhantes têm em comum.
A potência de um ponto P em relação a um círculo de centro O e Eles possuem os mesmos ângulos.
raio R é dada por Pot(0)P = d² – R², onde d é a distância de P ao centro Assim, para todos os triângulos retângulos semelhantes ao ∆ABC,
do círculo.
a razão entre o cateto oposto ao ângulo B e a hipotenusa é a mesma.
P exterior ao circulo  d  R  Pot OP  0 A essa razão damos o nome de seno de B . Da mesma forma, a razão
P pertence ao circulo  d  R  Pot OP  0 entre o cateto adjacente ao ângulo B e a hipotenusa é constante. A essa
P interior ao circulo  d  R  PotOP  0 razão damos o nome de cosseno de B . A razão entre o cateto oposto e
o cateto adjacente ao ângulo B é chamada de tangente de B .
Se um ponto está sobre uma circunferência, então a sua potência Seja o triângulo ABC retângulo em A, conforme a figura a seguir:
em relação à essa circunferência é nula.

Exercício Resolvido L DO PRO


I A FE
R
04. Considerando o círculo da figura de centro O, calcule
Pot O A  Pot OB  PotOC .

S
TE

SO
MA

R
Definem-se, para o ângulo agudo B :
cateto oposto b
Seno : sen B =
=
hipotenusa R a
cateto ad jacente c
Cosseno : cos B = =
MA

hipotenusa a
SO

cateto oposto b sen B


Tangente : tg B = = =
cateto adjacente c cos B
Resolução:
T

c ateto adjacente c cos B 1


R Co tan gente : cotg B = = = =
E

Pot O A  OA 2  R2  32  52  9  25  16 b sen B tg B


E

IA F cateto oposto

L DO PRO
Pot OB  OB2  R2  52  52  0 1
Secante : sec B =
Pot OC  OC  R  7  5  49  25  24
2 2 2 2
cos B
Pot O A  Pot OB  Pot OC  16  0  24  8 1
Cos sec ante : cossec B =
sen B

 = c;
 : senC
Analogamente, são definidas para o ângulo agudo C
a
TRIGONOMETRIA NO TRIÂNGULO
 = b , sec C
 = c , cotgC
 = b ; tgC
cosC  = 1 e cos sec C
= 1 .
RETÂNGULO a b c 
cos C 
sen C

Comparando os resultados obtidos para os ângulos B e C ,

concluímos que sen B = cos C  ; tg B = cot gC


 ; cosB = senC  ; cotg B = t gC
;
 e cossec B = sec C
sec B = cos sec C .

 são ângulos complementares,


Como Bˆ  Cˆ  90, ou seja, B e C
então, para   0, 90, temos:

sen   cos  90   


Considere os dois triângulos retângulos ABC e A’B’C’, então cos   sen  90   
AB AC BC
podemos escrever: = = . tg   cotg  90   
A ’B’ A ’C’ B’C’

132 PROMILITARES.COM.BR

PM_BOOK05_MAT.indb 132 01/11/2021 13:46:21


GEOMETRIA PLANA: POTÊNCIA DE PONTO, TRIGONOMETRIA NO TRIÂNGULO RETÂNGULO, LEIS DOS SENOS E DOS COSSENOS E ÁREAS

Assim, a razão trigonométrica de um ângulo agudo é igual à 2


2 2 2 2 x
corrazão do seu complemento. AM  BM  AB  AM     x 2 
 2
Como a hipotenusa a é o maior lado do triângulo retângulo,
concluímos que: 2 x 2 3x 2 x 3
 AM  x 2    AM 
4 4 2
0 < sen B < 1
x 3
e A altura de um triângulo equilátero de lado x é h = .
2
0 < cos B < 1 No triângulo retângulo AMB, temos:
x 3
AM 2 3 1
Exercício Resolvido = =
sen 60 
= = sen 30 cos 60 =
AB x 2 2
05. Seja o triângulo retângulo de lados 3, 4 e 5. Calcule as x
BM 2 1 3
linhas trigonométricas dos dois ângulos agudos desse triângulo. =
cos 60
= = =
cos 30 sen
= 60
AB x 2 2
sen 60 32 1 1 3
Resolução: = 60 =
tg= =
3 tg 30 = =
cos 60 12 tg 60 3 3

Exercício Resolvido

06. Calcule x e y na figura.

L DO PRO
I A FE
R

S
TE

SO
MA

4 3 4 3
sen   ;cos   ;tg   ;cotg  
5 5 3 4

R
1 5 1 5
sec    ;cossec   
cos  3 sen  4
3 4 3 4
sen   ;cos   ;tg   ;cotg  
5 5 4 3
1 5 1 5
sec    ;cossec    R
cos  4 sen  3
MA

SO

Resolução:
ÂNGULOS NOTÁVEIS
T

BC y 3
sen 60    y3
R
E

AC 2 3 2
E

ÂNGULOS DE 30° E 60°


IA F
L DO PRO
AB x 1
Seja o triângulo equilátero ABC de lado x, conforme a figura a cos 60    x 3
AC 2 3 2
seguir:

ÂNGULO DE 45°
Seja o quadrado ABCD de lado x, conforme a figura a seguir:

Seja M o ponto médio do lado BC, então AMB ˆ  90 . Aplicando


o teorema de Pitágoras no triângulo retângulo AMB, temos:

PROMILITARES.COM.BR 133

PM_BOOK05_MAT.indb 133 01/11/2021 13:46:25


GEOMETRIA PLANA: POTÊNCIA DE PONTO, TRIGONOMETRIA NO TRIÂNGULO RETÂNGULO, LEIS DOS SENOS E DOS COSSENOS E ÁREAS

Aplicando o teorema de Pitágoras no triângulo retângulo ABC,


temos:
2 2 2 2
AC  AB  BC  AC  x 2  x 2  2x 2  AC  x 2 .
A diagonal de um quadrado de lado x é d = x 2.
No triângulo retângulo ABC, temos:

  BC  x  2
sen 45  sen BAC
AC x 2 2
  AB  x  2
cos 45  cos BAC
AC x 2 2
sen 45 22
tg 45   1
cos 45 22
Note que sen 45  cos  90  45   cos 45 .

Exercício Resolvido
a b c
= = = 2R
07. Calcule os catetos de um triângulo retângulo isósceles de  sen B sen C
sen A 
hipotenusa 4.

D O PDemonstração:
RO
Resolução:
A L
I FE
R

S
TE

SO
MA

R
R
MA

BC x 2
SO

sen 45    x2 2


AC 4 2
a a
A ’BC  sen A    2R
T

2R sen A
R
E

QUADRO RESUMO
E

IA F
Adotando procedimento análogo para os outros vértices, temos
30° 45° L DO PRO
60°
a

b

c
 sen B sen C
sen A 
 2R C.Q.D. .

1 2 3
SENO
2 2 2
Exercício Resolvido

3 2 1 08. (EEAR 2008) Num triângulo ABC, são dados


=  45
A =  
, B 30
COSSENO
2 2 2 e AC = 6 cm. Então BC = _____ cm.
a) 4 3
3
TANGENTE 1 3 b) 6 2
3
3
c)
2
2
LEI DOS SENOS d)
2
Seja um triângulo ABC de lados BC = a, AC = b e AB = c e raio do
círculo circunscrito R, então

134 PROMILITARES.COM.BR

PM_BOOK05_MAT.indb 134 01/11/2021 13:46:34


GEOMETRIA PLANA: POTÊNCIA DE PONTO, TRIGONOMETRIA NO TRIÂNGULO RETÂNGULO, LEIS DOS SENOS E DOS COSSENOS E ÁREAS

Resolução: B b  m2  h2  a2  b2  2bm  m


 
2
 h2  a2  b2  2bm  c2  a2

c2

m
No triângulo retângulo ABD, temos: cos A   m  c  cos A .
c
 b2  2bm  c2  a2  a2  b2  c2  2bc cos A C.Q.D.

Exercício Resolvido

09. (EEAR 2001) Dois lados consecutivos de um paralelogramo


medem 8 m e 12 m e formam entre si um ângulo de 60º.
As medidas das diagonais desse paralelogramo são tais que o
número que expressa
Lei dos senos: a) o seu produto é racional.
BC AC BC 6 2 6 b) a sua razão é maior que 2.
    BC    6 2 cm
 sen B
sen A sen 45 sen 30 2 12 c) a sua soma é maior que 32.
d) a sua diferença é irracional.

Resolução D
LEI DOS COSSENOS
Seja um triângulo ABC de lados BC = a, AC = b e AB =L
DO PRO
IA FE
c, então

S
TE

SO
MA

R
Lei dos cossenos no triângulo ABD:
d2  82  122  2  8  12 cos 60  112  d  112  4 7
Lei dos cossenos no triângulo ABC:
R
MA

D2  82  122  2  8  12 cos 120  304  D  304  4 19



SO

a2  b2  c2  2bc  cos A
D  d  4 19  4 7  
b2  a2  c2  2ac  cos B

c2  a2  b2  2ab  cos C
T

R
E

Demonstração:
IA F CLAIRAUT
SÍNTESE DE
L D OladosSeja O ABC, onde a, b e c representam as medidas dos
Pe a éRo maior lado, então
um triângulo

∆ABC é acutângulo ↔ a² < b² + c²


∆ABC é retângulo ↔ a² = b² + c²
∆ABC é obtusângulo ↔ a² > b² + c²

Demonstração:
Se a é o maior lado do triângulo, então o ângulo  é o maior
ângulo.
Pela lei dos cossenos, temos:
  b2  c2   a2
  2bc  cos A
a2  b2  c2  2bc  cos A .
 0A
a2  b2  c2  cos A  é obtuso e o ∆ABC é obtusângulo
 0A
a2  b2  c2  cos A   90 e o ∆ABC é retângulo
Aplicando o teorema de Pitágoras no triângulo retângulo ABD,
temos: m² + h² = c².  0A
a2  b2  c2  cos A  é agudo e o ∆ABC é acutângulo
Aplicando o teorema de Pitágoras no triângulo retângulo BCD,
temos: (b – m)² + h² = a².

PROMILITARES.COM.BR 135

PM_BOOK05_MAT.indb 135 01/11/2021 13:46:42


GEOMETRIA PLANA: POTÊNCIA DE PONTO, TRIGONOMETRIA NO TRIÂNGULO RETÂNGULO, LEIS DOS SENOS E DOS COSSENOS E ÁREAS

Exercício Resolvido

10. Classifique os triângulos a seguir quanto aos ângulos.


a) triângulo de lados 8, 15 e 18.
b) triângulo de lados 8, 15 e 17.
c) triângulo de lados 8, 15 e 16.

Resolução:
a) 18² = 324 > 289 = 15² + 8², então o triângulo é obtusângulo.
b) 17² = 289 = 15² + 8², então o triângulo é retângulo.
c) 16² = 256 < 289 = 15² + 8², então o triângulo é acutângulo. Demonstração:

DEFINIÇÃO DE ÁREA
Cada figura plana está associada a um número positivo chamado
área que possui as seguintes propriedades:
P1. Figuras planas congruentes possuem a mesma área, ou seja,
são equivalentes.

então a área de P é a soma das áreas de P1 e de P2.


L DO PRO
P2. Se uma figura plana P for decomposta em duas outras P1 e P2

I A ao produto
P3. A área de um retângulo de base b e altura h é igual FE
b ⋅ h.
R S ADE  SBCF  SABCD  SDEFC  b  h

S
TE

ÁREAS DAS PRINCIPAIS FIGURAS PLANAS


Exemplo: Calcule a área do paralelogramo da figura.

SO
MA

RETÂNGULO
Como estabelecido na propriedade P1, a área de um retângulo de

R
base b e altura h é igual ao produto b ⋅ h.

R
MA

SO
T

SABCD = 5 ⋅ 3 = 15 u.a.
R
E

IA F
L D O Nessa O
TRIÂNGULOS
S=B·h
P Rseção serão apresentadas diversas fórmulas para o cálculo
da área de um triângulo.
Exemplo: Calcule a área do retângulo da figura.

Sret = 4 ⋅ 2 = 8 u.a.
a  hA b  hB c  hC
SABC   
2 2 2
PARALELOGRAMO
A área de um paralelogramo de base b e altura h é igual ao A área de um triângulo é igual à metade do produto de um dos
produto b ⋅ h, ou seja, é igual à área do retângulo de mesma base lados pela altura relativa a ele.
e altura.

136 PROMILITARES.COM.BR

PM_BOOK05_MAT.indb 136 01/11/2021 13:46:44


GEOMETRIA PLANA: POTÊNCIA DE PONTO, TRIGONOMETRIA NO TRIÂNGULO RETÂNGULO, LEIS DOS SENOS E DOS COSSENOS E ÁREAS

Demonstração:

bc   a  c sen B  a  b senC



SABC  sen A
Sejam AD||BC e CD||AB, então o #ABCD é um paralelogramo e 2 2 2
∆ABC = ∆CDA (L.L.L.).
1 1 a  hA Demonstração:
Logo, SABC = SCDA, então SABC  2 SABCD  2  a  hA  2 .

Note que, quando o triângulo é obtusângulo, o pé da altura pode

modo.
L DO PRO
estar no prolongamento do lado, mas a fórmula funciona do mesmo

I A FE
R

S
TE

SO
MA

R
Seja BE a altura relativa ao lado AC do ∆ABC, então
AC  BE
SABC  .
2
c  hC   BE  BE  AB  sen A
.
SABC  No triângulo retângulo ABE, temos sen A
R
2 AB
MA

AC  BE AC   b  c sen A

Logo, SABC    AB sen A
SO
.
Exemplo: Calcule a área dos triângulos das figuras a seguir. 2 2 2

Exemplo: Calcule a área do triângulo da figura a seguir.


T

R
E

IA F
L DO PRO

S = 3 ⋅ 3 = 9 u.a. S = 2 ⋅ 3 = 6 u.a.

A área de um triângulo é igual à metade do produto de dois lados


adjacentes multiplicado pelo seno do ângulo entre eles. 3 4 3
S  sen 60  6   3 3 u.a.
2 2

A área de um triângulo é igual ao produto de seu semiperímetro


pelo raio do círculo inscrito nesse triângulo.

PROMILITARES.COM.BR 137

PM_BOOK05_MAT.indb 137 01/11/2021 13:46:48


GEOMETRIA PLANA: POTÊNCIA DE PONTO, TRIGONOMETRIA NO TRIÂNGULO RETÂNGULO, LEIS DOS SENOS E DOS COSSENOS E ÁREAS

Demonstração:
Na figura AD = p – a e AT = p.
ID AD r pa
ADI ~ ATIA     
IA T AT ra p
 p  r  p  a  ra  SABC  p  a  ra

A área de um triângulo é igual ao produto dos três lados dividido


pelo quádruplo do raio do círculo circunscrito a esse triângulo.

SABC = p · r

Demonstração:
c r b r ar  a  b  c 
SABC  SABI  SACI  SBCI      r  pr
2 2 2  2 
Exemplo: Calcule a área do triângulo da figura a seguir, onde

r
7 3  39
6
.
L DO PRO
I A FE
R SABC 
ab c

S
TE

4R

SO
MA

Demonstração:

R
R
MA

SO

3  4  13 7  13
p 
2 2
T

7  13 7 3  39 49 3  7 39  7 39  13 3
S  pr   
R
 3 3 u.a.
E

2 6 12
IA F
A área de um triângulo é igual ao produto da diferença entre o
semiperímetro e um dos seus lados pelo raio do círculo ex-inscrito L DO PRO
relativo a esse lado.

  90.
AOF é um diâmetro do círculo circunscrito, então ACF

ABC   AC  BAD
  AFC   CAF

2
AB AD c hA bc
BDA ~ FCA      hA 
AF AC 2R b 2R

a  hA a bc abc
SABC    
2 2 2R 4R

Exemplo: Calcule a área do triângulo da figura a seguir, sabendo


39
que R = .
3
SABC  p  a  ra  p  b   rb  p  c   rc

138 PROMILITARES.COM.BR

PM_BOOK05_MAT.indb 138 01/11/2021 13:46:55


GEOMETRIA PLANA: POTÊNCIA DE PONTO, TRIGONOMETRIA NO TRIÂNGULO RETÂNGULO, LEIS DOS SENOS E DOS COSSENOS E ÁREAS

SABC  p  p  a  p  b   p  c 

abc
onde p  é o semiperímetro do triângulo.
2

Exemplo: Calcule a área do triângulo da figura.

abc 3  4  13
S   3 3 u.a.
4R 39
4
3

A área de um triângulo é igual ao dobro do quadrado do raio


do círculo circunscrito multiplicado pelo produto dos senos de seus
ângulos. L DO PRO
I A FE
R 3  4  13 7  13

S
TE

p 
2 2

SO
7  13  7  13   7  13   7  13 
MA

S   3    4   3 
 13
2  2   2   2 

R
7  13  13  1  13  1  7  13 
   
2  2   2   2 
1  1
 49  13  13  1  36  12  3 3
4 4
R
MA

QUADRADO
SO

A área do quadrado é igual ao seu lado elevado ao quadrado.


  sen B  senC
SABC  2R2  sen A 
T

Demonstração:
R
E

Aplicando a lei dos senos ao ∆ABC, temos:


IA F
a
= =
b c
 sen B sen C
sen A 
= 2R L DO PRO
  2R sen B  2R senC
a  b  c 2R sen A 
 SABC     sen B  sen C
 2R2  sen A 
4R 4R

Fórmula de Heron: A área de um triângulo é igual à raiz quadrada


do produto do semiperímetro pela diferença entre o semiperímetro e
cada um dos lados do triângulo.

Seja  o lado do quadrado ABCD, então sua área é SABCD = I².

Demonstração:
Basta utilizar a expressão da área do retângulo onde a base e a
altura são ambas iguais a . Assim, SABCD =  ⋅  = ².

PROMILITARES.COM.BR 139

PM_BOOK05_MAT.indb 139 01/11/2021 13:47:00


GEOMETRIA PLANA: POTÊNCIA DE PONTO, TRIGONOMETRIA NO TRIÂNGULO RETÂNGULO, LEIS DOS SENOS E DOS COSSENOS E ÁREAS

LOSANGO POLÍGONO REGULAR


A área do losango é igual à metade do produto de suas diagonais. A área de um polígono regular é igual ao produto do semiperímetro
pelo apótema.

Seja um polígono regular de semiperímetro p e apótema a, então


sua área é S = p ⋅ a.
Seja o losango ABCD de diagonais AC = p e BD = q, então sua
pq Demonstração:
área é SABCD 
2
D O ElePpode
Seja um polígono regular de gênero n, lado x e apótema a.
RserOdividido em n triângulos isósceles de vértice no centro
A L
Demonstração:
Inicialmente, cabe observar que o losango Ipossui diagonais
F E ao polígono, cuja base é o lado e a altura é
do círculo circunscrito
o apótema do polígono. A área de cada um desses triângulos
perpendiculares. Assim, R x a
isósceles é S  . Portanto, a área do polígono é dada por
2

S
TE

AC  BM AC  MD AC  BM  MD AC  BD p  q
SABCD  SABC  SACD     

SO
2 2 2 2 2 x  a nx nx
S  n  S  n    a  p  a , onde utilizamos p  .
MA

2 2 2

R
TRAPÉZIO
A área do trapézio é igual ao produto da semissoma de suas bases REGIÕES CIRCULARES
pela sua altura.
CÍRCULO
A área do círculo é o produto do quadrado do seu raio pelo
número irracional π. R
MA

SO
T

R
E

IA F
L DO PRO
Seja o trapézio ABCD de bases AB = B e CD = b, e altura h, então
Bb
sua área é S  h
2

Demonstração:
Seja um círculo de raio R, então sua área é S = π ⋅ R²

Demonstração:
Observe que a área do círculo pode ser calculada considerando-o
um polígono regular cujo número de lado tende ao infinito. Assim, sua
2R
área é p produto do seu semiperímetro  R pelo seu apótema
2
R, ou seja, S  R  R    R .
2

SETOR CIRCULAR
Um setor circular é a região da circunferência delimitada por
dois raios e um arco e é caracterizado pelo ângulo central por ele
determinado.
AB  h CD  h  AB  CD B  b 
SABCD  SABC  SACD    h  h A área do setor circular é igual à metade do produto do quadrado
2 2 2 2 do raio pelo ângulo central em radianos.

140 PROMILITARES.COM.BR

PM_BOOK05_MAT.indb 140 01/11/2021 13:47:06


GEOMETRIA PLANA: POTÊNCIA DE PONTO, TRIGONOMETRIA NO TRIÂNGULO RETÂNGULO, LEIS DOS SENOS E DOS COSSENOS E ÁREAS

Resolução: B

Seja um setor circular de ângulo central α em radianos e de raio R,


  R2
então sua área é S  .
2
Observe a figura representando a situação. No caso o triângulo
Observe que se o ângulo central estiver expresso em graus, a obtusângulo é isósceles de ângulos 120º, 30º e 30º.
R2   Aplicando a razão trigonométrica do seno no triângulo retângulo
expressão resultante é S  .
360 de cateto h e hipotenusa 40, temos:
Demonstração:
h  3

DO PRO
A área do setor circular é proporcional ao ângulo central. Assim,  sen60º  h  40.    h  20 3 m
40  2 
um setor circular de α radianos representa

A L
FE
da área total do
2
   R2 I
R
círculo, ou seja,  R 
2
.
2 2
Exercício Resolvido

S
TE

SEGMENTO CIRCULAR

SO
12. O triângulo ABC é retângulo em A B C e os segmentos BD e
Um segmento circular é uma região da circunferência delimitada
MA

AC são perpendiculares.
por uma corda e um arco e também é caracterizado pelo ângulo
central associado à corda.

R
R
MA

SO
T

R
E

IA F
Seja um segmento circular de α em radianos e de raioLR, então a) 10 R
O Assim, a medida do segmento DC vale:

sua área é
P
D O b) 6 3 3

R2  15
Ssegmento   Ssetor   Stringulo     sen   c) .
2 2
13
d) .
Exercício Resolvido 2

11. Um estudante de engenharia vê um prédio do campus da Resolução: C


UFSM construído em um terreno plano, sob um ângulo de 30º.
  90, ADB
Tem-se que ABC   90 e DÂB = 60º implicam
Aproximando-se do prédio mais 40 m, passa a vê-lo sob um ângulo
de 60º. Considerando que a base do prédio está no mesmo nível 
em DBC  60. Assim, do triângulo retângulo BCD, vem
dos olhos do estudante, então a altura h, em metros, do prédio é
igual a:   CD  CD  3  5 3  CD  15 .
sen DBC
BC 2 2
a) 30 3
b) 20 3
c) 10
d) 10 3
e) 28

PROMILITARES.COM.BR 141

PM_BOOK05_MAT.indb 141 01/11/2021 13:47:14


GEOMETRIA PLANA: POTÊNCIA DE PONTO, TRIGONOMETRIA NO TRIÂNGULO RETÂNGULO, LEIS DOS SENOS E DOS COSSENOS E ÁREAS

Exercício Resolvido a) 2,29. d) 3,50.


13. Uma rampa retangular, medindo 10 m² faz um ângulo de 25º b) 2,33. e) 4,80.
em relação ao piso horizontal. Exatamente embaixo dessa rampa, c) 3,16.
foi delimitada uma área retangular A para um jardim, conforme
figura. Resolução: D
Pela Lei dos Cossenos, obtemos:
2 2 2
  (0, 8)2  12  2  0, 8  1 cos 150 
BC  AC  AB  2  AC  AB  cos BAC
 3
 0, 64  1  2  0, 8      BC  1, 64  0, 8  1, 7  3.
 2 

Logo,BC ≅ 1,7 e, portanto, o resultado é 1 + 0,8 + 1,7 = 3,5.

EXERCÍCIOS DE

Considerando que cos 25º ≅ 0,9, a área A tem aproximadamente:


a) 3 m²
FIXAÇÃO
b) 4 m²
01. (EEAR) Na figura, são retângulos em E e em C, respectivamente,
c) 6 m²
d) 8 m² L D O os Ptriângulos
R OAEP e ACB. Se x = 30°, então a medida de PE, em cm, é
e) 9 m²
IA a) 10. FE
Resolução: E
R b) 5 3 .

S
TE

c) 10 3 .
Tem-se que x ⋅ y = 10 m². Logo, como z = y ⋅ cos25º e A = x ⋅ z,

SO
segue-se que A = x ⋅ y ⋅ cos25º ≅ 10 ⋅ 0,9 = 9 m². 20 3
d) .
MA

R
Exercício Resolvido

14. Seja um triângulo inscrito em uma circunferência de raio R. 02. (EEAR) O círculo da figura tem centro O e raio r. Sabendo-se que
Se esse triângulo tem um ângulo medindo 30º, seu lado oposto a 5r
PQ equivale a e é tangente ao círculo no ponto P, o valor de senα é
esse ângulo mede: 12
a) 5/12. R
R c) 2R
MA

a) b) 5/13.
2
SO

2R c) 12/13.
b) R d)
3
d) 0,48.
Resolução: B
T

R
Seja  a medida do lado do triângulo que é oposto ao ângulo de
E


IA
 2R    R. F
O
30º. Pela Lei dos Senos, tem-se que
sen 30
L D O03. (EEAR)RAs diagonais de um paralelogramo medem 10 m e 20 m
P entre si um ângulo de 60°. A área desse paralelogramo,
e formam
Exercício Resolvido em m2, é
a) 200. c) 50 3 .
15. A caminhada é uma das atividades físicas que, quando
b) 100.
realizada com frequência, torna-se eficaz na prevenção de doenças d) 25 3 .
crônicas e na melhora da qualidade de vida.
Para a prática de uma caminhada, uma pessoa sai do ponto A, 04. (EEAR) Em um triângulo retângulo, o quadrado da medida da
passa pelos pontos B e C e retorna ao ponto A, conforme trajeto hipotenusa é igual ao dobro do produto das medidas dos catetos. Um
indicado na figura. dos ângulos agudos desse triângulo mede
a) 15°. c) 45°.
b) 30°. d) 60°.

4
05. (EEAR) Um círculo é tal que a medida de seu raio é igual aos
7
medida do comprimento de um setor circular que ele contém. Se a
63
área desse setor é igual a π cm2, então a área do círculo, em cm², é
8
Quantos quilômetros ela terá caminhado, se percorrer todo o a) 9π. c) 6π.
trajeto? b) 9π2. d) 6π2.

142 PROMILITARES.COM.BR

PM_BOOK05_MAT.indb 142 01/11/2021 13:47:19


GEOMETRIA PLANA: POTÊNCIA DE PONTO, TRIGONOMETRIA NO TRIÂNGULO RETÂNGULO, LEIS DOS SENOS E DOS COSSENOS E ÁREAS

06. (EEAR) Na figura, os pontos M, N e P dividem o lado AB do 03. (EEAR) Dois círculos concêntricos têm 4 m e 6 m de raio. A área da
paralelogramo ABCD em 4 partes iguais, e os pontos E e F dividem a coroa circular por eles determinada, em m2, é
diagonal AC em 3 partes iguais. A área do triângulo APE é uma fração a) 2π. c) 20π.
da área do paralelogramo ABCD, equivalente a b) 10π. d) 52π.

a) 1/12. 04. (EEAR) S6 e S3 são, respectivamente, as áreas do hexágono regular


b) 1/16. e do triângulo equilátero, ambos inscritos na mesma circunferência.
Nessas condições, a relação verdadeira é
c) 1/20.
a) S6 = S3. c) S6 = 2S3.
d) 1/24.
b) S6 = 3S3. d) S3 = 2S6.

05. (EEAR) Os lados de um triângulo medem 7 cm, 8 cm e 9 cm. A


07. (EEAR) O trapézio ABCD é isósceles, e as medidas dos ângulos área desse triângulo, em cm2, é
D B A e DCB
 são 30° e 45°, respectivamente. Se BC = 12 cm, então
a) 12 3 . c) 8 2.
a medida de BD, em cm, é
b) 12 5 . d) 8 3 .
a) 6 2 . c) 10 2 .
b) 8 2 d) 12 2 . 06. (EEAR) Na figura, t é tangente à circunferência em B. Se AC = 8
cm e CD = 12 cm, então a medida de AB , em cm, é
08. (EEAR) Por um ponto P, distante 18 cm do centro de uma

DO PRO
circunferência de raio 12 cm, conduz-se um “segmento secante” que

L
determina na circunferência uma corda de 8 cm. A medida da parte

A
exterior desse segmento, em cm, é
a) 18. c) 8. I FE
b) 10. d) 6. R

S
TE

SO
09. (EEAR) Num triângulo ABC, BC = 10 cm e med(ABC) = 60°. Se
esse triângulo está inscrito numa semicircunferência e BC é seu menor
MA

a) 4 10 .
lado, então o raio dessa semicircunferência mede, em cm,
b) 2 5 .

R
a) 5. c) 10 2 .
b) 10. c) 10 .
d) 10 3 .
d) 5.
10. (EEAR) Num triângulo ABC, a razão entre as medidas dos lados AB
e AC é 2. Se  = 120° e AC = 1 cm, então o lado BC mede, em cm, 07. (EEAR) Um trapézio isósceles tem bases medindo 12 cm e 20 cm.
Se a medida de um de seus lados oblíquos é 5 cm, então sua área,
R
a) 7.
em cm2, é
MA

SO

b) 7 + 1. a) 25. c) 48.
c) 13 . b) 39. d) 54.
d) 13 – 1.
T

08. (EEAR) O triângulo cujos lados medem 6 cm, 7 cm e 10 cm é


R
E

F IA
classificado como
EXERCÍCIOS DE
L D Ob) escaleno O a) equilátero e retângulo.

TREINAMENTO P R e acutângulo.
c) isósceles e acutângulo.
d) escaleno e obtusângulo.
01. (EEAR) Um quadrado e um losango têm o mesmo perímetro. Se
as diagonais do losango estão entre si como 3 para 5, então a razão = 45°, =
09. (EEAR) Num triângulo ABC, são dados, Â B̂ 30° e
entre a área do quadrado e a do losango é AC = 6 cm. Então BC = _____ cm.
a) 17/15. c) 17/13. a) 4 3
b) 13/15. d) 11/13.
b) 6 2
02. (EEAR) Um triângulo, inscrito numa circunferência de 10 cm de c) 3/2
raio, determina nesta três arcos, cujas medidas são 90°, 120° e 150°.
d) 2 /2
A soma das medidas dos menores lados desse triângulo, em cm, é
a) 10 ( 2 + 3) . 10. (EEAR) No triângulo, cujos lados medem 5 cm, 10 cm e 6 cm, o
maior ângulo tem cosseno igual a
b) 10 (1 + 3 ) . 7 13
a) . c) − .
c) 5 ( 2 + 3 ). 10 20
9 8
5 (1 + 3 ) . −
d) b) . d) .
20 10

PROMILITARES.COM.BR 143

PM_BOOK05_MAT.indb 143 01/11/2021 13:47:22


GEOMETRIA PLANA: POTÊNCIA DE PONTO, TRIGONOMETRIA NO TRIÂNGULO RETÂNGULO, LEIS DOS SENOS E DOS COSSENOS E ÁREAS

EXERCÍCIOS DE
07. (EEAR) A figura mostra um quadro que

COMBATE possui quatro círculos de raio R e um de raio


r, ambos medidos em cm. Considerando
que os círculos não são secantes entre si,
R
01. (EEAR) Em um triângulo ABC, retângulo em A, a hipotenusa mede que r = e 4R + 2r = 30 cm, a área que os
2
1 círculos ocupam é _____ π cm2.
5 dm e senBˆ = .senCˆ . Nessas condições, o maior cateto mede, em
dm, 2
a) 120
a) 3. b) 4. c) 5. d) 2 5 . b) 138
c) 150
02. (EEAR) Seja a circunferência e duas de suas cordas, AB e CD. A d) 153
medida de CD, em cm, é
C 08. (EEAR) A área de um hexágono regular inscrito em um círculo de
a) 10.
b) 12. 6 cm de raio é _____ 3 cm2.
a
c) 14. A 16 cm 2 cm B a) 6
d) 16. b) 9
2a c) 12
d) 15

L D O 09.P(EEAR)
D
RO Na figura, os arcos que limitam a região sombreada são

IA ABCD. Se o ladoFdo quadrado mede 2R e considerando π = 3, então


arcos de circunferências
03. (EEAR) O segmento AT é tangente, em T, à circunferência de de raio R e centrados nos vértices do quadrado

a razão entre a áreaEsombreada e a área branca é


centro O e raio R = 8 cm. A potência de A em relação à circunferência
é igual a ______ cm2. R

S
TE

a) 16 a) 1/2

SO
b) 64 b) 1/3
MA

c) 192 c) 2

R
d) 256 d) 3

04. (EEAR) Se A, B, C e D são pontos da circunferência, o valor de x


é múltiplo de 10. (EEAR) Pelo triângulo ABC, o valor de x² + 6x é
R
MA

A
a) 76
SO

a) 5
b) 88 6 5
b) 6
c) 102 6
c) 7
T

d) 144 120º
d) 8
R
E

IA F B x C

L DO PRO
RESOLUÇÃO EM VÍDEO
05. (EEAR) Da figura, sabe-se que OB = r é raio do semicírculo de Abra o ProApp, leia o QR Code, assista à resolução
centro O e de diâmetro AC. Se AB = BC, a área hachurada da figura, de cada exercício e AVANCE NOS ESTUDOS!
em unidades quadradas, é

GABARITO
r 2.π
a) −1
2 EXERCÍCIOS DE FIXAÇÃO
π  01. A 04. C 07. D 10. A
b) r 2  −1
2  02. B 05. B 08. B

c) r 2 ( π −2) 03. C 06. D 09. B

1 EXERCÍCIOS DE TREINAMENTO
d) r2.π − 01. A 04. C 07. C 10. C
2
02. A 05. B 08. D
06. (EEAR) O piso de uma sala foi revestido completamente com 300 03. C 06. A 09. B
placas quadradas justapostas, de 20 cm de lado. Considerando que
todas as placas utilizadas não foram cortadas e que não há espaço EXERCÍCIOS DE COMBATE
entre elas, a área da sala, em metros quadrados, é 01. D 04. B 07. D 10. D
a) 120 c) 12 02. B 05. B 08. B
b) 80 d) 8 03. C 06. C 09. D

144 PROMILITARES.COM.BR

PM_BOOK05_MAT.indb 144 01/11/2021 13:47:23


TRIGONOMETRIA: CÍRCULO TRIGONOMÉTRICO
E FÓRMULAS DE ADIÇÃO, ARCO DOBRO E
METADE

SISTEMA CIRCULAR OU Exemplo:


π
RADIOMÉTRICO − RADIANOS (rad) Calcule o comprimento de um arco de circunferência de
3
rad
em uma circunferência de raio 3 cm.
O ângulo de 1 radiano (1 rad) é o ângulo central em uma
circunferência de raio R que determina um arco de comprimento R Resolução:
sobre essa circunferência. π
O comprimento do arco é  =α ⋅ R = ⋅ 3 =π cm.
3

RELAÇÕES ENTRE AS UNIDADES

L DO PRO 180 = π rad

IA Exemplo:
FE
R Converta o ângulo de 15° para radianos e o ângulo de
π
rad

S
TE

para graus. 8

SO
Resolução:
MA

π rad π
15°= 15° ⋅ = rad
180° 12

R
π π 180°
O sistema circular ou radiométrico adota como unidade de rad= rad ⋅ = 22,5°= 22°30'
8 8 π rad
medida 1 radiano (1 rad).
Como o comprimento de uma circunferência de raio R é 2πR,
então um ângulo de uma volta mede 2π rad. O CICLO TRIGONOMÉTRICO R
1 O ciclo trigonométrico é uma circunferência orientada de raio
MA

=
1rad ⋅ ( ângulo de uma volta)
SO

2π unitário e centrada na origem. A origem do sistema de medidas é no


 ponto de coordenadas A(1,0) e o sentido positivo é o sentido anti-
horário.
1ângulo de uma volta= 2π rad
T

1ângulo raso = π rad


R
E

1ângulo reto =
π
rad
IA F
L DO PRO
2

COMPRIMENTO DO ARCO DE
CIRCUNFERÊNCIA

A medida dos ângulos é feita por meio de uma função dos


números reais sobre a circunferência, que associa a cada número real
θ um único ponto P sobre a circunferência. O ponto P associado ao
número real θ é o ponto final de um percurso de comprimento θ sobre
2parco = α ⋅ R , onde α em radianos a circunferência, a partir de A, no sentido anti-horário para θ > 0 ou
θ πRθ no sentido horário para θ < 0. O Ponto P associado ao número real θ
2parco = 2πR ⋅ = , onde θ em graus
360 180 é chamado imagem de θ no ciclo trigonométrico.

PROMILITARES.COM.BR 145

PM_BOOK05_MAT.indb 145 01/11/2021 13:47:24


TRIGONOMETRIA: CÍRCULO TRIGONOMÉTRICO E FÓRMULAS DE ADIÇÃO, ARCO DOBRO E METADE

Isso é como se você enrolasse a reta real na circunferência com ARCOS CÔNGRUOS
o zero sobre o ponto A e o sentido positivo no sentido anti-horário.
Como o ciclo trigonométrico tem comprimento 2π, a função que
Observe que o ciclo trigonométrico tem comprimento 2π · 1 = 2π. define o ponto P do ciclo trigonométrico associado a um número
Assim, os arcos pertencentes ao intervalo [0,2π[ estão na primeira real θ é periódica de período 2π, ou seja, números que diferem por
volta; os arcos pertencentes ao intervalo [2π,4π[ estão na segunda múltiplos de 2π possuem a mesma imagem no ciclo trigonométrico.
volta; os arcos pertencentes ao intervalo [–2π,0[ estão na volta –1; e
Dois arcos são ditos côngruos quando possuem a mesma
assim por diante.
extremidade no ciclo trigonométrico.
π
O ciclo trigonométrico é divido em 4 quadrantes de radianos Assim, dois arcos α e β, expressos em radianos, são côngruos
2 (α ≡ β) se, e somente se, α – β = 2π · k para algum k ∈ .
enumerados no sentido anti-horário. Assim, arcos cujas extremidades
estão sobre o arco AB  estão no primeiro quadrante (Q ); sobre o Da mesma forma, dois arcos α e β, expressos em graus, são
I
  , no terceiro
arco BA', no segundo quadrante (QII); sobre o arco A'B' côngruos (α ≡ β) se, e somente se, α – β = 360° · k para algum k ∈ .

quadrante (QIII) e sobre o arco B'A , no quarto quadrante (QIV). α ≡ β ⇔ α − β= 2k ⋅ π, k ∈  (radianos )
Para arcos no intervalo [0,2π[ (primeira volta), os arcos do primeiro
 π β 360° ⋅ π, k ∈  ( graus )
α ≡ β ⇔ α −=
quadrante pertencem ao intervalo  0, , os arcos do segundo
 2
Exemplo:
π
quadrante pertencem ao intervalo  , π  , os arcos do terceiro Marque no ciclo trigonométrico a imagem de cada um dos
 2 
números a seguir e identifique os arcos côngruos.
 3π 
quadrante pertencem ao intervalo  π,  e os arcos do quarto 7π 11π
 2 a) ; d) − ;
 3π 
quadrante pertencem ao intervalo  ,2π .
L DO PRO 3
19π
4

A b) ; e) − ;
 2 
I
A figura a seguir traz a indicação dos quadrantes no ciclo
4 FE 3

trigonométrico. R c) −

4
; f)
14 π
3

S
TE

Resolução:

SO
7π π 6π π
a) = + = + 2π
MA

3 3 3 3
19π 3π + 16π 3π

R
b) = = + 2π ⋅ 2
4 4 4
5π 3π − 8π 3π
c) − = = + 2π ⋅ ( −1)
4 4 4
11π 5π − 16π 5π
d) − = = + 2π ⋅ ( −2)
4 4 R4
5π π − 6π π
MA

e) − = = + 2π ⋅ ( −1)
SO

3 3 3
14 π 2π + 12π 2π
f) = = + 2π ⋅ 2
3 3 3
T

R
E

IA F
Exemplo: L DO PRO
Identifique a que quadrante pertence cada um dos arcos a seguir:
3π d) 265°;
a) ;
4

b) 7π ; e) .
15
4

c) 9π ;
7
Resolução:
3π π 3π
a) ∈ QΙΙ , pois < < π.
4 2 4
b) 7π 3π 7π
∈ QΙV , pois < < 2π.
4 2 4 Os arcos das opções a) e e) são côngruos, assim como os das
opções b) e c).
c) 9π 5π 3π
∈ QΙΙΙ , pois π < < . A primeira determinação positiva de um arco β é o arco
7 4 2
α ∈ [0,2π
π[ côngruo a β.
d) 265° ∈ QΙΙΙ , pois 180° < 265° < 270°.
Para se identificar a imagem de um arco no ciclo trigonométrico
2π 2π π e os valores de suas linhas trigonométricas é sempre útil encontrar a
e) ∈ QΙ , pois 0 < < .
15 15 2 primeira determinação positiva.

146 PROMILITARES.COM.BR

PM_BOOK05_MAT.indb 146 01/11/2021 13:47:25


TRIGONOMETRIA: CÍRCULO TRIGONOMÉTRICO E FÓRMULAS DE ADIÇÃO, ARCO DOBRO E METADE

Exemplo: positiva, ou seja, quando essa imagem está à direita do eixo OY, e
Encontre a primeira determinação positiva dos seguintes arcos. valores negativos, caso contrário. Observe que a definição acima é
coerente com a que foi estabelecida nos triângulos retângulos para
35π 40π e) –1200°. ângulos agudos. Basta observar que o triângulo retângulo OPXP possui
a) ; c) − ;
4 3 hipotenusa 1 (raio do ciclo trigonométrico) e catetos OPx e OPy .
25π PPx OPy OPx OPx
b) ; d) 2880°;
6 Assim, temos: sen= θ = = OPy e cos= θ = = OPx .
OP 1 OP 1
Resolução: Denomina-se função seno, a função de  em  definida por
35π f(x) = sen x.
a) A primeira determinação positiva de é 3π , pois
35π 32π + 3π 3π 4 4 O domínio da função seno é Dsen =  e a imagem Imsen = [–1,1].
= = + 2π ⋅ 4 . Denomina-se função cosseno, a função de  em  definida por
4 4 4
25π π f(x) = cos x.
b) A primeira determinação positiva de é , pois
25π π + 24 π π 6 6 O domínio da função cosseno é Dcos =  e a imagem Imcos = [–1,1].
= = + 2π ⋅ 2.
6 6 6
40π 2π sen θ ∈ [ −1,1] , ∀θ ∈ 
c) A primeira determinação positiva de − é , pois
40π −42π + 2π 2π 3 3 cos θ ∈ [ −1,1] , ∀θ ∈ 
− = = ( )
+ 2π ⋅ −7 .
3 3 3
d) A primeira determinação positiva de 2730° é 210°, pois Seno e cosseno são funções periódicas de período 2π.
2730° = 360° · 7 + 210°. Os sinais das funções seno e cosseno em cada um dos quadrantes
e) A primeira determinação positiva de –1200° é 240°, pois estão representados nos diagramas a seguir
–1200° = 360° · (–4) + 240°.
L DO PRO
SENO E COSSENO I A FE
R
Seja P a imagem de um ângulo θ no ciclo trigonométrico. Define-

S
TE

se o seno do ângulo θ como a ordenada de P e o cosseno de θ como


a abscissa de P.

SO
Assim, para obter o seno de θ, devemos projetar P sobre o eixo
MA

vertical Oy, denominado eixo dos senos, e, para obter o cosseno de

R
θ, devemos projetar P sobre o eixo horizontal Ox, denominado eixo
dos cossenos. Na figura a seguir, temos

R
MA

SO
T

R
E

IA F
L DO PRO

TANGENTE E COTANGENTE
sen θ =OPy No ciclo trigonométrico, o eixo paralelo ao eixo Oy, com a mesma
orientação que este e passando pelo ponto A é denominado eixo das
tangentes, e o eixo paralelo ao eixo Ox, com a mesma orientação que
cos θ =OPx
este e passando pelo ponto B é denominado eixo das cotangentes.
Observe que, OPy e OPx são segmentos orientados. π
Seja θ um ângulo tal que θ ≠ + k ⋅ π , k ∈ , e cuja imagem
2
Dessa forma, o seno de um ângulo assume valores positivos
no ciclo trigonométrico é P. A tangente de θ é a medida algébrica
quando sua imagem no ciclo trigonométrico possui ordenada positiva,
do segmento AP1, onde P1 é a interseção da reta OP com o eixo das
ou seja, quando essa imagem está acima do eixo Ox, e valores
tangentes.
negativos, caso contrário.
π
Do mesmo modo, o cosseno de um ângulo assume valores Note que, se θ= + k ⋅ π , k ∈ , a reta OP não intersecta o eixo
2
positivos quando sua imagem no ciclo trigonométrico possui abscissa das tangentes e, portanto, a tangente de θ não está definida.

PROMILITARES.COM.BR 147

PM_BOOK05_MAT.indb 147 01/11/2021 13:47:27


TRIGONOMETRIA: CÍRCULO TRIGONOMÉTRICO E FÓRMULAS DE ADIÇÃO, ARCO DOBRO E METADE

Seja θ um ângulo tal que θ ≠ k · π, k ∈ , e cuja imagem no Tangente e cotangente são funções periódicas de período π.
ciclo trigonométrico é P. A cotangente de θ é a medida algébrica De acordo com a definição acima, os ângulos do 1º e 3º
do segmento BP2, onde P2 é a interseção da reta OP com o eixo das quadrantes possuem tangente e cotangente positivas e os ângulos
cotangentes. do 2º e 4º quadrantes possuem tangente e cotangente negativas,
Note que, se θ = k · π, k ∈ , a reta OP não intersecta o eixo conforme representado no diagrama a seguir.
das cotangentes e, portanto, a cotangente de θ não está definida. Na
figura a seguir, temos

L DO PRO
I A F
SECANTE EE COSSECANTE
R π
Seja θ um ângulo tal que θ ≠ + k ⋅ π , k ∈ , e cuja imagem

S
TE

SO
no ciclo trigonométrico é P. A secante de θ é a medida algébrica
do segmento OP’, onde P’ é a interseção da reta tangente ao ciclo
MA

tg θ =AP1
trigonométrico em P com o eixo dos cossenos.

R
cotg θ =BP2 π
Note que, se θ= + k ⋅ π , k ∈ , a reta tangente não intersecta o
2
eixo cos cossenos e, portanto, a secante de θ não está definida.
Observe que a definição acima é coerente com a que foi
estabelecida nos triângulos retângulos para ângulos agudos. Basta Seja θ um ângulo tal que θ ≠ k · π, k ∈ , e cuja imagem no
notar que, no triângulo retângulo OAP1, temos: ciclo trigonométrico é P. A cossecante de θ é a medida algébrica
do segmento OP’’, onde P’’ é a interseção da reta tangente ao ciclo
R
AP1 AP1 trigonométrico em P com o eixo dos senos.
tg θ= = ⇔ AP1= tg θ
MA

OA 1 Note que, se θ = k · π, k ∈ , a reta tangente não intersecta o eixo


SO

dos senos e, portanto, a cossecante de θ não está definida. Na figura


BP2 BP2 a seguir, temos
cotg=
θ = ⇔ BP
=2 cotg θ
OB 1
T

R
E

F
Note ainda que, se Px e Py são as projeções de P sobre os eixos dos
cossenos e dos senos, respectivamente, então temos: IA
PPx
∆OPPx  ∆OP1A ⇒ =
AP1 sen θ tg θ
⇔ = ⇔ tg=
θ
Lsen θ D O P R O
OPx OA cos θ 1 cos θ

PPy BP2 cos θ cotg θ cos θ


∆OPPy  ∆OP2B ⇒ = ⇔ = ⇔ cotg=
θ
OPy OB sen θ 1 sen θ

Portanto, para um ângulo θ qualquer, no qual as linhas


trigonométricas estejam definidas, valem as relações:
sen θ cos θ 1
=tg θ = cotg θ = cotg θ
cos θ sen θ tg θ

Denomina-se função tangente, a função de Dtg em  definida por


f(x) = tg x.

O domínio da função tangente é Dtg =


e a imagem Imtg = .
{x∈ | x ≠
π
2 }
+ kπ, k ∈ 

sec θ =OP'
Denomina-se função cotangente, a função de Dcotg em  definida
por f(x) = cotg x.
cossec θ =OP"
O domínio da função cotangente é Dcotg = {x ∈  | x ≠ kπ, k ∈ }
e a imagem Imcotg = .

148 PROMILITARES.COM.BR

PM_BOOK05_MAT.indb 148 01/11/2021 13:47:28


TRIGONOMETRIA: CÍRCULO TRIGONOMÉTRICO E FÓRMULAS DE ADIÇÃO, ARCO DOBRO E METADE

Observe que a definição acima é coerente com a que foi


estabelecida nos triângulos retângulos para ângulos agudos. Basta sen ( 2π ⋅ k + θ=
) sen θ, k ∈ 
notar que:
OP 1 1 cos ( 2π ⋅ k + θ=
) cos θ, k ∈ 
∆OPP' : cos=θ = ⇔ OP'
= ⇔ sec=
θ OP'
OP' OP' cos θ
tg ( 2π ⋅ k + θ )= tg θ, k ∈ 
OP 1 1
∆OPP" : sen= θ = ⇔ OP''
= ⇔ cossec
= θ OP''
OP'' OP'' sen θ
Dessa forma, ao efetuarmos a redução ao 1º quadrante, vamos
Denomina-se função secante, a função de Dsec em  definida por relacionar sempre ângulos na primeira volta.
f(x) = secx. A figura a seguir representa um ciclo trigonométrico no qual foi

{ π
O domínio da função secante é Dsec = x ∈  | x ≠ + kπ, k ∈  e
2 } π
marcada a imagem P do ângulo θ ∈  0,  .
 2
a imagem Imsec = ]−∞, −1] ∪ [1, +∞[ =  − ]−1,1[ .
Os pontos P2, P3 e P4 são simétricos do ponto P em relação ao eixo
Denomina-se função cossecante, a função de Dcossec em  definida Oy, à origem e ao eixo Ox, respectivamente. Assim, o quadrilátero
por f(x) = cossec x. ˆ ˆ ˆ ˆ
PP2P3P4 é um retângulo e = AOP =
A'OP 2 =
A'OP 3 =
AOP 4 θ.
O domínio da função cossecante é Dcossec = {x ∈  | x ≠ kπ, k ∈ }
e a imagem Imcossec = ]−∞, −1] ∪ [1, +∞[ =  − ]−1,1[ .
Secante e cossecante são funções periódicas de período 2π.
Os sinais da secante e da cossecante acompanham os sinais do
cosseno e do seno, respectivamente.

RELAÇÕES FUNDAMENTAIS L DO PRO


I
O seno e o cosseno de um mesmo ângulo θ são coordenadas de A FE
R
um ponto que dista 1 unidade da origem do sistema de eixos.
Se P é a imagem do ângulo θ ∈  no ciclo trigonométrico,

S
TE

então P = (cosθ, senθ). Assim, temos d(P,O) = 1 ⇔

SO
d (P,O ) = 1 ⇔ ( cos θ − 0 ) + ( sen θ − 0 ) = 1 ⇔ sen2 θ + cos2 θ = 1.
2 2
MA

Essa é a chamada relação fundamental da trigonometria.

R
sen2 θ + cos2 θ =1
Dividindo a relação acima por cos² θ, obtemos:

1 + tg=
2
θ sec2 θ O arco de imagem P2 na primeira volta é 180° – θ ou π – θ.
O arco de imagem P3 na primeira volta é 180° + θ ou π + θ.
Dividindo a relação acima por sen² θ, obtemos: R
O arco de imagem P4 na primeira volta é 360° – θ ou 2π – θ.
MA

1 + cotg
= 2
θ cossec2 θ Observe que, a menos do sinal, os arcos de imagem P, P2, P3 e P4
SO

têm as mesmas linhas trigonométricas, pois OT1 = OT2 e OT3 = OT4 .


Exemplo:
Analisando o sinal das linhas trigonométricas em cada um dos
3π 1
Sabendo que θ ∈  π,  e que sen θ = , calcule cos θ.
T

quadrantes, podemos concluir que:


S

 2 3
R
E

sen θ = sen ( π − θ ) = − sen ( π + θ ) = − sen ( 2π − θ ) ou


E

Resolução:
I −1= F
1 ⇔ cos θ = 1A
sen θ = sen (180° − θ ) = − sen (180° + θ ) = − sen ( 360° − θ )
O
2
 1 8
9 L cos θ R
sen2 θ + cos2 θ = 1 ⇒   + cos2 θ = 2

 3
D O
9
P = − cos π − θ ) = − cos ( π + θ ) = cos ( 2π − θ ) ou
(
cos θ = − cos (180° − θ ) = − cos (180° + θ ) = cos ( 360° − θ )
 3π  2 2
Como θ ∈  π,  , então cos θ < 0. Assim, temos: cos θ = − .
 2 3 tg θ = − tg ( π − θ ) = tg ( π + θ ) = − tg ( 2π − θ ) ou
tg θ = − tg (180° − θ ) = tg (180° + θ ) = − tg ( 360° − θ )

REDUÇÃO AO 1° QUADRANTE Para efetuar a redução ao 1º quadrante, é útil associar o valor


da linha trigonométrica de cada um dos ângulos à do ângulo
A redução ao primeiro quadrante é um método que permite o
correspondente no 1º quadrante.
cálculo das linhas trigonométricas de um ângulo qualquer por meio de
um ângulo do 1º quadrante associado a ele e que possui as mesmas
linhas trigonométricas, exceto pelo sinal. sen x cos x tg x
Antes de efetuar a redução ao 1º quadrante é interessante
determinar a primeira determinação positiva do ângulo.  π
x ∈  0,  sen (π – x) – cos (π – x) – tg (π – x)
 2

sen ( 360 ⋅ k + θ=
) sen θ, k ∈ 
 3π 
x ∈  π,  – sen (x – π) - cos (x - π) tg (x - π)
 2
cos ( 360 ⋅ k + θ=
 ) cos θ, k ∈ 

tg ( 360 ⋅ k + θ )= tg θ, k ∈   3π 
x ∈  , 2π  – sen (2π – x) cos (2π – x) – tg (2π – x)
2 

PROMILITARES.COM.BR 149

PM_BOOK05_MAT.indb 149 01/11/2021 13:47:30


TRIGONOMETRIA: CÍRCULO TRIGONOMÉTRICO E FÓRMULAS DE ADIÇÃO, ARCO DOBRO E METADE

sen ( α + β=
) sen α ⋅ cos β + sen β ⋅ cos α
sen ( α − β=
) sen α ⋅ cos β − sen β ⋅ cos α
cos ( α + β=
) cos α ⋅ cos β − sen α ⋅ sen β
cos ( α − β=
) cos α ⋅ cos β + sen α ⋅ sen β

Demonstração:

Observe que nessas relações o resultado apresenta sempre


D O primeira
Sejam Q, R e S a imagem no ciclo trigonométrico de arcos com
a mesma linha trigonométrica com o sinal dela no quadrante de
L P Rdeterminação positiva α, (α + β) e (–β), respectivamente.
=O
origem. Por exemplo, no 2º quadrante, o seno é positivo, o cosseno e

IA
 , o que implica AR = QS.
a tangente negativos. Logo, AR QS
F
As coordenadasEdesses pontos são dadas por:
Exemplos:
3
R Q = ( cos α,sen α ),
sen (180 − 60 =
) sen60=

S
TE

 
sen120=
2 R ( cos ( α + β ) ,sen ( α + β ) ) e

SO
= ( cos ( −β ) ,sen ( −β=
) ) ( cos β, − sen β ).
MA

1 S
cos (180 − 60 ) =
cos120 = − cos60 =

2
Aplicando a fórmula da distância entre pontos, temos:

R
1
sen150= sen (180 − 30 =
 ) sen30=

= QS ⇔
AR ( cos ( α + β ) − 1)
2
+ ( sen ( α + β ) − 0 ) ⇒
2
2
( cos α − cos β ) + ( sen α − ( − sen β ) ) ⇔
2 2
3 ⇔
cos (180 − 30 ) =
cos150 = − cos 30 =

2 ⇔ cos2 ( α + β ) − 2cos ( α + β ) + 1 + sen2 ( α + β ) ⇒
R
sen ( 360 − 45 ) =
sen315 = − sen 45 =

2 ⇔ cos2 α − 2cos α cos β + cos2 β + sen2 α + 2sen α sen β + sen2 β
MA

2 ⇔ 2 − 2cos ( α + β ) = 2 − 2cos α cos β + 2sen α sen β ⇔


SO

2 ⇔ cos ( α + β=
) cos α cos β − sen α sen β

cos 315= cos ( 360 − 45 =
) cos 45=

2
T

β ) cos ( α + ( −β=
cos ( α − = ) ) cos α cos ( −β ) − sen α sen ( −β=)
S

Exemplo:
R
E

cos α cos β − sen α ⋅ ( − sen β=


) cos α cos β + sen α sen β
E

A expressão sen(17π + θ) + sen(18π – θ) + sen(19π – θ) +


IA F
L DO PRO
sen(20π – θ), onde θ é um ângulo do 3° quadrante, vale: π   π  
sen ( α=
+ β ) cos  − ( α =+ β )  cos   − α =− β
a) senθ c) 2senθ e) 0 2   2  
b) –senθ d) –2senθ π  π 
cos  − α  cos β + sen  − α  sen=
β sen α cos β + sen β cos α
2  2 
Resolução: E
sen (17π + θ ) = sen ( 2π ⋅ 8 + π + θ ) = sen ( π + θ ) = − sen θ β ) sen ( α + ( −β=
sen ( α − = ) ) sen α cos ( −β ) + sen ( −β ) cos=
α
= sen α cos β − sen β cos α
sen (18π −
= θ ) sen ( 2π ⋅ 8 + 2π −
= θ ) sen ( 2π −
= θ ) − sen θ
As fórmulas a seguir permitem o cálculo da tangente da soma e
sen (19π − θ=
) sen ( 2π ⋅ 9 + π − θ=
) sen ( π − θ=
) sen θ π
da diferença de arcos, com α, β, α + β, α − β ≠ + kπ, k ∈ .
2
sen ( 20π + θ=
) sen ( 2π ⋅ 10 + θ=
) sen θ

sen (17π + θ ) + sen (18π − θ ) + sen ( sen19π − θ ) + sen ( 20π + θ ) = tg α + tg β


tg ( α + β ) =
( − sen θ ) + ( − sen θ ) + sen θ + sen θ =0 1 − tg α ⋅ tg β

Observe que o quadrante do ângulo θ não afeta em nada a tg α − tg β


tg ( α − β ) =
utilização das relações de redução ao 1º quadrante. 1 + tg α ⋅ tg β

FÓRMULAS DE ARCO SOMA E


DIFERENÇA FÓRMULAS DE ARCO DOBRO
As fórmulas a seguir permitem calcular o seno, o cosseno e a
As fórmulas a seguir permitem calcular o seno e o cosseno da
tangente do dobro de um arco.
soma e da diferença de arcos.

150 PROMILITARES.COM.BR

PM_BOOK05_MAT.indb 150 01/11/2021 13:47:31


TRIGONOMETRIA: CÍRCULO TRIGONOMÉTRICO E FÓRMULAS DE ADIÇÃO, ARCO DOBRO E METADE

=
cos 2α cos2 α − sen2 α p+q p−q
senp + senq =
2sen cos
2tg α 2 2
=
sen2α 2sen α ⋅ cos α = 2cos α − 1
2
tg2α =
1 − tg2 α
=
1 − 2sen2 α p−q p+q
senp − senq =
2sen cos
2 2
π
Note que a fórmula de tg2α só é válida se 2α ≠ + kπ, k ∈ .
2 p+q p−q
cosp + cos q =
2cos cos
2 2
FÓRMULAS DE ARCO METADE p+q p−q
cosp − cos q =
−2sen sen
As seguintes fórmulas permitem calcular o seno, o cosseno e a 2 2
tangente da metade de um arco, a menos do sinal.

α 1 − cos α α 1 + cos α α 1 − cos α sen (p + q)


sen = ± cos = ± tg = ± tgp + tgq =
2 2 2 2 2 1 + cos α cosp ⋅ cos q
sen(p − q)
tgp − tgq =
Exemplo: cosp ⋅ cos q
π
Calcule o seno, o cosseno e a tangente de .
8 As fórmulas a seguir permitem transformar produtos em soma.
π

D O P R Osenp ⋅ senq
Como ∈ QΙ , então todas as suas linhas trigonométricas são
8
L
positivas. 1
= cos (p − q) − cos (p + q) 
π 2
2− I2
A F 2

=
sen
π
=
sen
π4
1 − cos
= 4
1−
= 2
R E=q
cosp ⋅ cos
1
cos (p + q) + cos (p − q) 
8 2 2 2 2 2

S
TE

π 2 1

SO
1 + cos 1+ senp ⋅ cos
= q sen (p + q) + sen (p − q) 
π π4 4 2 2+ 2 2
MA

=
cos =
cos = =
8 2 2 2 2

R
π 2 Exercício Resolvido
1 − cos 1−
π π4 4 2 2− 2 2− 2
tg =tg = = = ⋅ =
8 2 π 2 2 + 2 2− 2 cos 45º ⋅tg45º − cos60º ⋅sen30º
1 + cos 1+ 01. O valor da expressão: é
4 2 cos2 30º + cos2 60º + cos2 45º
2− 2 2 −1
= = 2 −1 a) R
4−2 6
MA

2 2 +1
SO

b)
6
FÓRMULAS DE DUPLICAÇÃO
c) 2 2 −1
USANDO TANGENTE
T

6
S

R
As seguintes fórmulas permitem calcular o seno e o cosseno de
E

2 2 −1
E

um arco conhecendo-se a tangente do seu arco metade.


IA d)
3 F
2tg
x
1 − tg2
x L D O e) P R O 2 2 +1
3
sen x = 2 cos x = 2
x x
1 + tg2 1 + tg2
2 2 Resolução: C
sen45º
A fórmula seguinte permite calcular a tangente da metade de um cos 45º ⋅ − (sen30º ) ⋅ sen30º
cos 45º = sen45º − sen2 30º
ângulo conhecendo-se o seno e o cosseno do ângulo. =
sen 60º + cos 60º + cos 45º
2 2 2
1 + cos2 45º
2
x sen x 2  1 2 1 2 2 −1
tg = −  −
2 1 + cos x =
2  2
= 2 = 4 4= 2 2 − 1=

4 2 2 −1
 2
2
2 4+2 4 6 6
1 +  1+

 4 4
 2 
FÓRMULAS DE PROSTAFÉRESE
OU DE WERNER
As fórmulas de Prostaférese ou de Werner permitem transformar Exercício Resolvido
somas ou diferenças de senos, cossenos e tangentes em produtos ou
vice-versa. 12
02. Sendo senx = o valor de sen x + tg x é
13
As fórmulas a seguir permitem transformar somas e diferenças
em produtos.

PROMILITARES.COM.BR 151

PM_BOOK05_MAT.indb 151 01/11/2021 13:47:32


TRIGONOMETRIA: CÍRCULO TRIGONOMÉTRICO E FÓRMULAS DE ADIÇÃO, ARCO DOBRO E METADE

207 213 Resolução: C


a) d)
65 65  5 
2
144
cos2θ = 1 – sen2θ ⇒ cos2θ = 1 –   ⇒ cos2θ = ⇒
209 216  13  169
b) e) 12 −12
65 65 cosθ = ± (segundo quadrante) ⇒ cosθ =
13 13
211
c) 5  −5  −10
65 2⋅ 
senθ 3 −5 2 ⋅ tgθ  12  12 = 120
tg=
θ = = ⇒ tg2=
θ = =
Resolução: E cos θ −12 12 1 − tg²θ −
 5
2
−119 119
13 1−   144
 12 
 12
senx =
2
 12 
 13 ⇒   + cos2 x =
1⇒
sen2x + cos2 x = 13
 
 1
Exercício Resolvido
144 169 − 144 25 5
⇒ cos x =1 − = = =
169 169 169 13 6
05. Sabendo que cos θ − senθ = , então o valor de sen(2θ) é:
12 3
senx 13 12 13 12 a) –1
=
tgx = = .= e 1
cos x 5 13 5 5 d)
5 3
13 b) −
9 5
12 12 60 + 156 216 e)
senx + tgx = +
13 5
=
65
=
65
L DO PRO c)
1
6
6

I A Resolução: D FE
Exercício Resolvido R Elevando ao quadrado os dois membros da igualdade, temos:
2 2 1

S
TE

cos2θ + sen2θ – 2 · senθ · cosθ = ⇒ 1 – sen(2θ) = ⇒ sen(2θ) =


4 3 3 3
03. Se sen x =

SO
e tg x < 0, então tg2x vale:
5
MA

24 8
a) . d) .

R
7 3
EXERCÍCIOS DE

FIXAÇÃO
24 4
b) − . e) − .
7 3
8
c) − .
3
R 3π
Resolução: A 01. (EEAR) Gabriel verificou que a medida de um ângulo é rad.
MA

Essa medida é igual a 10


SO

2
4 16 9
cos2 x =
1−   ⇒ cos2 x =
1− ⇒ cos2 x =. Como a) 48° b) 54° c) 66° d) 72°
5 25 25
T

4 7π
S

02. (EEAR) O valor de rad em graus é


−3 5 4
R 30
E

tg x < 0, temos: cos x = e tgx = = − . Logo:


E

IA b)F 38.
5 3 3
− a) 36. c) 42. d) 46.
5
L D O03. (EEAR) O
=
tg(2x)
2 ⋅ tg x
=
 4
2⋅− 
 3
=

8
3
= =

8
3 24 P RUm arco de circunferência de 56π rad pode ser dividido em
1− tg2 x  4
2
16 7 7 _____ arcos de 30°.
1−  −  1 − −
 3 9 9 a) 6 b) 5 c) 4 d) 3

04. (EEAR) Considerando tg 25° = 1/2, o valor de tg 20° será


Exercício Resolvido a) 1/6. b) 1/5. c) 1/4. d) 1/3.

5  3π  05. (EEAR) O valor de cos 15° é


=
04. Se sen θ e θ ∈  , π  , então o valor de tg(2θ) é:
13 4 
2− 2 2+ 3 c) 2− 2 . d) 2+ 3 .
d) 1 a) . b) .
12 2 2
a) −
13 3
e)
120 3 06. (ESA) Sabendo que x pertence ao 4º quadrante e que cos x = 0,8,
b) − pode-se afirmar que o valor de sen2x é igual a:
119
120 a) 0,28 d) 0,96
c) b) –0,96 e) 1
119
c) –0,28

152 PROMILITARES.COM.BR

PM_BOOK05_MAT.indb 152 01/11/2021 13:47:34


TRIGONOMETRIA: CÍRCULO TRIGONOMÉTRICO E FÓRMULAS DE ADIÇÃO, ARCO DOBRO E METADE


07. (EEAR) Ao somar as medidas angulares 120° e rad, obtém- 03. (EEAR) Se x é um arco do 1º quadrante, com sen x = a e cos x = b,
2 senx.cos x
se a medida de um arco pertencente ao ___ quadrante então y = é
tgx.cos( π + x)
a) 1º
a) a
b) 2º
b) b
c) 3º
c) –a
d) 4º
d) –b

08. (EEAR) O valor de cos 735º é


3 4 a a
04. (EEAR) Sejam sen x = , cos x = e sen 2x = . Se é uma
1 5 5 b b
a)
4 fração irredutível, então b – a é igual a
3 a) 1
b)
4 b) 2
2+ 6 c) 3
c)
4 d) 4

d) 2+ 6
8 05. (EEAR) Dados sen a = x, cos a = y, sen b = z e cos b = w, então
sen(a + b) é igual a
tg x + cotg x a) xw + yz
09. Simplificando-se a expressão
cossec x
L D O b) Pxz R
, obtém-se
+ yw
O
a) cossec x.
b) cos x. I A c) xy – wz
FE
c) sec x. R d) xw – yz

S
TE

d) tg x. y 2cos180° − xysen270° + y 2sen90°


06. (EEAR) Para x·y ≠ 0, a expressão

SO
equivale a x 2cos0°
10. (EEAR) Seja x = 150°. Classifique em verdadeira (V) ou falsa (F)
MA

cada uma das sentenças, a seguir assinale a alternativa que apresenta y


a) .

R
o número de sentenças verdadeiras. x
3 1
I. cos x = b) .
2 x
II. sen 2x < 0 y
c) .
x2
x
III. tg > 0 y2
R
2 d) .
MA

x2
SO

a) 0.
b) 1. 07. (EEAR) Sendo tg x = 1/t e sen x = u, uma maneira de expressar o
c) 2. valor de cos x é
T

d) 3.
R
a) t.
E

IA b) u/t.
F
L D Od) uP+ t. R O
EXERCÍCIOS DE c) u · t.

TREINAMENTO 08. (EEAR) Se cos x = 2/3 e sen x > 0, então sen 2x é

4 5
01. (ESA) A soma dos valores de m que satisfazem a ambas as a)
9
m +1 m+2
igualdades senx = e cos x = é
m m 2 5
b)
a) 5 3
b) 6 5 3
c)
c) –4 2
d) 4 3
d)
e) –6 6

02. (EEAR) Ao simplificar a expressão (1 + cos x)(1 – cos x), tem-se 09. (EEAR) Se A = tg120° e B = tg240°, então
a) 1 a) B = A.
b) sen²x b) B = –A.
c) cos²x c) B = 2A.
d) 2 + cos²x d) B = –2A.

PROMILITARES.COM.BR 153

PM_BOOK05_MAT.indb 153 01/11/2021 13:47:35


TRIGONOMETRIA: CÍRCULO TRIGONOMÉTRICO E FÓRMULAS DE ADIÇÃO, ARCO DOBRO E METADE

sen x.sec x
10. (EEAR) Ao subtrair cos225° de sen420°, obtém-se 07. (EEAR) Seja A = . com tg x ≠ 0. Nessas condições, o
valor de A é tg x
3+ 2
a)
2 a) 2
2 2
b) 3−
2 b) 2
5 c) 2
c)
2 d) 1
1
d)
2 08. (EEAR) Se sen α ⋅ cos β =4 / 13 e sen β ⋅ cos α =36 / 65 , então
sen ( α +β ) é igual a
a) 56/65
EXERCÍCIOS DE b) 40/65

COMBATE c) 13/36
d) 13/56

2
01. (EEAR) Considere x um arco do 3º quadrante e cotangente de 09. (EEAR) Se a e b são arcos do 2º quadrante tais que sen a =
1 2
2 2 cos b = − , então sen (a + b) é
x igual a ctg x. Se senx = − , então o valor de =
A tgx + é 2
2 ctg2x
a) 3
L D O a) P 2R( − O
3+ 2 )
b) 2
I A 4 FE
c)
d) 3
2
R b)
− 2 1+ 3 ( )
4

S
TE

( 2 +1 )

SO
sec y 3
02. (EEAR) Se sen y = m e cos y = n, o valor de é c)
MA

cossec y 4
a) m.
(
3 3− 2 )

R
b) n². d)
c) mn. 4
m
d) . π
n 10. (AFA) Sabendo-se que 0 < α <β < , senα = a e senβ = b, então o
2
03. (EEAR) Simplificando a expressão sen ( 2π – x ) + sen ( 3π + x ) , valor da expressão sen (π + α) – cos (2π – β) será igual a
R
obtém-se
a) a + 1 − b2
MA

a) sen x
SO

b) −a + 1− b2
b) –sen x
c) 2sen x c) a − 1 − b2
T

d) –2sen x d) −a − 1− b2
R
E

04. (EEAR) O valor de sen 1270° é igual a IA F


O
L D O RESOLUÇÃO
PR
a) –cos 10°
EM VÍDEO
b) –sen 30° Abra o ProApp, leia o QR Code, assista à resolução
c) –sen 10° de cada exercício e AVANCE NOS ESTUDOS!

d) –cos 30°

05. (EEAR) Dados os ângulos de 30° e 150°, pode-se afirmar que GABARITO
a) sen 30° = cos 150°.
EXERCÍCIOS DE FIXAÇÃO
b) sen 30° = sen 150°.
01. B 04. D 07. A 10. C
c) cos 30° = cos 150°.
02. C 05. B 08. C
d) cos 30° = – sen 150°.
03. B 06. B 09. C
06. (EEAR) O valor correspondente ao cos 15º é EXERCÍCIOS DE TREINAMENTO
2+ 6 01. E 04. A 07. C 10. A
a)
4 02. B 05. A 08. A
2+ 3 03. D 06. A 09. B
b)
4 EXERCÍCIOS DE COMBATE
3 01. D 04. C 07. D 10. D
c)
4 02. D 05. B 08. A
d) 1 03. D 06. A 09. B

154 PROMILITARES.COM.BR

PM_BOOK05_MAT.indb 154 01/11/2021 13:47:36


TRIGONOMETRIA: FUNÇÕES
TRIGONOMÉTRICAS, EQUAÇÕES E
INEQUAÇÕES TRIGONOMÉTRICAS

FUNÇÕES TRIGONOMÉTRICAS FUNÇÃO COSSENO


Seja P a imagem de um ângulo θ no ciclo trigonométrico. Já vimos
FUNÇÃO SENO que o cosseno do ângulo θ é definido como a abscissa de P, ou seja,
cos θ =OPx . Assim, para obter o cosseno de θ, devemos projetar P
Seja P a imagem de um ângulo θ no ciclo trigonométrico. Já vimos sobre o eixo horizontal Ox, denominado eixo dos cossenos.
que o seno do ângulo θ é definido como a ordenada de P, ou seja,
sen θ =OPy . Assim, para obter o seno de θ, devemos projetar P sobre
o eixo vertical Oy, denominado eixo dos senos.

L DO PRO
I A FE
R

S
TE

SO
MA

sen θ =OPy R cos θ =OPx


A função seno é a função de  em  definida por f(x) = sen x. R
A função cosseno é a função de  em  definida por f(x) = cos x.
MA

O domínio da função seno é Dsen =  e a imagem Imsen = [–1,1]. O domínio da função cosseno é Dcos =  e a imagem Imcos = [–1,1].
SO

A função seno é periódica de período 2π. A função cosseno é periódica de período 2π.

Vamos analisar o gráfico da função seno, estudando os valores do Vamos analisar o gráfico da função cosseno, estudando os valores
T

seno de um ângulo de 0 a 2π. Assim, observe o que acontece com o


R do cosseno de um ângulo de 0 a 2π. Assim, observe o que acontece
E

F
segmento orientado OPy conforme o ponto P dá uma volta no ciclo com o segmento orientado OPx conforme o ponto P dá uma volta no
trigonométrico. IA O
ciclo trigonométrico.
π
L D O 1º)PDeRA até B, ou seja, de θ = 0 até θ = π2 , o cosseno decresce de
1º) De A até B, ou seja, de θ = 0 até θ = , o seno cresce de
2
π π π π
f(0) = sen 0 = 0 até f=   sen = 1. f(0) = cos 0 = 1 até   f= .= 0
  cos
2 2 2 2
π π
2º) De B até A’, ou seja, de θ = até θ = π, o seno decresce de 2º) De B até A’, ou seja, de θ = até θ = π, o cosseno decresce
π π
2 2
  π π
f=  sen = 1 até f(π) = sen π = 0. de f=  cos= 0 até f(π) = cos π = -1.
2 2 2 2
3π 3π
3º) De A’ até B’, ou seja, de θ = π até θ = , o seno decresce de 3º) De A’ até B’, ou seja, de θ = π até θ = , o cosseno cresce de
2 2

  3π  3π  3π
f(π) = sen π = 0 até f   = sen = −1 . f(π) = cos π = -1 até f=  cos = 0.
 2  2  2  2
3π 3π
4º) De B’ até A, ou seja, de θ = até θ = 2π, o seno cresce de 4º) De B’ até A, ou seja, de θ = até θ = 2π, o cosseno cresce
2 2
 3π  3π  3π  3π
f   = sen = −1 até f(2π) = sen 2π = 0. de f=  cos = 0 até f(2π) = cos 2π = 1.
 2  2  2  2

PROMILITARES.COM.BR 155

PM_BOOK05_MAT.indb 155 01/11/2021 13:47:38


TRIGONOMETRIA: FUNÇÕES TRIGONOMÉTRICAS, EQUAÇÕES E INEQUAÇÕES TRIGONOMÉTRICAS

FUNÇÃO TANGENTE FUNÇÃO COTANGENTE


Vimos que o eixo paralelo ao eixo Oy com a mesma orientação que Vimos que o eixo paralelo ao eixo Ox com a mesma orientação que
este e passando pelo ponto A é denominado eixo das tangentes. este e passando pelo ponto B é denominado eixo das cotangentes.
π Vimos também que, se um ângulo θ tal que θ ≠ k · π, k ∈ ,
Vimos também que, se um ângulo θ tal que θ ≠ + k ⋅ π, k ∈
2 tem imagem no ciclo trigonométrico P, então a cotangente de θ é a
, tem imagem no ciclo trigonométrico P, então a tangente de θ é a medida algébrica do segmento BP2 , onde P2 é a interseção da reta OP
medida algébrica do segmento AP1, onde P1 é a interseção da reta OP com o eixo das cotangentes.
OP com o eixo das tangentes.

L DO PRO
I A FE
R

S
TE

SO
MA

cotg θ =BP2
tg θ =AP1 A função cotangente é a função de Dcotg em  definida por

R
f(x) = cotg x.
A função tangente é a função de Dtg em  definida por f(x) = tg x.
{x ∈  | x ≠ kπ, k ∈ } e
{ }
O domínio da função cotangente é Dcotg =
π
O domínio da função tangente é Dtg = x ∈  | x ≠ + kπ, k ∈  a imagem Imcotg = .
e a imagem Im = . 2
tg A função cotangente é periódica de período π.
A função tangente é periódica de período π. R
Vamos analisar o gráfico da função cotangente, estudando os
MA

Vamos analisar o gráfico da função tangente, estudando os valores da cotangente de um ângulo de 0 a 2π. Assim, observe o que
SO

valores da tangente de um ângulo de 0 a 2π. Assim, observe o que


acontece com o segmento orientado BP2 conforme o ponto P dá uma
acontece com o segmento orientado AP1 conforme o ponto P dá uma
volta no ciclo trigonométrico.
volta no ciclo trigonométrico.
T

π π
1º) De A até B, ou seja, de θ = 0 (exclusive) até θ = , a cotangente
R
1º) De A até B, ou seja, de θ = 0 até θ = (exclusive), a tangente
E

2
E

cresce de f(0) = tg 0 = 0 até +∞. 2


IA decresce de +∞ até =F π
f   cotg = 0.
π

L DO PRO
2 2
π π
2º) De B até A’, ou seja, de θ = (exclusive) até θ = π, a tangente 2º) De B até A’, ou seja, de θ = até θ = π (exclusive), a
cresce de -∞ até f(π) = tg π = 0. 2 2
cotangente decresce de = π π
f   cotg= 0 até -∞.
3π 2 2
3º) De A’ até B’, ou seja, de θ = π até θ = (exclusive), a tangente 3π
cresce de f(π) = tg π = 0 até +∞. 2 3º) De A’ até B’, ou seja, de θ = π (exclusive) até θ = , a
2
3π cotangente decresce de +∞ até =  3π  3π
4º) De B’ até A, ou seja, de θ = (exclusive) até θ = 2π, a f   cotg = 0.
2  2  2

tangente cresce de –∞ até f(2π) = tg 2π = 0. 4º) De B’ até A, ou seja, de θ = até θ = 2π (exclusive), a
2
 3π  3π
cotangente decresce de = f   cotg = 0 até –∞.
 2  2

156 PROMILITARES.COM.BR

PM_BOOK05_MAT.indb 156 01/11/2021 13:47:40


TRIGONOMETRIA: FUNÇÕES TRIGONOMÉTRICAS, EQUAÇÕES E INEQUAÇÕES TRIGONOMÉTRICAS

FUNÇÃO SECANTE FUNÇÃO COSSECANTE


π Seja θ um ângulo tal que θ ≠ k · π, k ∈ , e cuja imagem no
Seja θ um ângulo tal que θ ≠ + k ⋅ π , k ∈ , e cuja imagem
2 ciclo trigonométrico é P. A cossecante de θ é a medida algébrica do
no ciclo trigonométrico é P. A secante de θ é a medida algébrica segmento OP”, onde P’’ é a interseção da reta tangente ao ciclo
do segmento OP’, onde P’ é a interseção da reta tangente ao ciclo trigonométrico em P com o eixo dos senos.
trigonométrico em P com o eixo dos cossenos.

L DO PRO
I A FE
R

S
TE

cossec θ =OP"

SO
MA

A função cossecante é a função de Dcossec em  definida por


sec θ =OP' f(x) = cossec x.

R
O domínio da função secante é Dcossec = {x ∈  | x ≠ kπ, k ∈ } e a
A função secante é a função de Dsec em  definida por f(x) = sec x.
imagem Imcossec = ]−∞, −1] ∪ [1, +∞[ =  − ]−1,1[ .
{ π
O domínio da função secante é Dsec = x ∈  | x ≠ + kπ, k ∈  e
2 } A função cossecante é periódica de período 2π.
a imagem Imsec = ]−∞, −1] ∪ [1, +∞[ =  − ]−1,1[ .
Vamos analisar o gráfico da função cossecante, estudando os
A função secante é periódica de período 2π.
valores da cossecante de um ângulo de 0 a 2π. Assim, observe o que
R
MA

acontece com o segmento orientado OP’’ conforme o ponto P dá uma


Vamos analisar o gráfico da função secante, estudando os valores
SO

volta no ciclo trigonométrico.


da secante de um ângulo de 0 a 2π. Assim, observe o que acontece
com o segmento orientado OP’ conforme o ponto P dá uma volta no π
1º) De A até B, ou seja, de θ = 0 (exclusive) até θ = , a cossecante
ciclo trigonométrico. 2
π π
T

decresce de +∞ até = = 1.
f   cossec
π
R 2 2
E

1º) De A até B, ou seja, de θ = 0 até θ = (exclusive), a secante


E

cresce de f(0) = sec 0 = 1 até +∞. 2


IA F π
2º) De B até A’, ou seja, de θ = até θ = π (exclusive), a cossecante
π
cresce de -∞ até f(π) = sec π = –1. 2
L DO PRO
2º) De B até A’, ou seja, de θ = (exclusive) até θ = π, a secante =
cresce de π π
= 1 até +∞.
f   cossec
2 2
2


3π 3º) De A’ até B’, ou seja, de θ = π (exclusive) até θ = , a
3º) De A’ até B’, ou seja, de θ = π até θ = (exclusive), a secante 2
2  3π  3π
decresce de f(π) = sec π = –1 até –∞. cossecante cresce de -∞ até f   = cossec = −1 .
 2  2

4º) De B’ até A, ou seja, de θ = (exclusive) até θ = 2π, a 3π
2 4º) De B’ até A, ou seja, de θ = até θ = 2π (exclusive), a
2
secante decresce de +∞ até f(2π) = sec 2π = 1. 3π 3π
cossecante decresce de f   = cossec = −1 até –∞.
 2  2

PROMILITARES.COM.BR 157

PM_BOOK05_MAT.indb 157 01/11/2021 13:47:41


TRIGONOMETRIA: FUNÇÕES TRIGONOMÉTRICAS, EQUAÇÕES E INEQUAÇÕES TRIGONOMÉTRICAS

ESTUDO DOS GRÁFICOS PERÍODO


Vamos estudar os gráficos de funções trigonométricas da forma 2π
A função f(x) = senBx possui período T = .
f(x) = A sen (Bx + C) + D. Para isso vamos analisar a influência de cada B
um dos coeficientes separadamente. Observe que o desenvolvimento Exemplo:
feito para a função cosseno se aplica de maneira similar às outras 2π
A função f(x) = sen2x possui período T = = π.
funções trigonométricas. 2

REFLEXÃO EM RELAÇÃO AO EIXO OX


A função f(x) = –sen x possui gráfico simétrico ao gráfico de
g(x) = senx em relação ao eixo Ox.

Exemplo:
x 2π
A função f ( x ) = sen possui período T= = 4π .
2 12

L DO PRO
AMPLITUDE I A FE
R
A função g(x) = sen x tem amplitude 1 e imagem Img = [–1,1].
A função f(x) = A sen x, com A > 0, tem amplitude A e imagem

S
TE

Imf = [–A,A].

SO
Exemplo:
MA

O gráfico da função f(x) = 2 sen x tem amplitude A = 2 e imagem DESLOCAMENTO HORIZONTAL

R
Im = [–2,2]. O gráfico da função f(x) = sen (Bx + C) é igual ao gráfico igual ao
C  C
gráfico de g(x) = senBx deslocado na horizontal de − . Se  −  > 0,
B  B
o gráfico se desloca para a direita e, se  −  < 0 , o gráfico se desloca
C
para a esquerda.  B
Exemplo: R
MA

 π
f ( x ) sen  x +  tem gráfico igual ao de f(x) = sen x
A função=
SO

π  4 
deslocado de unidades para a esquerda.
4
T

R
E

IA F
L DO PRO
Exemplo:
1 1
O gráfico da função f ( x ) = senx tem amplitude A = e
2 2
 1 1
imagem Im =  − , 
 2 2 Exemplo:
 π
f ( x ) sen  x −  tem gráfico igual ao de f(x) = sen x
A função=
π  4 
deslocado de unidades para a direita.
4

158 PROMILITARES.COM.BR

PM_BOOK05_MAT.indb 158 01/11/2021 13:47:42


TRIGONOMETRIA: FUNÇÕES TRIGONOMÉTRICAS, EQUAÇÕES E INEQUAÇÕES TRIGONOMÉTRICAS

Para encontrar o deslocamento na horizontal da função ProBizu


C O gráfico de f(x) = Asen (Bx + C) + D é tal que:
f(x) = sen (Bx + C), devemos fazer Bx + C = 0 ⇔ x =− . Se o
B A é a amplitude;

resultado for positivo, o deslocamento é para a direita e, se for
negativo, o deslocamento é para a esquerda. 2π
• T= é o período;
B
Exemplo: •  C  é o número de fase, ou seja, o deslocamento na
− 
 π  B
A função=f ( x ) sen  2x −  tem gráfico igual ao de f(x) = sen x horizontal (para direita, se positivo, ou para a esquerda, se
π  4 
deslocado de unidades para a direita. negativo); e
8
• D indica o deslocamento vertical (para cima, se positivo, ou
para baixo, se negativo).

Exemplo:
Construa o gráfico de f= ( x ) 2sen  2x − π  + 1 .
 4
1º) Constrói-se f1(x) = sen x.

2º) Constrói-se f2(x) = sen2x, a partir de f1, com período T = = π
2
 π
3º) Constrói-se=f3 ( x ) sen  2x −  , a partir de f2, deslocando-se
 4 
DO PRO
π
na horizontal para a direita.
DESLOCAMENTO VERTICAL
A L 8

I
O gráfico da função f(x) = senx + D é igual ao gráfico de f(x) = sen x FE
4º) Constrói-se= 

π
f4 ( x ) 2sen  2x −  , a partir de f3, com
4 
R
amplitude 2.
deslocado na vertical de D unidades. Se D > 0, o gráfico se desloca
para cima e, se D < 0, o gráfico se desloca para baixo.
5º) Constrói-se f=( x ) 2sen  2x − π  + 1 , a partir de f , deslocando-

S
TE

Exemplo: se 1 na vertical para cima.  4 4

SO
A função f(x) = sen x + 1 tem gráfico igual ao de g(x) = sen x
MA

deslocado de 1 unidade para cima.

R
R
MA

SO
T

R
E

IA CÁLCULO DO PERÍODO F
L= senDx O P R O
Exemplo: Seja f(x) uma função periódica de período P, então o período da
A função f(x) = sen x – 1 tem gráfico igual ao de g(x) P
função g(x) = A · f (Bx + C) + D é T = .
deslocado de 1 unidade para baixo. B
Note que as funções seno, cosseno, secante e cossecante são
periódicas de período 2π e as funções tangente e cotangente são
periódicas de período π.
Exemplo:
Calcule o período das seguintes funções.
a) y = sen2x  π
=
f) y cossec  2x − 
x  6
b) y = cos
2 x + π
=
g) y 2tg  +3
c) y = tg3x  6 
x  2π 
d) y = cotg cos  3x − 
3  3  −1
=h) y
 π 2
=
e) y sec  x + 
 3

PROMILITARES.COM.BR 159

PM_BOOK05_MAT.indb 159 01/11/2021 13:47:44


TRIGONOMETRIA: FUNÇÕES TRIGONOMÉTRICAS, EQUAÇÕES E INEQUAÇÕES TRIGONOMÉTRICAS

Resolução:
Propriedade fundamental:
2π π π
a) T= = π d) T= = 3π g) T= = 6π  π π
2 13 16 θ= arcsenk, k ∈ [ −1,1] ⇔ sen θ= k ∧ θ ∈  − , 
 2 2
2π 2π 2π
b) T= = 4π e) T= = 2π h) T=
12 1 3 Propriedades:
π 2π arcsen ( −x ) =− arcsenx, ∀x ∈ [ −1,1]
c) T= f) T= = π
3 2 sen ( arcsenx )= x; ∀x ∈ [ −1,1]
 π π
Sejam f1(x) e f2(x) duas funções periódicas de período P1 e P2, arcsen ( seny )= y; ∀y ∈  − , 
P n  2 2
respectivamente, com P1 ≠ P2. Se 1 = 1 , onde n1 e n2 são inteiros
P2 n2
positivos e primos entre si, então as funções ( f1 + f2 ) ( x ) =f1 ( x ) + f2 ( x ) A figura seguinte mostra o gráfico da função arco seno.
e ( f1 ⋅ f2 ) ( x ) =f1 ( x ) ⋅ f2 ( x ) são periódicas de período P = n2P1 = n1P2.

Exemplo:
Calcule o período das seguintes funções.
a) y = tg3x + cos4x
x
b)=y sen ⋅ cos 3x
2

DO PRO
c) y = secx – senx
Resolução:
A L
π
a) tg3x : P1 =; cos 4x : =
2π π
P2 = ; I FE
3
P1 π 3 2 π
4 2
π
R
= = ⇒ T =3 ⋅ =2 ⋅ =π

S
TE

P2 π 2 3 3 2

SO
MA

x 2π
b) sen : P1= = 4 π ; cos 3x : P2 = 2π ;
2 12 3

R
P1 4π 6 2π
= = ⇒ T = 1⋅ 4 π = 6 ⋅ = 4π
P2 2π 3 1 3
FUNÇÃO ARCO COSSENO
c) y=
sec x − sen x : P1 =
P2 =
2π Seja f : [0,π] → [–1,1] tal que f(x) = cos x uma função bijetora,
1 1 − sen x cos x então a sua inversa é f-1 : [–1,1] → [0,π] tal que f-1(x) = arccos x. Assim,
y = sec x − sen x = − sen x = = temos: y = f(x) = cos x ⇔ x = f-1(y) = arccos y.
R
cos x cos x
MA

1 1
1 − ⋅ 2sen x cos x 1 − sen2x Propriedade fundamental:
SO

2 = 2
cos x cos x θ= arccosk, k ∈ [ −1,1] ⇔ cos θ= k ∧ θ ∈ [0, π]
1 2π
T

1 − sen2x : P1 = =
π ; cos x : P2 = 2π ;
S

2 2
R Propriedades:
E

P1
=
π 1
= ⇒ T = 2 ⋅ π = 1⋅ 2π = 2π
IA F
arccos ( −x ) = π − arccos x, ∀x ∈ [ −1,1]
P2 2π 2
L D O arccos O
P R(cos y )=
cos ( arccos x )= x; ∀x ∈ [ −1,1]
y; ∀y ∈ [0, π]
FUNÇÕES TRIGONOMÉTRICAS
A figura seguinte mostra o gráfico da função arco cosseno.
INVERSAS
Vamos agora estudar as funções trigonométricas inversas. Todas
as funções trigonométricas que nós estudamos não são bijetoras. Para
podermos definir suas funções inversas, vamos restringir o domínio
das funções de maneira conveniente a fim de obter uma função
bijetora. O gráfico das funções trigonométricas inversas pode ser
obtido refletindo-se o gráfico da função trigonométrica em relação à
reta y = x (bissetriz dos quadrantes ímpares).

FUNÇÃO ARCO SENO


Seja f :  − π , π  → [ −1,1] tal que f(x) = sen x uma função bijetora,
 2 2 
 π π
então a sua inversa é f −1 : [ −1,1] →  − ,  tal que f-1(x)= arcsen x.
 2 2
Assim, temos: y = f(x) = sen x ⇔ x = f-1(y) = arcsen y.

160 PROMILITARES.COM.BR

PM_BOOK05_MAT.indb 160 01/11/2021 13:47:45


TRIGONOMETRIA: FUNÇÕES TRIGONOMÉTRICAS, EQUAÇÕES E INEQUAÇÕES TRIGONOMÉTRICAS

FUNÇÃO ARCO TANGENTE FUNÇÃO ARCO SECANTE


 π π  π π 
Seja f :  − ,  →  tal que f(x) = tgx uma função bijetora, Seja f : 0,  ∪  , π  → ]−∞, −1] ∪ [1, +∞[ tal que f(x) = sec x uma
 2 2 π π  2 2 
   π π 
então a sua inversa é f −1 :  →  − ,  tal que f-1(x) = arctgx. Assim, função bijetora, então a sua inversa é f −1 : ]−∞, −1] ∪ [1, +∞[ → 0,  ∪  , π 
 2 2  2 2 
temos: y = f(x) = tgx ⇔ x = f-1(y) = arctg y.  π π 
f −1 : ]−∞, −1] ∪ [1, +∞[ → 0,  ∪  , π  tal que f-1(x) = arcsec x. Assim, temos: y = f(x) = sec x
 2 2 
Propriedade fundamental: ⇔ x = f-1(y) = arcsec y.
θ= arc cotgk, k ∈  ⇔ cotg θ= k ∧ θ ∈ ]0, π[
Propriedade fundamental:
Propriedades:  π π 
θ= arcseck, k ∈  ⇔ sec θ= k ∧ θ ∈ 0,  ∪  , π 
arctg ( −x ) =− arctg x, ∀x ∈   2 2 

tg ( arctg x )= x; ∀x ∈ 
 π π Propriedades:
arctg ( tg y )= y; ∀y ∈  − , 
 2 2 arcsec ( −x ) = π − arcsec x, ∀x ∈ 
A figura seguinte mostra o gráfico da função arco tangente. sec ( arcsec x=) x; ∀x ∈ ]−∞, −1] ∪ [1, +∞[
 π π 
arcsec ( sec y )= y; ∀y ∈ 0,  ∪  , π 
 2 2 

L D O PA fiRguraOseguinte mostra o gráfico da função arco secante.


IA FE
R

S
TE

SO
MA

R
FUNÇÃO ARCO COTANGENTE R
Seja f: ]0,π[ →  tal que f(x) = cotg x uma função bijetora, então
MA

a sua inversa é f-1:  → ]0,π[ tal que f-1(x) = arccotg x. Assim, temos: y
SO

= f(x) = cotg x ⇔ x = f-1(y) = arccotg y.

Propriedade fundamental: FUNÇÃO ARCO COSSECANTE


T

R
θ = arc cotgk, k ∈  ⇔ cotg θ = k ∧ θ ∈ ]0, π[ π π
Seja f :  − ,0  ∪  0,  → ]−∞, −1] ∪ [1, +∞[ tal que f(x) = cossec x
E

IA F
 2   2

L DO PRO
 π   π
Propriedades: uma função bijetora, então a sua inversa é f −1 : ]−∞, −1] ∪ [1, +∞[ →  − ,0  ∪  0, 
 2   2
 π   π
arc cotg ( −x ) = π − arc cotgx, ∀x ∈  f −1 : ]−∞, −1] ∪ [1, +∞[ →  − ,0  ∪  0,  tal que f-1(x) = arccossec x. Assim, temos: y = f(x) =
 2   2
cotg ( arc cotgx )= x; ∀x ∈ 
cossec x ⇔ x = f-1(y) = arccossec y.
arc cotg ( cotgy )= y; ∀y ∈ ]0, π[
Propriedade fundamental:
A figura seguinte mostra o gráfico da função arco cotangente.
 π   π
θ= arccosseck, k ∈  ⇔ cossec θ= k ∧ θ ∈  − ,0  ∪  0, 
 2   2

Propriedades:
arccossec ( −x ) =− arccossec x, ∀x ∈ 
) x; ∀x ∈ ]−∞, −1] ∪ [1, +∞[
cossec ( arccossec x=
 π   π
arccossec ( cossec y )= y; ∀y ∈  − ,0  ∪  0, 
 2   2
A figura seguinte mostra o gráfico da função arco cossecante.

PROMILITARES.COM.BR 161

PM_BOOK05_MAT.indb 161 01/11/2021 13:47:46


TRIGONOMETRIA: FUNÇÕES TRIGONOMÉTRICAS, EQUAÇÕES E INEQUAÇÕES TRIGONOMÉTRICAS

EQUAÇÕES TRIGONOMÉTRICAS
BÁSICAS
Vamos mostrar como resolver equações trigonométricas básicas,
onde temos uma linha trigonométrica aplicada sobre uma função e
igual a um determinado valor. Um exemplo desse tipo de equação é
1
sen2x = . Normalmente, mesmo as equações trigonométricas mais
2
complexas, terminam com a resolução de uma equação dessa forma.

EQUAÇÃO EM SENO
Seja a ∈  tal que a ≤ 1, então
sen α = a ⇔ α = arc sena + 2kπ, k ∈  ∨ α = π − arc sena + 2kπ, k ∈ 

Notação resumida: α = kπ + ( −1) ⋅ arc sena, k ∈ 


k

Observe que representamos a solução da equação utilizando


QUADRO RESUMO a função arco seno por se tratar de um caso geral. Na maioria
dos problemas são apresentados ângulos cujos valores das linhas
FUNÇÃO INVERSA DOMÍNIO IMAGEM trigonométricas são conhecidos.
Ao resolver uma equação trigonométrica, é sempre útil identificar
D O as P
 π π
x ∈ [ −1,1]
y = arcsen x y ∈ − , 
L 2 2 RO
soluções no ciclo trigonométrico, como na figura seguinte.

y = arccos y x ∈ [ −1,1]
A
I y ∈ [0, π] FE
R
 π π

S
TE

y = arctg x x∈ y ∈ − , 
 2 2

SO
MA

y = arccotg x x∈ y ∈ ]0, π[

R
y = arcsec x x ∈ ]−∞, −1] ∪ [1, +∞[ y ∈ [0, π] − {}
π
2

x ∈ ]−∞, −1] ∪ [1, +∞[  π π


y = arccossec x y ∈  − ,  − {0}
 2 2 R
Exemplo:
MA

SO

1 k π kπ ( )k π
Exemplo: sen2x = ⇔ 2x = kπ + ( −1) ⋅ , k ∈  ⇔ x = + −1 ⋅ , k ∈ 
2 6 2 12
Calcule o valor das expressões a seguir:
Se, na equação sen α = a, o valor de a for tal que |a| > 1, então o
T

1  1 conjunto solução da equação é vazio.


a) arcsen
R
g) arcsen  − 
E

2  2
IA FExemplo:
b) arccos
2
2 
h) arccos  −
3
 2 

L DO PR O sen x = 2 ⇔ S = ∅

c) arctg 3 i) arctg (-1) EQUAÇÃO EM COSSENO


d) arccotg 1 Seja a ∈  tal que a ≤ 1, então
j) arc cotg ( − 3 )
e) arcsec 2 cos α = a ⇔ α = 2kπ ± arccos a, k ∈ 
2 3
f) arccossec
3 A identificação das soluções da equação no ciclo trigonométrico
encontra-se na figura seguinte.
Resolução:
1 π 2 3 π
a) arcsen = f) arccossec =
2 6 3 3
2 π  1 π
b) arccos = g) arcsen  −  =

2 4  2 6
π  3  5π
c) arctg 3 = arccos  −
3 h) =
 2  6
π π
d) arc cotg1 = arctg ( −1) =−
4 i)
4
π 5π
e) arcsec 2 =
3 j) arc cotg ( − 3 ) =
6

162 PROMILITARES.COM.BR

PM_BOOK05_MAT.indb 162 01/11/2021 13:47:48


TRIGONOMETRIA: FUNÇÕES TRIGONOMÉTRICAS, EQUAÇÕES E INEQUAÇÕES TRIGONOMÉTRICAS

Exemplo: EQUAÇÃO COM IGUALDADE DE COSSENOS


1 π π
cos 2x = ⇔ 2x = 2kπ ± , k ∈  ⇔ x = kπ ± , k ∈  cos α= cos β ⇔ α ± β= 2kπ, k ∈ 
2 3 6
Observe que dois arcos que possuem o mesmo cosseno ou são
Se, na equação cos α = a, o valor de a for tal que |a| > 1, então o
côngruos ( α − β= 2kπ, k ∈  ) ou suas imagens são simétricas em
conjunto solução da equação é vazio.
relação ao eixo Ox.
Exemplo:
No segundo caso, os ângulos seriam dados, sem perda de
cos x = 2 ⇔ S = ∅ generalidade, por α = θ + 2k1π, k1 ∈  e β = −θ + 2k 2π, k 2 ∈ , o que
implica α + β= 2 (k1 + k 2 ) π= 2kπ, k ∈  .
EQUAÇÃO EM TANGENTE A notação α ± β= 2kπ, k ∈  representa a união das soluções de
Seja a ∈ , então ambos os casos.
Outra maneira de resolver essa equação é usando as fórmulas
tg α = a ⇔ α = arc tga + kπ, k ∈ 
de Werner:
α +β α −β
Observe que a equação em tangente possui solução para qualquer cos α = cos β ⇔ cos α − cos β= 0 ⇔ −2sen   ⋅ sen  = 0 ⇔
valor de a real.  2   2 
A identificação das soluções da equação no ciclo trigonométrico α +β α −β
sen  = 0 ∨ sen  = 0
encontra-se na figura seguinte.  2   2 
α +β α+β
sen   =0 ⇔ =kπ, k ∈  ⇔ α + β =2kπ, k ∈ 
 2  2

DO PRO
α −β α −β
sen   =0 ⇔ =kπ, k ∈  ⇔ α − β =2kπ, k ∈ 

A L  2  2

I Exemplo: FE
R  π

3x +  2x +

π
=
3
2kπ, k ∈  ⇔ x =− +
π 2kπ
15 5
,k∈

S
TE

= cos  2x +  ⇔
cos 3x
 3  π π

SO
3x −  2x +  = 2kπ, k ∈  ⇔ x = + 2kπ, k ∈ 
 3 3
MA

Exemplo:

R
Exemplo: (ITA 1993) O conjunto das soluções da equação sen 5x = cos 3x
π π kπ contém o seguinte conjunto:
tg2x = 1 ⇔ 2x = + kπ, k ∈  ⇔ x = + , k ∈ 
4 8 2
a) {π
16 }
π
+k ,k∈
5

EQUAÇÕES COM IGUALDADE DE b) {π


16 }
π
+k ,k∈
3
R
LINHAS TRIGONOMÉTRICAS
MA



SO

c) +k ,k∈
4 3
EQUAÇÃO COM IGUALDADE DE SENOS


T

d) +k ,k∈
S

sen α = sen β ⇔ α − β = 2kπ, k ∈  ∨ α + β = π + 2kπ, k ∈ 


R 4 2
E

{ OF}
E

IA
Observe que dois arcos que possuem o mesmo seno ou são
côngruos ( α − β= 2kπ, k ∈  ) ou suas imagens são simétricas em
e)
π
4
+ 2kπ, k ∈ 

relação ao eixo Oy. L D O Resolução:


P E
R
No segundo caso, os ângulos seriam dados, sem perda de
generalidade, por α = θ + 2k1π, k1 ∈  e β = ( π − θ ) + 2k 2π, k 2 ∈ , o π  π 
= cos 3x ⇔ cos  − 5x=
sen5x  cos 3x ⇔  − 5x  ± 3x
= 2kπ
que implica α + β = π + 2 (k1 + k 2 ) π = π + 2kπ, k ∈ . 2  2 
Outra maneira de resolver essa equação é usando as fórmulas π kπ π
⇔x= + ,k∈ ∨ x = + kπ, k ∈ 
de Werner: 16 4 4
sen α = sen β ⇔ sen α − sen β= 0 ⇔
α −β α +β α −β α +β
{
S = x∈ | x =
π kπ π
+ , k ∈  ∨ x = + kπ, k ∈  ⊃
16 4 4
π
4 } {
+ 2kπ, k ∈  }
2sen   cos  =0 ⇔ sen  = 0 ∨ cos  = 0
 2   2   2   2 
α −β α −β EQUAÇÃO COM IGUALDADE DE TANGENTES
sen   =0 ⇔ =kπ, k ∈  ⇔ α − β =2kπ, k ∈ 
 2  2 tg α = tg β ⇔ α − β = kπ, k ∈ 
α +β α+β π
cos  = 0⇔ = + kπ, k ∈  ⇔ α + β = π + 2kπ, k ∈ 
 2  2 2 Observe que dois arcos que possuem o mesmo cosseno ou são
côngruos ( α − β= 2kπ, k ∈  ) ou suas imagens são simétricas em
Notação resumida: α = kπ + ( −1) ⋅ β, k ∈ 
k
relação à origem dos eixos ordenados.
Exemplo:
No segundo caso, os ângulos seriam dados, sem perda de
 π π generalidade, por α = (π + θ) + 2k1π, k1 ∈  e β = θ + 2k 2π, k 2 ∈ , o
2x −  x −  = 2kπ, k ∈  ⇔ x = − + 2kπ, k ∈ 
 π  4 4 que implica α − β = π + 2 (k1 − k 2 ) π = π + 2kπ, k ∈  .
sen2x= sen  x −  ⇔
 4  π 5π 2kπ A notação α − β = kπ, k ∈  representa a união das soluções de
2x +  x −  = π + 2kπ, k ∈  ⇔ x = + ,k∈
 4 12 3 ambos os casos

PROMILITARES.COM.BR 163

PM_BOOK05_MAT.indb 163 01/11/2021 13:47:50


TRIGONOMETRIA: FUNÇÕES TRIGONOMÉTRICAS, EQUAÇÕES E INEQUAÇÕES TRIGONOMÉTRICAS

Outra maneira de resolver essa equação é usando as fórmulas Marcamos na circunferência trigonométrica os pontos que são
de Werner: extremidade final dos arcos x cujo senx ≥ 1/2.
sen ( α − β )
tgα= tgβ ⇔ tgα − tgβ= 0 ⇔ = 0 ⇔ sen ( α − β )= 0 ⇔
cos α ⋅ cos β
α − β = kπ, k ∈ 

Exemplo:
 π  π π kπ
tg  4x −  =tg2x ⇔  4x −  − 2x =kπ, k ∈  ⇔ x = + , k ∈ 
 4  4 8 2

INEQUAÇÕES TRIGONOMÉTRICAS
Para simplificar uma inequação trigonométrica são utilizadas as
mesmas técnicas utilizadas nas equações trigonométricas., obtendo-
se ao final uma inequação básica. Para resolver essa inequação,
basta marcar as soluções da “equação” no ciclo trigonométrico e,
posteriormente, identificar os intervalos que satisfazem à inequação.
Exemplo:  π 5π 
Logo, S = x ∈  | ≤ x ≤ 
1  6 6
Resolva a inequação senx ≤ em [0,2π].
2

D O P2º R
Inicialmente, vamos marcar no ciclo trigonométrico as raízes de
caso
1
sen x = que são
π
e

.
A L O 2
2 6 6
I F
Resolver no intervalo 0 ≤ x ≤ 2π, a inequação cos x <
E no eixo dos cossenos o comprimento menor que
2
R Devemos marcar
2

S
TE

SO
MA

R
R
MA

SO

Os valores que satisfazem à inequação são aqueles cujo seno é


T

1  π   5π 
menor ou igual a , ou seja, S =
R
0, 6  ∪  6 ,2π .
E

2
IA F
L DO PRO
Marcamos na circunferência trigonométrica os pontos que são
Vamos ver alguns outros casos
2
extremidade final dos arcos x cujo cos x <
1º caso 2
1
Resolver no intervalo 0 ≤ x ≤ 2π , a inequação senx ≥ .
2
Devemos marcar no eixo dos senos os comprimentos maiores que
1
.
2

π 7π
Logo, S = x ∈  | ≤ x ≤ 
 4 4

164 PROMILITARES.COM.BR

PM_BOOK05_MAT.indb 164 01/11/2021 13:47:51


TRIGONOMETRIA: FUNÇÕES TRIGONOMÉTRICAS, EQUAÇÕES E INEQUAÇÕES TRIGONOMÉTRICAS

3º caso Exercício Resolvido


3
Resolver no intervalo 0 ≤ x ≤ 2π , a inequação senx < 02. Seja a função real definida por f(x) = 2 + 2sen(x), no intervalo
2
3 0 ≤ x ≤ 2π. O ponto de mínimo de f(x), nesse intervalo, tem
Devemos marcar no eixo dos senos o comprimento menor que . coordenadas
2

π   3π 
a)  , 0 . c)  , − 2.
2   2 
π   3π 
b)  , − 2. d)  , 0 .
2   2 

Resolução: D
Calculando:
−1 ≤ sen x ≤ 1
3π  3π 
Marcamos na circunferência trigonométrica os pontos que são f(x) = 2 + 2 ⋅ ( −1) = 0 ⇒ senx = −1 ⇒ x = 270° = rad ⇒  , 0 
2  2 
3
extremidade final dos arcos x cujo senx < .
2
Exercício Resolvido

L D O 03.
P NaRequação
O
π
tan(x) = cot(x) em , onde 0 < x < , o valor de x é
2

I A F
R
a)
b) 1
–1
E d)
4
π

S
TE

π π
c) e)

SO
3 6
MA

A dificuldade dessa resolução e a maneira de dar a resposta. Para


entendermos melhor o porquê da resposta, vamos retificar a circunferência Resolução: D

R
trigonométrica, indicando a resposta gráfica no Universo dado. 1
Sabendo que cotgx = , temos
tgx
1
tgx =
cotgx ⇒ tgx = ⇒ tg2 x = 1⇒ tgx =
1 ou tgx =
−1.
tgx
π 2π
Logo, S = x ∈  | 0 ≤ x < ou 
≤ x ≤ 2π Portanto, como x é um arco do primeiro quadrante, só pode ser
 3 3  π R
x= .
MA

4
Exercício Resolvido
SO

x Exercício Resolvido


01. Seja f : ( −π, π) →  definida por f(x) = cos   , então, é
T

 2
S

verdade que
R 04. A inequação sen(x)cos(x) ≤ 0, no intervalo de 0 ≤ x ≤ 2π e x
E

a) A função é crescente no intervalo (–π,0] decrescente no


intervalo [0,π) e não possui raízes reais. IA F
real, possui conjunto solução

L DO PR
b) A função é crescente no intervalo (–π,0] decrescente no O a)
π
2
≤ x ≤ π ou

2
≤ x ≤ 2π
intervalo [0,π) e possui duas raízes reais.
π 3π
c) A função é decrescente no intervalo (–π,0] crescente no b) 0 ≤ x ≤ ou π ≤ x ≤
intervalo [0,π) e possui duas raízes reais. 2 2
π 3π 5π 7π
d) A função é decrescente no intervalo (–π,π) e não possui raízes c) ≤x≤ ou ≤x≤
reais. 4 4 4 4
e) A função é crescente no intervalo [0,π) e possui uma raiz real. 3π 5π 7π
d) ≤x≤ ou ≤ x ≤ 2π
4 4 4
Resolução: A π 2π
Calculando:
e) 0 ≤ x ≤ ou ≤x≤π
3 3
f : ( −π, π) → 
Resolução: A
 −π  1
=
f( −π) cos
=   0 sen x cos x ≤ 0 ⇔ sen2x ≤ 0 ⇔ sen2x ≤1 0 ⇔
 2  → crescente Tem-se que 2 sen x cos x ≤ 0 ⇔ sen2x ≤ 0 ⇔ sen2x ≤ 0 ⇔
2
= =
f(0) cos ( 0) 1 π
π + 2kπ ≤ 2x ≤ 2π + 2kπ⇔ + kπ ≤ x ≤ π + kπ, com
π k ∈ . Assim,
π + 2k2π ≤ 2x ≤ 2π + 2kπ⇔ + kπ ≤ x ≤ π + kπ,
= =
f(0) cos ( 0) 1 π 2 3π
→ decrescente como para k = 0 vem ≤ x ≤ π, e para k = 1 temos ≤ x ≤ 2π,
π 2 2
=
f( π) cos =
  0 segue que o conjunto solução da inequação no intervalo [0, 2π] é
 2
x
cos   = 0 → x = ±π → x ⊄ ( −π, π)
 2
{ π
S = x ∈  | ≤ x ≤ π ou
2

2
≤ x ≤ 2π . }

PROMILITARES.COM.BR 165

PM_BOOK05_MAT.indb 165 01/11/2021 13:47:52


TRIGONOMETRIA: FUNÇÕES TRIGONOMÉTRICAS, EQUAÇÕES E INEQUAÇÕES TRIGONOMÉTRICAS

Exercício Resolvido 04. (EEAR) As funções f(x) = senx e g(x) = cosx, no segundo quadrante,
são, respectivamente,
05. O conjunto solução (S) para a inequação 2·cos²x + cos(2x) > 2, a) decrescente e decrescente
em que 0 < x < π, é dado por:
b) decrescente e crescente
 π 5π  c) crescente e decrescente
a) S = x ∈ (0, π) | 0 < x < ou < x < π
 6 6  d) crescente e crescente
 π 2π 
b) S = x ∈ (0, π) | < x <  1
 3 3 05. No ciclo trigonométrico os valores de x, tais que cos x ≤ , são
2
 π 2π   π 5π 
c) S = x ∈ (0, π) | 0 < x < ou < x < π a) x ∈  | < x < 
 3 3   3 3

 π 5π   π 5π 
d) S = x ∈ (0, π) | < x <  b) x ∈  | ≤ x ≤ 
6 6  3 3

 π 11π 
e) S = {x ∈ (0, π)} c) x ∈  | ≤ x < 
 6 6 
Resolução: A  π 7π 
d) x ∈  | 0 ≤ x ≤ ou ≤ x ≤ 2π
 6 6 
2cos x + cos ( 2x ) > 2 ⇒ 2cos x + cos x – sen x > 2 ⇒
2 2 2 2

2cos2x + cos2x – (1– cos2x ) > 2 ⇒ 4cos2x – 3 > 0 ⇒ 1 π 


DO PRO
06. (EEAR) Se senx = e x ∈  , π  , então tg x é igual a ______.
3 3 3 2 
cos x < − ou cosx >
A L
2 2
I a) −
2
4
FE c) −
2 2
9
R b) −
2 d)
2

S
TE

2 2

SO
2
MA

07. (EEAR) Se x é um arco do terceiro quadrante tal que tg x = , o


valor de sen x é 3

R
13 −2 13
a) . c) .
13 13

b) − 13 . −3 13
d) .
13 13
R
3 π
MA

08. (EEAR) Se cos x = , com 0 < x < , então o valor numérico da


SO

Logo, o conjunto solução será: 5 2


expressão 3 sen x + cos x é
 π 5π 
S = x ∈ (0, π) | 0 < x < ou < x < π 7 1
 6 6  a) 1. b) 3. c) . d) – .
T

5 5
R
E

IA F
09. (EEAR) É dito que um valor x anula uma função f se f(x) = 0. Assim,
EXERCÍCIOS DE L D Oa) πP. R O
um valor de x que anula a função f(x) = cos x é

FIXAÇÃO
c) 2π.
3 d) π.
π
b) .
2
01. (EEAR) No intervalo [0,π], a soma das raízes da equação
 π 3π 
3cos²x – 7sen²x + 2 = 0 é igual a 10. (EEAR) A imagem da função f(x) = cos x no intervalo  ,  é
2 2 
a) 4π c) 2π
a) 0. b) [0,1]. c) [–1,1]. d) [–1,0].
b) 3π d) π

− 3 EXERCÍCIOS DE

TREINAMENTO
02. (EEAR) Se cos α = é um arco cuja extremidade pertence ao
2 π
2º quadrante, então α pode ser________ rad.
6
a) 7 b) 17 c) 27 d) 37
3
01. Se senx = e 0 ≤ x < 2π, então a soma dos valores possíveis
3 2
03. (EEAR) Se 0°≤ x ≤ 90° e se sen4x = − , um dos possíveis para x é
2
valores de x é π 3π π
a) c) e)
a) 30° c) 75° 2 2 3
b) 45° d) 85° b) π d) 2π

166 PROMILITARES.COM.BR

PM_BOOK05_MAT.indb 166 01/11/2021 13:47:54


TRIGONOMETRIA: FUNÇÕES TRIGONOMÉTRICAS, EQUAÇÕES E INEQUAÇÕES TRIGONOMÉTRICAS

02. Se sen x + cos 2x = 1, então um dos valores de sen x é 09. (ESPCEX) A figura abaixo representa o gráfico da função definida
1 por f(x) = a · cos(bx). Os valores de a e b são respectivamente
a) 1 2 e) −
c)
2 2 y
1 − 3
b) d)
2 3 1

π
03. Se θ é um ângulo tal que 0 < θ < e o dobro do seu seno é igual
2 3π 4π
ao triplo do quadrado da sua tangente, então o valor do seu cosseno é −π 0 π 2π x
3 2
a) d)
3 3
-1
2 e) 1
b)
2
a) 1 e 2 1 e) –1 e 2
3 c) 1e
c) 1 2
2 b) –1 e d) –1 e 1
2
π
04. Os valores de x que satisfazem a equação cos  3x −  =
0 , são
 5 3
7π π 10. (ESPCEX) A soma das raízes da equação sen2x − =0 , onde
a) x = + k ; k = 0, ±1, ±2,… 0 < x < 360°, é 4
30 3

b) x=
7π π
+ k ; k = 0, ±1, ±2,… L D O a)b) P240°
RO
60° d) 720°
15

3
π I A c) 180° FE
e) 360°

c) x=
2
+ k ; k = 0, ±1, ±2,…
4 R

S
TE

7π π
d) x= + k ; k = 0, ±1, ±2,…

SO
5 2 EXERCÍCIOS DE

COMBATE
MA

7π π
e) x= + k ; k = 0, ±1, ±2,…
4 6

R
05. O número de raízes da equação cos x + sen x = 0 no intervalo
[π,3π] é: 01. (ESPCEX) Considere as seguintes proposições
a) 2 d) 4  π π
I. f ( x ) tg  2x +  é periódica, de período
A função = .
b) 1 e) 0  6 2
3
R
c) 3 II. A equação sen x = tem infinitas soluções.
MA

2
SO

06. A soma das raízes da equação 1 – 4cos² x = 0, compreendias 3 3π −3 4


entre 0 e π é: III. Sendo tg x = e π<x< , temos sen x = e cotg x = .
4 2 5 3
T

π 3π 7π
S

a) c) e) Sobre as proposições, pode-se afirmar que


3 4
R 6
E

a) todas são verdadeiras.


E

b) π 5π
IA F
d)
6
L DO PR c)O
b) todas são falsas.
apenas I e II são verdadeiras.
07. A solução da inequação sen²x < 2senx, no intervalo fechado d) apenas I e III são verdadeiras.
[0, 2π] é:
e) apenas II e III são verdadeiras.
a) 0 < x < 2π π
d) 0 < x <
3π 2 1
b) π < x < e) π 02. (ESPCEX) Se sen x + cos x = , com 0 ≤ x ≤ π , então o valor de
2 5
sen2x é
c) 0<x<π
12 12 24
a) − c) e)
25 25 25
1 2
08. A solução da inequação ≤ cos x ⋅ senx ≤ , onde x ∈ [0,π] 24 16
4 2 b) − d)
25 25
 π π
a) S= ,  2
12 6  625cos x
03. (ESPCEX) A soma das soluções da equação = 1 , para
 π 5π  π 25cos x
b) S= ,  0≤x≤ é
 12 3  2
 5π 5π  π π 5π
c) S= ,  a) c) e)
 12 6  6 2 6
π 2π
 π 5π  b) d)
d) S= ,  3 3
12 12 

PROMILITARES.COM.BR 167

PM_BOOK05_MAT.indb 167 01/11/2021 13:47:56


TRIGONOMETRIA: FUNÇÕES TRIGONOMÉTRICAS, EQUAÇÕES E INEQUAÇÕES TRIGONOMÉTRICAS

1 − sen2x
04. (ESPCEX) Dada a função f ( x ) = =I
e o intervalo [0, 2π],
1 + sen x
pode-se afirmar que RESOLUÇÃO EM VÍDEO
a) f é definida para todo x ∈ I e a imagem de f em I é [0,2]. Abra o ProApp, leia o QR Code, assista à resolução
3π de cada exercício e AVANCE NOS ESTUDOS!
b) f é definida para todo x ∈ I | x ≠ e a imagem de f em I é [0,2].
2
c) f não é definida x = –1 e a imagem de f em I é ]–1,1[.
π GABARITO
d) f não é definida x = e a imagem de f em I é [0,2].
2
EXERCÍCIOS DE FIXAÇÃO

e) f não é definida x = e a imagem de f em I é [0,1[. 01. D 04. A 07. C 10. D
2
02. B 05. B 08. B
05. (ESPCEX) A equação f(x) = –5 tem solução real se 03. C 06. A 09. B
a) f(x) = x² + 2x + 1 d) f(x) = tg x EXERCÍCIOS DE TREINAMENTO
b) f(x) = 10x e) f(x) = log3(|x| + 1) 01. B 04. A 07. C 10. D
c) f(x) = cos x 02. B 05. A 08. D
03. C 06. B 09. B
06. (ESPCEX) O conjunto de valores de x em [0,2π], em que a função EXERCÍCIOS DE COMBATE
1
f (x) = está definida, é 01. A 04. B 07. D 10. B
tg x − 1

 π   3π   π π   5π 3π  L D O 03.PE R O
02. B 05. D 08. D

A
06. D 09. C
a)  0, 2  ∪  π, 2  d)  4 , 2  ∪  4 , 2 
I FE
b)
 π   3π 
0, 2  ∪  π, 2  e)  π   5π  R
 0, 4  ∪  π, 4 
ANOTAÇÕES

S
TE

SO
 π   5π 
c) 0, 4  ∪  π, 4 
MA

R
07. (AFA) Considere as afirmativas abaixo:
I. cos x= cos 33°→ x= 33° ± k 360° (k ∈z ) ;
II. sen = x 43° ± k 360° (k ∈z ) ;
x sen 43°→ =
III. tg x= tg 36°→ x= 36° + k180° (k ∈ z )
Podemos dizer que são verdadeiras
R
MA

a) I e II c) II e III
SO

b) I e III d) I, II e III
T

08. (AFA) A soma das raízes da equação 1 – 4 cos²x = 0, 0 ≤ x ≤ π ,


S

é igual a
R
E

a) π/3 c) 5π/6
IA F
b) 3π/4 d) π
L DO PRO
09. (AFA) A solução da equação cos²x + senx + 1 = 0 é
π
a) x= + 2kπ, k ∈ Z
2
b) x = π + 2kπ, k ∈ Z

c) x= + 2kπ, k ∈ Z
2

d) x= + 2kπ, k ∈ Z
4

2sen2x + sen2x  π
10. A solução da inequação 0 < < 1 para x ∈ 0,  é
o conjunto 1 + tgx  2

 π   π 
a)  0, 4  . d)  0, 2  .

b)  π  π π 
 0, 4  . e)  4 , 2  .

c)  π .
 0, 2 

168 PROMILITARES.COM.BR

PM_BOOK05_MAT.indb 168 01/11/2021 13:47:57


NÚMEROS COMPLEXOS:
FORMA ALGÉBRICA E MÓDULO

NÚMEROS COMPLEXOS Números Complexos


a + bi; i² = –1
DEFINIÇÕES
Vimos na resolução de uma equação do 2º grau que se o
discriminante é negativo, ela não admite raízes reais. Por exemplo, Números Reais Números Imaginários Puros
a equação b=0 a=0
x2 + 9 = 0
não admite raízes reais. Se usarmos os métodos que conhecemos
para resolvê-la, obtemos
x2 = –9 L D O IGUALDADE
POsRnúmeros
O
DE NÚMEROS COMPLEXOS
complexos a + bi e c + di são iguais se suas partes
x = ± −9 IA reais são iguais eFsuas partes imaginárias são iguais, isto é:
E a = c
não têm raiz quadrada.
R
mas é inaceitável tal resultado para x; os números negativos

a + bi = c + di se  e

S
TE

Para superar tal impossibilidade e poder, então, resolver todas b = d


SO
equações do 2º grau, os matemáticos ampliaram o sistema de
MA

Exemplos:
números, inventando os números complexos.
10

R
Primeiro, eles definiram um novo número 2 + 5i = 4+ i
2
i= −1 Se x e y são números reais e x + yi = 7 – 4i, então x = 7 e y = – 4.
Isso conduz a i2 = –1. Um número complexo é então um número
da forma a + bi onde a e b são números reais.
Para a equação acima fazemos
ADIÇÃO E SUBTRAÇÃO
Adição R
x=± −9
MA

SO

x=± 9 ⋅ ( −1) Para adicionarmos dois


números complexos,
x = ± 9 ⋅ −1 (a + bi) + (c + di) = (a + c) + (b + d)i
adicionamos as partes reais e
T

x=±3i as partes imaginárias


S

As raízes da equação x2 + 9 = 0 são 3i e – 3i.


R
E

IA
Subtração
F
L DO PRO
DEFINIÇÃO
Para subtrairmos dois números
Um número complexo é uma expressão da forma (a + bi) – (c + di) = (a – c) + (b – d)i complexos, subtraímos as partes
a + bi reais e as partes imaginárias
onde a e b são números reais e i2 = –1.
Exemplos:
No número complexo a + bi, a é a parte real e b é a parte
imaginária. (3 + 4i) + (– 7 + 8i) = (3 – 7) + (4 + 8) i

Exemplos: = – 4 + 12i

Na prática, fazemos
2 + 5i parte real 2 parte imaginária 5 +
(3 + 4i) + (–7 + 8i) = 3 + 4i – 7 + 8i = – 4 + 12i
1 5 1 5 +
− i parte real parte imaginária −
3 7 3 7 (– 5 + 6i) – (4 – 2i) = (– 5 – 4) + [6 – (– 2)] i
12i parte real 0 parte imaginária 12 = – 9 + 8i

–9 parte real –9 parte imaginária 0 Na prática fazemos

Um número como 12i, com parte real 0, chama-se número


imaginário puro. Um número real como –9, pode ser considerado (–5 + 6i) – (4 – 2i) = –5 + 6i – 4 + 2i = –9 + 8i
como um número complexo com parte imaginária 0.

PROMILITARES.COM.BR 169

PM_BOOK05_MAT.indb 169 01/11/2021 13:47:58


NÚMEROS COMPLEXOS: FORMA ALGÉBRICA E MÓDULO

MULTIPLICAÇÃO Usamos essa propriedade para expressar o quociente de dois


números complexos na forma a + bi.
Multiplicamos números
complexos como
(a + bi)⋅(c + di) = (ac – bd) + (ad + bc)i
multiplicamos binômios, DIVIDINDO DOIS NÚMEROS COMPLEXOS
usando i2 = – 1 a + bi
Para escrevermos o quociente na forma A + Bi, multiplicamos
Exemplos: c + di
o numerador e o denominador pelo conjugado do denominador.

(2 + 3i)·(3 – 4i) = 6 – 8i + 9i – 12i2 Distributiva Exemplo:


3−i
Vamos escrever o quociente na forma a + bi.
2+i
= 6 + i – 12 ⋅ (–1) –8i + 9i = i e i2 = – 1 Multiplicamos o numerador e o denominador pelo conjugado do
denominador, para obter um número real no denominador.
= 6 + i + 12
3 − i (3 − i).(2 − i) 6 − 3i − 2i + i2 6 − 5i − 1 5 − 5i
= = = = = 1− i
= 18 + i 2 + i (2 + i).(2 − i) 4 − i2 4 +1 5

(–4 + 2i)·(2 + i) = – 8 – 4i + 4i + 2i2 Distributiva POTÊNCIAS DE i


Temos:

= – 8 + 2 ⋅ (–1) – 4i + 4i = 0 e i = – 1
D O P Ri O
2
i0 = 1 i4 = i2 ⋅ i2 = (–1) ⋅ (–1) = 1
=–8–2
A L =i i5 = i4 ⋅ i = 1 ⋅ i = i
F
1

= – 10 I i = –1 E
R i6 = i4 ⋅ i2 = 1 ⋅ (–1) = 1
2

i3 = i2 ⋅ i = –1 ⋅ i = –i i7 = i4 ⋅ i3 = 1(–i) = – i

S
TE

–3i·(4 – 2i) = – 3i ⋅ (4) – 3i ⋅ (–2i)

SO
Observe que as quatro potências de i na coluna da esquerda,
= – 12i + 6i2
MA

repetem-se nos quatro casos seguintes na coluna da direita. Este ciclo


= – 12i + 6 ⋅ (–1)
1, i, –1, –i

R
= – 6 – 12i
repete-se indefinidamente. Então, para simplificar ix para x > 4,
buscamos o maior múltiplo de 4 contido em x; por exemplo
O CONJUGADO E A DIVISÃO i26 = i24 · i2 = (i4)6 · i2 = 16 · (–1) = –1
Divisão de números complexos é semelhante à racionalização i43 = i40 · i3 = (i4)10 · i3 = i10 · (–i) = –i
do denominador de uma fração com radicais. Assim, se temos o R
3 −i
REPRESENTAÇÃO DOS NÚMEROS
MA

quociente nosso objetivo é escrevê-lo na forma a + bi. Para


2+i
SO

isso, introduziremos inicialmente o conceito de conjugado de um COMPLEXOS


número complexo. Um número complexo é constituído por duas componentes: a
parte real e a parte imaginária. Isso sugere a utilização de dois
T

COMPLEXOS CONJUGADOS
R
eixos para representá-lo: um para a parte real e o outro para a parte
E

O conjugado de um número complexo a + bi é a – bi, e o


IA F
imaginária. Esses dois eixos chamam-se eixo real e eixo imaginário,

O
respectivamente. O plano determinado por esses dois eixos chama-se
conjugado de a – bi é a + bi.
L D O Para
Os números complexos a + bi e a – bi são chamados complexos PR
plano complexo ou plano de Argand-Gauss.
desenharmos o gráfico do número complexo a + bi,
conjugados.
marcamos o ponto (a; b) no plano.
Para um número complexo z, seu conjugado é representado com
z; então, se z = a + bi escrevemos z = a – bi.
Exemplos:
O conjugado de z = 2 + 3i é z = 2 – 3i
O conjugado de z = 2 – i é z = 2 + i
O conjugado de z = 5i é z = – 5i
O conjugado de z = 10 é z = 10

Quando multiplicamos um número complexo z = a + bi pelo seu


conjugado z = a – bi, o resultado que se obtém é um número real
não negativo:

z ⋅ z = (a + bi) ⋅ (a – bi)

= a2 – abi + abi – b2i2

= a2 – b2 ⋅ (–1)
A soma dos quadrados de dois
= a2 + b2 números reais nunca é negativa

170 PROMILITARES.COM.BR

PM_BOOK05_MAT.indb 170 01/11/2021 13:47:58


NÚMEROS COMPLEXOS: FORMA ALGÉBRICA E MÓDULO

Exemplo:
a) 1
b) 1/2
c) 2
d) –1
e) 3/2

Resolução: A
2 + i 1 − i 2 − 2i + i − i2 3 − i 3 1
= ⋅ = ⇒ a+b = − = 1
1+ i 1− i 1 − i2 2 2 2

Exercício Resolvido

02. O conjunto solução da equação (1 + i)x + (1 – i) = 0, onde i é


a unidade imaginária é

a) 1 c) i
b) – 1 d) – i
MÓDULO DE NÚMERO COMPLEXO
L D O Resolução:
P R OC
IA F
O módulo (ou valor absoluto) do número complexo a + bi é
(1 + i) x + (1 − i) = 0E⇒ x =
distância de a + bi à origem do plano complexo. −1 + i 1 − i −1 + i + i − i2
⋅ = = i
R 1+ i 1− i 1 − i2

S
TE

SO
MA

Exercício Resolvido

R
03. z é um número complexo tal que =
z7 1, z ≠ 1 . O valor de
1+ z + z + z + z + z + z é
2 3 4 5 6

a) 1 + i
b) 1 – i
c) i R
MA

d) – i
SO

e) 0

Resolução: E
T

1( z7 − 1)
R
E

Soma da P.G.: S =1 + z + z2 + z3 + z 4 + z5 + z6 = . Como


E

IA F z −1

L DO PRO
z7 = 1 e z ≠ 1 ⇒ S = 0.

Usando o Teorema de Pitágoras, concluímos que a distância de Exercício Resolvido


(a; b) a (0; 0) é
|z|² = a² + b² 04. Considere um número complexo z, tal que o seu módulo é 10,
e a soma dele com o seu conjugado é 16. Sabendo que o afixo de
|z| = a² + b² z pertence ao 4º quadrante, pode-se afirmar que z é igual a:
a) 6 + 8i
DEFINIÇÃO b) 8 + 6i
O módulo (ou valor absoluto) do complexo z = a + bi é c) 10
|z| = a² + b² d) 8 – 6i
e) 6 – 8i

Exercício Resolvido Resolução: D


z = a + bi ⇒ |z|= 10 ⇒ a + b = 100 ⇒ z + z = 16 = 2a ⇒ a = 8
2 2
2+i
01. Se = a + bi , onde i = −1 , então o valor de a + b é ⇒ b² = 100 – 64 = 36
1+ i
4º quadrante ⇒ b = –6. Então, z = 8 – 6i.

PROMILITARES.COM.BR 171

PM_BOOK05_MAT.indb 171 01/11/2021 13:47:59


NÚMEROS COMPLEXOS: FORMA ALGÉBRICA E MÓDULO

Exercício Resolvido 06. (EEAR) Dado x ∈ , para que o número z = (2 – xi)(x + 2i) seja real,
o valor de x pode ser
2 + 3i a) 4. c) –1.
05. Seja z o número complexo . Determine o valor de α ∈ 
α +i b) 0. d) –2.
para que z seja um imaginário puro é
a) 1 07. (EEAR) O módulo do complexo z = –3 + 4i é
3 a) 3. c) 5.
b) −
2 b) 4. d) 6.
1
c) −
2 08. (EEAR) Calculando i2053, obtém-se
3 a) 1. c) –i.
d)
2 b) i. d) –1.
1
e) 09. (EEAR) Seja z = bi um número complexo, com b real, que satisfaz a
2
condição 2z2 − 7iz − 3 = 0. Assim, a soma dos possíveis valores de b é
Resolução: B
a) 7/2 c) 1
2 + 3i 2 + 3i ( α − i) 2α − 2i + 3αi − 3i2 (2α + 3) + i(3α − 2) (2α + 3) b)
(3α 5/2
− 2) d) –1
= ⋅ = = = 2 + 2 i
α +i α + i ( α + i) α2 − i2 α2 + 1 α +1 α +1
i2 (2α + 3) + i(3α − 2) (2α + 3) (3α − 2) 10. (EEAR) Sejam ρ1 e ρ2, respectivamente, os módulos dos números
=
α +1
2
= 2
α +1
+ 2
α +1
i , como deve ser imaginário puro teremos
L DO PRO complexos Z1 = 2 − 5i e Z2 = 3 + 4i. Assim, é correto afirmar que
α) ρ1 < ρ2
(2α + 3)
α +1
2
= 0 ⇒ 2α + 3= 0 ⇒ α = −
3
2 I A b) ρ2 < ρ1 FE
R c) ρ1 + ρ2 = 10
d) ρ1 − ρ2 = 2

S
TE

SO
MA

EXERCÍCIOS DE EXERCÍCIOS DE

FIXAÇÃO TREINAMENTO
01. (EEAR) A soma dos possíveis números complexos z1 e z2, tais que
z² = 5 + 12i, é
R
01. (EEAR) Se i é a unidade imaginária dos números complexos, o
valor de i15 + i17 é
a) 6. c) 4i. a) –i
R c) 0
MA

b) 0. d) 3 + 2i. b) –1 d) 1
SO

02. (EEAR) Sendo i a unidade imaginária, simplificando-se a expressão 02. (EEAR) Sejam Z1 = 3 + 3i, Q e R as respectivas representações,
( 3 + i) ⋅ ( 3 − i)
71 30
no plano de Argand-Gauss, dos números complexos Z2 e Z3. Assim, é
T

, obtém-se
R
(i − 3) ⋅ ( −3 − i)
29 70 correto afirmar que Z1 =
E

IA F y

L DO PRO
a) –10. c) 8.
b) –8. d) 10.

03. (EEAR) Sendo m – ni = i e mi – n = 1 + 3i, os números complexos


“m” e “n” são tais, que sua soma é igual a -2 1
x
1 3 1 3 Q
a) − − i c) − i
2 2 2 2 -2
1 3 1 3
b) − + i d) + i
2 2 2 2
-5
R
3 3 + 2i
=
04. (EEAR) A forma algébrica do número complexo z + é
3−i i−2 a) Z2 – Z3
a) 0,1 – 3i
b) Z2 + Z3
b) 0,1 – 1,1i
c) – Z2 + Z3
c) 1,7 + 1,1i
d) – Z2 – Z3
d) 1 – 1,7i
03. (EEAR) Dado o número complexo z = a + bi, se z + z = 10 e
05. (EEAR) O quadrante em que se representa, no plano de Argand- z – z = –16i, então a + b é
Gauss, o número complexo z = 1 + i³ é o
a) – 6 c) 2
a) 1º. c) 3º.
b) – 3 d) 8
b) 2º. d) 4º.

172 PROMILITARES.COM.BR

PM_BOOK05_MAT.indb 172 01/11/2021 13:48:00


NÚMEROS COMPLEXOS: FORMA ALGÉBRICA E MÓDULO

04. (EEAR) Sejam os números complexos z1 = 1 – i, z2 = 3 + 5i e 1


04. (EEAR) O inverso do número complexo z = –2i, z’ = é
z3 = z1 + z2. O módulo de z3 é igual a z
i
a) c) -2
a) 2 2 c) 2 3 2
b) 4 2 d) 2 3 1 d) 2i
b)
2
05. (EEAR) Considere z1= (2 + x) + (x2 – 1)i e z2 = (m – 1) + (m2 – 9)i.
Se z1 é um número imaginário puro e z2 é um número real, é correto 05. (EEAR) Seja o número complexo z = 1 + i. Se z’ é o conjugado de
afirmar que x + m pode ser igual a z, então o produto |z| · |z’| é igual a
a) 1 c) 3 a) 1. c) 3.
b) 2 d) 4 b) 2. d) 2 3.

06. (EEAR) Se i é a unidade imaginária, então 2i³ + 3i² + 3i + 2 é um 06. (EEAR) O número complexo z = (a – 4) + (b – 5)i será um número
número complexo que pode ser representado no plano de Argand- imaginário puro se
Gauss no ___________ quadrante. a) a = 4 e b = 5. c) a ≠ 4 e b = 5.
a) primeiro c) terceiro b) a = 4 e b ≠ 5. d) a ≠ 4 e b ≠ 5.
b) segundo d) quarto
07. (EEAR) Dado x ∈  para que o número z = (2 – xi)(x + 2i) seja real,
07. (EEAR) Sejam os números complexos z1 = 1 + 2i, z2 = 2 – i e o valor de x pode ser
z3 = 3i. O valor de z1 + z2 – z3 é a) 4. c) –1.
a) 1 – 2i.
b) 2 + 3i.
c) 3 – 2i.
d) 4 – i. L DO PRO
b) 0. d) –2.

I A FE 1
R
08. (EEAR) Sabe-se que os números complexos Z1 = [2m (3+ m)] + (3n + 5)i
e Z2 = (2m² + 12) + [4(n +1)]i são iguais. Então, os valores de m e n
08. (ESA) A parte real do número complexo


1
( 2i)
2 é:

a) b) –2 c) 0 d) 2

S
TE

são, respectivamente 4

SO
a) 3 e 1 c) 2 e –1
MA

b) 2 e 1 d) 3 e –1 09. (ESA) O número complexo i102, onde i representa a unidade


imaginária,

R
09. (EEAR) Sejam Z1 e Z2 dois números complexos. Sabe-se que o a) é positivo. d) está na forma trigonométrica.
produto de Z1 e Z2 é –10 + 10i. Se Z1 = 1 + 2i, então o valor de Z2 é b) é imaginário puro. e) está na forma algébrica.
igual a
c) é real.
a) 5 + 6i c) 2 + 15i
b) 2 + 6i d) –6 + 6i 10. (ESA) Com relação aos números complexos Z1 = 2 + i e Z2 = 1 – i,
R
onde i é a unidade imaginária, é correto afirmar
MA

10. (EEAR) Sejam z um número complexo e z’ o conjugado de z. Se a) z1 ⋅ z2 =−3 + i z1 ⋅ z2 =10


d)
SO

z1 = z + z’ e z2 = z – z’, pode-se garantir que


b) z1 = 2 e) z1 + z2 =3
a) z1 é um número real e z2 é um imaginário puro.
b) z1 é um imaginário puro e z2 é um número real c) z2 = 5
T

c) z1 e z2 são imaginários puros.


R
E

d) z1 e z2 são números reais. IA F


L DO PR O EM VÍDEO
RESOLUÇÃO
Abra o ProApp, leia o QR Code, assista à resolução
de cada exercício e AVANCE NOS ESTUDOS!
EXERCÍCIOS DE

COMBATE
GABARITO

01. (EEAR) Se i³ + 2i² é um número complexo do tipo a + bi, com a e EXERCÍCIOS DE FIXAÇÃO
b reais, pode-se afirmar, corretamente, que 01. B 04. B 07. C 10. B
a) a > 0 e b > 0 c) a>0eb<0 02. A 05. D 08. B
b) a < 0 e b < 0 d) a < 0 e b > 0 03. C 06. D 09. A
EXERCÍCIOS DE TREINAMENTO
02. (EEAR) A soma dos conjugados dos números complexos z1 = 2 – 3i
01. C 04. B 07. C 10. A
e z2 = 3 + i é o número complexo
02. A 05. A 08. B
a) 5 + 2i c) 3–i
03. B 06. B 09. B
b) 3 + 2i d) 5 – i
EXERCÍCIOS DE COMBATE
03. (EEAR) O valor de i11 – i21 – i38 é 01. B 04. A 07. D 10. D
a) 1 – 2i. c) –2. 02. A 05. B 08. A
b) 2 – i. d) 1. 03. A 06. B 09. C

PROMILITARES.COM.BR 173

PM_BOOK05_MAT.indb 173 01/11/2021 13:48:01


NÚMEROS COMPLEXOS: FORMA ALGÉBRICA E MÓDULO

ANOTAÇÕES

L DO PRO
I A FE
R

S
TE

SO
MA

R
R
MA

SO
T

R
E

IA F
L DO PRO

174 PROMILITARES.COM.BR

PM_BOOK05_MAT.indb 174 01/11/2021 13:48:01


NÚMEROS COMPLEXOS:
FORMA TRIGONOMÉTRICA

FORMA TRIGONOMÉTRICA Onde podemos isolar a e b.


A forma trigonométrica dos números complexos é uma =b Z senθ
parametrização em função de seu módulo e do seno e do cosseno de =a Z cosθ
seu ângulo com o eixo real.
Assim, nosso complexo Z, que na forma algébrica é Z = a + bi
pode ser escrito como
Z = a + bi = Z cosθ + Z senθ.i

Podemos colocar em |Z| evidência

L D O P R O =Z Z (cosθ + i ⋅ senθ)
IA F Ede definir a forma trigonométrica de um número
Onde acabamos
R complexo Z.
Para o dia a dia dos cálculos, afim de se não fazer várias vezes

S
TE

a escrita de ( cosθ + i ⋅ senθ ) é aceita a abreviatura de cis θ para

SO
( cosθ + i ⋅ senθ ) .
MA

Então cis=
θ ( cosθ + i ⋅ senθ )

R
A forma de escrita mais completa é Z = |Z|(cos(θ + 2kπ) + i · sen(θ
+ 2kπ))=ou Z Z ( cos ( θ + 360° ⋅ k ) + i ⋅ sen ( θ + 360° ⋅ k ) ) , porém será
mais importante para a operação de radiciação e dessa forma no dia
a dia será utilizada majoritariamente a 1ª determinação positiva do
ângulo.
R
A tangente é uma ótima maneira de se descobrir somente o
Olhando para o complexo Z = a + bi no plano de Argand-Gauss
MA

ângulo, ou como definiremos a partir de agora, o argumento de um


SO

acima podemos identificar coeficiente da sua parte real a sobre o eixo b


real e o coeficiente da sua parte imaginária b sobre o eixo imaginário. complexo. Assim utilizando somente tgθ = podemos encontrar o
a
Dessa forma temos que seu módulo (distância do afixo a origem do argumento de um complexo, basta somente ter atenção ao quadrante
plano) é dada por = a2 + b2 .
T

Z que ele está através dos sinais de a e de b.


S

R
E dessa forma, quando identificamos o ângulo θ formado entre o
E

a > 0 e b > 0 → 1° quadrante


E

IA
módulo de Z e o eixo real, podemos no triângulo retângulo destacado Fa < 0 e b > 0 → 2° quadrante
L DO PRO
abaixo fazer mais algumas deduções.
a < 0 e b < 0 → 3° quadrante
a > 0 e b < 0 → 4° quadrante
A base para trabalhar com números complexos na forma
trigonométrica é o conhecimento do ciclo trigonométrico, os valores
dos ângulos notáveis em todos os quadrantes e suas reduções.
Exemplo:
Passar para a forma trigonométrica o número complexo Z = 1 – i
Teremos que

12 + ( −1) =
2
Z= 1 + 1= 2

Assim teremos que


Podemos montar algumas relações trigonométricas −1 1 2
senθ = =− =−
b 2 2 2
senθ =
Z 1 2
cos=
θ =
a 2 2
cosθ =
Z Dessa forma, como a > 0 e b < 0 sabemos que o afixo de Z está
b no 4° quadrante.
tgθ =
a

PROMILITARES.COM.BR 175

PM_BOOK05_MAT.indb 175 01/11/2021 13:48:02


NÚMEROS COMPLEXOS: FORMA TRIGONOMÉTRICA

Pelos valores de senθ e cosθ vemos que é o ângulo que quando Tanto o produto quanto a divisão de números complexos na forma
reduzido do 4° ao 1° quadrante tem seus valores de seno e cosseno trigonométrica resultam em translações no plano de Argand-Gauss.
iguais ao ângulo de 45°, ou como é mais utilizado, o ângulo em
π
radianos rad. A relação entre o 4° e o 1° quadrantes é de ângulos
4 7π
replementares, assim θ = rad.
4
 7π 
= Z Z ( cosθ + i ⋅ sen
= θ) 2  cis 
 4 

Uma forma mais rápida, que com mais experiência será mais
fluída, é colocar |Z| em evidência em relação a Z.

Z=
1− i e Z =2

 1 i   2 2   7π 7π 
=Z=
1− i 2 −=  2  −= i  2  cos + i ⋅ sen 
 2 2  2 2   4 4 

ProBizu
I. Conjugado de Z na forma trigonométrica
= e Z Z ( cis ( −θ ) )
Z Z ( cisθ )=

L DO PRO
IA ZF> 1 e Z
II. Argumentos de reais e imaginários puros
> 1 → Z1 ⋅ Z2 > max { Z1 , Z2 }

R E 1 2

Z1 > 1 e Z2 < 1 ou Z1 < 1 e Z2 > 1 → Z1 ⋅ Z2 < max { Z1 , Z2 }


Z1 < 1 e Z2 < 1 → Z1 ⋅ Z2 < min{ Z1 , Z2 }

S
TE

SO
MA

III. Potenciação (1ª Fórmula de Moovrie)

( Z1 ) (| Z1 |) ⋅ cis (n ⋅ θ )
n n

R
=

Exemplo:
Sendo Z =−1 + 3i calcular o valor de Z16
Primeiramente iremos colocar Z na forma trigonométrica
R
( 3)
2
( −1)
2
Z = + = 1+ 3 = 2
MA

SO

Assim
 1 3   2π 
Z =−1 + 3i =2  − + i  =2cis (120° ) =2cis  
 2 2   3 
T

R
E

Utilizando a 1ª fórmula de Moovrie


E

IA F ⋅ 2π  2 ⋅ cis =
L DO PR O 
⋅ cis 16=
=
Z16

( 2)
3


16


32π 
3 
 30π 2π 
 2 ⋅ cis 
 3
+
16

3 
16

 2π   2π   2π 
=Z 2 ⋅ cis 10π + =16
 2 ⋅ cis  5 voltas + =  2 ⋅ cis  
16 16

OPERAÇÕES NA FORMA  3   3   3 
TRIGONOMÉTRICA Se quisermos novamente a forma algébrica
A forma trigonométrica se torna mais usual para operações
  2π   2π    1 3 
de produto e divisões e quase que fundamental para potências e Z =216 ⋅  cos   + isen    =216 ⋅  − + i  =−215 + 215 3i
radiciações.   3 3
   2 2 
Vamos supor para todos os casos
IV. Radiciação (2ª Fórmula de Moovrie)
=Z1 Z1 ( cisθ1 )
Para a radiciação será imprescindível a utilização de
=Z2 Z2 ( cisθ2 )
( θ + 2kπ ) e ( θ + 360° ⋅ k ) .
I. Produto
Z1.Z2 Z1 Z2 ( cis ( θ1 + θ2 ) )
1
= n Z1 = ( Z1 ) n
1
II. Divisão Z n é uma potência e daí poderemos fazer como a 1ª fórmula de
Moovrie.
Z1 Z1
= cis ( θ1 − θ2 )
Z2 | Z2 |

176 PROMILITARES.COM.BR

PM_BOOK05_MAT.indb 176 01/11/2021 13:48:03


NÚMEROS COMPLEXOS: FORMA TRIGONOMÉTRICA

 θ + 360° ⋅ k 
(| Z1 |)n ⋅ cis 
1
1 
=
n Z
1 ⋅ ( θ + 360° = ⋅ k )  n | Z1 | ⋅ cis  
n   n 
 θ + 2k π 
=
n Z
1
n | Z | ⋅ cis
1  
 n 

 θ + 2kπ   θ + 360° ⋅ k 
Do argumento   ou   podemos perceber que
 n   n 
 θ + 2kπ   θ 2kπ   θ + 360° ⋅ k   θ 360° ⋅ k 
 = + e = + 
 n  n n   n  n n 

Onde para k ∈ {1,2,3,...} o argumento de n Z1 é uma progressão


θ 2π 360°
aritmética de 1º termo e razão ou .
n n n
Para encontrarmos todas as raízes de n Z1 faremos k variar dentro
do conjunto ∈ {1,2,3,...} até que as raízes comecem a se repetir. Dessa maneira sempre que executamos a radiciação de um
número complexo, no plano de Argand-Gauss teremos formado um
Exemplo:
polígono regular.
Para Z =−1 − 3i encontrar todas as raízes de 4 Z
Quando fazemos a raiz cúbica temos o triângulo equilátero,
Primeiramente encontraremos Z na forma trigonométrica. quando fazemos a raiz quarta temos o quadrado e assim por diante.

Z= ( −1)
2
(
+ − 3 )
2
= L D O porP1,Sabendo
1+ 3 = 2 R
... naO
disso, não a necessidade de substituirmos o k por 0,

IA
operação de radiciação. Basta fazermos somente k = 0 e
 1 3 
depois somarmos FoEvalor dopara
ângulo central do nosso polígono regular
Z = −1 − 3i = 2  − −
2 2  R
i  = 2 ⋅ cis ( 240° + 360° ⋅ k ) no argumento encontrado
raízes.
k = 0 até que encontremos todas as

S
TE

 240° + 360° ⋅ k  No exemplo anterior, quando encontramos Para


4
=
Z 4
2 ⋅ cis  

SO
 4   240° + 360° ⋅ 0  4  240°  4
k = 0 → Z0 = 2 ⋅ cis   = 2 ⋅ cis   = 2 ⋅ cis ( 60° )
4
MA

 4   4 
 240° + 360° ⋅ 0  4  240°  4
Para k = 0 → Z0 = 2 ⋅ cis   = 2 ⋅ cis   = 2 ⋅ cis ( 60° ) bastava sabermos que essas raízes, quando representadas no plano,

R
4

 4   4  iriam gerar um quadrado e dessa forma basta somar 90° em cada


 240° + 360° ⋅ 1 4  600°  4 argumento a partir do argumento 60°, assim
Para k = 1 → Z1 = 4 2 ⋅ cis   = 2 ⋅ cis   = 2 ⋅ cis (150° )
 4   4 
 240° + 360° ⋅ 2  4  960°  4 Z1 =⋅
4
2 cis (150° )
Para k = 2 → Z2 = 4 2 ⋅ cis   = 2 ⋅ cis   = 2 ⋅ cis ( 240° )
 4   4  Z2 =⋅
4
2 cis ( 240° )
 240° + 360° ⋅ 3  4  1320°  4 R
Para k= 3 → Z3 =⋅ 4
2 cis   =⋅2 cis   =⋅2 cis ( 330° ) Z3 =⋅
4
2 cis ( 330° )
 4   4 
MA

SO

Quando utilizamos k = 4 percebemos que encontramos


novamente Z0.
T

 240° + 360° ⋅ 4   1680° 


S

Para k = 4 → Z 4 = 4
2 ⋅ cis  =
4
2 ⋅ cis  =
R
E

 4   4 
E

4
2 ⋅ cis ( 420° )= 4
2 ⋅ cis ( 360° + 60° )= 4
IA
2 ⋅ cis ( 60° ) F
L R O
DO
Neste exemplo ainda podemos encontrar todas as raízes na forma P
algébrica Z = a + bi, mas isso não é uma regra visto que nem sempre
encontraremos ângulos notáveis ou ângulos que são redutíveis a
notáveis, mesmo que por transformações trigonométricas.

RAÍZES COMPLEXAS NO PLANO DE


ARGAND-GAUSS
A parte mais brilhante do estudo talvez se dê agora. A
representação das raízes complexas no plano de Argand-Gauss (A
patir da raíz de índice 3) sempre resulta em um polígono regular.
Vimos que sempre que fazemos a radiciação de um número
complexo temos que

 θ + 2kπ   θ 2kπ   θ + 360° ⋅ k   θ 360° ⋅ k 


 = + e = + 
 n  n n   n  n n 

Se lembrarmos da geometria plana, mais precisamente do estudo


dos polígonos regulares, lembraremos que para qualquer polígono
360°
regular representa o seu ângulo central.
n

PROMILITARES.COM.BR 177

PM_BOOK05_MAT.indb 177 01/11/2021 13:48:04


NÚMEROS COMPLEXOS: FORMA TRIGONOMÉTRICA

Olhando para o polígono regular acima vemos que ele pode ser
dividido em n triângulos isósceles onde os lados congruentes
são todos iguais ao raio da circunferência circunscrita, n Z e com
360°
ângulos do vértice iguais a .
n
Assim a área do nosso polígono pode ser calculada como
 360° 
( Z)  360° 
2
n Z ⋅ n Z ⋅ sen   n ⋅ sen   ⋅n
S= n ⋅  n =  n 
2 2
Para o nosso exemplo das raízes quartas de Z =−1 − 3 i se
quisermos encontrar a área basta fazermos
4
2 ⋅ 4 2 ⋅ sen ( 90° )
S =⋅
4 =
2 ⋅ 4 22 =
2 2
2

Exercício Resolvido

01. O número complexo Z = 1 + i representado na forma


trigonométrica é

L DZ . O b)a)P 2(cos
O polígono sempre será inscrito numa circunferência de raio n
RO
2 (cos 45° + isen 45°).
12

I A F E 60°).
90° + isen 90°).
c) 4 (cos 60° + isen
R d) 4 (cos 60° − isen 60°).

S
TE

e) 2(cos 90° − isen 90°).

SO
MA

Resolução: A

R
a 1 2
=
ρ 12 + 12 →=
ρ 2 ⇒ cos=
θ = → cos=
θ →=
θ 45°
ρ 2 2
b 1 2
sen=
θ = → sen=
θ →=
θ 45° ⇒
ρ 2 2
=
Z 2 ⋅ ( cos 45° + i ⋅ sen 45°=
) 21 2 ⋅ ( cos 45° + isen 45° )
R
MA

SO

Exercício Resolvido

02. Seja o número complexo z =−1 − 3i, onde i é a unidade


T

imaginária. O valor de z8 é:
S

R
E

 4π 4π 
E

IA =
a) z 256  cos
 3
+ isen 
F
3 
ProBizu
L DO PRO
Muitas questões nos pedem a área do polígono gerado pelas =
 π π
b) z 256  cos + isen 
 3 3
raízes n-ésimas de um complexo. Vamos definir uma fórmula bem
simples, que inclusive não depende das raízes mas sim somente de  5π 5π 
=
c) z 256  cos + isen 
360°  3 3 
n Z e .
n  2π 2π 
=
d) z 256  cos + isen 
 3 3 
e) z 256 ( cos 2π + isen2π )
=

Resolução: D
O módulo de z é ρ= ( −1)2 + ( − 3)2= 2. Logo, se θ é o
1 3
argumento de z, então cos θ = − e sen θ = − . Em conse-
4π 2 2
quência, temos θ = rad. Daí, a forma trigonométrica de z é
3
 4π 4π 
=z 2  cos + isen  .
 3 3 
Portanto, pela Fórmula de Moivre, segue que
  4π   4π    2π 2π 
=
z8 28  cos  8 ⋅  + isen  8 ⋅ =   256  cos + isen  .
  3   3   3 3 

178 PROMILITARES.COM.BR

PM_BOOK05_MAT.indb 178 01/11/2021 13:48:05


NÚMEROS COMPLEXOS: FORMA TRIGONOMÉTRICA

Exercício Resolvido Exercício Resolvido

03. Considere as raízes complexas w0, w1, w2, w3 e w4 da equação 05. No plano Argand-Gauss estão indicados um quadrado ABCD e
w5 = z, onde z ∈  representadas graficamente por os afixos dos números complexos Z0, Z1, Z2, Z3, Z4, e Z5.

O número complexo z é
a) 16i.
L DO PRO
b) 32i.
I A FE
c) 16 + 16i.
d) 16 + 16 3i.
R Se o afixo do produto de Z0 por um dos outros cinco números

S
TE

e) 32 + 32 3i. complexos indicados é o centro da circunferência inscrita no

SO
quadrado ABCD, então esse número complexo é
MA

a) Z1.
Resolução: D
b) Z2.

R
Tem-se que w0 = 2 · (cos12° + i · sen 12°). Logo, sabendo que
w5 = z, pela Primeira Fórmula de Moivre, vem c) Z3.
d) Z4.
1 3
z= 25 ⋅ (cos60° + i ⋅ sen60°)= 32 ⋅  + i ⋅ = 16 + 16 3i. e) Z5.
 2 2 

Resolução: B R
MA

É fácil ver que o centro da circunferência inscrita no quadrado


Exercício Resolvido
SO

ABCD é o ponto (–1,5;–1,5). Desse modo, queremos calcular Zk,


04. Na figura abaixo, o ponto A é o afixo de um número complexo tal que Z0 ⋅ Zk =
−1,5 − 1,5 ⋅ i. Assim, como Z0 = –1 + i, temos
z no plano de Argand-Gauss. −1,5 − 1,5 ⋅ i −1,5 − 1,5 ⋅ i −1 − i 1,5 + 1,5 ⋅ i + 1,5 ⋅ i − 1,5
T

Zk= = ⋅ = = 1,5 ⋅ i= Z2.


S

−1 + i −1 + i −1 − i 1+ 1
R
E

IA F
L DO PRO EXERCÍCIOS DE

FIXAÇÃO
 5π 5π 
=
01. (EEAR) Se z 2  cos + isen , então z7 é igual ao produto
Se a distância do ponto A até a origem O é 4, então a diferença de 8 2 por  4 4 
entre z e o seu conjugado é igual a π π
a) cos + isen
a) −4 2 − 4 2i d) 4 2i 4 4
b) −4 2 + 4 2i e) 4 2 5π 5π
b) cos + isen
4 4
c) −4 2i
7π 7π
c) cos + isen
4 4
Resolução: D
3π 3π
De acordo com as informações, segue que d) cos + isen
4 4
z = 4 ⋅ (cos135° + i ⋅ sen135°) = −2 2 + 2 2 ⋅ i.

Logo, sendo z o conjugado de z, temos


z−z= −2 2 + 2 2 ⋅ i − ( −2 2 − 2 2 ⋅ i) =
4 2 ⋅ i.

PROMILITARES.COM.BR 179

PM_BOOK05_MAT.indb 179 01/11/2021 13:48:06


NÚMEROS COMPLEXOS: FORMA TRIGONOMÉTRICA

02. (EEAR) Seja M o afixo de um número complexo z. A forma polar 07. Passe o complexo Z = 1 + 3i para a forma trigonométrica.
de z é
08. Passe para forma trigonométrica o complexo −1 − 3i

09. Encontre o argumento do número complexo – 1 – i.

3 i
10. Qual o argumento do número complexo=
Z − ?
2 2

EXERCÍCIOS DE

TREINAMENTO
 4π 4π   7π 7π 
a) 2  cos + isen  c) 2  cos + isen 
 3 3   6 6  01. Passe para a forma trigonométrica o número complexo Z = 1 – i.
4π 4π 7π 7π
b) cos + isen d) cos + isen 1 3
3 3 6 6 02. Sendo Z1= + i e Z2 = –1 – i, calcule Z1Z2.
2 2

cosx + isenx D O 03.PSendo


03. (EEAR) Sendo i a unidade imaginária, simplificando-se a expressão
L RO Z=
1
+
3
i e Z2 = –1 – i, calcule (Z1)².
cosx − isenx
.
I A 1

FE
2 2

a) i ( cos 2x – sen 2x ) . R 04. Sendo Z1=


1
+
3 (Z )
i e Z2 = –1 – i, calcule 1 .
3

i ( cos 2x + sen 2x ) .

S
TE

b) 2 2 Z2

SO
c) cos 2x –isen 2x.
MA

d) cos 2x + isen 2x. 05. Sendo Z = 1 + i, qual deve ser o menor número n, com n ∈ *,
para que Zn seja um número real positivo?

R
04. (EEAR) Seja Q a imagem geométrica de um número complexo. O
argumento desse número é 3 i 6
06. Para o complexo Z =
− + encontre todas as raízes de Z.
2 2

07. Determine a forma trigonométrica do número complexo dado:


a) z = 1 + i
R d) z = 2i
MA

b) z = 1 + i 3 e) z = –3
SO

c) z = –1 + i

08. Dados z = 2 · cis50° e w = 3 · cis40°, calcule:


T

R a) z · w
E

 1  1 IA b) w3 F
a) arc sen  
 3
c) arc cos  
 3 L DO PR c) O
z6
2 2  2 2
b) arc sen   d) arc cos  −  09. (FUVEST) Seja z um número complexo de módulo 2 e argumento
 3   3  principal 120°. O conjugado de z é:
a) 2 – 2i 3 d) –1 + i 3
 5π 5π  b) 2 + 2i 3 e) 1 + i 3
=
05. (EEAR) O produto z · z’, sendo z 2  cos + isen  e
 4 4  c) –1 – i 3
 3π 3π 
=z′ a  cos + isen 
 4 4 
 π π 10. (EFOMM) Seja o número complexo z =−1 − 3i, onde i é a
a) 2a(cos 0 + isen 0) c) a  cos + isen  unidade imaginária. O valor de z8 é:
 2 2 
 π π  4π 4π 
b) 2a  cos + isen 
2 2 d) a ( cos 2π + i sen 2π ) =
a) z 256  cos + isen 
  3 3 
 π π
=
b) z 256  cos + isen 
=
06. (EEAR) Seja z 3 ( cos 20° + i ⋅ sen20° ) um número complexo na  3 3
forma trigonométrica. Assim, z² é igual a  5π 5π 
=
c) z 256  cos + isen 
a) 3(cos 20° + i.sen20°).  3 3 
b) 3(cos 40° + i.sen 40°).  2π 2π 
=
d) z 256  cos + isen 
c) 2 3 ( cos 20° + i ⋅ sen20° ) .  3 3 
d) 2 3 ( cos 40° + i ⋅ sen40° ) . e) z 256 ( cos 2π + isen2π )
=

180 PROMILITARES.COM.BR

PM_BOOK05_MAT.indb 180 01/11/2021 13:48:08


NÚMEROS COMPLEXOS: FORMA TRIGONOMÉTRICA

EXERCÍCIOS DE 07. (AFA) O número complexo z = a + bi é vértice de um triângulo

COMBATE equilátero, como mostra a figura abaixo.

 π π
01. (ESPCEX) Se (1 + i)  cos + isen  = x + iy, em que i é a unidade
 12 12 
imaginária e x e y são números reais, o valor de 3 ⋅ x + y é
a) 6 d) 3 6
b) 3 3
e)
2 2 É correto afirmar que o conjugado de z² tem afixo que pertence ao
c)
2 a) 1º quadrante.

02. Um quadrado ABCD está inscrito num círculo com centro na b) 2º quadrante.
origem do plano de Gauss. O vértice “A” é imagem do complexo c) 3º quadrante.
3 + 4i. Os afixos dos outros três vértices são os complexos: d) 4º quadrante.
a) − 3 + 4i; − 3 − 4i; 3 − 4i .
b) − 4 + 3i; − 3 − 4i; 4 − 3i . 08. (ESPCEX) No plano complexo, temos uma circunferência λ de raio
2 centrada na origem. Sendo ABCD um quadrado inscrito à λ, de
c) − 4 + 3i; − 3 − 4i; 3 − 4i .
acordo com a figura abaixo, podemos afirmar que o número complexo
d) − 3 + 4i; − 3 − 4i; 4 − 3i .
L DO PRO que representa o vértice B é
e) i, − i,1+ i .
I A a)
1
− +
3
i.
FE
n = 45, 46, 47, ..., 149, 150. Pode-se afirmar que
R
03. (AFA) Seja P o produto dos fatores (sen nO + cos nO), onde 2 2
b) − 3 − i.

S
TE

a) P = 0

SO
c) −1 + 3 i.
b) P = 290
MA

1 3
c) 1≤P<8 d) − − i.
2 2

R
d) 8 ≤ P ≤ 2 90

3 1
e) − + i.
 2 2 2
04. (AFA) Considere no campo complexo uma curva tal que Im   ≥ k,
z
onde z é um complexo não nulo. Se k = 2, tem-se sua representação
gráfica dada pelo R
1
MA

a) círculo de raio e tangente ao eixo real.


SO

4
1
b) círculo de raio e tangente ao eixo imaginário.
2
T

c) conjunto de pontos do plano complexo exterior ao círculo de raio 09. (AFA) Resolva a equação z³ – 1 = 0 no conjunto dos números
R
E

1 complexos. Considerando as raízes encontradas, analise as proposições


e centro  − , 0 
1
E

2  2  IA F
abaixo e classifique-as em V (VERDADEIRA) ou F (FALSA).

d) círculo de raio
1
e tangente ao eixo real.
O
L D O( ) OsPafiRxos das raízes formam um triângulo equilátero cuja área é
( ) A equação possui três raízes de multiplicidade 1.
2
3 3
05. (AFA) Dado o número complexo z tal que z + 2z − 9 =
3i , é correto unidades de área.
2
afirmar que:
( ) Duas das raízes são conjugadas.
a) z = 3 10
( ) Todas as raízes têm o mesmo módulo.
 7π 7π  A sequência correta é
=
b) z 3 2  cos + isen 
 4 4  a) V – F – V – V
c) z = 9 – 3i
b) V – V – F – V
d) 3
c) F–F–V–F
4 d) V – F – V – F
a b  π π
06. (ESPCEX) Seja a igualdade − =i  cos + isen  , onde i é
3 5  6 6
10. (EN) Qual valor de n, n inteiro maior que zero, para que (1 + i)n
a unidade imaginária. Se a e b são números reais, então o quociente
seja um número real?
a
é igual a a) 2
b
b) 3
3 3 3 e)
15 3
a) . c) − . . c) 4
5 5 4
d) 5
b)
3 3 3
. d) − . e) 6
5 5

PROMILITARES.COM.BR 181

PM_BOOK05_MAT.indb 181 01/11/2021 13:48:09


NÚMEROS COMPLEXOS: FORMA TRIGONOMÉTRICA

ANOTAÇÕES
RESOLUÇÃO EM VÍDEO
Abra o ProApp, leia o QR Code, assista à resolução
de cada exercício e AVANCE NOS ESTUDOS!

GABARITO
EXERCÍCIOS DE FIXAÇÃO
01. D
02. C
03. D
04. B
05. A
06. B
π
07. z 2 ( cos60° + isen60°=
= ) ou z 2cis
3

DO PRO

08. z 2 ( cos 240° + isen240=
= ° ) ou z 2cis
3

A L
09. =
θ

ou=
θ 225°
I FE
R
4
11π
10. =
θ ou=
θ 330°

S
TE

SO
EXERCÍCIOS DE TREINAMENTO
MA


=
01. z 2 ( cos 315° + isen315
= ° ) ou z 2cis
4

R
19π
=
02. z 2 ( cos 285° + isen285
= ° ) ou z 2cis
15
03. z = 1( cos180° + isen180° ) = −1

2 2
=
04. z
2
( cos ( −45°) + isen ( −45
= °))
2
( cos 315° + isen315° ) R
MA

05. n = 8
SO

06. cis25°, cis85°, cis145°, cis205°, cis265°, cis325°


 π π
=
07. a) z 2  cos + i sen 
T

 3 3
R
E

=
 π π
b) z 2  cos + i sen 
IA F
L DO PRO
 3 3
 3π 3π 
=c) z 2  cos + i sen 
 4 4 
 π π
=
d) z 2  cos + i sen 
 2 2
e) z 3 ( cos π + i sen π )
=

08. a) z=
⋅ w 6cis90
= 0
6i.
27 27 3
b) w 3 =
27cis1200 =
− + i.
2 2
c) z= 64cis300= 32 − 32 3i.
6 0

09. C
10. D
EXERCÍCIOS DE COMBATE
01. A 06. A
02. B 07. C
03. A 08. C
04. D 09. A
05. B 10. C

182 PROMILITARES.COM.BR

PM_BOOK05_MAT.indb 182 01/11/2021 13:48:10


POLINÔMIOS

DEFINIÇÃO Exemplos:
Uma função polinomial ou simplesmente polinômio, é toda a) Encontre os valores de m e n para que os polinômios
função definida pela relação P ( x ) = 4x 4 + (m − 1) x 2 + 3x − n e H ( x )= 4x 4 − 2x 2 + 3x + m + n sejam
iguais.
P(x) = an ⋅ xn + an −1 ⋅ xn −1 + an − 2 ⋅ xn − 2 + ... + a1x + a0 .
Igualando os coeficientes dos monômios de graus equivalentes
Onde an, an-1, an-2, ..., a1, a0 são números reais chamados teremos
coeficientes.
 m − 1 =−2 1
n ∈  e x ∈ C (nos complexos) é a variável.  ⇒ m =−1 e − n =−1 + n ⇒ n =
 −n= m + n 2
xn, xn-1, xn-2, ..., x são ditos termos algébricos.
A “junção” an · xn é chamada de monômio e o grau de um
L D O (bxP+b)c)(xR
Calcular a, b e c, sabendo-se que x² – 2x + 1 ≡ a(x² + x + 1) +
+O
1).
A
monômio é dado pela soma dos expoentes de seus termos algébricos.
Exemplos: I Eliminando Fos Eparênteses e somando os termos semelhantes do
2x³ → Monômio de grau 3 R segundo membro temos:
x² − 2x + 1 ≡ ax² + ax + a + bx² + bx + cx + c

S
3xy²z → Monômio de grau 4 (1 + 2 + 1)
TE

1x² − 2x + 1 ≡ (a + b)x² + (a + b + c)x + (a + c)

SO
GRAU DE UM POLINÔMIO
MA

Agora igualamos os coeficientes correspondentes:


O grau de um polinômio é dado pelo grau de seu maior monômio. Substituindo a 1ª equação na 2ª:

R
No caso dos polinômios de apenas uma variável, que são o alvo do a + b = 1
nosso estudo, se o coeficiente an ≠ 0, então o expoente máximo n é 
 a + b + c =−2
dito grau do polinômio e indicamos gr(P) = n. a + c =
 1
Exemplos:
1 + c =−2 ⇒ c =−3.
P(x) = 5 ou P(x) = 5x0 é um polinômio constante, ou seja, gr(P) = 0.
Colocando esse valor de c na 3ª equação, temos:
P(x) = 3x + 5 é um polinômio do 1º grau, isto é, gr(P) = 1.
R
MA

a – 3 = 1 ⇒ a = 4.
P(x) = 4x5 + 7x4 é um polinômio do 5º grau, ou seja, gr(P) = 5.
SO

Colocando esse valor de a na 1ª equação, temos:


4 + b = 1 ⇒ b = –3.
VALOR NUMÉRICO
T

Resposta: a = 4, b = –3 e c = –3.
S

O valor numérico de um polinômio P(x) para x = a, é o número


R
E

que se obtém substituindo x por a e efetuando todas as operações


indicadas pela relação que define o polinômio. IA
Observação
F
L R O Um polinômio é dito identicamente nulo se tem todos os seus

P
Exemplo:
Se P(x) = x³ + 2x² + x – 4, o valor numérico de P(x), para x = 2, D
coeficientes nulos.
é: O
P ( x ) = x 3 + 2x 2 + x − 4
OPERAÇÕES COM POLINÔMIOS
P ( 2) = 23 + 2 ⋅ 22 + 2 − 4
P ( 2) = 14 SOMA E SUBTRAÇÃO DE POLINÔMIOS
Da álgebra elementar, temos que só podemos somar e/ou subtrair
Observação termos semelhantes, ou seja, termos que possuam expoentes iguais.
Se P(a) = 0, o número a chamado raiz ou zero de P(x). Exemplo:
a é raiz de P(x) ⇔ P(a) = 0 P(x) = 3x4 – 7x3 + 5x2 + 12x – 8 e Q(x) = x4 – 12x2 + 7x + 2
P(x) + Q(x) = 4x4 – 7x3 – 7x2 + 19x – 6
P(x) – Q(x) = 2x4 – 7x3 + 17x2 + 5x – 10
POLINÔMIOS IGUAIS
Dizemos que dois polinômios A(x) e B(x) são iguais ou idênticos (e PRODUTO DE POLINÔMIOS
indicamos A(x) ≡ B(x)) quando assumem valores numéricos iguais para O produto de polinômios também segue da álgebra elementar se
qualquer valor comum atribuído à variável x. A condição para que dando pela distributiva.
dois polinômios sejam iguais ou idênticos é que os coeficientes dos
Exemplo:
termos correspondentes sejam iguais.
Sejam P(x) = x³ – 1 e H(x) = x² – x + 1. Calcular P(x) · H(x)
P(x) ⋅ H(x) = (x 3 − 1)(x² − x + 1) = x5 − x 4 + x 3 − x 2 + x − 1

PROMILITARES.COM.BR 183

PM_BOOK05_MAT.indb 183 01/11/2021 13:48:11


POLINÔMIOS

DIVISÃO DE POLINÔMIOS x 4 + 2x 3 − x + 3 x2 − x
Sejam dois polinômios P(x) e D(x), com D(x) não nulo.
−x + x
4 3
x 2 + 3x
Efetuar a divisão de P por D é determinar dois polinômios Q(x) e
R(x), que satisfaçam as duas condições abaixo: 3x 3 − x + 3
1ª) Q(x) · D(x) + R(x) = P(x) − 3x 3 + 3x 2
2ª) gr(R) < gr(D) ou R(x) = 0 3x 2 − x + 3
P(x) D(x) x 4 + 2x 3 − x + 3 x2 − x
R(x) Q(x)
−x 4 + x 3 x 2 + 3x + 3
Nessa divisão: 3x 3 − x + 3
P(x) é o dividendo. − 3x 3 + 3x 2
D(x) é o divisor.
3x 2 − x + 3
Q(x) é o quociente.
R(x) é o resto da divisão. x 4 + 2x 3 − x + 3 x2 − x
−x 4 + x 3 x 2 + 3x + 3
Observação
3x 3 − x + 3
Quando temos R(x) = 0 dizemos que a divisão é exata, ou seja, P(x)
é divisível por D(x) ou D(x) é divisor de P(x). − 3x 3 + 3x 2

D O P R− 3x3x −+ x3x+ 3 2

Se D(x) é divisor de P(x) ⇔ R(x) = 0

A L O 2
Vamos ver como funciona a divisão pelo método das chaves
I x + 2x − x +E
F3
Vamos dividir x + 2x − x + 3 por x² – x
R x2 − x
4 3 4 3

x 4 + 2x 3 − x + 3 x2 − x −x 4 + x 3 x 2 + 3x + 3

S
TE

3x 3 − x + 3

SO
O processo se dá sempre iniciando a divisão do monômio de
− 3x 3 + 3x 2
MA

maior grau do dividendo pelo monômio de maior grau do divisor.


x 4 + 2x 3 − x + 3 x² − x 3x 2 − x + 3

R
− 3x 2 + 3x
x4
Faremos a divisão de x por x², ou seja 2 = x 2 4
2x + 3
x
x 4 + 2x 3 − x + 3 x 2 − x
Exemplo:
x2 Determinar o quociente de P(x) = x4 + x³ – 7x² + 9x – 1 por
D(x) = x² + 3x – 2.
R
Posteriormente o resultado da divisão será multiplicado por todos
MA

Resolução:
SO

os monômios do divisor e será subtraído do dividendo (sinal trocado


após o produto) Aplicando o método da chave, temos:
x (x − x) = x − x
2 2 4 3
x 4 + x 3 − 7x 2 + 9x − 1 x 2 + 3x − 2
T

Ao trocarmos os sinais será –x4 + x³


R −x 4 − 3x 3 + 2x 2 x 2 − 2x + 1 → Q(x)
E

x + 2x − x + 3 x − x
4 3 2
IA − 2x − 5x + 9x − 1
3
F
2

−x 4 + x 3 x2 L D O P R Ox + 2x 3 + 6x 2 − 4x
2
+ 5x − 1
Nessa parte do processo o monômio de maior grau do dividendo,
no momento, sempre será anulado. − x 2 − 3x + 2
E dessa forma o processo se repete até que o resultado da 2x + 1 → R(x)
subtração tenha grau inferior ao grau do divisor. Verificamos que
x 4 + 2x 3 − x + 3 x2 − x 4
+ x 3 −
x 7x 2 + 9x - 1 ≡ (x

2
+
3x −
2) (x 2
− 2x + 1) + (2x
+
1)
−x + x
4 3
x 2 P(x ) D(x ) Q(x ) R(x )

+ 3x − x + 3
3

DIVISÃO DE UM POLINÔMIO POR


x 4 + 2x 3 − x + 3 x2 − x
UM BINÔMIO DA FORMA ax + b
−x 4 + x 3 x 2 + 3x Vamos calcular o resto da divisão de P(x) = 4x² – 2x + 3 por
3x − x + 3
3 D(x) = 2x – 1.
4x 2 − 2x + 3 2x − 1
x 4 + 2x 3 − x + 3 x2 − x
− 4x + 2x
2
2x
−x 4 + x 3 x 2 + 3x
3
3x 3 − x + 3
Utilizando o método da chave temos:
− 3x 3 + 3x 2
Logo: R(x) = 3

184 PROMILITARES.COM.BR

PM_BOOK05_MAT.indb 184 01/11/2021 13:48:12


POLINÔMIOS

1
A raiz do divisor é 2x –1= 0 ⇒ x = . ca + d =
r1
2 
cb + d =r2
1
Agora calculamos P(x) para x = .
2 Resolvendo o sistema obtemos
 1  1   1  1 r1 − r2 ar2 − ar1
=
P   4   – 2   + 3=
⇒ P  3 =c = e d , com a ≠ b
 2  4   2  2 a−b a−b
r1 − r2 ar − ar1
Logo: R(x) = x+ 2 , com a ≠ b
Observe que R(x)= 3= P  
1 a−b a−b
 2
Portanto, mostramos que o resto da divisão de P(x) por D(x) é igual Observações
1
ao valor numérico de P(x) para x = , isto é, a raiz do divisor. 1ª) Se P(x) for divisível por (x – a) e por (x – b), temos:
2
P(a) = r1 = 0
P(b) = r2 = 0
TEOREMA DO RESTO
Portanto, P(x) é divisível pelo produto (x – a)(x – b), pois:
O resto da divisão de um polinômio P(x) pelo binômio ax + b é
r1 − r2 ar − ar1
igual a P  −  =
b R(x) = x+ 2 =0+0=0
0
 a a−b a−b
b 2ª) Generalizando, temos
Note que − é a raiz do divisor.
a Se P(x) é divisível por n fatores distintos ( x − a1 )( x − a2 ) ,...., ( x − an )
Exemplo:
L D O então
P RP(x)Oé divisível pelo produto ( x − a )( x − a ) ,...,( x − a ) .
1 2 n

IA F
Calcule o resto da divisão de x² + 5x – 1 por x + 1.
Exemplo:
Um polinômio P(x)Edividido por x dá resto 6 e dividido por (x – 1) dá
Resolução:
Achamos a raiz do divisor: R resto 8. Qual o resto da divisão de P(x) por x(x – 1)?

S
TE

x + 1 = 0 ⇒ x = –1 Resolução:

SO
Pelo teorema do resto sabemos que o resto é igual a P(–1) 0 é a raiz do divisor x, portanto P(0) = 6 (eq. 1)
MA

P(–1) = (–1)² + 5 · (–1) – 1 ⇒ P(–1) = –5 = R(x) 1 é a raiz do divisor x – 1, portanto P(1) = 8 (eq. 2)
Resposta: R(x) = –5.

R
E para o divisor x(x – 1) temos P(x) = x(x – 1) Q(x) + R(x) (eq. 3)
O resto da divisão de P(x) por x(x – 1) é no máximo do 1º grau, pois
TEOREMA DE D’ALEMBERT o divisor é do 2º grau; logo: R(x) = ax + b
Da eq.3 vem P(x) = x(x – 1) Q(x) + ax + b
Um polinômio P(x) é divisível pelo ax + b se P  −  =
b
0
 a Fazendo:
Exemplo: R
x=0 ⇒ P (0) = a (0) + b ⇒ P (0) =b (eq. 4)
Determinar o valor de p, para que o polinômio P(x) = 2x³ + 5x² –
MA

1 ⇒ P (1) =
x= a (1) + b ⇒ P (1) =a + b (eq. 5)
SO

px + 2 seja divisível por x – 2.


Resolução: Das equações 1, 2, 4 e 5 temos:
Se P(x) é divisível por x – 2, então P(2) = 0. b = 6
T


S

P(2) =0 ⇒ 2 ⋅ 8 + 5 ⋅ 4 − 2p + 2 =0 ⇒ 16 + 20 − 2p + 2 =0 ⇒ p =19 a + b =8
R
E

Resposta: p = 19.
IA F
Logo, b = 6 e a = 2.

O
Agora achamos o resto: R(x) = ax + b = 2x + 6
L D O Resposta:
DIVISÃO DE UM POLINÔMIO
PELO PRODUTO (x – a)(x – b)
P RR(x) = 2x + 6.
Vamos resolver o seguinte problema: calcular o resto da divisão
do polinômio P(x) pelo produto (x – a)(x – b), sabendo-se que os restos O DISPOSITIVO DE BRIOT-RUFFINI
da divisão de P(x) por (x – a) e por (x – b) são, respectivamente, r1 e r2. Serve para efetuar a divisão de um polinômio P(x) por um binômio
Temos: da forma (ax + b).
a é a raiz do divisor x – a, portanto P(a) = r1 (eq. 1) Exemplo:
b é a raiz do divisor x – b, portanto P(b) = r2 (eq. 2) Determinar o quociente e o resto da divisão do polinômio
P(x) = 3x³ – 5x² + x – 2 por (x – 2).
E para o divisor (x – a)(x – b) temos
Resolução:
P(x) = (x – a)(x – b) Q(x) + R(x) (eq. 3)
Observe que o grau de Q(x) é uma unidade inferior ao de P(x), pois
O resto da divisão de P(x) por (x – a)(x – b) é no máximo do 1º
o divisor é de grau 1.
grau, pois o divisor é do 2º grau; logo R(x) = cx + d

 
RAIZ DO DIVISOR 
COEFICIENTES DE P(x)
Da eq.3 vem P(x) = (x – a)(x – b) Q(x) + cx + d
2 3 −5 1 −2
Fazendo
↓ 3 ⋅ (2) − 5 1⋅ (2) + 1 3 ⋅ (2) − 2
x = a ⇒ P(a) = c(a) + d (eq. 4)
3 1
 3  4 

x = b ⇒ P(b) = c(b) + d (eq. 5) COEFICIENTES DO QUOCIENTE Q(x) RESTO
Das equações 1, 2, 4 e 5 temos:
Resposta: Q(x) = 3x² + x + 3 e R(x) = 4.

PROMILITARES.COM.BR 185

PM_BOOK05_MAT.indb 185 01/11/2021 13:48:13


POLINÔMIOS

Para a resolução desse problema seguimos os seguintes passos:


Resolução: D
1º) Colocamos a raiz do divisor e os coeficientes do dividendo
ordenadamente na parte de cima da “cerquinha”. Um polinômio é identicamente nulo se todos os coeficientes são
nulos. Vamos analisar cada item.
2º) O primeiro coeficiente do dividendo é repetido abaixo.
a) Não será identicamente nulo, pois se a = 1 e b = 7 anula-se
3º) Multiplicamos a raiz do divisor por esse coeficiente repetido somente os termos em x2 e x. O termo com x3 terá coeficiente
abaixo e somamos o produto com o 2º coeficiente do dividendo, a2 = (1)2.
colocando o resultado abaixo deste.
b) Não será identicamente nulo, pois se a = 1 e b = 7 anula-se
4º) Multiplicamos a raiz do divisor pelo número colocado abaixo somente os termos em x2 e independente. O termo com x2 terá
do 2º coeficiente e somamos o produto com o 3º coeficiente, coeficiente (a + 1) = (1 + 1)2 = 4.
colocando o resultado abaixo deste, e assim sucessivamente.
c) Não será identicamente nulo, pois, se a = 1 o termo em x3 será
5º) Separamos o último número formado, que é igual ao resto da (a2 + 1) e não se anula.
divisão, e os números que ficam à esquerda deste serão os coeficientes
do quociente. d) Poderá ser identicamente nulo para a = 1 e b = –3.
e) Não será identicamente nulo, pois o termo em “x” é diferente
Exercício Resolvido de zero e não se anula.

01. O valor numérico do polinômio P(x) = x3 – 7x2 + 3x – 4 para


x=2é Exercício Resolvido
a) –16
04. Se o polinômio P(x) = x5 + 2ax4 + 2b é divisível por (x + 1)2,
b) –18 então a + b vale:
c)
d) –22
–20
L D O a)b)P -11R O
I A c) 2 FE
Resolução: B R d) −
1
O valor de P(x) para x = 2 é encontrado calculando P(2).

S
TE

2
Substituindo, temos:

SO
1
e)
P(2) =(2)3 − 7(2)2 + 3(2) − 4 ⇒ P(2) =8 − 28 + 6 − 4 =−18
MA

R
Resolução: E
Exercício Resolvido Pelo teorema do resto, P(–1) = 0.

02. Sabendo 2 é uma raiz de P(x) = 2x – 2x – (m – 1)x – m, o


3 2 2 P(x) = x5 + 2ax 4 + 2b
 ⇒ ( −1)5 + 2a( −1)4 + 2b =0 ⇒
maior valor de m é P( −1) =0
a) 1 d) 4 1
−1 + 2a + 2b = 0 ⇒ 2(a + b) = 1 ⇒ a + b =
R
b) 2 e) 5 2
MA

SO

c) 3

Exercício Resolvido
Resolução: B
T

Se (2) é raiz de P(x), então P(2) = 0. Substituindo em P(x) o valor 05. A divisão de um polinômio P(x) por um polinômio k(x) tem
R
E

Q(x) = x3 + 3x2 + 5 como quociente e R(x) = x2 + x + 7 como resto.


F
x = 2, temos:
IA Sabendo-se que o resto da divisão de k(x) por x é 2, o resto da

L D O a) P10 R O
P(2)
= 2(2)3 − 2(2)2 − (m2 − 1) ⋅ (2) − m divisão de P(x) por x é:
 ⇒ 16 − 8 − 2m2 + 2 − m = 0 ⇒
P(2) = 0
−( −1) ± ( −1)2 − 4( −2)(10) 1 ± 81 b) 12
−2m2 − m + 10 = 0 ⇒ m = = ⇒
2( −2) −4 c) 17
 1+ 9 10 5 d) 25
m =−4
=

4
=

2
 e) 70
1− 9
m = −8
=
− =
2
 −4 −4
Resolução: C
De acordo com a informação, P(x) = k(x) · (x3 + 3x2 + 5) + x2 + x + 7.
Exercício Resolvido Pelo teorema do resto se o resto da divisão de k(x) por x é 2, então,
k(0) = 0, pois dividir por x é o mesmo que dividir por (x – 0). Logo,
03. Dos polinômios abaixo, qual o único que pode ser identica- o resto da divisão de P(x) por x é P(0).
mente nulo?
P(0) = k(0) · (03 + 302 + 5) + 02 + 0 + 7 ⇒ P(0) = 2 · (5) + 7 ⇒
a) a2x3 + (a – 1)x2 – (7 – b)x P(0) = 17.
b) (a + 1)x2 + (b2 – 1)x + (a – 1)
c) (a2 + 1)x3 – (a – 1)x2
d) (a – 1)x3 – (b + 3)x2 + (a – 1)
e) a2x3 – (3 + b)x2 – 5x

186 PROMILITARES.COM.BR

PM_BOOK05_MAT.indb 186 01/11/2021 13:48:13


POLINÔMIOS

EXERCÍCIOS DE
09. (ESA) Sendo o polinômio P(x) = x³ + 3x² + ax + b um cubo perfeito,

FIXAÇÃO então a diferença a – b vale:


a) 3
b) 2
01. (ESA) O grau do polinômio (4x – 1) · (x² – x – 3) · (x + 1) é c) 1
a) 6 c) 3 d) 0
b) 5 d) 4 e) –1

02. (EEAR) O resto da divisão de kx² + x + 1 por x – k é 10. (EEAR) Ao comparar o valor de f(1) e f(–1) da função polinomial
a) k² + 1. f(x) = 5x6 + 4x² + 3x – 1, obtém-se

b) k² + k + 1. a) f(1) < f(–1).

c) k³ + k² + 1. b) f(1) = f(–1).

d) k³ + k + 1. c) f(1) > 2f(–1).


d) f(1) = 2f(–1).
03. (EEAR) Sejam os polinômios A(x) = a(x² + x + 1) + (bx + c)(x + 1) e
B(x) = x² – 2x + 1. Se A(x) ≡ B(x), então a + b – c =
EXERCÍCIOS DE

TREINAMENTO
a) 4.
b) 3.
c)
d) 1.
2.
L DO PRO
I A+ 5x – 30 por 01. (EEAR) Se (xF+ b)² – (x – a)(x + a) ≡ 2x + 17, sendo a e b números
E o valor de a + b é
Q(x) = x – 2 é igual a 44, então n é igual a: R
04. (ESA) Se o resto da divisão do polinômio P(x) = 2xn reais positivos, então
a) 2. c) 5.

S
TE

a) 4 d) 2 b) 3. d) 6.

SO
b) 5 e) 6
MA

c) 3 02. (EEAR) Sejam os polinômios A(x) = x³ + 2x² – x – 4,


B(x) = ax³ – bx² – 4x + 1 e P(x) = A(x) – B(x). Para que P(x) seja de grau

R
2, é necessário que
05. (EEAR) O polinômio (m – n – 3) x 2 + (m + n – 5) x =
0 será
identicamente nulo, se o valor de m² – n² for a) a ≠ -1 e b = -2
a) – 12. b) a = 1 e b = -2
b) – 5. c) a = 1 e b ≠ -2
c) 10. d) a ≠ 1 e b ≠ 2 R
MA

d) 15.
03. (EEAR) Ao dividir 3x³ + 8x² + 3x + 4 por x² + 3x + 2 obtém-se
SO

como resto.
06. (ESA) Para que o polinômio do segundo grau A(x) = 3x² – bx + c,
com c > 0 seja o quadrado do polinômio B(x) = mx + n, é necessário que a) 6 c) 4
T

a) b² = 4c b) 5 d) 3
R
E

b) b² = 12c
IA F
04. (EEAR) O resto da divisão de 4x³ + 2x² + x – 1 por x² – 3 é igual a

L D Ob) 11xP – R3 O
c) b² = 12
a) 13x + 5
d) b² = 36c
e) b² = 36
c) 2x + 5
07. (CFOE) Qual é o número que deve ser somado ao polinômio d) 6x – 3
3
x 2 + x para que ele se torne um quadrado perfeito? 05. (EEAR) Considere P(x) = 2x³ + bx² + cx, tal que P(1) = –2 e
2
P(2) = 6. Assim, os valores de b e c são, respectivamente,
3 9
a) c) a) 1 e 2
2 4
b) 1 e –2
3 9
b) d) c) –1 e 3
8 16
d) –1 e –3
08. (EEAR) Seja um polinômio P(x) = ax² + bx² + cx + d. Se os
coeficientes de P(x) são diferentes de zero, então, para todo x ∈ , 06. (EEAR) Dado o polinômio: ax³ + (2a + b)x² + cx + d – 4 = 0, os
“P(x) + P(–x)” tem grau valores de a e b para que ele seja um polinômio de 2º grau são
a) 4. a) a = 0 e b = 0
b) 3. b) a = 1 e b ≠ 0
c) 2. c) a=0eb≠0
d) 1. d) a = –1 e b = 0

PROMILITARES.COM.BR 187

PM_BOOK05_MAT.indb 187 01/11/2021 13:48:14


POLINÔMIOS

07. (EEAR) Assinale o polinômio que representa o comprimento do 04. (CFOE) Na divisão do polinômio x³ – 5x² + 12x – 18 por x – 3,
segmento AE. obtemos como quociente o polinômio:
3x² + 4x 2x² – 2x 3x² – 3x 2x a) x² – 6x + 2. c) x² – 2x + 2.
A B C D E b) x² + 2x – 6. d) x² – 2x + 6.

a) 8x² + 11x 05. (VUNESP) O resto da divisão do polinômio P(x) = x4 + 2x³ + mx² – 2
b) 3x² + 6x pelo binômio x + 1 é igual a 8, sendo m uma constante real. Portanto
m vale
c) 8x² + x
a) 8. d) 7.
d) 5x² – 5x
b) 10. e) 9.
08. (CFOE) O resto da divisão do polinômio P(x) = 3x² – 3x + 1 por c) 11.
G(x) = 3x – 1 é um número:
a) negativo. 06. (CBMSE) O quociente entre os polinômios P(x) = 2x³ – 7x² + 7x – 2
e Q(x) = 2x – 1, respectivamente, é
b) menor que 1.
a) x² – 4x + 1 c) x² – 3x + 4
c) natural.
b) x² + 3x – 4 d) x² – 3x + 2
d) maior que 1.
07. (AFA) O polinômio P(x) é divisível por x² – a² (a ≠ 0), se, e somente se:
09. (EEAR) O resto da divisão do polinômio P(x) = x4 – x³ + 4x² – 4x + 3
pelo binômio Q(x) = x – 1 é a) P(a) = 0 c) P(a) = P(–a) = 0

a) 1.
L DO PRO b) P(–a) = 0 d) P(a) = 0 e P(–a) ≠ 0

IA F do polinômio P(x) do 2° grau, de raiz nula, tal


b) 2.
08. (AFA) A expressão
que P(x) – P(x – 1) =Ex para todo x real é
R
c) 3.
d) 4. a) x² + x

S
TE

b) x² – x

SO
10. (EEAR) Sejam A(x) e B(x) dois polinômios cujos graus são 5 e 4,
respectivamente. Assim, têm o mesmo grau os polinômios resultantes x2 x
MA

c) +
de 2 2

R
a) A(x) – B(x) e A(x) · B(x). x2 x
d) −
b) A(x) + B(x) e A(x) · B(x). 2 2
c) A(x) + B(x) e A(x) – B(x).
09. (AFA) Se o polinômio P(x) = x³ – x² + mx + n é divisível por
d) A(x) – B(x) e A(x) ÷ B(x). Q(x) = x² – 2x + 1, então o valor de m² + n² é
a) 0 b) 1 R c) 2 d) 3
MA

SO
EXERCÍCIOS DE x+2 A B C

COMBATE
10. (AFA) Se =+ + , então A² + BC vale
x ( x + 1)( x − 2) x x + 1 x − 2
a) 7/9 b) 11/9 c) 5/3 d) 19/9
T

R
E

IA
01. (EEAR) Se o polinômio P(x) = ax³ – 3x² – bx – 3 é divisível por (x – 3) F EM VÍDEO
RESOLUÇÃO
L DO PRO
(x + 1), então o valor de a + b é
Abra o ProApp, leia o QR Code, assista à resolução
a) 10. de cada exercício e AVANCE NOS ESTUDOS!
b) 8.
c) 7.
d) 5.
GABARITO
02. (CFOE) Para que o polinômio 3x³ + px + q seja divisível por x² + 3x + 5, EXERCÍCIOS DE FIXAÇÃO
p e q devem ser, respectivamente,
01. D 04. B 07. D 10. C
a) –9 e –15
02. D 05. D 08. C
b) 9 e 15
03. A 06. B 09. B
c) –12 e –45
EXERCÍCIOS DE TREINAMENTO
d) 12 e 45
01. C 04. A 07. C 10. C
03. Calcule o valor de r que torna o polinômio p(x) = x² + r · x – 7 02. C 05. D 08. B
divisível por q(x) = x + 1. 03. A 06. C 09. C
a) –2 EXERCÍCIOS DE COMBATE
b) –4 01. A 04. D 07. C 10. B
c) –5 02. C 05. C 08. C
d) –6 03. D 06. D 09. C

188 PROMILITARES.COM.BR

PM_BOOK05_MAT.indb 188 01/11/2021 13:48:14


EQUAÇÕES POLINOMIAIS

M.M.C E M.D.C. DE POLINÔMIOS Então escrevemos que P(x) ≡ G(x) (mod (D(x) ) ) ⇒ x 3 − x 2 + 2x − 1 ≡ x 4 + x
P(x) ≡ G(x) ( mod (
O máximo divisor comum (M.D.C.) entre polinômios é o polinômioD(x) ) ) ⇒ x 3
− x 2
+ 2x − 1 ≡ x 4
+ x 2
+ x mod ( x 2
+ 1) . ( )
unitário formado pelos fatores comuns aos polinômios elevados aos Vamos ver um exemplo de como utilizar a congruência em divisões
seus menores expoentes, de forma que ele é o polinômio de maior de polinômios de graus elevados.
grau que divide todos aqueles. Exemplo:
As raízes comuns aos polinômios são também raízes de seu MDC, Sendo R(x) o resto da divisão de
com a menor multiplicidade.
= 100x100 + 99x 99 + 98x 98 + 5x5 + 4x 4 + 3x 3 + 2x 2 + x + 1 por
P(x)
Se o MDC de dois polinômios é 1, diz-se que eles são primos
D(x) = x² – x + 1. O valor de R(–1) será...
D O chaves
entre si.
Quando os polinômios não estão na forma fatorada, o seu MDC
L PMuitos
Rencontrando
O
tentam resolver este tipo de questão pelo método das
um padrão de decrescimento nos expoentes nas
pode ser obtido pelo método das divisões sucessivas.
I A F
Um outro fatoEque é importante de ser observado é que o
divisões sucessivas, mas esse padrão nem sempre aparece.
Exemplo: R
Obtenha o MDC dos polinômios p(x) = x4 − 3x3 + 3x2 − 3x + 2 e divisor geralmente não pode ser decomposto em outros fatores, mas

S
TE

q(x) = x2 − 4x +3. geralmente é fator de um outro polinômio conhecido.

SO
D(x) = x² – x + 1 não pode ser decomposto em outros fatores de
MA

polinômios mas lembrando que x³ + 1 = (x + 1)(x² – x + 1) sabemos que


1 3
x2 + x +4 x− ← quocientes x³ + 1 é divisível por x² – x + 1, assim x³ + 1 ≡ 0 mod(x² – x + 1)

R
10 10 ⇒ x³ ≡ –1 mod(x² – x + 1).
Então faremos aparecer x³ no polinômio P(x) = 100x100 + 99x99 +
x4 − 3x3 + 3x2 − 3x +2 x2 − 4x +3 10x −10
98x98 + 5x5 + 4x4 + 3x3 = 2x2 + x + 1.
10x −10 0 ← restos
P(x) 100 ( x 3 ) ⋅ x + 99 ( x 3 ) + 98 ( x 3 ) ⋅ x 2 + 5 ( x 3 ) ⋅ x 2 + 4 ( x 3 ) ⋅ x + 3x 3 + 2x 2 + x + 1
33 33 32
=

=
⇒ mdc(p,q)
P(x) =100 ((10x
) − 10) =(x −1) ( ) ( ) ( )
1 x 3 33 ⋅ x + 99 3 33 + 98 x 3 32 ⋅ x 2 + 5 x 3 ⋅ x 2 + 4 x 3 ⋅ x + 3x 3 + 2x 2 + x + 1 , como x³ ≡ -1 mod(x² – x + 1) então
R
10
MA

100 ( −1) ⋅ x + 99 ( −1) + 98 ( −1) ⋅ x 2 + 5 ( −1) ⋅ x 2 + 4 ( −1) ⋅ x + 3( −1) + 2x 2 + x + 1 ⇒


33 33 32
SO

vale notar que a divisão por 10 se faz necessária para que o mdc seja
( −1) ⋅ x + 99 ( −1) + 98 ( −1) ⋅ x 2 + 5 ( −1) ⋅ x 2 + 4 ( −1) ⋅ x + 3( −1) + 2x 2 + x + 1 ⇒
33 33 32
100unitário.
um polinômio
−100x − 99 + 98x 2 − 5x 2 − 4x − 3 + 2x 2 + x + 1= 95x 2 − 103x − 101 ⇒
T

O mínimo múltiplo comum entre polinômios é o polinômio mod (D(x) )


R 100x100 + 99x 99 + 98x 98 + 5x5 + 4x 4 + 3x 3 + 2x 2 + x + 1 ≡ 95x 2 − 103x − 101
E

unitário formado por todos os fatores que aparecem nos polinômios,


E

comuns ou não, elevados ao seu maior expoente, de forma que ele é IA ≡ 95x² – 103x – 101
F mod(D(x))
o polinômio de menor grau que é múltiplo de todos aqueles.
Todas as raízes dos polinômios são raízes do seu mmc. L DO PRO Como o grau de R(x) ainda está igual ao grau de D(x) devemos
dividir R(x) por D(x)
Exemplos:
P(x) = x(x – 1)2(x – 2)3 e Q (x) = x3(x – 1)(x – 3)2. 95x² – 103x – 101 x² + 1
mdc (P,Q) = x(x – 1)
mmc (P,Q) = x3(x – 1)2(x – 2)3(x – 3)2 –95x² –95 95

Logo R(x) = -95x² – 95 = –95(x² + 1) ⇒


CONGRUÊNCIA POLINOMIAL R(–1) = –95 ((–1)² + 1) = –95 · 2 = –190.
A congruência entre polinômios se dá da mesma forma dos
números inteiros. Todos os polinômios que ao serem divididos por um
polinômio D(x) gerarem um mesmo resto R(x) serão ditos congruentes EQUAÇÕES ALGÉBRICAS
e formarão um conjunto de polinômios. Sendo P(x) um polinômio em C, chama-se equação algébrica à
igualdade P(x) = 0. Portanto, as raízes da equação algébrica, são as
Exemplo: mesmas do polinômio P(x). O grau do polinômio, será também o grau
P(x) = x 3 − x 2 + 2x − 1 e G(x) = x 4 + x 2 + x são congruentes para o da equação.
mesmo divisor D(x) = x² + 1 pois ambos geram o mesmo resto R(x) = x Exemplo:
quando divididos por D(x).
3x 4 − 2x 3 + x + 1 =0 é uma equação do 4º grau.
P(x) = x 3 − x 2 + 2x − 1 = Q1(x).D(x) + R(x) = ( x − 1) ( x 2 + 1) + x
G(x) = x 4 + x 2 + x = Q2 (x).D(x) + R(x) = ( x )( x
2 2
+ 1) + x

PROMILITARES.COM.BR 189

PM_BOOK05_MAT.indb 189 01/11/2021 13:48:15


EQUAÇÕES POLINOMIAIS

RAÍZES DE UMA EQUAÇÃO x1 + x 2 + x 3 =−


b
POLINOMIAL a
c
Dada uma equação polinomial P(x) = 0, chama-se raiz da equação x1.x 2 + x1.x 3 + x 2.x 3 =
a
todo número que, substituído no lugar de x torna a sentença
d
verdadeira. x1.x 2.x 3 = −
a
Assim r é dito raiz de P(x) se e somente se P(r) = 0.
Chama-se conjunto-solução ou conjunto-verdade em C da
equação polinomial P(x) = 0 o conjunto S cujos elementos são as raízes Para uma equação do 4º grau, da forma ax4 + bx² + cx² + dx + e = 0,
complexas da equação (não esquecer que R ⊂ C). sendo as raízes iguais a x1, x2, x3 e x4, temos as seguintes relações de
Girard.
Exemplo b
x1 + x 2 + x 3 + x 4 =−
O conjunto-solução da equação polinomial x³ – 2x – x + 2 = 0 é a
S = {1,2,–1}. c
x1.x 2 + x1.x 3 + x1.x 4 + x 2.x 3 + x 2.x 4 + x 3.x 4 =
a
TEOREMA FUNDAMENTAL DA x1.x 2.x 3 + x1.x 2.x 4 + x1.x 3.x 4 + x 2.x 3.x 4 =−
d
ÁLGEBRA a
e
x1.x 2.x 3.x 4 =
Toda equação polinomial an .xn + an −1.xn −1 + an − 2.xn − 2 + … + a1.x + a0 = 0 a
admite n raízes complexas.

L D O P(x)P=DeaRmaneira
x O
geral sendo um polinômio
+ a x + ... + a x + a teremos
n n −1
FORMA FATORADA
I A em n
Todo polinômio de grau n, (n ≥ 1), pode ser decomposto S =
F + x =− a
x + x + ...E
n n −1 1 0

R fatorada de um
n −1
1 1 2 n
binômios de grau 1, onde essa forma se chama forma a n
polinômio e expõe todas as suas raízes. an − 2

S
TE

P ( x= S2= x1.x 2 + x 2.x 3 + ... + xn −1.xn=


) an ( x − r1 )( x − r2 )…( x − rn ) an

SO
MA

Exemplo: a0
Assim até P = x1.x 2..xn = ( −1)n
an

R
a) Fatorar o polinômio P ( x ) = 5x − 5x − 80x + 80 sabendo que
5 4

suas raízes são 1, –2, 2, –2i, 2i. O que importa é que sempre a soma das raízes tomadas uma a
Assim P(x) = 5(x – 1)(x + 2)(x – 2)(x + 2i)(x – 2i) a
uma (soma simples) será − n −1 enquanto o produto sempre será o
an
termo independente sobre o coeficiente do monômio de maior grau,
b) Qual o conjunto solução da equação 2(x – 2)³(x – 1)²(x – 3) = 0? com sinal de menos na fórmula quando for um polinômio de grau
R
De que grau é esse polinômio? ímpar e de sinal positivo na fórmula quando for um polinômio de grau
MA

O grau do polinômio é dado pela soma dos expoentes dos fatores, a


SO

assim 3 + 2 + 1 = 6. par, assim ( −1)n 0 .


an
2(x – 2)³(x – 1)²(x – 3) = 0 é equivalente a 2(x – 2)(x – 2)(x – 2)
(x – 1)(x – 1)(x – 3) = 0. ProBizu
T

R
Dessa forma igualando cada fator a 0 teremos as raízes
E

Sempre a partir da soma das raízes uma a uma as fórmulas vão


E

{1,1,2,2,2,3} e conjunto-solução {1,2,3}.


IA F
alternando sinal, – , +, – , ...

L D O Exemplo: O
MULTIPLICIDADE DE RAÍZES PR
Quando temos an ( x − r1 ) . ( x − r2 ) …( x − rn )
α β γ
=
0 temos que r é
1 Sendo o polinômio P(x) = x³ + 6x² + 11x + 6 cujas raízes são -1,
raiz um número α de vezes, que r2 é raiz um número β de vezes e -2 e -3, então temos:
assim para todos os fatores que possuem expoentes.
6
No exemplo visto anteriormente 2(x – 2)³(x – 1)²(x – 3) = 0, a raiz s1 =( −1) + ( −2) + ( −3) = ( − 1)1 ⋅ =−6
1
1 é uma raiz de multiplicidade 2, a raiz 2 é uma raiz de multiplicidade
11
3 enquanto a raiz 3 é apenas de multiplicidade 1. s2 = ( −1)( −2) + ( −1)( −3) + ( −2)( −3) = ( −1)2 = 11
1
6
s3 =P =( −1)( −2)( −3) =( −1)3 =−6
RELAÇÕES DE GIRARD 1
São as relações existentes entre os coeficientes e as raízes de uma
equação algébrica.
Para uma equação do 2º grau, da forma ax² + bx + c = 0, já RAÍZES COMPLEXAS
conhecemos as seguintes relações entre os coeficientes e as raízes x1 Se uma equação polinomial de coeficientes REAIS admite como
e x2. raiz o número complexo a + bi então, obrigatoriamente, a equação
b polinomial também admite como raiz o conjugado a - bi.
c
x1 + x 2 =
− e x1.x 2 = . Assim sempre que um número complexo z é raiz então z também é.
a a
Para uma equação do 3º grau, da forma ax³ + bx² + cx + d = 0, Da mesma forma se z é uma raiz de multiplicidade n então z
sendo x1, x2 e x3 as raízes, temos as seguintes relações de Girard. também será raiz de mesma multiplicidade n.

190 PROMILITARES.COM.BR

PM_BOOK05_MAT.indb 190 01/11/2021 13:48:16


EQUAÇÕES POLINOMIAIS

Dessa forma podemos concluir que raízes que não são números Exemplo:
reais puros sempre são raízes em pares de uma equação polinomial P(x). A equação x³ – x = 0 possui 3 raízes a saber: x = 0 ou x = 1 ou
Assim podemos também enunciar que uma equação polinomial de x = –1. Dizemos então que o conjunto verdade ou conjunto solução
grau ímpar sempre admite pelo menos uma raiz real pura. da equação dada é S = {0,1,–1}.

II. Se b for raiz de P(x) = 0, então P(x) é divisível por x – b.


TEOREMA DAS RAÍZES RACIONAIS Esta propriedade é muito importante para abaixar o grau de uma
O teorema das raízes racionais, também conhecido por teorema equação, o que se consegue dividindo P(x) por x - b, aplicando Briot-
do teste das raízes racionais, estabelece uma condição sobre as Ruffini.
soluções racionais de uma equação polinomial.
III. Se o número complexo a + bi for raiz de P(x) = 0, então o
Sendo an .xn + an −1.xn −1 + an − 2.xn − 2 + … + a1.x + a0 = 0 uma equação conjugado a – bi também será raiz
polinomial de coeficientes inteiros o teorema estabelece que se a0 e an
são diferentes de zero, então, cada solução racional x, quando escrita Exemplo:
p Qual o grau mínimo da equação P(x) = 0, sabendo-se que três de
como uma fração irredutível x = .
q suas raízes são os números 5, 3 + 2i e 4 – 3i. Ora, pela propriedade
P3, os complexos conjugados 3 - 2i e 4 + 3i são também raízes. Logo,
Se a equação polinomial admite raízes racionais elas são frações pela propriedade I, concluímos que o grau mínimo de P(x) é igual a 5,
divisores inteiros de a0 ou seja, P(x) possui no mínimo 5 raízes.
dentre todas as possibilidades de .
divisores inteiros de a1
Exemplo: IV. Se a equação P(x) = 0 possuir k raízes iguais a m então

D O PExemplo:
Para a equação polinomial 2x³ + x – 1 = 0 se utilizarmos o teorema
L
veremos que an = 2 e a0 = –1, assim os divisores de an são {–2,–1,1,2} RO
dizemos que m é uma raiz de grau de multiplicidade k.

I A
e de a0 são {–1,1}. Assim todas as frações possíveis que podem ser FE
a) A equação (x – 4)10 = 0 possui 10 raízes iguais a 4. Portanto 4 é
formadas com esses 2 conjuntos da forma
1 1
R
divisores inteiros de a0
divisores inteiros de a1
raiz décupla ou de multiplicidade 10.
b) a equação x³ = 0, possui três raízes iguais a 0 ou seja três raízes

S
TE

são −1, − , e 1. Onde testando os 4 candidatos verificamos que


2 2 nulas com ordem de multiplicidade 3 (raízes triplas).

SO
nenhum é raiz e concluímos dessa forma que 2x³ + x – 1 = 0 não c) A equação do segundo grau x² – 8x + 16 = 0, possui duas raízes
MA

possui nenhuma raiz racional. reais iguais a 4, (x’ = x” = 4). Dizemos então que 4 é uma raiz
dupla ou de ordem de multiplicidade dois.

R
ProBizu
V. Toda equação de termo independente nulo, admite um
I. Como consequência desse teorema temos que número de raízes nulas igual ao menor expoente da variável.
Se a soma dos coeficientes de uma equação algébrica Exemplo
P(x) = 0 for nula, então a unidade é raiz da equação (1 é raiz).
a) A equação 3x5 + 4x 2 =
0 possui duas raízes nulas.
R
Exemplo:
b) A equação x100 + x12 =
0 , possui 100 raízes, das quais 12 são nulas!
MA

1 é raiz de 40x5 −10x 3 +10x − 40 =


0 , pois a soma dos coeficientes
SO

é igual a zero.
Se x1, x2, x3, ..., xn são raízes da equação an · xn + an-1 · xn-1 +
an-2 · xn-2 + ... + a1 · x + a0 = 0 , então ela pode ser escrita na
T

II. Todo teste de candidatos a raízes do teorema das raízes


S

forma fatorada an(x – x1)(x – x2) · (x – xn)


R
racionais deve ser feito no dispositivo prático de Briot-Ruffini
E

pois faz com que o trabalho seja reduzido.


IA
Exemplo:
F
L DO PRO
Se –1, 2 e 53 são as raízes de uma equação do 3º grau, então
podemos escrever (x + 1)·(x – 2)·(x – 53) = 0, que desenvolvida fica
x³ – 54x² + 51x + 106 = 0.
TEOREMA DE BOLZANO
Sejam P(x) = 0 uma equação polinomial com coeficientes reais e Exercício Resolvido
]a,b[ um intervalo real aberto.
01. Uma fábrica utiliza dois tanques para armazenar combustível.
I. Se P(a) e P(b) tem os mesmos sinais então existe um número par
Os níveis de combustível, H1 e H2, em cada tanque, são dados
de raízes reais ou não existem raízes reais da equação no intervalo
]a,b[. pelas expressões: H1(t) = 150t3 − 190t + 30 e H2 (t) = 50t3 + 35t + 30,
sendo t o tempo em hora. O nível de combustível de um tanque é
II. Se P(a) e P(b) tem sinais contrários então existe um número ímpar igual ao do outro no instante inicial (t = 0) e, também, no instante:
de raízes reais da equação no intervalo ]a,b[.
a) t = 0,5 h
b) t = 1,0 h
RAÍZES COMUNS c) t = 1,5 h
As raízes comuns de P(x) e Q(x) são as raízes do mdc (P(x),Q(x))
d) t = 2,0 h
Algumas propriedades importantes que sempre são válidas e) t = 2,5 h
estarem na ponta da língua:
I. Toda equação algébrica de grau n possui exatamente n Resolução: C
raízes. H= H2 (t) ⇔ 150t 3 − 190t + 30 = 50t 3 + 35t + 30 ⇒
1(t)

0 ⇒ t (100t 2 − 225) =
100t 3 − 225t = 0 ⇒ t = 0 ou t = 1,5.

PROMILITARES.COM.BR 191

MAT144 - EQUACOES POLINOMIAIS.indd 191 01/11/2021 15:43:25


EQUAÇÕES POLINOMIAIS

Exercício Resolvido Exercício Resolvido

02. Determine todos os valores possíveis de m ∈ , de modo que 05. Os zeros do polinômio a seguir formam uma P.A.
o polinômio p(x) = x³ + (m – 1)x² + (4 – m)x – 4 tenha três raízes p(x) =x 3 − 12x 2 + 44x − 48
distintas, sendo x = 1 a única raiz real. O conjunto solução da equação p(x) = 0 pode ser descrito por:
a) –4 < m < 4 a) {0, 4, 8}
b) m < 4 b) {2, 4, 6}
c) m>4 c) {–1, 4, 9}
d) m < -4 d) {–2, –4, –6}

Resolução: A Resolução: B
p(x) = x 3 + (m − 1) x 2 + ( 4 − m ) x − 4 então Raízes: α – r, α, α + r ⇒ Soma = 3α = 12 ⇒ α = 4.
x = 1 ⇒ 1+ m − 1+ 4 − m − 4 = 0 ⇒ 0 = 0.

x 3 − 12x 2 + 44x − 48 = ( x − 4 ) ( x 2 − 8x + 12) ⇒ raízes: 4, 2, 6.


p(x) =( x − 1) ( x + mx + 4 )
2

x² + mx + 4 possui 2 raízes complexas distintas


⇒ ∆ < 0 ⇒ m2 − 16 < 0 ⇒ −4 < m < 4 .
L DO PRO EXERCÍCIOS DE

I A FE
FIXAÇÃO
Exercício Resolvido R

S
TE

03. O polinômio p(x) =x − 2x + 5x − 8x + 4 também pode ser


4 3 2

SO
01. (EEAR) Seja a equação x³ – 5x² + 7x – 3 = 0. Usando as relações de
( x − 1)
escrito sob a forma: p(x) =
n
(x 2
+ s ) , n ∈ ℕ e s ∈ ℝ. O valor Girard, pode-se encontrar como soma das raízes o valor
MA

de n + s é:
a) 12.

R
a) 1
b) 7.
b) 4
c) 5.
c) 0
d) 2.
d) 6
e) 2 02. (EEAR) A equação (x² + 3)(x – 2)(x + 1) = 0 tem ____ raízes reais.
R
a) 3
MA

Resolução: D
SO

b) 2
x 4 − 2x 3 + 5x 2 − 8x + 4 ≡ ( x − 1)
n
(x 2
+ s) ⇒ n =
2; c) 1
( x − 1)
2
(x 2
+ s ) ≡ ( x 2 − 2x + 1) ( x 2 + s ) d) 0
T

≡ x 4 − 2x 3 + x 2 (1 + s ) − 2sx + s . Então, s = 4 ⇒ n + s = 2 + 4 = 6.
R
E

03. (EEAR) Dado P(x) = x³ – (2m + 4)x² + 9x + 13, o valor de m, para


E

IA que 3i seja raiz de P(x), éF


Exercício Resolvido L D Oa) P R O −
49
18
.
04. Entre as equações abaixo, a que tem o número complexo 23
2 + 3i como uma de suas raízes é: b) − .
18
a) x 2 + 13x + 1 =0 25
c) − .
b) x 2 − 4x − 5 =0 6

x 3 − 4x 2 + 13x =
23
c) 0 d) .
18
d) x 4 + 81 =
0
04. (EEAR) Se o polinômio x³ – 9x² + 14x + 24 tem uma raiz igual a 6,
e) x 4 + x 2 + 13 =
0 decompondo-o em fatores, obtém-se

Resolução: C
a) ( x –6 )( x – 4 )( x +1) .
2 + 3i e 2 – 3i são raízes; Soma das duas raízes: S’ = 4. Produto das b) ( x –6 )( x + 4 )( x –1) .
duas raízes: P’ = 13. c) ( x + 6 )( x – 4 )( x +1) .
a) Não; S’ = –13 e P’ = 1 d) ( x + 6 )( x + 4 )( x –1) .
b) Não; S’ = 4 e P’ = -5
Sim; x ( x − 4x + 13) = 0 ⇒ S' = 4 e P’ = 13
2
c)
d) Não; P’ = 81
e) Não; Raízes: ± −1 + i 51 , ± −1 − i 51

192 PROMILITARES.COM.BR

PM_BOOK05_MAT.indb 192 01/11/2021 13:48:18


EQUAÇÕES POLINOMIAIS

05. (EEAR) A equação, cujas raízes são –2, +2, –5 e +5, é x4 + ax² + b = 0. 02. (EEAR) Seja r a maior raiz da equação x(x + 2)(x – 1)³ = 0. Se m é
O valor de |a + b| é a multiplicidade de r, então r·m é igual a
a) 2. a) 6.
b) 3. b) 5.
c) 4. c) 4.
d) 5. d) 3.

06. (EEAR) Para que o polinômio P(x) = 2x4 + x³ – 6x² + αx + β 03. (EEAR) Seja a equação polinomial x³ + bx² + cx + 18 = 0. Se –2
tenha como raiz dupla o número 1, os valores de α e β devem ser, e 3 são suas raízes, sendo que a raiz 3 tem multiplicidade 2, o valor
respectivamente, de “b” é
a) 1 e 2. a) 8
b) 2 e 1. b) 6
c) –2 e 1. c) -3
d) 1 e –2. d) -4

07. (EEAR) Uma equação polinomial de coeficientes reais admite como 04. (EEAR) Dada a equação 3x³ + 2x² – x + 3 = 0 e sabendo que a, b
raízes os números 3 + i, 7 e 2 – 3i. Essa equação tem, no mínimo, grau e c são raízes dessa equação, o valor do produto a·b·c é
a) 6. a) 1
b) 5. b) –1
c) 4.
L D O c) P1 R O
IA FE
d) 3. 3
1

as outras raízes são


R x – 5x² – 36 = 0, d) − 3
08. (EEAR) Se 3 e –3 são duas das raízes da equação 4

S
TE

a) 3i e 2i. 05. (EEAR) Resolvendo-se a equação x³ – 5x² + 6x = 0, é correto

SO
afirmar que sua maior raiz é ____.
b) 2i e -2i.
MA

a) 2
c) –i e –3i.

R
b) 3
d) 3i e –3i.
c) 4

09. (EEAR) Dada a equação x ⋅ ( x – 2) ⋅ ( x + 2x +1) =


4 2 d) 6
0 , o número de
elementos reais de seu conjunto solução é
06. (ESA) Uma equação polinomial do 3° grau que admite as raízes
a) 2.
1 R
b) 3. –1, − e 2 é
2
MA

SO

c) 4.
a) x 3 − 2x 2 − 5x − 2 =0.
d) 5.
b) 2x 3 − x 2 − 5x + 2 =0.
T

10. (ESA) O conjunto solução da equação x³ – 2x² – 5x + 6 = 0 é c) 2x 3 − x 2 + 5x − 2 =0.


R
E

F
a) S = {-3; -1; 2}
b) S = {–0,5; -3; 4} IA d)
O
2x 3 − x 2 − 2x − 2 =0.

c) S = {–3; 1; 2} L DO PR e) 2x 3 − x 2 − 5x − 2 =0.

d) S = {–2; 1; 3} 07. (EEAR) Sabendo que 1 é raiz dupla da equação x – 4x³ + 5x² – 2x = 0,
4

e) S = {0,5; 3; 4} a maior das outras duas raízes é um número múltiplo de


a) 2.
EXERCÍCIOS DE b) 3.

TREINAMENTO
c) 5.
d) 7.

08. (EEAR) Seja a equação polinomial 2x³ + 4x² – 2x + 4 = 0. Se S e P


01. (EEAR) Da equação x³ + 11x² + kx + 36 = 0, sabe-se que o produto são, respectivamente, a soma e o produto de suas raízes, então
de duas de suas raízes é 18. Assim, o valor de k é a) S = P.
a) 6 b) S = 2P.
b) 8 c) S = 2 e P = – 4.
c) 18 d) S = – 2 e P = 4.
d) 36

PROMILITARES.COM.BR 193

PM_BOOK05_MAT.indb 193 01/11/2021 13:48:19


EQUAÇÕES POLINOMIAIS

09. (EEAR) Uma equação polinomial de coeficientes reais admite 07. Um polinômio q(x) = –4 + 5x + 3x² – 5x³ + x4 possui o 1 como uma
como raízes os números –2, 0, 2 e 1 + i. O menor grau que essa raiz dupla. Assim, este polinômio possui
equação pode ter é a) duas raízes complexas e duas reais positivas.
a) 6. b) 5. c) 4. d) 3. b) um outro par de raiz dupla.
c) uma raiz complexa, duas reais positivas e uma real negativa.
10. (EEAR) Seja A = {−2, −1, 1, 2} o conjunto formado pelas raízes
de um polinômio P(x) do 4° grau. Se o coeficiente do termo de maior d) as quatro raízes maiores que –2.
grau de P(x) é 1, então o termo independente é e) uma, de suas quatro raízes, nula.
a) 3. b) 4. c) 5. d) 6.
08. Em relação à função f(x) = x5 + 4x³ + 2x + 3 pode-se afirmar:
a) não tem raízes reais.
EXERCÍCIOS DE b) tem cinco raízes reais.

COMBATE c) tem três raízes reais e duas complexas.


d) tem uma raiz real e quatro complexas.
e) tem duas raízes reais e três complexas.

01. (EEAR) Se a maior das raízes da equação x³ – 6x² + 11x – 6 = 0 é 09. (AFA) O polinômio P(x) = x4 – 75x² + 250x tem uma raiz dupla.
igual à soma das outras duas, então seu valor é divisor de
Em relação à P(x) é correto afirmar que
a) 10.
a) apenas uma de suas raízes é negativa.

D O b)c) PatrêssuaRderaizO
b) 16.
dupla é negativa.
c) 18.
L
IA
suas raízes são negativas.
d) nenhuma deFsuas raízes é negativa.
d) 20.

R
02. (EEAR) Uma das raízes da equação 2x³ + x² – 7x – 6 = 0 é x = 2.
E
1
10. (AFA) As raízes da equação algébrica 2x³ – ax² + bx + 54 = 0

S
TE

Pode se afirmar que:


a
formam uma progressão geométrica. Se a, b ∈ , b ≠ 0, então

SO
a) as outras raízes são números imaginários puros.
é igual a b
MA

b) as outras raízes são – 3 e – 2.


c) só uma das outras raízes é real. 2
a)

R
3
d) as outras raízes estão entre – 2 e 0.
b) 3
03. Na equação polinomial x³ – 2x² – 5x + k = 0, uma das raízes é 3
c) −
–2. Sabendo que a soma das outras duas raízes é raiz da equação 2
x² + mx + 4 = 0, o valor de m + k é 1
d) − R
a) 1. 3
MA

b) 0.
SO

c) –2.
d) –1.
RESOLUÇÃO EM VÍDEO
T

e) –3.
R
E

F
Abra o ProApp, leia o QR Code, assista à resolução

IA
04. (EEAR) Se 3, 5 e –2, são as raízes da equação 4(x – a)(x – b)(x – 5) = 0 de cada exercício e AVANCE NOS ESTUDOS!

o valor de a + b é
a) 0 b) 1 c) 2 d) 3
L DO PRO
GABARITO
05. (BARRO BRANCO) Considere o polinômio P(x) = x4 – 2x3 – 3x² + 8x – 4.
Sabendo-se que ele é divisível por x – 1 mais de uma vez, a soma entre EXERCÍCIOS DE FIXAÇÃO
a maior e a menor raízes da equação P(x) = 0 é igual a 01. C 04. A 07. B 10. D
a) 4. d) 1. 02. B 05. B 08. B
b) 3. e) 0. 03. A 06. A 09. B
c) 2. EXERCÍCIOS DE TREINAMENTO
01. D 04. B 07. A 10. B
06. Sabendo-se que x = 2 é uma raiz de multiplicidade 3 do polinômio
02. D 05. B 08. A
x³ + ax² + bx + c, os valores de a, b e c são
03. D 06. E 09. B
a) a = –6, b = –12 e c = –8
EXERCÍCIOS DE COMBATE
b) a = –6, b = –12 e c = –8
01. C 04. B 07. D 10. D
c) a = –16, b = 12 e c = 8
02. D 05. E 08. D
d) a = 6, b = –12 e c = 8
03. A 06. E 09. A
e) a = –6, b = 12 e c = –8

194 PROMILITARES.COM.BR

PM_BOOK05_MAT.indb 194 01/11/2021 13:48:19


GEOMETRIA ANALÍTICA:
COORDENADAS NO PLANO E DISTÂNCIA

O PLANO CARTESIANO IV. Todos os pontos da bissetriz dos quadrantes pares possuem
abscissas e ordenadas opostas e vice-versa.
B ∈ bp ⇔ B = (b, −b)

bi ⇒ bissetriz dos quadrantes ímpares


bp ⇒ bissetriz dos quadrantes pares

L D O PONTO
P R OMÉDIO E DISTÂNCIA
IA FE
Vamos começar nossos estudos em apenas uma dimensão.

S
TE

SO
Vamos primeiramente definir distância em apenas uma dimensão.
MA

A cada ponto P do plano cartesiano corresponde um par ordenado


(x,y) de números reais e escrevemos P(x,y) para indicar este ponto.

R
Dois eixos orientados (x e y) são dispostos ortogonalmente, dando
a origem à divisão do plano em quatro partes, cada uma denominada
Em um eixo, a distância entre xA e xB será xB – xA, o que chamamos
quadrante. Os quatro quadrantes são numerados no sentido anti-
de ponto final menos o ponto inicial. Sendo assim dAB = xB – xA = ∆x.
horário, e os eixos e a intersecção entre eles são denominados,
respectivamente, eixo das abscissas (x), eixo das ordenadas (y) e Dessa forma podemos encontrar o ponto médio de um segmento.
origem (0) do sistema de coordenadas cartesianas. R
A reta que divide ao meio os quadrantes ímpares é chamada de
MA

bissetriz dos quadrantes ímpares e a que divide os quadrantes


SO

pares é a bissetriz dos quadrantes pares.

xB − x A
Teremos que metade do segmento é . Dessa
T

Qualquer ponto P no plano cartesiano terá coordenadas .


S

2
R
E

Se P ∈ ao 1º quadrante: x > 0 e y > 0. IA F


forma a coordenada de xM será xA acrescida de
xB − x A
, assim

L D O x = Px +Rx −2O
2
Se P ∈ ao 2º quadrante: x < 0 e y > 0. x 2x + xB − x A x A + xB .
M =
A
B A A
=
Se P ∈ ao 3º quadrante: x < 0 e y < 0. 2 2
Se P ∈ ao 4º quadrante: x > 0 e y < 0.
Assim para 2 dimensões, para o plano xy, basta fazermos o ponto
médio em relação ao eixo x e o ponto médio em relação ao eixo y.
A equação de reta da bissetriz dos quadrantes ímpares é x = y
enquanto a equação da bissetriz dos quadrantes pares é x = –y.

Observações:

I. Os pontos pertencentes ao eixo 0x possuem ordenadas nulas
P ∈ OX ⇔ P = ( x, 0)


II. Os pontos pertencentes ao eixo 0y possuem abscissas nulas.
P ∈ OY ⇔ P = (0, y )

III. Todos os pontos da bissetriz dos quadrantes ímpares possuem Assim o ponto médio M terá 2 coordenadas xM e yM.
abscissas iguais à ordenada e vice-versa.
 x + xB y A + yB 
A ∈ bi ⇔ A = (a, a) M ( xM , yM ) ⇒ M  A , 
 2 2 

PROMILITARES.COM.BR 195

PM_BOOK05_MAT.indb 195 01/11/2021 13:48:24


GEOMETRIA ANALÍTICA: COORDENADAS NO PLANO E DISTÂNCIA

Sendo M(xM,yM) o ponto médio do segmento cujas extremidades x A + xB + xC


são A(xA,yA) e B(xB,yB), tem-se: xG =
3
x A + xB y + yB e
xM = e yM = A
2 2 y A + yB + yC
yG =
3
Para calcularmos a distância entre 2 pontos no plano xy basta
observarmos a figura abaixo. Assim, o baricentro do triângulo ABC será:

 x + xB + xC y A + yB + yC 
G A , 
 3 3

ÁREA DO TRIÂNGULO

L DO PRO
I A FE
R
Temos a formação de um triângulo retângulo onde a hipotenusa
é a distância entre os pontos A e B e os catetos são as distâncias em

S
TE

apenas uma dimensão, xB – xA = ∆x em relação ao eixo x e yB – yA = ∆y

SO
em relação ao eixo .
MA

Sendo d a distância entre os pontos A(xA,yA) e B(xB,yB),

R
aplicando o teorema de Pitágoras teremos d = ∆ x 2 + ∆ y 2 , onde
∆ x = xB − x A e ∆ y = yB − y A . xA yA 1
1
A tri = . xB yB 1
dAB = ( xB − x A )
2
+ ( yB − y A )
2
2
xC yC 1
R
BARICENTRO DE UM TRIÂNGULO
MA

CONDIÇÃO DE ALINHAMENTO DE
SO

Baricentro é o ponto de encontro de todas as medianas de um


triângulo. Lembre-se que o baricentro sempre divide a mediana na TRÊS PONTOS
razão de 2 : 1. A distância do vértice ao baricentro (G) é o dobro da Podemos pensar que se temos 3 pontos alinhados esses pontos
T

distância do baricentro ao lado. não podem formar um triângulo, logo se não podem formar
R
E

F
um triângulo não há área (s = 0). Dessa forma devemos igualar o
IA determinante a0.

L DO PRO

ÁREA DE UM POLÍGONO QUALQUER


Podemos encontrar a área do triângulo a partir das coordenadas
dos seus 3 vértices através do seguinte algoritmo.

xA yA
1 xB yB
SABC =
2 xC yC
xA yA

196 PROMILITARES.COM.BR

PM_BOOK05_MAT.indb 196 01/11/2021 13:48:30


GEOMETRIA ANALÍTICA: COORDENADAS NO PLANO E DISTÂNCIA

Perceba que a última linha será a coordenada do mesmo ponto Exercício Resolvido
utilizado na primeira linha. Assim para calcularmos a área de um
triângulo teremos 4 linhas, um quadrilátero 5 linhas, ..., e assim para 02. Considere os pontos A(2,8) e B(8,0). A distância entre eles é de
qualquer quantidade de vértices do polígono.
a) 14
b) 3 2
c) 3 7
d) 10

Resolução: D
A distância d entre os pontos A e B será dada por:
d = (2 − 8)2 + (8 − 0)2 = 36 + 64 = 100 = 10

Exercício Resolvido

03. O triângulo ABC formado pelos pontos A(7,3), B(4,3) e


C(–4,–2) é
xA yA
a) escaleno
D O b)P isósceles
xB yB
SABC
1
= xC yC
L RO
2
xD yD
I A c) equiângulo
F
d) obtusângulo E
xA yA
R

S
TE

O processo se dá de forma semelhante ao cálculo do determinante Resolução: A

SO
Calculando os quadrados das medidas dos lados do triângulo ABC,
xA yA
MA

encontramos d²(A,B) = (–4 – 7)² + (3 – 3)² = 121, d²(A,C) = (–4 – 7)²


1 xB yB 1
SABC = = ( x y + x B y C + x Cy A − y A x B − y B x C − y Cx A ) + (–2 – 3)² = 146 e d²(B,C) = (–4 + 4)² + (–2 – 3) = 25.

R
2 xC yC 2 A B
Portanto, sendo d2 ( A, C) = d2 ( A, B) + d2 (B, C), podemos concluir
xA yA que o triângulo ABC é retângulo escaleno.

Exemplo: Calcular a área do quadrilátero A(2,–1), B(4,5), C(3,–3) Exercício Resolvido


e (–4,0). R
04. No plano cartesiano, a reta s : 4x – 3y + 12 = 0 intersecta o
MA

2 −1
eixo das abscissas no ponto A e o eixo das ordenadas no ponto B.
SO

1 4 5 1 Nessas condições, qual é a distância entre os pontos A e B?


SABC = = (2.5 + 4.( −3) + 3.0 − ( −1).4 − 5.3 − ( −3)( −4 ))
2 3 −3 2 a) 5
−4 0
T

b) 5
1 1
R
−25 25
E

= (10 − 12 + 4 − 15 − 12) = .( −25) ⇒ SABC =


E

SABC
2 2 2
=
2
IA
c) 2 2
F
L D O e) P2 R O
d) 2

Exercício Resolvido

01. Seja ABC um triângulo tal que A(1,1), B(3,4) e C(5,3) O ponto Resolução: A
_____ é o baricentro desse triângulo.
Intersecção com o eixo x(y = 0)
a) (2,1).
4 x − 3 ⋅ 0 + 12 = 0 ⇒ 4 x = −12 ⇒ x = −3 ⇒ A( −3, 0)
b) (3,3).
c) (1,3), Intersecção com o eixo y(x = 0)
d) (3,1). 4 ⋅ 0 − 3y + 12 = 0 ⇒ −3y = −12 ⇒ y = 4 ⇒ B(0, 4 )

Logo, a distância entre os pontos A e B será dada por:


Resolução: D d = (0 − ( −3))2 + ( 4 − 0)2 = 25 = 5
Sabendo que as coordenadas do baricentro correspondem à média
aritmética simples das coordenadas dos vértices do triângulo, vem
 1 + 3 + 5 1 − 1 + 3

,  = (3, 1).
3 3 

PROMILITARES.COM.BR 197

PM_BOOK05_MAT.indb 197 01/11/2021 13:48:37


GEOMETRIA ANALÍTICA: COORDENADAS NO PLANO E DISTÂNCIA

Exercício Resolvido 05. (EEAR) A área do triângulo cujos vértices são os pontos A, B e C
é, em unidades de área,
05. Os pontos (0,–1), (1,2) e (3,k) do plano são colineares. O valor
de k é igual a a) 4.
a) 0 b) 3.
b) 2 c) 2.
c) –2 d) 1.
d) 8
e) –8

06. (EEAR) Os pontos A(3,5), B(4,3), C(1,0) e D(0,4) são vértices de um


Resolução: D quadrilátero ABCD. A área desse quadrilátero é
Do enunciado, temos: 15
a)
2
7
b)
2
c) 11.
d) 15.

L D O 07.
P(EEAR)
RN (3,−
M (1,1),
O baricentro de um triângulo, cujos vértices são os pontos
O 4) e P (− 5,2), tem coordenadas cuja soma é

I A a) 2. FE
R b) 1.
= =
mr m mAC 2

S
TE

 
AB c) − .
3

SO
Então,
MA

1
2 − ( −1) k − ( −1) 3 k +1 d) − .
= ⇒ = ⇒ 3 ⋅ 3 = k + 1⇒ k = 8 3
1− 0 3−0

R
1 3
08. (EEAR) Sejam A( −3, 3), B(3, 1), C(5, − 3) e D(–1,–2) vértices de um
quadrilátero convexo. A medida de uma de suas diagonais é
EXERCÍCIOS DE a) 15 c) 12

FIXAÇÃO
b) 13 d) 10
R
MA

09. (EEAR) Seja ABC um triângulo tal que A(1,1), B(3,–1) e C(5,3) O
SO

ponto _____ é o baricentro desse triângulo.


01. (EEAR) Sejam os pontos D (k,–3), E (2,t) e F (–1,1). Se F divide DE a) (2,1).
em duas partes iguais, então os números k e t são tais que a soma b) (3,3).
T

deles é
R c) (1,3).
E

a) – 1. c) 1.
IA F
d) (3,1).
b) 0. d) 2.
L e O10. (EEAR) O
02. (EEAR) O baricentro do triângulo de vértices A(–5,6), B(–1,–4)D P RO triângulo ABC formado pelos pontos A(7,3), B(4,3) e
C(–4,–2) é
C(3,2) é o ponto
a) escaleno
 7 3  7 4 b) isósceles
a)  ,  c)  , 
4 2 4 3
c) equiângulo
 3  4 d) obtusângulo
b)  −1,  d)  −1, 
2 3

EXERCÍCIOS DE

TREINAMENTO
03. (EEAR) Seja um ponto Q, de ordenada –3, equidistante dos pontos
A(0,1) e B(2,3). O produto das coordenadas do ponto Q é:
a) 3. c) 12.
b) –6. d) –18.
01. (EEAR) Considere os pontos A(2,8) e B(8,0). A distância entre eles
04. (EEAR) Em um plano cartesiano desenhado no chão, uma formiga, é de
andando em linha reta, se deslocou do ponto A(2,–1) para o ponto
a) 14
B(–1,3), e depois para o ponto C(2,3). Se cada unidade deste plano
representa 1 cm, então a distância percorrida pela formiga, em cm, foi b) 3 2
a) 4. c) 10. c) 3 7
b) 8. d) 12. d) 10

198 PROMILITARES.COM.BR

PM_BOOK05_MAT.indb 198 01/11/2021 13:48:44


GEOMETRIA ANALÍTICA: COORDENADAS NO PLANO E DISTÂNCIA

02. (EEAR) Considere os segmentos de retas AB e CD, onde A(0,10), EXERCÍCIOS DE


B(2,12), C(–2,3) e D(4,3). O segmento MN, determinado pelos
pontos médios dos segmentos AB e CD é dado pelos pontos M e N,
pertencentes respectivamente a AB e a CD. Assinale a alternativa que
COMBATE
corresponde corretamente a esses pontos.
1  01. (EEAR) O valor de a para que os pontos A (–1,3 – a ), B (3,a + 1) e
a) M  , 1 e N(–1,3)
2  C(0,–1) sejam colineares é um número real
b) M(–2, 10) eN(–1,3) a) primo
c) M(1,–2) e N(1,3) b) menor que 1.
d) M(1,11) e N(1,3) c) positivo e par.
d) compreendido entre 2 e 5.
03. (EEAR) O triângulo determinado pelos pontos A(–1,–3), B(2,1) e
C(4,3) tem área igual a 02. (EEAR) Existe uma reta passando pelos pontos (1,4), (t,5) e (–1,t).
a) 1 c) 3 A soma dos possíveis valores de t é
b) 2 d) 6 a) 3. c) 5.
b) 4. d) 6.
04. (EEAR) O ponto M é o ponto de intersecção das diagonais AC e
BD de um quadrilátero ABCD. Sendo A(0,0), B(3,0), C(4,2) e D(0,5) 03. (EEAR) Se M(a,b) é o ponto médio do segmento de extremidades
as coordenadas dos vértices do quadrilátero, as coordenadas do ponto A(1,–2) e B(5,12), então é correto afirmar que
M são
D O a)b) Paa eeRbb são
são pares. c) a é par e b é primo.
a)
 15 30 
 ,  c)
 30 15 
 , 
A L O primos. d) a é primo e b é par.
13 13 13 13
I F E ABCD, A(–2,3) e B(0,2) são vértices de um
b)
 180 90 
 , 
13 13 
d) 
 7
,
7
R
 30 15 

04. (CFT) Num quadrado
dos lados. Assim, o lado desse quadrado mede ___.

S
TE

a) 2 c) 3

SO
b) 5 d) 2
05. (EEAR) Se um ponto móvel se deslocar, em linha reta, do ponto
MA

A(0,0) para o ponto B(4,3) e, em seguida, para o ponto C(7,7), então


05. (EEAR) Sejam os pontos A(x,1), M(1,2) e B(3,y). Se M é ponto

R
ele percorre uma distância de ___________ unidades de comprimento.
médio de AB, então x ⋅ y é igual a
a) 10 c) 8
a) –3. c) 1.
b) 9 d) 7
b) –1. d) 3.
06. (EEAR) Para que os pontos A(x,3), B(–2x,0) e C(1,1) sejam
colineares, é necessário que x seja 06. (EEAR) Se a distância entre A(2 3 ,y) e B(4 3 ,1) é 4, o valor de y
R
a) –2 c) 2 pode ser
MA

SO

b) –1 d) 3 a) 1. c) –1.
b) 0. d) –2.
07. (EEAR) Sejam A(–3,3), B(3,1), C(5,–3) e D(–1,–2) vértices de um
T

quadrilátero convexo. A medida de uma de suas diagonais é 07. (EEAR) Considerando que os pontos A(–2,6), B(2,4) e C(x,3) estão
R
E

alinhados, ou seja, são pontos de uma mesma reta, o valor de x é


E

a) 15 c) 12
IA F
O
b) 13 d) 10 a) 1. c) 3.
L o O PR
08. (EEAR) Se os pontos A(a,2), B(b,3) e C(–3,0) estão alinhados,D
b) 2. d) 4.

valor de 3a – 2b é 08. (EEAR) Para que os pontos A(2,0), B(a,1) e C(a + 1,2) estejam
a) 3 c) –3 alinhados, é necessário que o valor de a seja
b) 5 d) –5 a) 5. c) 3.
b) 4. d) 2.
09. (EEAR) Seja ABC um triângulo tal que A(1,1), B(3,–1) e C(5,3). O
ponto ______ é o baricentro desse triângulo. 09. (EEAR) Seja um triângulo ABC, tal que A(1,3), B(9,9), AC = 8 e
a) (2,1) BC = 5. Sendo assim, o perímetro desse triângulo é
b) (3,3) a) 19 c) 23
c) (1,3) b) 20 d) 26
d) (3,1)
10. (ESA) Os pontos M(–3,1) e P(1,–1) são equidistantes do ponto
S(2,b). Desta forma, pode-se afirmar que b é um número:
10. (EEAR) O triângulo ABC formado pelos pontos A(7,3), B(–4,3) e
C(–4,–2) é a) primo
a) escaleno b) múltiplo de 3
b) isósceles c) divisor de 10
c) equiângulo d) irracional
d) obtusângulo e) maior que 7

PROMILITARES.COM.BR 199

PM_BOOK05_MAT.indb 199 01/11/2021 13:48:47


GEOMETRIA ANALÍTICA: COORDENADAS NO PLANO E DISTÂNCIA

RESOLUÇÃO EM VÍDEO
Abra o ProApp, leia o QR Code, assista à resolução
de cada exercício e AVANCE NOS ESTUDOS!

GABARITO
EXERCÍCIOS DE FIXAÇÃO
01. C 04. B 07. C 10. A
02. D 05. B 08. D
03. D 06. C 09. D
EXERCÍCIOS DE TREINAMENTO
01. D 04. C 07. D 10. A
02. D 05. A 08. C
03. A 06. B 09. D
EXERCÍCIOS DE COMBATE
01. A 04. B 07. D 10. B
02. C 05. A 08. C
L DO PRO
03. B 06. C 09. C
I A FE
ANOTAÇÕES R

S
TE

SO
MA

R
R
MA

SO
T

R
E

IA F
L DO PRO

200 PROMILITARES.COM.BR

PM_BOOK05_MAT.indb 200 01/11/2021 13:48:47


GEOMETRIA ANALÍTICA: RETA

EQUAÇÃO REDUZIDA DA RETA


A reta r do gráfico a seguir é dada pela equação y = mx + n.
Coeficiente angular (declividade): o número real m.
Coeficiente linear: o número real n.

L DO PRO
I A FE
R ∆x
Logo m = tgα = .

S
TE

∆y

SO
n → Coeficiente linear
MA

O coeficiente linear (n) será o responsável por determinar a


 
intersecção da reta com e eixo Oy . Toda intersecção com o eixo Oy

R
Interseção da reta r com o eixo Oy:
se dá num ponto da forma (0,y).
x = 0 ⇒ y = n ⇒ ponto P(0,n)
Assim y = m ⋅ 0 + n ⇒ y = n, então o ponto de intersecção será
Propriedades do coeficiente angular: (0,n).
∆x Vamos ver um exemplo
m = tg α = , onde ∆x = xB – xA e ∆y = yB – yA.
∆y Exemplo: encontrar equação de reta que passa pelos pontos
R
A(2,1) e B(–3,4)
MA

Reta vertical: A equação de uma reta paralela ao eixo Oy é x = k,


SO

onde k é constante. Neste caso, não existem os coeficientes angular Sendo y = mx + n teremos
e linear. 4 −1 3
m= =−
Sabemos que para determinar uma reta é necessário, no mínimo, −3 − 2 5
T

que tenhamos 2 pontos distintos ou também podemos determinar


R 3
E

uma reta conhecendo um ponto e o ângulo formado pela reta e o Assim nossa reta terá a equação y = − x + n .
E


eixo Ox . IA F 5

É conhecido, desde o estudo da função afim, que uma reta é L DO PR O Podemos usar o seguinte: (y – y0) = m(x – x0) onde x0 e y0 são as
definida pela equação y = ax + b. Se conhecermos 2 pontos distintos coordenadas de um ponto que pertença a reta.
A(xA,yA), B(xB,yB) podemos fazer Ou podemos também simplesmente substituir as coordenadas do
3
y A = a.x A + b ponto A ou do ponto B na equação y = − x + n .
 − ⇒ y A − y B = a.x A − a.xB ⇒ a ( xB − x A ) = ( y B − y A ) ⇒ 5
 yB = a.xB + b Substituindo as coordenadas do ponto A na nossa reta

a=
( yB − y A ) ⇒ a = ∆x 3 6 11
1 = − .2 + n ⇒ n = 1 + =
( xB − x A ) ∆y 5 5 5

3 11
Se utilizarmos as nomenclaturas utilizadas em geometria analítica Assim y = − x + .
5 5
a nossa equação será y = mx + n, sendo chamada de equação reduzida.
Nessa equação teremos 2 elementos muito importantes. Utilizando (y – y0) = m(x – x0) e ponto B teríamos
m → Coeficiente angular
3 3 3 3 9
O coeficiente angular (m) será o responsável pelo ângulo (y − 4 ) = − ( x − ( −3)) ⇒ y = − x − .3 + 4 ⇒ y = − x − + 4 ⇒
 5 5 5 5 5
formado pela reta e o eixo Ox Como vimos o coeficiente angular 3 11
∆x y=− x+
será m = , que ao olharmos para a figura abaixo veremos que 5 5
∆y
∅x
é a tangente do ângulo α.
∅y

PROMILITARES.COM.BR 201

PM_BOOK05_MAT.indb 201 01/11/2021 13:48:55


GEOMETRIA ANALÍTICA: RETA

ProBizu ÂNGULO ENTRE DUAS RETAS


Tenha bastante atenção com pegadinhas que são colocadas. A reta
só está na sua forma reduzida quando y está realmente isolado.
Por exemplo 2y = x + 2 não tem seu m = 1 e n = 2 pois o y não
x+2 x
está isolado na equação. Temos de fazer y = = + 1 para só
2 2
1
assim determinarmos m = e n = 2. Atenção!
2

EQUAÇÃO DE RETA POR DETERMINANTE

θ=β–α
Aplicando a função tangente em ambos os lados da igualdade
teremos tgθ = tg(β − α ) e como mr = tgβ e ms = tgα teremos pela
fórmula de subtração de tangente:

L D O P Rtgθ O
= tg(β − α ) =
tgβ − tgα
⇒ m = tgβ e m = tgα ⇒
A F m −m
r s
1 + tgβ.tgα
I pontos A e B
Introduzindo o ponto (x,y) na reta formada pelos
⇒ tgθE=
 m −m 
R para estarem
teremos 3 pontos alinhados, onde vimos que a condição ⇒ θ = arctg 
r s
1 + mr .ms 
r s
 1 + mr .ms 
alinhados é

S
TE

SO
x y 1
MA

xA yA 1 = 0 PARALELISMO E
xB yB 1
PERPENDICULARISMO

R
A equação reduzida da reta é y = mx + n, onde m é o coeficiente
Exemplo: A equação de reta que passa pelos pontos A(2,1) e angular e n é coeficiente linear. Onde m = tgα e α é o ângulo formado
B(–3,4) é 
entre a reta e o eixo OX .
x y 1
yB − y A ∆y
2 1 1 = 0 ⇔ x + 8 − 3y + 3 − 4 x − 2y = 0 ⇔ m = tgα =
R =
xB − x A ∆x
MA

−3 4 1
SO

3 11 
⇔ −5y − 3x + 11 = 0 ⇔ y = − x + n é o ponto de intersecção da reta com o eixo OY .
5 5
T

RETAS PARALELAS
S

R
E

IA F
INTERSECÇÃO DE DUAS RETAS
A teoria da intersecção entre duas retas é a mesma paraL R O
D
duas curvas, que é a resolução de um sistema entre as equações que O
quaisquer P
representam cada uma das curvas.

y = mx + n

 y = px + q

A forma de resolução do sistema é a mesma da álgebra, podendo


se dar pelo método da adição ou da substituição.
Exemplo: O ponto de intersecção entre as retas de equações
y = x – 3 e 2y – 3x = 4 é 
Como as retas r e s possuem o mesmo ângulo α com o eixo OX
 y  x3 y  x  3 x (- 3) 3y  3x  9
     y  13  y  13 então mr = ms.
2y  3x  4  2y  3x  4  2y  3x  4
13  x  3  x  10
RETAS PERPENDICULARES
P  10,  13
mr − ms
Na equação , do ângulo entre 2 retas para
1 + mr .ms
mr − ms
termos θ = 90º temos que não pode existir, daí
1 + mr .ms = 0 ⇒ mr .ms = −1 . 1 + mr .ms

202 PROMILITARES.COM.BR

PM_BOOK05_MAT.indb 202 01/11/2021 13:49:03


GEOMETRIA ANALÍTICA: RETA

x
Exemplo: As equações de retas r : y = – 2x + 3, s : y = − 4 e
2
DISTÂNCIA DE PONTO A RETA
t : y = – 2x +3.
As restas r e t possuem o mesmo coeficiente angular, mr = mt. Daí
r e t são paralelas.
As retas r e s ou s e t possuem os coeficientes angulares
 1
inversos simétricos entre si, mr .ms = −2.   = −1. Daí r // t que são
 2
perpendiculares a s.

OUTRAS EQUAÇÕES DE RETA


EQUAÇÃO COMPLETA DA RETA
Da equação reduzida y = mx + n quando isolamos todos os termos
de um mesmo lado da igualdade passamos a ter os termos ax + by + c = 0
Sendo a equação da reta r : ax + by + c = 0 e P(x0,y0) temos
com a, b e c ∈ Z , que é a equação completa da reta.
x x a.x 0 + b.y 0 + c
Exemplo: y=− + 4 ⇒ y + − 4 = 0 ⇒ x + 3y − 12 = 0 , daí dP,r =
3 3 a2 + b2
a = 1, b = 3 e c = – 12.
Exemplo: A distância do ponto P(–1,1) a reta de equação

D O Py =R–2xO+ 3 ⇒ 2x + y – 3 = 0 ⇒ a = 2, b = 1 e c = –3 e x = –1 e
y = –2x + 3 é
EQUAÇÃO SEGMENTÁRIA DA RETA
L
Da equação completa da reta ax + by + c = 0 isolaremos o termo

IA
0
y =1
c de um lado da igualdade, ax + by = –c e dividiremos todos os termos F y  c 2  (1)  1 1 (3) 4 4 5
a  x  b E
0

a b
por –c, assim − x − y = 1 .
c c
R d  
P ,r
0 0

a2  b2
 
(2)2  (1)2 5 5

S
TE

Assim teremos por exemplo 2x – y – 6 = 0 e 2x – y = 6

SO
DISTÂNCIA ENTRE DUAS RETAS PARALELAS
MA

2 1 x y
⇒ x − y = 1⇒ − = 1 Se r // s de equações r : ax + by + c1 = 0 e s : ax + by + c2 = 0
6 6 3 6

R
Com a equação segmentária, podemos determinar os pontos de c1 − c2
dr ,s =
interseção da reta com os eixos ordenados do plano. O termo que a2 + b2
divide x na equação segmentária é abscissa do ponto de intercessão
da reta com o eixo x, e o termo que divide y é abscissa do ponto de
interseção da reta com o eixo y. Assim: Exercício Resolvido
 R
Intersecção com o eixo OX é (3,0)
01. Considere as retas r e s definidas por r : kx – (k + 2)y = 2 e
MA


s : ky – x = 3k. Determine k de modo que:
SO

Intersecção com o eixo OY é (0,–6)


a) r e s sejam concorrentes
EQUAÇÃO PARAMÉTRICA DA RETA b) r e s sejam paralelas
T

As equações paramétricas são duas equações que representam a c) r e s sejam coincidentes.


R
E

mesma reta utilizando uma incógnita t. Essa incógnita recebe o nome


E

IA
de parâmetro e faz a ligação entre as duas equações que representam F
a mesma reta.
L D O PelaPdefiR O
Resolução:
nição, retas concorrentes possuem um ponto de interseção,
Vamos a um exemplo prático.
as paralelas nenhum ponto de interseção e as coincidentes, todos
x+3 seus pontos são de interseção. Nos termos de Geometria Analítica,
Seja a equação x − 2y + 3 = 0 ⇒ 2y = x + 3 ⇒ y = . Vamos
2 analisamos os coeficientes angulares das retas:
escolher uma parametrização arbitrária para y, por exemplo y = 2t + 1.
k 2 k
x+3 r : kx − (k + 2)y = 2 ⇒ y = x+ ⇒ mr =
Assim sendo y = 2t + 1 e y = teremos que k+2 k+2 k+2
2 1 1
x+3 s : ky − x = 3k ⇒ y = x + 3 ⇒ ms =
= 2t + 1 ⇒ x + 3 = 4t + 2 ⇒ x = 4t – 1. k k
2
x = 4t − 1 a) retas concorrentes possuem os coeficientes angulares diferentes:
Assim temos a equação paramétrica 
 y = 2t + 1 k ≠ 2
Para voltarmos para equação completa ou reduzida basta k 1 
mr ≠ ms ⇒ ≠ ⇒ k 2 − k − 2 ≠ 0 ⇒ (k − 2).(k + 1) ≠ 0 ⇒ e
isolarmos t em ambas equações e igualá-los. k+2 k k ≠ −1

x +1
x = 4t − 1 ⇒ t =
4 b) retas paralelas possuem os coeficientes angulares iguais:
y −1 k = 2
y = 2t + 1 ⇒ t = k 1 
2 mr = ms ⇒ = ⇒ k 2 − k − 2 = 0 ⇒ (k − 2).(k + 1) = 0 ⇒ ou
x +1 y −1 k+2 k k = −1
= ⇒ 2x + 2 = 4 y − 4 ⇒ 
4 2
⇒ 2x − 4 y + 6 = 0 ⇒ x − 2y + 3 = 0

PROMILITARES.COM.BR 203

PM_BOOK05_MAT.indb 203 01/11/2021 13:49:14


GEOMETRIA ANALÍTICA: RETA

c) retas coincidentes possuem os coeficientes angulares e os Exercício Resolvido


lineares iguais:
04. As retas r, s e t são definidas respectivamente, por 4x – 7y + 18 = 0,
 k = 2 2x – y – 6 = 0 e 4x + 3y – 2 = 0. A área da região limitada por
 k 1  essas retas é
 m r = m s ⇒ = ⇒ k 2
− k − 2 = 0 ⇒ (k − 2).(k + 1) = 0 ⇒ ou
k+2 k k = −1
  a) 30 d) 15

n = n ⇒ 2 = 3 ⇒ 3k + 6 = 2 ⇒ 3k = −4 ⇒ k = − 4 b) 25 e) 10
 r s
k+2 3 c) 20
Não é possível conciliar os valores de “k”. Logo, não existe um
valor de “k” que satisfaça a condição pedida. Resolução: C
As retas são concorrentes duas a duas. Logo suas interseções
formarão um triângulo. Sejam os pontos P, Q e R as interseções
Exercício Resolvido respectivamente r ∩ s, r ∩ t e s ∩ t.

02. A reta r determina um ângulo de 120° com a reta s, cujo


1
coeficiente angular é − . O coeficiente angular de r vale:
3
a) 3
6+5 3
b)
3

c) -
6+5 3
L DO PRO
6-5 3
3
I A FE
d)
3 R

S
TE

Resolução: C

SO
O ângulo agudo formado entre duas retas é dado pela fórmula:
MA

ms − mr . Como o ângulo indicado é 120º (obtuso), o


tga =

R
1 + ms .mr
agudo interno será de 60º.
1 −1 − 3mr 4 x − 7y + 18 = 0 4 x − 7y + 18 = 0
−− mr i)  ⇒ ⇒1
ms − mr 3 3
42xx −− 7yy−+618
= =0 0→ ×( −7) 4x−14 − 7xy ++ 18
7y += 42
0 =0
g60º = ⇒ 3= ⇒ 3= ⇒ i)  ⇒ ⇒1
1 + ms .mr  1 3 − mr 2x − y − 6 = 0 → ×60 ( −7)  −14 x + 7y + 42 = 0
1 +  −  .mr x = =6
 3 3 ⇒ 0x = 60 ⇒x = 60
R
10= 6 ⇒ P = (6, 6)

⇒ 0x = 60 ⇒ y =10 ( ) ⇒ P = (6, 6)
MA

−1 − 3mr 3 −1 − 3mr −1 − 3mr  2 6 − 6 = 6


3= ⇒ 3= ⇒ 3= ⇒ y = 2 (6) − 6 = 6
SO
.
3 3 − mr 3 − mr 3 − mr 
4 x − 7y + 18 = 0 4 x − 7y + 18 = 0
ii) 4 x − 7y + 18 = 0 ⇒ 4x − 7y + 18 = 0 ⇒
−1 − 3 3 −1 − 3 3 3 + 3 ii) 4 x + 3y − 2 = 0 → ×( −1⇒ )   −4 x − 3y + 2 = ⇒ 0
⇒ − 3mr + 3 3 = −1 − 3mr ⇒ mr = = . ⇒
4 x + 3y − 2 = 0 → ×( −1)  −4 x − 3y + 2 = 0
T

3− 3 3− 3 3+ 3
S

 20
y =20 = 2
−3 − 3 − 9 3 − 9 −12 − 10 3 6+5 3
R
E

y = 10= 2
E

⇒ Q = ( −1, 2)
⇒ mr =
9−3
=
6
=−
3
IA
⇒ 10y = 20 ⇒ 10
⇒ 10y = 20 ⇒  F 7 (2) − 18 ⇒ Q = ( −1, 2)

L DO PRO
 x =7 (2) − 18 = −1
x= 4 = −1
 4
2 x − y − 6 = 0 → × ( 3 ) 6x − 3y − 18 = 0
Exercício Resolvido iii)) 2x − y − 6 = 0 → ×(3) ⇒⇒6x − 3y − 18 = 0 ⇒⇒
iii 4 x + 3y − 2 = 0  4 x + 3y − 2 = 0
4 x + 3y − 2 = 0 4 x + 3y − 2 = 0
03. As retas r e (s) de equações 3x – y + 7 = 0 e 4x – y – 5 = 0  2020
x = ==22
respectivamente passam pelo ponto P(a, b). O valor de (a + b) é ⇒ 10
⇒ 10xx == 2020⇒⇒x = 10 10 ⇒⇒ RR (−22,−−) 2)
= (=2,−
yy==22
(2(2)− ) −66= =−2−2
a) 51 d) 53
66 66 11
b) 52 e) 55 11 11 1 1 40 40
A= −−11 22 11== 66 ) −) −6(6−(3−)3+) 1
( 4( 4 +( −1(2−) 2=) = 2424 + 18
+ 18 − 2−) =
2) = = 20
= 20
c) 53 22 22 2 2 2 2
22 −−22 11
Resolução: E
Se ambas passam pelo ponto P, então ele é um ponto de interseção.
Basta resolver o sistema Exercício Resolvido
3x − y + 7 = 0 3x − y + 7 = 0
 ⇒ ⇒ 05. A equação da reta que é perpendicular à reta 4x + y – 1 = 0
 4 x − y − 5 = 0 → × ( −1)  −4 x + y + 5 = 0 e que passa pelo ponto de interseção das retas 2x – 5y + 3 = 0 e
 x = 12 x – 3y – 7 = 0 é:
⇒ − x + 12 = 0 ⇒ 
y = 3(12) + 7 = 36 + 7 = 43 a) x – 4y – 24 = 0 d) x + y + 24 = 0
a = 12 b) 4x – y – 24 = 0 e) x – 4y + 24 = 0
Logo, P(a,b) = P( −12, −29) ⇒  ⇒ a + b = 12 + 43 = 55
b = 43 c) x – y – 24 = 0

204 PROMILITARES.COM.BR

PM_BOOK05_MAT.indb 204 01/11/2021 13:49:21


GEOMETRIA ANALÍTICA: RETA

05. (EEAR) A distância do ponto P(–3,–2) à bissetriz dos quadrantes


Resolução: A ímpares do plano cartesiano é
i) Interseao  : 2x − 5y + 3 = 0 ⇒
2x − 5y + 3 = 0
⇒ a) 2
 x − 3y − 7 = 0 → × ( −2)  −2x + 6y + 14 = 0
b) 5 2 .
y = −17

⇒ 5( −17) − 3 ⇒ P = ( −44, −17) 5 2
x= = −44 c)
 2 2
ii) r : 4 x + y − 1 = 0 ⇒ y = −4 x + 1 ⇒ mr = −4 2
d)
 x 2
1 s : y = + ns
iii) s ⊥ r ⇒ ms = ⇒  4 ⇒
4 P = ( −44, −17) ∈ S 06. (EEAR) A equação segmentária da reta que passa pelos pontos
( −44 ) A(–2,–7) e B(1,–5) é
⇒ −17 = + ns ⇒ −17 = −11 + ns ⇒ ns = −6
4 3y 2x 3x 2y
a) − =1 c) _ =1
x 17 17 17 17
Logo, s : y = − 6 ou s : x − 4 y − 24 = 0
4 2x 3y 3y 2x
b) − =1 d) + =1
17 17 17 17

07. (EEAR) Dada a reta (s) 2x – y + 3 = 0, a equação da reta r,


EXERCÍCIOS DE
perpendicular à s, que intercepta o eixo y no ponto de ordenada 2, é

FIXAÇÃO L D O a)b) P2y2y R++ xx ––O24 == 0.0.


IA c) 2x + y + 4 =F0.
E
R
01. (EEAR) Uma reta r passa pelo ponto A (–1,4) e é perpendicular à
reta s de equação 3x + 5y – 2 = 0. Nessas condições, a equação da
d) 2x + y + 2 = 0.

S
TE

reta r é 08. (EEAR) Se uma reta passa pelo ponto P(3,4) e tem coeficiente

SO
a) 3x + 5y – 23 = 0. angular 2, então o coeficiente linear dessa reta é
MA

b) 5x + 3y – 17 = 0. a) −4. c) 1.

R
c) 3x + 5y – 17 = 0 b) –2. d) 3.
d) 5x – 3y + 17 = 0.
09. (EEAR) A equação geral da reta que passa por P(0,3) e Q(1,5) é
02. (EEAR) Considere as afirmações: a
representada por ax + by + c = 0. Assim, o valor de é
c
I. As retas (r) x – 3y + 1 = 0 e (s) – 2x + 6y + 1 = 0 são paralelas 2 1
distintas. a) . R c) − .
3 5
MA

II. As retas (t) – 2x + y + 5 = 0 e (u) – 6x + 3y + 15 = 0 são coincidentes.


3 5
SO

III. As retas (v) – 5x– 4y – 3 = 0 e (w) – 10x + 8y + 6 = 0 são b) . d) − .


4 6
concorrentes.
Das afirmações anteriores, é(são) verdadeira(s)
T

10. (EEAR) Se (r) x + 6y – 2 = 0 e (s) 8x + (t – 1) y – 2 = 0 são duas retas


S

a) apenas duas.
R paralelas, então t é múltiplo de
E

b) apenas uma.
IA a) 3. F c) 7.
c) nenhuma.
d) todas.
L DO PRO b) 5. d) 9.

EXERCÍCIOS DE
03. (EEAR) Seja α o ângulo formado por duas retas cujos coeficientes
1 1
angulares são − e . O valor de tg α é
3 3
TREINAMENTO
3 5
a) . c) .
4 4 01. (EEAR) Para que os pontos A(x,3), B(–2x,0) e C(1,1) sejam
colineares, é necessário que x seja
3
b) 1. d) . a) –2 c) 2
2
b) –1 d) 3

 7 5  5 7
04. (EEAR) Os pontos A  ,  e B  − , −  definem uma reta de 02. (EEAR) Considere os pontos A(2,3) e B(4,1) e a reta r : 3x + 4y = 0.
 2 2  2 2 Se dA, r e dB, r são, respectivamente, as distâncias de A e de B até a
C reta r, é correto afirmar que
equação ax + by + c = 0. O valor de é
B
a) dA, r > dB, r
a) 3.
b) dA, r < dB, r
b) 2.
c) dA, r = dB, r
c) 1.
d) 0. d) dA, r = 2dB, r

PROMILITARES.COM.BR 205

PM_BOOK05_MAT.indb 205 01/11/2021 13:49:34


GEOMETRIA ANALÍTICA: RETA

03. (EEAR) A equação reduzida da reta que passa pelos pontos A(0,1) 09. (CFOE) As retas de equações y = 2x – b e y = cx + d se interceptam
e B(6,8) é dada por perpendicularmente no ponto (8,2). Qual o valor do produto b ⋅ c ⋅ d?
a) y = 7x + 1 a) – 42
b) y = 6x + 1 b) – 63
7 c) – 36
c) =
y x +1
6 d) – 27
6
d) =
y x +1
7 10. (EEAR) Dois pontos sobre a reta y = 2 distam 4 unidades da reta
4x – 3y + 2 = 0. A distância, em unidades, entre as abscissas dos pontos é
04. (EEAR) A reta s que passa por P(1,6) e é perpendicular a
2 a) 10
r :=
y x+3 é
3 b) 2
3 c) 6
a) y = x
2 d) 4
b) y = x + 5
2 20
c) y=
− x+
3 3 EXERCÍCIOS DE

d) y=
3
− x+
2
15
2 COMBATE
DO PRO
05. (EEAR) Dada a reta r : 2x – 3y + 5 = 0 e o ponto P(5,6), a distância
L
IA
de P à reta r é
a) 91 F E(t) com a reta (r) 3x – y = 0 e B, a intersecção de (t)
01. (AFA) Seja P(3,1) o ponto médio do segmento AB, onde A é
intersecção da reta
b) 30 13 R com a reta (s) x + 5y = 0. O coeficiente angular de (t) é
a) negativo.

S
TE

3 91
c) b) par positivo.

SO
91
c) 5, pois (t) é perpendicular à (s).
MA

3 13
d) d) nulo, isto é, a reta é do tipo y = k, k = constante.

R
13
02. (EEAR) Sejam o ponto C e a reta s de equação(s) x − y − 2 = 0,
06. (CFOE) Dadas as retas perpendiculares de equações y = ax + b e representados na figura. O quadrado do raio da circunferência de
y = mx + n, a afirmativa correta é: centro C e tangente à reta s é
a) a = m
b) a = – m y R s
c) a = 1/m
MA

C
SO

d) a.m = – 1 1
07. (EEAR) A reta 3x – 2y – 5 = 0 é perpendicular à reta
T

–1
S

a) 2x – 3y = 5. 2 x
R
E

b) 4x + 6y = 1.
IA F
L DO PRO
c) 3x + 2y = 0.
–2
d) 6x – 4y = 10.

08. (EEAR) Cada equação a seguir representa uma reta do plano


cartesiano. Associe cada uma dessas equações ao coeficiente angular a) 24
da reta que a mesma representa. b) 16
(1) – x + 2y – 2 = 0 ( ) 1/2 c) 8
(2) 4x + 8y – 9 = 0 ( )–2 d) 4
(3) – 10x + 5y – 3 = 0 ( )2
(4) 6x + 3y – 5 = 0 ( ) – 1/2 03. (EEAR) Sejam r : y = 3x + 6 e s : y = – 4x – 1 as equações de duas
retas cuja interseção é o ponto A. A área do triângulo cujos vértices
Assinale a alternativa que apresenta a sequência correta. são os pontos A, B(0,0) e C(7/2,0) é igual a
a) 1 – 3 – 4 – 2 a) 16
b) 3 – 4 – 2 – 1 b) 21
c) 4–2–1–3 c) 16/3
d) 1 – 4 – 3 – 2 d) 21/4

206 PROMILITARES.COM.BR

PM_BOOK05_MAT.indb 206 01/11/2021 13:49:34


GEOMETRIA ANALÍTICA: RETA

04. (EEAR) Seja a equação geral da reta ax + by + c = 0. Quando a = 0, 09. Considerando as retas r e s da figura, o valor de a é
b ≠ 0 e c ≠ 0, a reta
a) passa pelo ponto (c,0) 3
a)
b) passa pelo ponto (0,0) 2
c) é horizontal b) 3
d) é vertical c) 2 3
d) 3 3
05. (EEAR) As retas de equações y + x – 4 = 0 e 2y = 2x – 6 são, entre si,
a) paralelas
b) coincidentes
c) concorrentes e perpendiculares
d) concorrentes e não perpendiculares

x y
06. (CFT) O coeficiente angular da reta de equação + =1 é igual a
−2 6 10. (CFT) Na figura abaixo, o ponto O é origem do sistema de
a) –2. coordenadas cartesianas ortogonais. O triângulo equilátero OAB e o
quadrado BCDE têm lados medindo 8 unidades. M e N são pontos
b) –3. médios de OB e DE, respectivamente.
c) 3.
d) 6.
L DO PRO
I Acom
07. (EEAR) Analisando o gráfico, temos que a reta forma os eixos FE
R
coordenados um triângulo de 4 unidades de área. Marque a alternativa
correspondente à equação da reta que passa pelos pontos P e Q.

S
TE

SO
MA

Q(0,q)

R
2
A equação geral da reta que passa por M e N, é ______________.
a) x + y + 2 = 0
P(p,0) b) x + y – 6 = 0
0 1 x c) x–y–4=0 R
d) x – y – 8 = 0
MA

SO

a) 2x + y – 4 = 0
b) – 2x + y = 4
RESOLUÇÃO EM VÍDEO
T

c) 2x + y = – 4
R Abra o ProApp, leia o QR Code, assista à resolução
E

d) 2x – y = 4
IA F
de cada exercício e AVANCE NOS ESTUDOS!


08. (EEAR) Dada a reta DG , conforme ilustração abaixo, e, sabendo
que a área do quadrado ABCD é igual a 9 m2 e a área do quadrado
L DO PRO

BEFG é 25 m2, a equação da reta DG é GABARITO
Y EXERCÍCIOS DE FIXAÇÃO
01. D 04. C 07. A 10. C
G F 02. D 05. D 08. B
D C 03. A 06. B 09. A
EXERCÍCIOS DE TREINAMENTO
A B E X 01. B 04. D 07. B 10. A
02. A 05. D 08. D
03. C 06. D 09. A
a) −2x − 3y − 9 =
0
EXERCÍCIOS DE COMBATE
b) 2x − 3y − 9 =
0
01. A 04. C 07. A 10. C
c) −2x − 3y =
−9
02. C 05. C 08. D
d) 2x − 3y =
−9
03. D 06. C 09. C

PROMILITARES.COM.BR 207

PM_BOOK05_MAT.indb 207 01/11/2021 13:49:35


GEOMETRIA ANALÍTICA: RETA

ANOTAÇÕES

L DO PRO
I A FE
R

S
TE

SO
MA

R
R
MA

SO
T

R
E

IA F
L DO PRO

208 PROMILITARES.COM.BR

PM_BOOK05_MAT.indb 208 01/11/2021 13:49:35


GEOMETRIA ANALÍTICA:
CIRCUNFERÊNCIA

DEFINIÇÃO Nos produtos notáveis o 1º termo é x e o 2º termo é 2 e também


o 1º termo é y e o 2º termo é 3, daí vamos acrescentar o quadrado
do 2º termo em cada caso e ao mesmo tempo subtraí-lo, para não
“desiquilibrar” a igualdade.

x 2 − 2.2x + 4 − 4 + y 2 + 2.3y + 9 − 9 − 3 = 0 ⇒
⇒ ( x − 2)2 − 4 + ( y + 3)2 − 9 − 3 = 0 ⇒
( x − 2)2 + ( y + 3)2 = 16 ⇒ ( x − 2)2 + ( y + 3)2 = 42

D O POTambém
centro é O(2,–3) e o raio é R = 4.
R Opodemos fazer o seguinte, desde que a equação
A L
I F
completa da circunferência possua os coeficientes de x² e y² iguais
a 1 (caso não sejaEpodemos dividir toda a equação pelo valor do
R coeficiente).

S
TE

Utilizando a propriedade da circunferência como o lugar ( x − x 0 )2 + ( y − y 0 )2 = R2 ⇔ x 2 + y 2 + Bx + Cy + D = 0

SO
geométrico dos pontos do plano equidistantes de um único ponto x 2 − 2x 0 x + x 20 + y 2 − 2y 0 y + y 20 − R2 = 0 ⇔ x 2 + y 2 + Bx + Cy + D = 0
MA

teremos, P(x,y) como um ponto qualquer de raio R e O(x0,y0). Assim


x 2 + y 2 − 2x 0 x − 2y 0 y + x 20 + y 20 − R2 = 0 ⇔ x 2 + y 2 + Bx + Cy + D = 0
( x − x 0 )2 + ( y − y 0 )2

R
dO,P = R ⇒ =R⇒  B
x 0 = − 2
⇒ R2 = ( x − x 0 ) + ( y − y 0 )
2 2

 C
y 0 = −
( x − x 0 )2 + ( y − y 0 )2 = R2 ⇒ Equação reduzida da circunferência  2
x 20 + y 20 − R2 = D
O(x0,y0) ⇒ Centro da circunferência 
R
R ⇒ Raio da circunferência
MA

SO

Exemplo: a equação reduzida de uma circunferência de centro Exemplo: 2x² + 2y² + 6x – 8y – 15 = 0 , teremos
O(2,–3) e raio 4 é
15
2x 2 + 2y 2 + 6x − 8y − 15 = 0 ÷ 2 ⇒ x 2 + y 2 + 3x − 4 y − =0
(x − 2)2 + (y − ( −3))2 = 42 ⇒ (x − 2)2 + (y + 3))2 = 16 2
T

R  3
E

x 0 = − 2
E

EQUAÇÃO COMPLETA IA  F
L DO PRO
y = − ( −4 ) = 2
Desenvolvendo os produtos notáveis da equação reduzida  0 2
teremos Ax + Ay + Bx + Cy + D = 0 , onde para voltarmos para
2 2
 2

equação reduzida, para se determinar o centro e o raio devemos x + y − R2 = − 15 ⇒  − 3 + 22 − R2 = − 15


2 2
 0 0
2  2  2
fazer o processo de completar quadrados, mas antes iremos fazer o 
reconhecimento de uma equação de circunferência. ⇒ −R2 = − 15 − 9 − 4 ⇒ R2 = 30 + 9 + 16 ⇒ R = 55
 2 4 4 2
Ax 2 + By 2 + Cx + Dy + Exy + F = 0

Esses são todos os elementos que uma cônica pode apresentar,


para que essa equação represente uma circunferência devemos ter
POSIÇÕES RELATIVAS
sempre
ENTRE PONTO E CIRCUNFERÊNCIA
A = B e A e B ≠ 0

E = 0

Ou seja x² e y² devem ter o mesmo coeficiente e não pode haver


o termo xy.
Completando os quadrados da equação x² + y² – 4x + 6y – 3 = 0
x 2 + y 2 − 4 x + 6y − 3 = 0 ⇒ x 2 − 4 x + y 2 + 6y − 3 = 0 ⇒
⇒ x 2 − 2.2x + y 2 + 2.3y − 3 = 0 ⇒ • O ponto P(x,y) pertence à circunferência de centro C(a,b) e
raio R se e somente se CP = R ⇔ (x – a)² + (y – b)² = R².

PROMILITARES.COM.BR 209

PM_BOOK05_MAT.indb 209 01/11/2021 13:49:42


GEOMETRIA ANALÍTICA: CIRCUNFERÊNCIA

• O ponto I(x,y) é interior à circunferência de centro C(a,b) e ENTRE DUAS CIRCUNFERÊNCIAS


raio R se e somente se CI < R ⇔ (x – a)² + (y – b)² < R².
• O ponto E(x,y) é exterior à circunferência de centro C(a,b) e EXTERIORES
raio R se e somente se CE > R ⇔ (x – a)² + (y – b)² > R².

ENTRE RETA E CIRCUNFERÊNCIA

RETA EXTERIOR

dO1,O2 > R + r

( x 2 − x1)2 + ( y 2 − y1)2 >R+r

TANGENTES EXTERIORES

L DO PRO
I A FE
ax o + by o + c
R
d= >R

S
TE

a2 + b2

SO
MA

RETA TANGENTE

R
( x 2 − x1)2 + ( y 2 − y1)2 =R+r

SECANTES
R
MA

SO
T

R
E

IA F
ax o + by o + c
L DO PRO
d= =R
a2 + b2

RETA SECANTE R −r < ( x 2 − x1)2 + ( y 2 − y1)2 <R+r

TANGENTES INTERIORES

ax o + by o + c
d= <R
a2 + b2
( x 2 − x1)2 + ( y 2 − y1)2 =R−r

210 PROMILITARES.COM.BR

PM_BOOK05_MAT.indb 210 01/11/2021 13:49:48


GEOMETRIA ANALÍTICA: CIRCUNFERÊNCIA

INTERIORES

( x 2 − x1)2 + ( y 2 − y1)2 <R −r

RETA TANGENTE A CIRCUNFERÊNCIA


Sempre que desejamos encontrar a intersecção entre 2 curvas
devemos resolver um sistema envolvendo as equações das 2 curvas.
O sistema encontrado entre uma equação de circunferência e
 y = mx + n L DO PRO
IA F E aencontrar
uma equação de reta seria  2 , assim por Também é possível a reta tangente utilizando a fórmula
 x + y 2
+ Bx + cy + D = 0
substituição teremos
R de distância de ponto
o raio da circunferência.
reta. Para isso devemos encontrar o centro e

S
TE

x 2 + (mx + n) + Bx + c (mx + n) + D = 0 x 2 + y 2 − 12x + 20 = 0 ⇒ x 2 − 2.6.x + 36 − 36 + (y − 0) + 20 = 0 ⇒


2 2

SO
x 2 + m2x 2 + 2mnx + n2 + Bx + cmx + cn + D = 0 (x − 6)2 + (y − 0)2 − 16 = 0 ⇒ (x − 6)2 + (y − 0)2 = 16
MA

(m2
)
+ 1 x + (B + cm + 2mn) x + (n + cn + D) = 0
2 2
Centro (6,0) e R² = 16 ⇒ R = 4

R
Logicamente isso não foi exposto para ser decorado mas sim para A reta como vimos é y = mx + m ⇒ mx – y + m = 0.
mostrar que todo sistema entre uma equação de reta e uma equação de Assim pela fórmula de distância de ponto a reta sendo a reta na
circunferência resulta numa equação do 2° grau onde daí sabemos que sua forma completa e seus coeficientes a = m, b = –1 e c = m.
∆ > 0 → 2 pontos de intersecção → reta secante m  6  1 0  m 7m
 
2

∆ = 0 → 1 ponto de inteersecção → reta tangente 4  4  49m2  4 m2  1 


m   1
2 2 R m2  1
∆ < 0 → Nenhum ponto de intersecção → reta exterior
MA

16 4 33
SO

49m2  16m2  16  33m2  16  m2  m


( )
Daí da equação m2 + 1 x 2 + (B + cm + 2mn) x + (n2 + cn + D) = 0 33 33
resultante do sistema basta fazermos o delta corresponde a situação
de reta que nos for pedida.
T

Exemplo: Encontre a equação de reta que passa pelo ponto


R Exercício Resolvido
E

(–1,0) e é tangente a circunferência x² + y² – 12x + 20 = 0.


IA F
Reta y = mx + n que passa por (–1,0) → L DO PR
0 = m(–1) + n → O
01. A equação da circunferência de centro (3,2) que é tangente

ao eixo OX é:
m = n, assim y = mx + m.
a) x² + y² - 6x – 4y + 12 = 0
 y = mx + m
⇒ x 2 + (mx + m) − 12x + 20 = 0 ⇒
2
 2 b) x2 + y2 – 6x – 4y + 9 = 0
x + y − 12x + 20 = 0
2
c) x² + y² – 6x – 4y + 7 = 0
x 2 + m2x 2 + 2m2x + m2 − 12x + 20 = 0
d) x² + y² + 4x + 4y + 9 = 0
( ) () ( )
x 2 m2 + 1 + 2m2 − 12 x + m2 + 20 = 0
Resolução: B
tangente → ∆ = 0 → (2m − 12) − 4.(m + 1).(m )
2
2 2 2
+ 20 = 0
A circunferência está afastada da origem de uma unidade no
4m − 48m + 144 − 4 (m + 21m + 20) = 0
4 2 4 2
sentido positivo de x.
4m4 − 48m2 + 144 − 4m4 − 84m2 − 80 = 0 O raio portanto vale 2.
64 16 4 33 Equação reduzida é (x – 3)2 + (y – 2)2 = 4.
−132m2 + 64 = 0 ⇒ m2 = = ⇒m= ±
132 33 33 Equação geral: x2 – 6x + 9 + y2 – 4y + 4 – 4 = 0 ⇒
x2 + y2 – 6x – 4y + 9 = 0.
4 33 4 33
Assim nossas retas tangentes são y= x+ e
33 33
4 33 4 33
y=− x− .
33 33

PROMILITARES.COM.BR 211

PM_BOOK05_MAT.indb 211 01/11/2021 13:49:55


GEOMETRIA ANALÍTICA: CIRCUNFERÊNCIA

Exercício Resolvido Exercício Resolvido

02. Considerando uma circunferência de centro (2,1), que passa 05. Na figura abaixo, a reta (r) dada pela equação x + y – 10 = 0
pelo ponto (2,–2), assinale a opção correta. se intercepta com a reta (t) no ponto P(x,y).
a) A equação da circunferência é (x – 2)² + (y – 1)² = 3.
b) O interior da circunferência é representado pela inequação
x² + 4x + y² + 2y < 4.
c) O interior da circunferência é representado pela inequação
x² – 4x + y² – 2y < 4.
d) O exterior da circunferência é representado pela inequação
x² + 4x + y² – 2y > –2.
e) O ponto (5,–1) pertence à circunferência.

Resolução: C
O centro é (2,1) e o raio é a distância entre (2,1) e
(2, − 2) ⇒ R = (2 − 2) 2 + (1 + 2) 2 = 3.
Equação da circunferência:
(x − 2) 2 + (y − 1) 2 = 9 ⇔ x 2 + y 2 − 4x − 2y = 4 .

DO PRO
Interior da circunferência:
(x − 2) 2 + (y − 1) 2 < 9 ⇔ x 2 + y 2 − 4x − 2y < 4 .
A L
Exterior da circunferência:
I FE
Então, a soma das coordenadas do ponto P é igual a:
R
(x − 2) 2 + (y − 1) 2 > 9 ⇔ x 2 + y 2 − 4x − 2y > 4 .
a) 11.

S
TE

b) 12.

SO
c) 9.
Exercício Resolvido
MA

d) 10.
03. Se (p,q) são as coordenadas cartesianas do centro da

R
circunferência x 2 + y 2 − 4 x + 2y − 4 = 0, então é correto afirmar Resolução: D
que 5p – 3q é igual a: Percebe-se que o ponto P pertence à reta t e também à reta r, logo
a) 7 deve obedecer a equação x + y – 10 = 0. Essa mesma pode ser
escrita como: x + y = 10. Logo, a soma das coordenadas será igual
b) 10 a 10. Ou ainda pode-se resolver o exercício calculando, ou seja:
c) 13 chamando os pontos de intersecção da reta r com a circunferência
R
de A e B, pode-se escrever:
MA

d) 16
SO

e) 19. A (0, y ) ⇒ x + y − 10 = 0 ⇒ 0 + y − 10 = 0 ⇒ y = 10 ⇒ A (0, 10)


B ( x, 0) ⇒ x + y − 10 = 0 ⇒ x + 0 − 10 = 0 ⇒ x = 10 ⇒ B (10, 0)
Resolução: C
T

x 2 + y 2 − 4 x + 2y − 4 = 0 ⇒ ( x − 2) + ( y + 1) = 32 Centro = C(0,0)
R
2 2
E

p = 2 e q = −1 ⇒ 5p − 3q = 10 + 3 = 13
IA F
Raio = distância entre C e A ⇒ R = 10

O
Ponto de intersecção entre a reta t e a circunferência = T(6,b).
L D O Circunferência:
PR /
Exercício Resolvido 
x 2 + y 2 = R2 ⇒ 62 + b2 = 102 ⇒ 36 + b2 = 100 ⇒ b = 8 (nao conv em)
b = −8
04. As retas 2x – y – 4 = 0 e 2x + 3y – 12 = 0 interceptam-se no
centro de uma circunferência de raio igual a 3. Então podemos Reta s||t com pontos C(0,0) e T(6,–8):
dizer que:
4 4
a) a circunferência possui centro no ponto (2,3). ms = − ⇒ y = − x (eq. reta s )
3 3
b) a circunferência corta o eixo y em dois pontos. 3
s  t ⇒ mt =
c) a circunferência corta o eixo x em um ponto. 4
d) a circunferência é tangente ao eixo x.
3
e) a circunferência é tangente ao eixo y. Reta t : y + 8 = (x − 6) ⇒ 3x − 4y − 50 = 0
4

Resolução: E
3x − 4 y = 50 x = 90 7 70
Calculando as coordenadas do centro da circunferência, tem-se: Ponto P :  ⇒ ⇒x+y= = 10
y + 4 = −3y + 12 → 4 y = 8 → y = 2 x + y = 10 y = − 20 7 7

2x − 2 − 4 = 0 → 2x = 6 → x = 3

→ Centro Circunferencia (3 , 2)
Sabendo-se as coordenadas do centro e o raio, é possível desenhar
a circunferência no plano cartesiano. Esta tangencia o eixo y e
corta o eixo x em dois pontos.

212 PROMILITARES.COM.BR

PM_BOOK05_MAT.indb 212 01/11/2021 13:50:02


GEOMETRIA ANALÍTICA: CIRCUNFERÊNCIA

EXERCÍCIOS DE EXERCÍCIOS DE

FIXAÇÃO TREINAMENTO
01. (EEAR) Uma circunferência tem centro (4,3) e passa pela origem. 01. A equação da circunferência, em que os pontos M(–3,2) e N(5,4)
A equação dessa circunferência é são extremos de um diâmetro, é
a) x² + y² = 25. a) x² + y² – 5 = 0.
b) x² + y² + 8x + 6y = 0. b) x² + y² – 17 = 0.
c) x² + y² − 8x − 6y = 25. c) x² + y² – 2x – 6y – 7 = 0.
d) x² + y² − 8x − 6y = 0. d) x² + y² – 2x – 6y – 5 = 0.
e) x² + y² – 2x – 6y – 12 = 0.
02. (EEAR) O raio da circunferência de equação x² + y² – 2x + 10y +1 = 0
é igual a
3x
a) 5 c) 6. 02. Dadas a reta de equação y = e a circunferência de equação
3
b) 4. d) 7. x² + y² – 4x = 0. A área do triângulo determinado pelo centro da
circunferência e os pontos de intersecção entre a reta e ela, em
03. (EEAR) Se a circunferência de equação x² + by² + cx + dy + k = 0 unidades de área, é igual a
tem centro C(1,–3) e raio 3, então “b + c + d + k” é igual a a) 3 c) 3 3
a) 12.
b) 11.
c)
d) 9.
10.
L DO PRO b) 3 d) 6

IA 03. (EEAR) Se F A(x,y) pertence ao conjunto dos pontos do plano


E d do ponto C(x ,y ), sendo d > 2, então
tem 45° de inclinação é R
04. (EEAR) A equação da reta que passa pelo ponto E(–1,–3) e que cartesiano que distam
a) (x – x0)2 + (y – y0)2 + d2 = 0
0 0

S
TE

a) x – y + 2 = 0. c) x + y + 2 = 0. b) (x – x0)2 + (y – y0)2 = d2

SO
b) x – y – 2 = 0. d) x + y – 2 = 0. c) (x – x0)2 + (y – y0)2 = 2d
MA

d) y – y0 = d(x – x0)
05. (EEAR) Se uma circunferência tem centro C(1,0) e raio 1 e outra

R
tem equação x² + y² – 2x – 8y + 8 = 0, então essas circunferências são 04. (EEAR) Seja (x – 1)² + (y – 6)² = 25 a equação reduzida de uma
a) secantes. c) tangentes internas. circunferência de centro C (a,b) e raio R. Assim, a + b + R é igual a
b) externas. d) tangentes externas. a) 18 c) 12
b) 15 d) 9
06. (EEAR) Para que a reta de equação y = 3 x + n seja tangente à R
circunferência de equação x² + y² = 4, o valor de n deve ser 05. (ESA) A equação da circunferência de centro (1,2) e raio 3 é:
MA

SO

a) – 3 ou 3. c) – 3 ou 3. a) x² + y² – 2x – 4y + 14 = 0
b) – 2 ou 2. d) – 4 ou 4. b) x² + y² – 2x – 4y – 4 = 0
c) x² + y² – 4x – 2y – 4 = 0
T

07. (EEAR) Se a distância entre uma reta t e o centro da circunferência


R d) x² + y² – 4x – 2y – 14 = 0
E

(λ) x² + (y – 2)² = 16 é 17 , então t e λ são


IA F
e) x² + y² – 2x – 4y – 14 = 0
a) secantes. c) exteriores.
L D O06. (EEAR) O
RPara que uma circunferência λ : x² + y² – mx – 4y – c = 0
tenhaPcentro C(1,2) e raio R = 5, os valores de m e de c são
b) tangentes. d) interiores.

08. (EEAR) As posições dos pontos A(1,7) e B(7,1) em relação à respectivamente


circunferência de equação (x – 6)² + (y – 2)² = 16 são, respectivamente, a) –1 e –10 c) 1 e –20
a) interna e interna. b) –2 e 25 d) 2 e 20
b) interna e externa.
c) externa e interna. 07. (CFT) A circunferência de centro (a,0) passa pela origem do sistema
de eixos cartesianos. Sendo a > 0, a equação da circunferência é
d) externa e externa.
a) x² + y² – 2ax = 0.
09. O maior valor inteiro de k para que a equação b) x² + y² – 2ay = 0.
x 2 + y 2 + 4 x − 6y + k = 0 represente uma circunferência é c) x² + y² + 2ax = 0.
a) 14 c) 12 e) 8 d) x² + y² + 2ay = 0.
b) 13 d) 10
08. (EEAR) Seja O o centro da circunferência α : (x – 1)² + (y – 3)² = 9.
10. Uma corda é determinada pela reta x – y = 0 sobre a circunferência O ponto P(3,2) é
(x – 2)² + (y + 2)² = 16. A área da menor região determinada por essa a) interior a α, estando mais próximo de α do que de O.
corda e o círculo é: b) interior a α, estando mais próximo de O do que de α.
a) 4π – 8 c) 4π – 2 c) pertencente a α.
b) 4π – 16 d) 4π – 4 d) exterior a V.

PROMILITARES.COM.BR 213

PM_BOOK05_MAT.indb 213 01/11/2021 13:50:04


GEOMETRIA ANALÍTICA: CIRCUNFERÊNCIA

09. (CFT) O centro e o raio da circunferência de equação x² + (y – 1)² = 4 06. (ESA) Em um sistema de coordenadas cartesianas no plano,
são, respectivamente, considere os pontos O(0,0) e A(8,0). A equação do conjunto dos
a) (–1,0) e 4. c) (0,1) e 4. pontos P(x,y) desse plano sabendo que a distância de O a P é o triplo
da distância de P a A, é uma
b) (1,0) e 2. d) (0,1) e 2.
a) circunferência de centro (9,0) e raio 3.
10. (ESA) A reta y = mx + 2 é tangente à circunferência de equação b) elipse de focos (6,0) e (12,0), e eixo menor 6.
(x – 4)² + y² = 4. A soma dos possíveis valores de m é c) hipérbole de focos (3,0) e (15,0), e eixo real 6.
a) 0. d) parábola de vértice (9,3), que intercepta o eixo das abscissas nos
4 pontos (6,0) e (12,0).
b) .
3 e) reta que passa pelos pontos (6,0) e (9,3).
4
c) − . 07. (AFA) A equação da circunferência de raio 5, concêntrica à
3
circunferência de Equação x2 + y2 – 4x – 2y + 3 = 0, é:
3
d) − . a) x2 + y2 – 4x – 2y + 20 = 0
4
e) 2. b) x2 + y2 – 4x – 2y – 15 = 0
c) x2 + y2 – 4x – 2y = 0
d) x2 + y2 – 4x – 2y – 20 = 0
EXERCÍCIOS DE

COMBATE
08. (AFA) A reta s: y = –x + 4 intercepta a circunferência C : x2 + y2 +

L DO PRO 2x – 4y – 4 = 0 nos pontos P e Q. Se O é o centro de C, então a área


do triângulo OPQ, em unidades de área, é

I A a) 4 Fb)E 5 c) 4,5 d) 5,5

R
01. (ESA) As equações (x + 1)² + (y – 4)² = 64 e (x – 4)² + (y + 8)² = 25
representam duas circunferências cuja posição relativa no plano 09. (ESPCEX) O ponto da circunferência x² + y² + 2x + 6y + 1 = 0 que

S
TE

permite afirmar que são: tem ordenada máxima é

SO
a) interiores (sem ponto de intersecção). a) (0,–6) d) (2,3)
MA

b) tangentes interiores. b) (–1,–3) e) (2,–3)


c) (–1,0)

R
c) secantes.
d) tangentes exteriores.
10. (AFA) Os vértices de um triângulo ABC são os centros das
e) exteriores (sem ponto de intersecção). circunferências:
(λ1) x2 + y2 + 2x – 4y – 1 = 0
02. (ESA) Dada a equação da circunferência (x – a)² + (y – b)² = r²,
sendo (a;b) as coordenadas do centro e r a medida do raio, identifique (λ2) 4x2 + 4y2 + 12x – 8y – 15 = 0
R
a equação geral da circunferência de centro (2;3) e raio igual a 5. (λ3) (x – 7)2 + (y + 3)2 = 8
MA

SO

a) x² + y² – 4x – 6y – 12 = 0 O tetraedro cuja base é o triângulo ABC e cuja altura, em metros, é


b) x² – 4x = –16 igual à média aritmética dos quadrados dos raios das circunferências
acima, também em metros, possui volume, em m3, igual a
c) x² + y² = 25
T

a) 21/2 c) 49/2
R
d) y² – 6y = –9
E

b) 21/4 d) 49/4
E

e) x² + y² – 4xy – 12 = 0
IA F
L λD O
03. (EEAR) Dados os pontos B(1,2) e C(0,1) uma circunferência
equação x² + y² – 3x – 4 = 0, é correto afirmar que
de
PR
O RESOLUÇÃO EM VÍDEO
Abra o ProApp, leia o QR Code, assista à resolução
a) B é interior a λ e C é exterior a λ. de cada exercício e AVANCE NOS ESTUDOS!
b) B é exterior a λ e C é interior a λ.
c) B e C são exteriores a λ.
d) B e C são interiores a λ. GABARITO
EXERCÍCIOS DE FIXAÇÃO
04. (EEAR) Considere a circunferência (x – 2)² + (y – 4)² = 9 e uma
reta t secante a ela. Uma possível distância entre r e o centro da 01. D 04. B 07. C 10. A
circunferência é 02. A 05. D 08. C
a) 5,67 03. A 06. D 09. C
b) 4,63 EXERCÍCIOS DE TREINAMENTO
c) 3,58 01. C 04. C 07. A 10. C
d) 2,93 02. A 05. B 08. A
03. B 06. D 09. D
05. (EEAR) Seja a circunferência de centro (0,–2) e raio 5 . Se (k, 0) EXERCÍCIOS DE COMBATE
pertence à circunferência, sendo k > 0, o valor de k é 01. D 04. D 07. D 10. D
a) 0 c) 2 02. A 05. B 08. C
b) 1 d) 3 03. D 06. A 09. C

214 PROMILITARES.COM.BR

PM_BOOK05_MAT.indb 214 01/11/2021 13:50:05


GEOMETRIA ANALÍTICA:
CÔNICAS

TRANSLAÇÃO DE EIXOS Daí nós percebemos que se, a partir do plano cartesiano x.y,
voltarmos x0 em x e voltarmos y0 em y estaremos novamente
COORDENADOS trabalhando com o centro do eixo cartesiano x.y. Sendo assim se
Antes de introduzirmos os conceitos e equações gerais das cônicas tivermos por exemplo uma elipse com centro em (x0,y0) esse será o
vamos primeiramente entender como podemos facialmente transladar centro do plano cartesiano xA·yA e para chegarmos no centro do plano
nossos eixos coordenados de forma que sempre possamos trabalhar x − x 0
cartesiano x·y teremos que fazer  .
no centro (0,0) do eixo cartesiano. Assim nossas demonstrações e y − y 0
entendimento será facilitado.
Imagine que temos uma cônica que o centro não se encontra no

DO PRO
centro (0,0) do nosso eixo cartesiano, mas sim possui centro no ponto
(x0,y0).

A L
I FE
R

S
TE

SO
MA

R
R
MA

SO
T

R
E

IA F
Assim para demonstrações podemos considerar qualquer centro

L DO PRO
Sendo assim vamos criar os eixos cartesianos xA e yA que terá x − x 0
como centro o ponto (x0,y0).. (x0,y0) que sabemos que na verdade em relação a x.y teremos  .
y − y 0

ELIPSE
A elipse é o lugar geométrico dos pontos plano cuja soma das
distâncias a 2 pontos fixos (focos) é sempre constante e igual a 2a
(eixo maior).

PROMILITARES.COM.BR 215

PM_BOOK05_MAT.indb 215 01/11/2021 13:50:05


GEOMETRIA ANALÍTICA: CÔNICAS

Sendo assim, ao escolhermos qualquer ponto que esteja RELAÇÃO FUNDAMENTAL DA ELIPSE
sobre a elipse, por exemplo P1, ao traçarmos a distância de P1 Utilizando a propriedade da elipse como lugar geométrico.
a um dos focos e a distância de P1 ao outro foco e somarmos
essas distâncias encontraremos sempre o mesmo valor 2a, que
é exatamente o valor do eixo maior da elipse.

1 1 + PF
PF 12 =
2a
P2F1 + P2F2 =
2a
P3F1 + P3F2 =
2a

PnF1 + PnF2 =
2a

ELEMENTOS DA ELIPSE
Toda elipse possui um centro que chamaremos de (x0,y0), paralelos
 
aos eixos Ox e Oy passando pelo centro (x0,y0) teremos o eixo
maior A1A 2 , no mesmo segmento de reta a distância focal FF 1 2 e no
segmento de reta perpendicular o eixo menor B1B2 . Considerando o ponto B1 teremos que B1F1 + B1F2 = 2a , porém o

L D O PObservação: 1 = F2O e A1A 2 ⊥ B1B2 .


triângulo F1B1F2 é isósceles, FO
R O O = centro (x ,y ) da elipse
I A Assim comF o triângulo F B F sendo isósceles teremos
0 0

R B F = B F ⇒ B F = BEF = a .Daí podemos aplicar o teorema de


1 1 2

11 12 11 12
Pitágoras no triângulo retângulo encontrando a relação fundamental

S
TE

da elipse.

SO
MA

2
a= b2 + c2

R
EQUAÇÃO FUNDAMENTAL DA ELIPSE
Fazendo a elipse com centro em (0,0) de um eixo cartesiano
1 1 + PF
transladado e aplicando que PF 12 =
2a , onde o ponto P1 = (x,y) é
um ponto qualquer do plano, teremos:
R
1 1 + PF
1 2 = 2a ⇒ (x − ( −c)) + (y − 0) + (x − (c)) + (y − 0) = 2a ⇒
2 2 2 2
PF
MA

Observação, também podemos ter a elipse “de pé”, falaremos


SO

(x + c))2 + (y − 0)2 = 2a − (x − c)2 + (y − 0)2 ⇒


mais sobre ela mais a frente.
A1A 2 → Eixo maior, representado por 2a. (x 2
+ 2xc + c2 ) + y 2 = [2a − (x 2
− 2xc + c2 ) + y 2 ]2 ⇒

(x ) ( )
y 2  4a2 + x 2 − 2xc + c2 + y 2 − 4a ( x 2 − 2xc + c2 ) + y 2  ⇒
T

2
+ 2xc + c2 +=
S

B1B2 → Eixo menor, representado por 2b.  


R
E

( )
E

4 xc − 4 a= [4a x − 2xc + c ) + y ] ⇒ ( xc − a2 )= a2 ( x 2 − 2xc + c2 ) + y 2 ⇒


( F
2
1 2 → Distância focal, representada por 2c.
FF
I A a partir
2 2 2 2

L D O ( P )R O (
Perceba que na elipse deitada esse será o funcionamento x c −2a xc + a = a x −2a xc + a c + a y ⇒ a x − c x + a y =a − a c ⇒
2 2 2 4 2 2 2 2 2 2 2 2 2 2 2 2 2 4 2 2

do centro (x ,y ).
0 0
x 2 a2 − c2 + a2y 2= a2 a2 − c2 ) ⇒
A= x − a
1 0
Lembre que a2 = b2 + c2 ⇒ x 2b2 + a2y 2 = a2b2 ÷ a2b2 ⇒
A=
2 x0 + a
x2 y2
B= yo + b + =
1
1 a2 b2
B=
2 y0 − b Voltando para o plano xy original
F=
1 x0 − c
F= x0 + c
( x − x 0 )2 + ( y − y 0 )2 =
1
2
a2 b2
Na elipse em pé
A= y0 − a Essa é a equação da elipse de eixo maior horizontal

1

A=
2 y0 + a (A A
1 2  Ox ) , percebemos que o valor a (o maior valor), irá
“aparecer” no denominador de x.
B=
1 x0 − b
B=
2 x0 − b
F=
1 y0 − c
F=
2 y0 + c

216 PROMILITARES.COM.BR

PM_BOOK05_MAT.indb 216 01/11/2021 13:50:07


GEOMETRIA ANALÍTICA: CÔNICAS

Assim ficamos com a equação 4x 2 + 9y 2 − 16x − 18y − 11 =


0 que
da forma que está não conseguimos identificar as coordenadas do
centro ou os valores de a e de b.
Para chegarmos novamente a equação reduzida deveremos
usar o mesmo artifício que utilizamos na equação da circunferência,
completar quadrados.
4x 2 + 9y 2 − 16x − 18y − 11 =0 ⇒ 4x 2 − 16x + 9y 2 − 18y − 11 =0⇒

( ) ( ) ( ) (
4 x 2 − 4x + 9 y 2 − 2y − 11 =0 ⇒ 4 x 2 − 4 x + 4 − 4 + 9 y 2 − 2y + 1− 1 − 11 =
0⇒ )
( ) ( ) ( ) (
4 x 2 − 4x + 9 y 2 − 2y − 11 =0 ⇒ 4 x 2 − 4 x + 4 − 4 + 9 y 2 − 2y + 1− 1 − 11 =
0⇒ )
(( ) ) (( ) ) ( ) (
4 x 2 − 4x + 4 − 4 + 9 y 2 − 2y + 1 − 1 − 11 =0 ⇒ 4 ( x − 2) − 4 + 9 ( y − 1) − 1 − 11 =
2
0⇒
2
)
(( ) ) (( ) ) ( ) (
4 x 2 − 4x + 4 − 4 + 9 y 2 − 2y + 1 − 1 − 11 =0 ⇒ 4 ( x − 2) − 4 + 9 ( y − 1) − 1 − 11 =
2
0⇒
2
)
4 ( x − 2) − 16 + 9 ( y − 1) − 9 − 11 =0 ⇒ 4 ( x − 2) + 9 ( y − 1) = 36 ⇒
2 2 2 2

A equação da elipse de eixo maior vertical ( A1A 2  Oy ) é
( x − x0 ) 2
( y − y0 ) 2
4 ( x − 2) + 9 ( y − 1) = 36 ÷ 36 ⇒
2 2 ( x − 2)2 + ( y − 1)2 = 1
+ =
1
D O PAssim
9 4
b2 a2
L R teríamos
O
O valor a (o maior valor), irá “aparecer” no denominador de y.
I A FE
R

S
TE

SO
MA

R
R
MA

SO

O(2,1)
A1( −1, −1)
A 2 (5, −1)
T

B1(2, −1)
R
E

IA B2 (2,3)
F
L DO (
F1 2 − 5,1
R O)
P
F ( 2 + 5,1)
2

Lembrando:
EQUAÇÃO COMPLETA
I. Elipse centrada na origem com eixo maior paralelo ao eixo x
=
( x − 2)2 + ( y − 1)2
1,
Temos por exemplo a equação da elipse x2 y2
9 4 + =
1
de onde podemos perceber que possui o centro em (2,1) e a² = 9 ⇒ a2 b2
a = 3 e b² = 4 ⇒ b = 2. Assim vemos que é uma elipse horizontal onde
o eixo maior mede 6 e o eixo menor mede 4. Vamos desenvolver a II. Elipse centrada na origem com eixo maior paralelo ao eixo y
equação.
( x − 2)2 + ( y − 1)2 x2 y2
1 x 36 ⇒ 4 ( x − 2) + 9 ( y − 1)= 36 ⇒
=
2 2 + =
1
9 4 b2 a2

( ) ( )
4 x − 4x + 4 + 9 y 2 − 2y + 1 =36 ⇒ 4x 2 − 16x + 16 + 9y 2 − 18y + 9 =36 Para você que gosta de fórmulas, vamos tentar comprimir a
2

equação completa de forma totalmente literal.


) ( )
+ 4 + 9 y 2 − 2y + 1 =36 ⇒ 4x 2 − 16x + 16 + 9y 2 − 18y + 9 =36 Ax 2 + By 2 + Cx + Dy + E = 0 ⇒ Ax 2 + Cx + By 2 + Dy + E = 0 ⇒

4x 2 + 9y 2 − 16x − 18y + 25 =36 ⇒ 4x 2 + 9y 2 − 16x − 18y − 11 =0  C   D 


A  x2 + x  + B  y2 + y  + E = 0 ⇒
 A   B 

PROMILITARES.COM.BR 217

PM_BOOK05_MAT.indb 217 01/11/2021 13:50:08


GEOMETRIA ANALÍTICA: CÔNICAS

 C 
2
C2   D
2
D2  ELEMENTOS DA HIPÉRBOLE
A x +  −  + B  y +  − 2  + E = 0 ⇒
 2A  4A 2  2B  4B 
  Eixo real – O segmento que une os pontos mais próximos da
2 2
hipérbole.
 C  C2  D  D2
Ax +  − + B y +  − +E = 0 ⇒ Distância focal – Distância entre os focos principais da hipérbole.
 2A  4A  2B  4B
Eixo imaginário – Traçamos uma circunferência de centro no centro
2 2
 C   D C2 D2 da hipérbole e raio igual a OF1 . Por A1 traçamos uma perpendicular
Ax +  + B y +  = + −E ⇒
 2A   2B  4A 4B até que intercepte a circunferência, esse será o tamanho do eixo
imaginário. O segmento paralelo passando pelo centro O será o eixo
2 2
 C   D imaginário.
Ax +  B y + 
 2A   2B 
+ =
1⇒
BC2 + AD2 − 4ABE BC2 + AD2 − 4ABE
4AB 4AB
2 2
 C   D
x +  B y + 
 2A   2B 
+ =
1⇒
BC2 + AD2 − 4ABE BC2 + AD2 − 4ABE
2 2
4A B 4AB
C D
x0 =
− ;y 0 =

2A 2B

Parece complicado? Mas encontramos o centro de maneira


rápida, vamos ver com o exemplo anterior.
L DO PRO
4x 2 + 9y 2 − 16x − 18y − 11 =
0⇒A =
4;B =
9;C =
−16;D =
−18 e E =
−11
I A FE
x0 =

C
2A
=

( −16) 16
2.4
==
8
2 ; y0 =

D
2B
=

( −18) 18
2.9
= =
18
1 R

S
TE

BC2 + AD2 − 4ABE 9( −16)2 + 4( −18)2 − 4.4.9.( −11) 2304 + 1296 + 1584

SO
= = = 9
4A 2B 4(4)2.9 576
MA

BC2 + AD2 − 4ABE 9( −16)2 + 4( −18)2 − 4.4.9.( −11) 2304 + 1296 + 1584
= = = 4

R
4AB2 4.4.92 1296

Assim a² = 9 e b² = 4.

RELAÇÃO FUNDAMENTAL DA HIPÉRBOLE


HIPÉRBOLE
Da figura que vemos os elementos da hipérbole percebemos que
A hipérbole é o lugar geométrico dos pontos do plano cuja o raio da circunferência (c) será a hipotenusa do triângulo retângulo,
R
diferença das distâncias a 2 pontos fixos (focos) é sempre constante e dessa forma para a hipérbole nossa relação será
MA

igual a 2a (eixo real).


SO

2
c= a2 + b2
T

R EQUAÇÃO FUNDAMENTAL DA HIPÉRBOLE


E

IA F
Fazendo da mesma maneira que na elipse teremos a hipérbole

L DO PRO centrada na origem (0, 0) e aplicaremos a definição PF


partir de um ponto qualquer P(x, y).
1 2 = 2a a
1 1 - PF

PF 12 =
1 1 - PF 2a ⇒ ( x - (-c) )2 + ( y - 0 )2 - ( x - (c) )2 + ( y - 0 )2 =
2a

Deixo então o desenvolvimento para que seja feito por vocês da


mesma maneira que na elipse, lembrando apenas que na hipérbole
c² = a² + b².
Depois de desenvolver nossa equação teremos:

Hipérbole centrada na origem de eixo real paralelo a Ox

x2 y2
− =
1
a2 b2


Hipérbole centrada na origem de eixo real paralelo a Oy

y2 x2
− =
1
a2 b2
1 1 − PF
PF 1 2 = P2F1 − P2F2 = P3F1 − P3F2 = ... = PnF1 − PnF2 = 2a
Fazendo a translação de eixos coordenados teremos as equações
de hipérboles com centro em qualquer ponto.

218 PROMILITARES.COM.BR

PM_BOOK05_MAT.indb 218 01/11/2021 13:50:10


GEOMETRIA ANALÍTICA: CÔNICAS


Eixo real paralelo a Ox EQUAÇÃO COMPLETA
( x − 3)2 − ( y + 2)2
( x − x 0 )2 − ( y − y 0 )2 =
1
Temos por exemplo a equação da hipérbole
9 16
=1
a2 b2 de onde podemos perceber que possui o centro em (3,-2) e a² = 9 ⇒
a = 3 e b² = 16 ⇒ b = 4. Assim vemos que é uma hipérbole com eixo
 
Eixo real paralelo a Oy real paralelo a Ox . Vamos desenvolver a equação.
( x − 3)2 − ( y + 2)2 1 x 144 ⇒ 16 ( x − 3) − 9 ( y + 2) =
2 2
( y − y 0 )2 − ( x − x 0 )2 =
1 9 16
= 144 ⇒
a2 b2
( ) ( )
16 x 2 − 6x + 9 − 9 y 2 + 4 y + 4 = 144 ⇒ 16x 2 − 96x + 144 − 9y 2 − 36y − 36 = 144 ⇒
Na equação da hipérbole não devemos ter atenção quanto ao
tamanho dos eixos, mas sim quanto 16 aos (
x 2 −sinais.
6x + 9 Aquele ) (
− 9 y 2 +que )
4 = o144 ⇒ 16x 2 − 96x + 144 − 9y 2 − 36y − 36 = 144 ⇒
4 y +tiver
sinal positivo terá o eixo real e o que tiver sinal negativo terá o eixo
imaginário. Dessa forma quando o sinal negativo “estiver no x” o eixo 16x 2 − 9y 2 − 96x − 36y − 36 = 0
imaginário estará paralelo ao eixo x e quando o sinal negativo “estiver
no y” o eixo imaginário estará paralelo ao eixo x, independente dos
Assim ficamos com a equação 16x 2 − 9y 2 − 96x − 36y − 36 = 0
valores dos denominadores.
 que da forma que está não conseguimos identificar as coordenadas
Hipérbole de eixo real paralelo a Ox . do centro ou os valores de a e de b.
Para chegarmos novamente à equação reduzida deveremos

DO PRO
usar o mesmo artifício que utilizamos na equação da circunferência,

L completar quadrados.

IA 16x − 9y − 96xF− 36y − 36 =⇒


2 2 2 2
0 16x − 96x − 9y − 36y − 36 =⇒ 0

R E
( 2
) ( 2
) (
16 x − 6x − 9 y + 4y − 36 =0 ⇒ 16 x − 6x + 9 − 9 − 9 y + 4 y + 4 − 4 − 36 =0 ⇒2
) ( 2
)

S
TE

( ) ( ) ( ) (
16 x 2 − 6x − 9 y 2 + 4y − 36 =0 ⇒ 16 x 2 − 6x + 9 − 9 − 9 y 2 + 4 y + 4 − 4 − 36 =0 ⇒ )

SO
MA

(( ) ) (( ) ) (
16 x 2 − 6x + 9 − 9 − 9 y 2 + 4y + 4 − 4 − 36 =0 ⇒ 16 ( x − 3) − 9 − 9 ( y + 2) − 4 − 36 =
2
) ( 2
)

R
(( ) ) (( ) ) ( ) (
16 x 2 − 6x + 9 − 9 − 9 y 2 + 4y + 4 − 4 − 36 =0 ⇒ 16 ( x − 3) − 9 − 9 ( y + 2) − 4 − 36 =
2
0⇒
2
)
16 ( x − 3) − 144 − 9 ( y + 2) + 36 − 36 =0 ⇒ 16 ( x − 3) − 9 ( y + 2) =144 ⇒
2 2 2 2

16 ( x − 3) − 9 ( y + 2) =
2 2
144 ÷ 144 ⇒
R ( x − 3)2 − ( y + 2)2 =
1
MA

A1 = x0 – a B2 = y0 + b 9 16
SO

A2 = x0 + a F1 = x0 – c O processo é o mesmo da elipse, só devemos ter atenção com o


sinal negativo, pois não deveremos subtrair o quadrado do segundo
B1 = y0 – b F2 = x0 + c termo mas sim somar, como visto no processo (-9)(-4) = 36.
T


R Assim nossa hipérbole será
E

Hipérbole de eixo real paralelo a Oy .


IA F
c2 = a2 + b2 ⇒ c2 = 9 + 16 = 25 ⇒ c = 5

L DO PRO

A1 = y0 – a B2 = X0 + b
A2 = y0 + a F1 = y0 – c
B1 = X0 – b F2 = y0 + c

PROMILITARES.COM.BR 219

PM_BOOK05_MAT.indb 219 01/11/2021 13:50:12


GEOMETRIA ANALÍTICA: CÔNICAS

O(3, −2)
A1(0, −2)
A 2 (6, −2)
B1(3, −6)
B2 (3,2)
F1 ( −2, −2)
F2 ( 8, −2)

HIPÉRBOLE EQUILÁTERA
A hipérbole equilátera é aquela em que os eixos real e imaginário
possuem as mesmas dimensões. Por exemplo x2 – y2 = 1 é um
x2 y2
hipérbole equilátera pois − =1 e daí a² = b² = 1 ⇒ a = b = 1 e
1 1 =
PO
1 P1A1; P=
2O P2A 2 ;P= = P=
3O P3A 3 = ... nO PnAn
c2 = a2 + b2 = 1+ 1 ⇒ c2 = 2 ⇒ c = 2 , assim temos A1A 2 = B1B2 .

ELEMENTOS DA PARÁBOLA

L DO PRO
I A FE
R

S
TE

SO
MA

R
R
MA

SO
T

R
E

EXCENTRICIDADE
E

IA F
A excentricidade mede o achatamento de uma cônica e é
c
simbolizado por e, o seu cálculo se dá por e = . L D O PF R O
– foco
I. Pontos principais:

a
V – vértice
e= 1 → circunferência
II. Segmentos:
e < 1 → elipse
V'F = p – parâmetro (semicorda focal mínima)
e > 1 → hipérbole 
FP – raio vetor

III. Relação:
PARÁBOLA p
A parábola é o lugar dos geométricos dos pontos de um plano VF =
2
equidistantes de um ponto fixo e de uma reta fixa do mesmo plano. O
ponto fixo denomina-se foco (F) e a reta fixa, diretriz (d). IV. Reta e eixo: a reta fixa (d) é a diretriz e e, eixo que passa
Definição geométrica: é a cônica obtida mediante a secção de um pelo foco e é perpendicular à diretriz, eixo de simetria da
plano secante a um cone quadrático, sendo o plano paralelo a uma e parábola.
somente uma geratriz do cone.

220 PROMILITARES.COM.BR

PM_BOOK05_MAT.indb 220 01/11/2021 13:50:13


GEOMETRIA ANALÍTICA: CÔNICAS

EQUAÇÃO DA PARÁBOLA 


• Reta diretriz paralela a Ox

*Concavidade para baixo

Fazendo o ponto genérico (x,y) e utilizando um sistema cartesiano V ( x 0 ,y 0 )


adequado de forma que o vértice esteja centrado na origem(0,0) o  p
F  x 0 ,y 0 + 
p
foco F será F  0,  . Vamos aplicar distância entre 2 pontos.  2
 2 p
2 =
d y0 −
 p 2
DO PRO
d= (x − 0)2 +  y − 
 2

A L 
−p I FE
• Reta diretriz paralela a Oy
A reta diretriz tem equação y =
2 R distância do
, vamos aplicar
ponto (x,y) a reta diretriz e igualar a mesma distância d.

S
TE

SO
p p p
y =− ⇒ y + =0 ⇒ a =0,b =1e c =
MA

2 2 2
p

R
0.x + 1.y +
ax 0 + by 0 + c 2 p
d= = = y+
a2 + b2 0 + 12 2

Igualando as distâncias

 p
2
p  p p2
2 R
(x − 0)2 +  y −  =y + ⇒ x 2 +  y − =
 y 2 + py + ⇒
MA

 2 2  2 4
SO

p2 p2
x 2 + y 2 − py + = y 2 + py + ⇒ x2 =
2py
4 4
T

R
E

Dessa forma nossas equações de parábolas com vértice na origem


E

serão IA F
I. Parábola com reta diretriz paralela a Ox

L DO PRO
x 2 = 2py


II. Parábola com reta diretriz paralela a Oy
*Concavidade para esquerda
y 2 = 2px V ( x 0 ,y 0 )
 p 
Fazendo a translação de eixos adequada teremos F  x 0 + ,y 0 
 2 

III. Parábola com reta diretriz paralela a Ox p
=
d x0 −
2
( x − x 0 )2 = 2p ( y − y 0 )
Por exemplo a parábola de equação (x + 1) = (y – 3)², teremos o
 1 p 1
IV. Parábola com reta diretriz paralela a Oy número 1 multiplicando (y – 3)², daí 2p =1 ⇒ p = ⇒ = . Teremos
2 2 4

uma parábola com reta diretriz paralela ao eixo Oy .
( y − y 0 )2 = 2p ( x − x 0 )
V ( −1,3)
Vamos encontrar os elementos da parábola.

PROMILITARES.COM.BR 221

PM_BOOK05_MAT.indb 221 01/11/2021 13:50:15


GEOMETRIA ANALÍTICA: CÔNICAS

 p   1   3  CIRCUNFERÊNCIA
F  −1+ ,3  = F  −1+ ,3  = F  − ,3  A equação de uma circunferência sempre possui os coeficientes
 2   4   4 
de x2 e de y2 iguais, dessa forma toda equação que E = 0 e A = B será
equação de uma circunferência.
p 1 5
d =−1− =−1− =−
2 4 4
ELIPSE
A equação de uma elipse sempre possui os coeficientes de x2 e
de y2 diferentes, porém sempre com os mesmos sinais. Dessa forma
toda equação que E = 0; A ≠ B e A·B > 0 será equação de uma elipse.

HIPÉRBOLE
A hipérbole pode-se dizer que é a mais fácil de se identificar pelo
sinal negativo, dessa forma devemos ter E = 0; A·B < 0. Perceba que
não necessariamente devemos ter A ≠ B pois na hipérbole equilátera
A = B.

PARÁBOLA
A parábola também é fácil de ser identificada, pois não possui x2
e y2 ao mesmo tempo na equação, apenas um ou outro, dessa forma

D O Exercício
E = 0; A = 0 ou B = 0.

L P R OResolvido
IA F E plano cartesiano, considere a circunferência de
R 01. (UNICAMP) No
equação x² + y² –4y + 3 = 0 e a parábola de equação 3x² – y + 1 = 0.

S
TE

Essas duas curvas se interceptam em

SO
a) um ponto.
MA

b) dois pontos.
c) três pontos.

R
d) quatro pontos.

EQUAÇÃO COMPLETA Resolução: C


Vamos expandir a equação do exemplo anterior. Se y = 3x² + 1, então
(x + 1) = ( y − 3)2 ⇒ x + 1 = y 2 − 6y + 9 ⇒ x = y 2 − 6y + 8 R
x 2 + (3x 2 + 1)2 − 4(3x 2 + 1) + 3= 9x 4 − 5x 2= x 2 (9x 2 − 5) ⇒
MA

Dessa forma final x = y² – 6x + 8 não sabemos as coordenadas


SO

do vértice ou do foco ou o valor do parâmetro p, apenas sabemos  5  5


 9x 2  x −   x +  .
que se trata de uma parábola de reta diretriz paralela a Oy (o y está  3  3 
ao quadrado) e possui concavidade para direita (o coeficiente de y² é
T

Portanto, as duas curvas se intersectam em três pontos cujas


S

positivo). Vamos usar a mesma técnica usada em todas as equações de


R
E

5 5
E

F
cônicas, completar quadrados. abscissas são x = 0, x =
IA
x = y 2 − 6y + 8 + 1− 1 ⇒ x + 1 = y 2 − 6y + 9 ⇒ (x + 1) = (y − 3)2
O
3
e x= −
3
.

L Da O P R
O que para a equação da parábola se torna bem mais imediato,
partir daí identificamos os valores necessários. Exercício Resolvido

ProBizu 02. (UECE) No plano, com o sistema de coordenadas cartesianas


usual, a equação x² + 4y² = 4x representa
As fórmulas de x do vértice e y do vértice aprendidas no módulo
de função quadrática também funcionam em geometria analítica, a) uma circunferência.

porém cuidado com a parábola de reta diretriz paralela ao eixo Oy, b) duas retas.
pois nela as nossas coordenadas do vértice serão trocadas. c) uma parábola.

Parábola de reta diretriz paralela ao eixo Oy d) uma elipse.
∆ b
xv =
− e yv =

4a 2a Resolução: D
Completando os quadrados, obtemos
x 2 + 4y 2 =
4x ⇔ x 2 − 4x + 4y 2 =
0 ⇔ (x − 2)2 + 4y 2 =
4
IDENTIFICAÇÃO DE UMA CÔNICA
(x − 2)2 (y − 0)2
De maneira geral podemos ter a seguinte equação para as cônicas. ⇔ + =
1.
2
2 12
2 2
Ax + By + Cx + Dy + Exy + F =0
Trata-se de uma elipse com centro em (2,0) e eixo maior paralelo
ao eixo das abscissas.
No nosso nível de estudo não haverá cônicas com termo xy, então
dessa forma E = 0.

222 PROMILITARES.COM.BR

PM_BOOK05_MAT.indb 222 01/11/2021 13:50:16


GEOMETRIA ANALÍTICA: CÔNICAS

Exercício Resolvido

03. (UDESC) A área delimitada por uma elipse cuja equação é


x2 y2
+ = 1 é dada por A = abπ. Então, a área da região situada
a2 b2
entre as elipses de equações 16x² + 25y² = 400 e 16x² + 9y² = 144
é:
a) 12 µ u.a.
b) 20 µ u.a.
c) 8 µ u.a.
d) 256 µ u.a.
e) µ u.a.

Resolução: C
Reescrevendo as equações das elipses, obtemos
x2 y2
16x 2 + 25y 2 = 400 ⇔ 2 + 2 = 1 e
5 4
x2 y2
16x 2 + 9y 2 = 144 ⇔ + = 1.
32 42
L DO PRO
A
Logo, traçando os gráficos dessas elipses, vem
I FE
Podemos ver que se r = 1 a circunferência e a hipérbole serão
R tangentes nos pontos (1,0) e (-1,0), fazendo com que a opção C
esteja incorreta.

S
TE

Para r = 17 que é maior que 1 teremos intersecções em 4 pontos,

SO
fazendo com que a resposta seja a opção E, mas antes vamos ver
MA

porque a opção D está incorreta.


x + y =

R
2 2
17
( )
2
x 2 + y 2 = 17 ⇒ x2 + y2 =
17 ⇒  + ⇒ 2x 2 =
18 ⇒ x 2 =⇒
9 x=
±3
2 2
x − y =1
x 2 + y 2 = 17
( )
2
x 2 + y 2 = 17 ⇒ x2 + y2 =
17 ⇒  + ⇒ 2x 2 =
18 ⇒ x 2 =⇒
9 x= ±3
2 2
 x − y =
1
3 + y =17 ⇒ y =17 − 9 ⇒ y 2 =8 ⇒ ±2 2
2 2 2

Assim intersecções em 4 pontos


R
MA

(3,2 2 ) ;( −3,2 2 ) ;(3, −2 2 ) e ( −3, −2 2 ) .


SO
T

Exercício Resolvido
R
e, portanto, a área sombreada é dada por π ⋅ (5 ⋅ 4 − 4 ⋅ 3) = 8π u.a.
E

IA F
05. (UDESC) Analise as afirmações dadas a seguir, classifique-as

Exercício Resolvido
O
como verdadeiras (V) ou falsas (F).
L D O ( )PA equação
R x - 2x + y + 2y + 1 = 0 representa uma
2 2

circunferência que é tangente, tanto ao eixo das abscissas quanto


04. (UEL) Considere o círculo x² + y² – r² = 0 de raio r e a hipérbole ao eixo das ordenadas.
x² – y² = 1. Nesse caso, pode-se afirmar que:
( ) A elipse de equação 9x2 + 4y2 = 36 intercepta a hipérbole de
a) Se r < 1, então as curvas se intersectam em quatro pontos. equação x2 - 4y2 = 4 em apenas dois pontos, que são os vértices
b) Se r = 1, então as curvas tem quatro pontos em comum. da hipérbole.
c) Se r = 1 , as curvas se intersectam em (0,1) e (0,-1). ( ) O semieixo maior da elipse 9x2 + 4y2 = 36 é paralelo ao eixo
real da hipérbole x2 - 4y2 = 4.
d) Se r = 17 então as curvas se intersectam apenas nos pontos
(3,2 2) e ( −3, −2 2) .
Assinale a alternativa que contém a sequência correta, de cima
e) Se r > 17 e então as curvas se intersectam em quatro pontos. para baixo.
a) V - V - V
Resolução: E b) V - V - F
A Circunferência é (x – 0)² + (y – 0)² = r² ⇒ O(0,0) e Raio = r, c) F-V-F
circunferência centrada na origem e raio r.
d) F - F - V
(x − 0)2 (y − 0)2 e) V - F - F
A hipérbole − =1 ⇒ a2 =b2 =1 ⇒ a =b =1 é uma
1 1
hipérbole equilátera centrada na origem.

PROMILITARES.COM.BR 223

PM_BOOK05_MAT.indb 223 01/11/2021 13:50:17


GEOMETRIA ANALÍTICA: CÔNICAS

Se desejarmos que as elipses de luz se tangenciem nas extremidades


Resolução: B dos eixos maiores, a distância, em metros, entre dois postes
I) Verdadeira. consecutivos deverá ser de aproximadamente:
x 2 − 2x + y 2 + 2y + 1 = 0 ⇔ (x − 1)2 + (y + 1)2 = 1 ⇒ C(1, − 1) e r = 1 Dado: 0,943² ≈ 0,889 e 0,111 ≈ 0,333
II) Verdadeira.

9x + 4y =
2
36
 2 2
⇒ ( −2, 0) e (2, 0)
x − 4y =4

x2
− y 2 =1 ⇒ a2 = 4 ⇒ a = 2 ⇒ A1( −2, 0) e A 2 (2, 0)
4
III) Falsa.
a) 35. c) 25. e) 15.
x2 y2
+ =1 ⇒ a2 =9 e b2 =4 ⇒ A1A 2 ⊂ Oy e B1B2 ⊂ Ox b) 30. d) 20.
4 9

x2 03. (ESPM) Seja A o vértice da parábola de equação y = x2 – 4x + 6.


− y 2 =1 ⇒ a2 =4 e b2 =1 ⇒ A'1 A'2 ⊂ Ox A reta que passa pela origem O do plano cartesiano e pelo ponto A
4
intercepta a parábola também num ponto B.
Portanto, A1A 2 ⊥ A'1 A'2 . Pode-se afirmar que:
d) AB = 3 . OA
D O b) PAOR= 2O. AB
a) AO = AB

L e) AO = 3 . AB

IA c) AB = 2 . OAF
EXERCÍCIOS DE

FIXAÇÃO R E
04. (UECE) No plano, com o sistema de coordenadas cartesiano

S
TE

usual, a área do quadrilátero convexo cujos vértices são os pontos de


interseção das elipses representadas pelas equações X2 + 2Y2 = 2 e

SO
01. (UFPB) A secretaria de infraestrutura de um município contratou 2X2 + Y2 = 2 é:
MA

um arquiteto para fazer o projeto de uma praça. Na figura a seguir, está u. a. ≡ unidade de área
o esboço do projeto proposto pelo arquiteto: uma praça em formato

R
retangular medindo 80 m x 120 m, onde deverá ser construído um 9 7
a) u. a. c) u. a.
jardim em forma de elipse na parte central. 2 3
8 5
b) u. a. d) u. a.
3 3
05. (UFRN) Um arquiteto projetou, para um salão de dimensões 22 m
R
por 18 m, um teto de gesso em formato de elipse com o eixo maior
MA

SO

medindo 20 m e o eixo menor, 16 m, conforme ilustra a figura abaixo.


T

R
E

IA F
L DO PRO
Estão destacados na figura os segmentos AC e BD que são,
respectivamente, o eixo maior e o menor da elipse, bem como os
pontos F1 e F2, que são os focos da elipse onde deverão ser colocados
dois postes de iluminação. Com base nessas informações, conclui-se
que a distância entre os postes de iluminação será, aproximadamente,
de:
a) 68 m c) 76 m e) 84 m O aplicador do gesso afirmou que saberia desenhar a elipse, desde
que o arquiteto informasse as posições dos focos. Para orientar o
b) 72 m d) 80 m
aplicador do gesso, o arquiteto informou que, na direção do eixo
maior, a distância entre cada foco e a parede mais próxima é de
02. (UNESP) A figura mostra a representação de algumas das ruas
de nossas cidades. Essas ruas possuem calçadas de 1,5m de largura, a) 3 m. c) 5 m.
separadas por uma pista de 7m de largura. Vamos admitir que: b) 4 m. d) 6 m.
I. os postes de iluminação projetam sobre a rua uma área iluminada
na forma de uma elipse de excentricidade 0,943; 06. (FGV) Sendo m o maior valor real que x pode assumir na equação
analítica (x – 2)² + 4(y + 5)² = 36 e n o maior valor real que y pode
II. o centro dessa elipse encontra-se verticalmente abaixo da
assumir nessa mesma equação, então, m + n é igual a
lâmpada, no meio da rua;
a) 8. c) 6. e) 3.
III. o eixo menor da elipse, perpendicular à calçada, tem exatamente
a largura da rua (calçadas e pista). b) 7. d) 4.

224 PROMILITARES.COM.BR

PM_BOOK05_MAT.indb 224 01/11/2021 13:50:18


GEOMETRIA ANALÍTICA: CÔNICAS

07. (FGV) Na representação gráfica do sistema de equações 10. (MACKENZIE) Com relação às equações das elipses
x 2 + y 2 =
4 16x² + 16y² + 150x + 256y – 351 = 0 e 16x² + 25y² – 96x – 200y +144 = 0,
 2 no plano cartesiano, uma das soluções é (0,-2). A podemos afirmar que
4x − y = 2
a) as elipses têm centros coincidentes.
distância entre os pontos que representam as duas outras soluções
desse sistema é igual a b) as elipses têm a mesma distância focal.
c) as elipses têm a mesma excentricidade.
a) 14.
d) as elipses têm focos sobre o eixo das abscissas.
7
b) . e) o eixo maior de uma delas é o dobro do eixo menor da outra.
2
15
c) .
2 EXERCÍCIOS DE

d)
14
2
. TREINAMENTO
3
e) .
2
01. (AFA) Sobre a circunferência de menor raio possível que
08. (UNIFOR) Uma bola é jogada dentro de uma cesta cuja superfície é circunscreve a elipse de equação x² + 9y² – 8x – 54y – 88 = 0 é correto
obtida girando a parábola y = x2 em torno do eixo y. O centro da bola afirmar que
ocupa um ponto de altura y = 3. O raio da bola é: a) tem raio igual a 1.

D O c) Pé secante
a) 11 b) tangencia o eixo das abscissas.
11
L R Oao eixo das ordenadas.
b)
2
I A d) intercepta aF
reta de equação 4x – y = 0.

c)
11
R E
02. (MACKENZIE) Dadas as cônicas de equações (I) x² + y² – 2x +
3

S
TE

8y + 8 = 0 e (II) 4x² + y² - 8x + 8y + 16 = 0 assinale a alternativa


11 INCORRETA.

SO
d)
4 a) Os gráficos de (I) e (II) são, respectivamente, uma circunferência e
MA

11 uma elipse.
e)

R
5 b) As duas cônicas têm centro no mesmo ponto.
c) As duas cônicas se interceptam em dois pontos distintos.
3x 2 + y 2 ≤ 2
09. (FGV) A solução gráfica do sistema de inequações  d) O gráfico da equação (I) é uma circunferência de raio 3.
2 2
é a região sombreada em x + y ≤ 1
e) O gráfico da equação (II) é uma elipse com centro C = (1,-4).
a) d)
R
03. (AFA) No plano cartesiano, os pontos P(x,y) satisfazem a equação
MA

SO

(x − 1)2 (y + 2)2
+ 1 da curva Se F1 e F2 são os focos de λ, tais que a
=
25 9
abscissa de F1 é menor que a abscissa de F2, é INCORRETO afirmar que
T

R a) a soma das distâncias de P a F1 e de P a F2 é igual a 10.


E

IA F
b) F1 coincide com o centro da curva x² + y² + 6x – 4x = 0.
c) O
L D Od) oPpontoRde abscissa máxima de λ pertence à reta y = x - 8.
F2 é exterior a x² + y² = 25.
b) e)

04. (ESPCEX) Os valores reais de n para os quais a reta (t)y = x + n seja


tangente à elipse de equação 2x2 + 3y2 = 6 são iguais a
a) − 5 e 5
b) − 3 e 3
c) –3 e 3
d) –2 e 2
e) –5 e 5
c)
05. (EN) A equação 4x2 – y2 – 32x + 8y + 52 = 0 no plano xy, representa
a) duas retas
b) uma circunferência
c) uma elipse
d) uma hipérbole
e) uma parábola

PROMILITARES.COM.BR 225

MAT238 - GEOMETRIA ANALÍTICA - CÔNICAS.indd 225 01/11/2021 15:49:22


GEOMETRIA ANALÍTICA: CÔNICAS

06. (ESPCEX) Uma elipse tem centro na origem e vértices em (2a,0) e EXERCÍCIOS DE

COMBATE
(0,a), com a > 0. A área do quadrado inscrito nessa elipse é
16a2 12a2 e)
20a2
a) . c) . .
5 5 5
4a2 8a2
b) . d) . 01. (AFA 2004) Sobre o triângulo PF1F2 onde P(2,2) e F1F2 são da elipse
5 5
x2 y2
07. (ESPCEX) Considere as afirmações:
+ = 1 , é correto afirmar que:
9 25
I. Uma elipse tem como focos os pontos F1(-3,0), F2(3,0) e a medida a) é isósceles.
x2 y2 b) é obtusângulo
do eixo maior é 8. Sua equação é + = 1.
16 7
c) tem área igual a 16
II. Os focos de uma hipérbole são F1(-10,0), F2(10,0) e sua
d) tem perímetro igual a 2√2 + 8.
5
excentricidade é . Sua equação é 16x2 – 9y2 = 576.
3
02. (UFT 2010) Considere  o conjunto dos números reais e b ∈ .
III. A parábola 8x = -y2 + 6y – 9 tem como vértice o ponto V(3,0).
Encontre os valores de b, tais que no plano cartesiano xy, a reta y = x + b
Com base nessas afirmações, assinale a alternativa correta. x²
intercepta a elipse + y² = 1 em um único ponto. A soma dos
a) Todas as afirmações são falsas. valores de b é: 4
b) Apenas as afirmações I e III são falsas. a) 0
c) Apenas as afirmações I e II são verdadeiras.
L D O b) P2 R O
IA FE
d) Todas as afirmações são verdadeiras. c) 2√5
e) Apenas a afirmação III é verdadeira.
R d) √5
e) –2√5
08. (UFTM) Os pontos P e Q estão na parábola dada por y = 4x2 + 7x – 1,

S
TE

e a origem do sistema de coordenadas cartesianas está no ponto

SO
médio de PQ. Sendo assim, P e Q são pontos que estão na reta 03. (ESPCEX) Num estádio de futebol em forma de elipse, o gramado
MA

é o retângulo MNPQ, inscrito na cônica, conforme mostra a figura.


15x Escolhendo o sistema de coordenadas cartesianas indicado e tomando
a) y= .

R
2 x² y²
b) y = 7x. o metro como unidade, a elipse é descrita pela equação + =
1.
36² 60²
13x Sabe-se também que os focos da elipse estão situados em lados do
c) y= .
2 retângulo MNPQ.
d) y = 6x.
11x
R
e) y= .
MA

2
SO

09. (ITA) Sobre a parábola definida pela equação x² + 2xy + y² - 2x +


T

4y + 1 = 0 pode-se afirmar que


S

R
E

a) ela não admite reta tangente paralela ao eixo Ox.


E

IA F
L DO PRO
b) ela admite apenas uma reta tangente paralela ao eixo Ox.
c) ela admite duas retas tangentes paralelas ao eixo Ox.
d) a abscissa do vértice da parábola é x = -1.
2
e) a abscissa do vértice da parábola é x = − .
3 Assim, a distância entre as retas MN e PQ é
10. (ESPCEX) Uma hipérbole tem focos F1(-5,0) e F2(5,0) e passa pelos a) 48 m
pontos P(3,0) e Q(4,y), com y > 0. O triângulo com vértices em F1, P e b) 68 m
Q tem área igual a c) 84 m
16 7 d) 92 m
a) .
3 e) 96 m
16 7
b) .
5 04. (ESPCEX) Sobre a curva 9x² + 25y² – 36x + 50y – 164 = 0, assinale
a alternativa correta.
32 7
c) . a) Seu centro é (– 2,1).
3
b) A medida do seu eixo maior é 25.
8 7
d) . c) A medida do seu eixo menor é 9.
3
d) A distância focal é 4.
8 7
e) . e) Sua excentricidade é 0,8.
5

226 PROMILITARES.COM.BR

MAT238 - GEOMETRIA ANALÍTICA - CÔNICAS.indd 226 01/11/2021 15:49:23


GEOMETRIA ANALÍTICA: CÔNICAS

05. O gráfico da equação x2 – y2 = 4 representa uma hipérbole.


Os focos dessa hipérbole são:
RESOLUÇÃO EM VÍDEO
1   1 
a)  ,0  e  − ,0  Abra o ProApp, leia o QR Code, assista à resolução
de cada exercício e AVANCE NOS ESTUDOS!
2   2 
b) (2, 0) e (–2, 0)
c) (2 ,√2, 0) e (–2 √2, 0)
d) (0, √2) e (0, –√2) GABARITO
 1  1 EXERCÍCIOS DE FIXAÇÃO
e)  0,  e  0, − 
 2   2  01. D 04. B 07. C 10. C
06. (AFA) A equação reduzida da hipérbole, cujos focos são os 02. B 05. C 08. B
extremos do eixo menor da elipse de equação 16x2 + 25y2 = 625, 03. B 06. C 09. C
e cuja excentricidade é igual ao inverso da excentricidade da elipse EXERCÍCIOS DE TREINAMENTO
dada, é
01. B 04. A 07. C 10. A
a) 16y2 – 9x2 = 144 c) 9x2 – 16y2 = 144
02. C 05. D 08. B
b) 9y2 – 16x2 = 144 d) 16x2 – 9y2 = 144
03. B 06. A 09. B
07. (ESPCEX) A representação no sistema cartesiano ortogonal da EXERCÍCIOS DE COMBATE
equação 9x2 – y2 = 36x + 8y – 11 é dada por
D O 01.
P B
R
04. E 07. E 10. B
a) duas retas concorrentes.
A L 02. A
O 05. C 08. A
b) uma circunferência.
I 03. E F EA
06. 09. A
c) uma elipse.
d) uma parábola.
R ANOTAÇÕES

S
TE

e) uma hipérbole.

SO
MA

 1 y² + 3
08. (ESPCEX) O ponto P  a,  pertence à parábola x = A

R
 3 3
equação da reta perpendicular à bissetriz dos quadrantes ímpares que
passa por P é:
a) 27x + 27y – 37 = 0
b) 37x + 27y – 27 = 0
c) 27x + 37y – 27 = 0
R
MA

d) 27x + 27y – 9 = 0
SO

e) 27x + 37y – 9 = 0
T

09. (AFA) Com relação ao conjunto de pontos P(x, y) equidistantes da


S

R
reta y = 3 e da origem do sistema cartesiano ortogonal, é INCORRETO
E

afirmar que é uma curva


IA F
L DO PRO
a) representada por x² – 6y – 9 =0.
b) cujas coordenadas do vértice tem soma 1,5.
c) que representa uma função par.
d) cujo parâmetro é igual a 3.

10. (AFA) Classifique em VERDADEIRO ou FALSO cada item a seguir.


(2) A parábola cuja equação é x² – 4y = 0 tem diretriz representada
pela reta y + 1 = 0 e foco coincidente com o baricentro do triângulo
ABC, onde A é a origem do sistema cartesiano, B (2, 3) e C (–2, 0).
(3) O conjunto de pontos representados pela equação x² – y² + x +
y = 0 é uma hipérbole equilátera que NÃO tem centro na origem do
sistema cartesiano.
(8) Na elipse 16x² + 64y² = 1 a medida do eixo vertical é 50% da
medida do eixo horizontal.
(16) Existem apenas 4 números inteiros entre os valores de k, para os
quais o vértice da parábola y² = 4x + 1 é ponto exterior à circunferência
x² + y² – 2x + 4y + k = 0.
A soma dos itens VERDADEIROS é um número do intervalo
a) [2, 10[ c) [16, 22[
b) [10, 16[ d) [22, 30[

PROMILITARES.COM.BR 227

MAT238 - GEOMETRIA ANALÍTICA - CÔNICAS.indd 227 01/11/2021 15:49:23


GEOMETRIA ANALÍTICA: CÔNICAS

ANOTAÇÕES

L DO PRO
I A FE
R

S
TE

SO
MA

R
R
MA

SO
T

R
E

IA F
L DO PRO

228 PROMILITARES.COM.BR

MAT238 - GEOMETRIA ANALÍTICA - CÔNICAS.indd 228 01/11/2021 15:49:24


GEOMETRIA ESPACIAL: CONCEITOS,
PARALELEPÍPEDO E CUBO

FUNDAMENTOS d) Duas retas paralelas distintas.


Conceitos primitivos: ponto, reta e plano.

POSTULADOS PRINCIPAIS
• Dois pontos distintos determinam uma única reta que passa
por eles.
• Três pontos não colineares determinam um único plano que
passa por eles.

DO PRO
Se dois pontos distintos de uma reta pertencem a um plano,
então essa reta está contida nesse plano. L

I A
Se dois planos possuem um ponto comum, então possuem FE três pontos não colineares
R
pelo menos algum outro ponto comum. Isso indica que a
interseção de dois planos distintos que se interceptam é uma uma reta e um ponto fora dela

S
TE

reta. DETERMINAÇÃO DE
UM PLANO

SO
• Por um ponto não pertencente a uma reta, passa uma, e duas retas concorrentes
MA

apenas uma, reta paralela à primeira. (Euclides)


duas retas paralelas distintas

R
DETERMINAÇÃO DO PLANO
Um único plano fica determinado por:
POSIÇÕES RELATIVAS ENTRE RETAS
a) Três pontos não colineares.

RETAS COPLANARES
R
a) Concorrentes: um ponto de interseção
MA

SO
T

R
E

IA F
b) Uma reta e um ponto exterior. L DO PRO
b) Paralelas Coincidentes: infinitos pontos de interseção

c) Duas retas concorrentes.

r≡s

PROMILITARES.COM.BR 229

PM_BOOK05_MAT.indb 229 01/11/2021 13:50:22


GEOMETRIA ESPACIAL: CONCEITOS, PARALELEPÍPEDO E CUBO

c) Paralelas Distintas: não há pontos de interseção. b) Reta e plano paralelos: não há pontos de interseção.

RETAS NÃO COPLANARES


a) Retas Reversas: não há pontos de interseção
c) Reta e plano secantes: um único ponto de interseção.
α ≡β

L DO PRO
I A FE
R

S
TE

SO
MA

POSIÇÕES RELATIVAS ENTRE UMA RETA E UM PLANO

R
todos os pontos da reta pertencem ao
contida
plano

paralela não têm ponto em comum

concorrente
têm somente um ponto em comum
(ou secante) R
MA

SO

POSIÇÕES RELATIVAS ENTRE DUAS RETAS INTERSEÇÃO

concorrentes 1 ponto POSIÇÕES RELATIVAS ENTRE PLANOS


T

coplanares coincidentes toda a reta POSIÇÕES RELATIVAS ENTRE DOIS PLANOS


R
E

paralelas
E

distintas
Ivazia
A F
POSIÇÕES RELATIVAS ENTRE DOIS

L DO PRO
INTERSEÇÃO
PLANOS
não coplanares reversas vazia
coincidentes todo o plano
paralelos
distintos vazia
POSIÇÕES RELATIVAS ENTRE RETA E PLANO
a) Reta e plano contidos: infinitos pontos de interseção. secantes (ou concorrentes) uma única reta (traço)

a) Planos paralelos coincidentes: a interseção é todo o plano.

α ∩ β = {r}

230 PROMILITARES.COM.BR

PM_BOOK05_MAT.indb 230 01/11/2021 13:50:23


GEOMETRIA ESPACIAL: CONCEITOS, PARALELEPÍPEDO E CUBO

b) Planos paralelos distintos: não há pontos de interseção. • Dois planos são paralelos se, e somente se, não têm ponto em
α||β comum ou são coincidentes.
• Por um ponto fora de um plano passa um único plano paralelo
a esse plano.
• TEOREMA: A condição necessária e suficiente para que dois
planos sejam paralelos, é que um deles contenha duas retas
concorrentes, paralelas ao outro.

c) Planos secantes (ou concorrentes): a interseção é uma reta. r,s ∈ α


D O P R O r e s concorrentes ⇔ α  β

α ∩ β = {r}
L
IA F Er  β e s  β 

R Se dois planos são secantes e uma reta de um deles é paralela ao

S
TE

outro, então essa reta é paralela à interseção.

SO
MA

R
R
MA

SO
T

PARALELISMO
R
E


IA
Se duas retas são paralelas a uma terceira, então elas são F α e β secantes
paralelas entre si.
Lnão têm R O 
• Uma reta é paralela a um plano se, e somente se, eles
ponto em comum.
DO P s ∈β
sα
 ⇔ sr


• TEOREMA: A condição necessária e suficiente para que uma
reta não contida em um plano seja paralela a esse plano é que • Se duas retas distintas são paralelas entre si e um plano
ela seja paralela a uma reta do plano. paralelo à primeira contém um ponto da segunda, então esse
plano contém a segunda.
• Se uma reta é paralela a dois planos secantes, então ela é
paralela à interseção desses planos.

PROMILITARES.COM.BR 231

PM_BOOK05_MAT.indb 231 01/11/2021 13:50:24


GEOMETRIA ESPACIAL: CONCEITOS, PARALELEPÍPEDO E CUBO

Retas ortogonais são retas reversas que formam ângulo reto.

α e β secantes
 ⇔ sr
sα e sβ 

RETA E PLANO PERPENDICULARES


• Se dois planos são paralelos a um terceiro, então eles são
Uma reta e um plano são perpendiculares se, e somente se, a reta
paralelos entre si.
é perpendicular ou ortogonal a todas as retas do plano.

DO PRO
• Se dois planos paralelos entre si são interceptados por um

L
terceiro, então as interseções são paralelas entre si.
A
I FE
R

S
TE

SO
MA

R
Se uma reta e um plano são perpendiculares, o traço (P) da reta
no plano é o pé da perpendicular.
R
• Se uma reta é perpendicular a um plano, então ela é
MA

perpendicular ou ortogonal a qualquer reta do plano.


SO

• Uma reta é perpendicular a um plano se, e somente se, é


αβ  perpendicular (ou ortogonal) a duas retas concorrentes desse
 plano.
T

α ∩=δ {r}  ⇒ r  s
S

β ∩ δ ={s} R
E

IA F
PLANOS PERPENDICULARES

L DO PRO • Um plano α é perpendicular a um plano β se, e somente se, α


contém uma reta perpendicular a β.
PERPENDICULARIDADE
RETAS ORTOGONAIS
Ângulo entre retas reversas é o ângulo formado por duas retas
concorrentes paralelas às retas dadas.

232 PROMILITARES.COM.BR

PM_BOOK05_MAT.indb 232 01/11/2021 13:50:25


GEOMETRIA ESPACIAL: CONCEITOS, PARALELEPÍPEDO E CUBO

r ∈ α DISTÂNCIAS
α ⊥β⇔ 
r ⊥ β
DISTÂNCIA ENTRE DOIS
• Por uma reta r não perpendicular a um plano α, existe um segmento de reta AB
PONTOS A E B
único plano perpendicular a α.
• Se dois planos são perpendiculares entre si e uma reta de um distância do ponto ao pé da
DISTÂNCIA ENTRE UM
deles é perpendicular à interseção dos planos, então essa reta perpendicular à reta conduzida pelo
PONTO E UMA RETA
é perpendicular ao outro plano. ponto

DISTÂNCIA ENTRE
distância entre um ponto qualquer
DUAS RETAS
de uma das retas e a outra reta
PARALELAS

distância entre o ponto e o pé da


DISTÂNCIA ENTRE
perpendicular ao plano conduzida
PONTO E PLANO
pelo ponto

DISTÂNCIA ENTRE RETA distância entre um ponto qualquer


E PLANOS PARALELOS da reta e o plano

DISTÂNCIA ENTRE distância entre um ponto qualquer


PLANOS PARALELOS de um deles e o outro plano

L D O PDISTÂNCIA
R O REVERSAS
ENTRE
distância entre um ponto qualquer
de uma delas e o plano que passa

I A DUAS RETAS
FE pela outra e é paralelo à primeira

R • PERPENDICULAR COMUM: dadas duas retas reversas, existe

S
TE

uma única reta que é perpendicular comum a essas retas. De

SO
todos os segmentos que têm extremidades em cada uma das
MA

retas reversas, o menor é o da perpendicular comum.

R
α ⊥β 
⇒r ⊥β
r ⊥ s = α ∩ β

• Dois planos secantes são perpendiculares se, e somente


se, toda reta de um deles, perpendicular à interseção, é
perpendicular ao outro. R
• Se uma reta é perpendicular a um plano, qualquer outro
MA

SO

plano que a contenha é perpendicular ao primeiro.


• Se um plano é perpendicular a dois planos secantes, então ele
é perpendicular à interseção desses planos.
T

γ ⊥ α
R
E

 ⇒ γ ⊥ r = α ∩β
E

γ ⊥β
IA F
L DO PRO

Todo plano que passa pelo ponto médio de um segmento é


equidistante das extremidades do segmento.

PROJEÇÃO ORTOGONAL
Projeção ortogonal de um ponto sobre um plano: é o pé da
perpendicular ao plano conduzida pelo ponto.
Projeção ortogonal de uma figura sobre um plano: é o conjunto
das projeções ortogonais dos pontos da figura sobre o plano.
Projeção ortogonal de uma reta sobre um plano:
1) reta perpendicular ao plano: traço da reta sobre o plano
2) reta não perpendicular ao plano: A projeção ortogonal da reta
r sobre o plano α é o traço em α do plano β, perpendicular a
α, conduzido por r.

PROMILITARES.COM.BR 233

PM_BOOK05_MAT.indb 233 01/11/2021 13:50:25


GEOMETRIA ESPACIAL: CONCEITOS, PARALELEPÍPEDO E CUBO

• α → plano de projeção Na figura r ⊥ γ e o ângulo a ^ b é a seção reta do diedro.


• β → plano projetante A medida de um diedro é a medida de sua seção reta (a
• Projeção ortogonal de um segmento de reta AB sobre um medida de um diedro é igual ao ângulo entre as faces)
plano α, contido numa reta não perpendicular a esse plano é Diedro reto é aquele cuja seção normal é um ângulo reto.
o segmento A’B’ de α onde A’ e B’ são as projeções ortogonais
de A e B, respectivamente, sobre o plano α.
TRIEDROS
ÂNGULO DE UMA RETA Sejam três semi-retas de mesma origem e não coplanares. Três
planos podem ser formados, um a partir de cada par de retas. Cada
COM UM PLANO plano determina um semi-espaço que contém a terceira semi-reta.
• O ângulo entre uma reta e um plano oblíquos é o ângulo que Assim, o triedro V (a, b, c) ou V (A, B, C) formado pelas três semi-retas
a reta forma com sua projeção ortogonal sobre o plano. é a interseção desses três semi-espaços.
• O ângulo entre uma reta e um plano perpendiculares é reto.
• Se a reta é paralela ou está contida no plano, o ângulo entre
a reta e o plano é nulo.
• Se uma reta é oblíqua a um plano α e o intercepta em A,
então o ângulo agudo de r com sua projeção ortogonal r’
sobre α é menor que o ângulo agudo de r com qualquer outra
reta de α que passe por A.
• Se dois planos α e β são oblíquos, r é sua intersecção, e por

L DO PRO
um ponto P de α, não pertencente a r, são conduzidas duas
retas concorrentes a e b, sendo a perpendicular a r, então o

I A
ângulo entre a reta a e β é maior que o ângulo entre a reta
b e β.
FE
• Rrelação a outro: se
Reta de maior declive de um plano em

S
TE

dois planos α e β são oblíquos, toda reta de α perpendicular


à interseção dos planos é chamada reta de maior declive de

SO
α em relação a β. A reta de maior declive é a reta de α que
MA

forma ângulo máximo com β.

R
DIEDROS
Ângulo diedro ou diedro ou ângulo diédrico é a reunião de dois A origem comum V é chamada vértice do triedro e cada uma das
semi-planos α e β de mesma origem não contidos num mesmo plano semi-retas a, b e c, aresta.
e é denotado por αβ.
Um triângulo que possui um vértice em cada aresta do triedro é
R
uma seção do triedro, como por exemplo o triângulo ABC.
MA

• Triedro tri-retângulo: faces são ângulos retos e os diedros


SO

são diedros retos.


T

R
E

IA F
L DO PRO
A reta r chama-se aresta do diedro e os semiplanos α e β, faces.
Seção reta ou seção normal de um diedro é uma seção cujo
plano é perpendicular à aresta do diedro.

Triedro
tri-retângulo

CONDIÇÃO DE EXISTÊNCIA
1) A medida de cada face está compreendida entre 0° e 180°.
2) Em todo triedro, qualquer face é menor que a soma das
outras duas.
3) A soma das medidas das faces de um triedro qualquer é
menor que 360°.

234 PROMILITARES.COM.BR

PM_BOOK05_MAT.indb 234 01/11/2021 13:50:26


GEOMETRIA ESPACIAL: CONCEITOS, PARALELEPÍPEDO E CUBO

ÂNGULOS POLIÉDRICOS CONVEXOS


O conceito de ângulos poliédricos convexos é uma extensão do
conceito de triedros. Dado um número finito n ≥ 3 de semirretas
Va1, Va2, ... Van, de mesma origem V, tais que o plano de duas
consecutivas deixa as demais num mesmo semiespaço, consideremos
n semiespaços E1, E2, ... En, cada um deles com origem no plano de
duas semirretas consecutivas e contendo as restantes. Então o ângulo
poliédrico convexo determinado por Va1, Va2, ... Van é a interseção dos
semiespaços E1, E2, ... En.

CONDIÇÃO DE EXISTÊNCIA
1) A medida de cada face está compreendida entre 0° e 180°.
2) Em todo ângulo poliédrico convexo, qualquer face é menor
que a soma das demais
3) A soma das medidas das faces de um ângulo poliédrico
convexo qualquer é menor que 360°.
RELAÇÃO DE EULER
POLIEDROS CONVEXOS E Seja um poliedro convexo com V vértices, A arestas e F faces,
então
RELAÇÃO DE EULER
D O PR V +F = A +2

DEFINIÇÕES
AdeLpolígonos poliedros
Poliedro convexo é uma reunião de um número fiInito
O que satisfazem a relação de Euler são chamados
FE
Os poliedros
eulerianos.
planos convexos chamados faces onde: R Todo poliedro convexo é euleriano, mas nem todo poliedro
euleriano é convexo.
1) dois polígonos não estão no mesmo plano;

S
TE

2) cada lado de polígono é comum a dois e somente dois A soma dos ângulos de todas as faces de um poliedro convexo

SO
polígonos; e de V vértices é
MA

3) o plano de cada polígono deixa os demais polígonos no S= 360 ⋅ ( V − 2)

R
mesmo semi-espaço.
Seja fn o número de faces de gênero n, então:
3f3 + 4f4 +  =
2A

Seja Vp o número de vértices onde concorrem p arestas, então:


3v 3 + 4v 4 +  =
2A
R
Seja ∑df o total de diagonais das faces, então o número de
MA

diagonais do poliedro D é:
SO

Os vértices das faces são também os vértices do poliedro e os


lados das faces são chamadas arestas do poliedro.
= D
V(V − 1)
2
−A− df∑
T

Cada vértice do poliedro corresponde a um ângulo poliédrico, no


R POLIEDROS DE PLATÃO
E

qual está contido todo o poliedro.


IA F
• Diagonal do poliedro é qualquer segmento com
L DO PR
extremidades em dois vértices do poliedro e não contido em O Um poliedro é chamado poliedro de Platão se, e somente se,
• todas as faces têm o mesmo número de arestas;
uma face.
• todos os ângulos poliédricos têm o mesmo número de
arestas; e
• vale a relação de Euler (V − A + F = 2).
Existem exatamente cinco poliedros de Platão.
Seja n o mesmo número de arestas de cada face e m o número de
arestas dos ângulos poliédricos, temos:

POLIEDROS DE PLATÃO

m n A V F NOME

3 3 6 4 4 tetraedro

3 4 12 8 6 hexaedro

4 3 12 6 8 octaedro
No poliedro acima, por exemplo, BH é uma diagonal.
Os poliedros são classificados de acordo com o seu número de faces. 3 5 30 20 12 dodecaedro
O poliedro com menor número de faces é o tetraedro e possui 4 faces. 5 3 30 12 20 icosaedro

PROMILITARES.COM.BR 235

PM_BOOK05_MAT.indb 235 01/11/2021 13:50:27


GEOMETRIA ESPACIAL: CONCEITOS, PARALELEPÍPEDO E CUBO

Tetraedro Hexaedro Octaedro OCTAEDRO REGULAR

• Quantidade de faces: 8
• Formato das faces: triângulos
equiláteros
• Quantidade de vértices: 6
• Quantidade de arestas: 12
Dodecaedro Icosaedro
DODECAEDRO REGULAR

• Quantidade de faces: 12
• Formato das faces: pentágonos
• Quantidade de vértices: 20
• Quantidade de arestas: 30
POLIEDROS REGULARES
Um poliedro convexo é regular quando suas faces são polígonos
regulares e congruentes e seus ângulos poliédricos são congruentes.
ICOSAEDRO REGULAR
DO PRO
Todo poliedro regular convexo é um poliedro de Platão, mas nem
todo poliedro de Platão é um poliedro regular.
L
A regular e o
Os poliedros convexos regulares são cinco: o tetraedro
I
o hexaedro regular, o octaedro regular, o dodecaedro
regular,
FE • Quantidade de faces: 20

icosaedro regular. R • Formato das faces: triângulos


equiláteros

S
TE

• Quantidade de vértices: 12

SO
• Quantidade de arestas: 30
MA

R
PARALELEPÍPEDO
É o prisma cuja base é um paralelogramo.
Tetraedro regular Hexaedro regular Octaedro regular

R
MA

SO
T

R
E

Dodecaedro regular Icosaedro regular


IA F
TETRAEDRO REGULAR
L DO PRO PARALELEPÍPEDO RETO RETÂNGULO

• Quantidade de faces: 4
• Formato das faces: triângulos
equiláteros
• Quantidade de vértices: 4
• Quantidade de arestas: 6
DIAGONAL DO PARALELEPÍPEDO
Do Pitágoras no triângulo retângulo de catetos a e b e hipotenusa
d teremos:
HEXAEDRO REGULAR d=
2
a2 + b2

Do Pitágoras no triângulo retângulo hachurado, de catetos d e c


• Quantidade de faces: 6
e hipotenusa D teremos:
• Formato das faces: quadrados
D=
2
d2 + c2
• Quantidade de vértices: 8
D= a2 + b2 + c2
2
• Quantidade de arestas: 12
D = a2 + b2 + c2

236 PROMILITARES.COM.BR

PM_BOOK05_MAT.indb 236 01/11/2021 13:50:27


GEOMETRIA ESPACIAL: CONCEITOS, PARALELEPÍPEDO E CUBO

ÁREA TOTAL DO PARALELEPÍPEDO Diagonal da face: AC = a 2.


Utilizaremos as áreas dos 6 retângulos, retângulos a × b, a × c e
Diagonal do cubo: CE = a 3.
b × c.
ST = 2ab + 2ac + 2bc = 2(ab + ac + bc) Centro do cubo: Ponto O, interseção das quatro diagonais do
poliedro. O ponto O equidista dos vértices, equidista das arestas e
equidista das faces do cubo.
VOLUME DO PARALELEPÍPEDO
V = Sb ⋅ H = abc
Exercício Resolvido

HEXAEDRO REGULAR (CUBO) 01. Um poliedro convexo possui 8 (oito) faces, todas triangulares.
Nestas condições, assumindo que tal poliedro exista, o número
esperado de vértices para este será
a) 10
b) 9
c) 8
d) 7
e) 6

Resolução: E

D O AP= 8.3
R2 =O12 e F = 8. Logo, V – A + F = 2 ⇒ V – 12 + 8 = 2 ⇒ V = 6
Para encontrar os elementos do cubo basta utilizarmos as mesmas
L
IA
definições do paralelepípedo reto-retângulo fazendo a = b = c.
FE
DIAGONAL DO CUBO R

S
TE

D= a2 + b2 + c2 Exercício Resolvido

SO
Com a = b = c: 02. Um poliedro convexo tem 32 faces, sendo 20 hexágonos e 12
MA

pentágonos. O número de vértices deste polígono


=D a2 + a2 + a2

R
a) 90
=D =
3a2 a 3
b) 72
c) 60
ÁREA LATERAL DO CUBO d) 56
Teremos as áreas dos 4 quadrados laterais.
Resolução: C R
Sl = 4a2
MA

F: número de faces
SO

A: número de arestas
V: número de vértices
ÁREA TOTAL DO CUBO
T

20 ⋅ 6 + 12 ⋅ 5
S

Áreas de todas as faces, ou seja, dos 6 quadrados. =A = 90


R 2
E

ST = 6a2
IA F = 32 F
VOLUME DO CUBO
L D O V =P2 + 90R – O
V=2+A–F
32
V = 60.
V = Sb ⋅ H = a3

Exercício Resolvido
RELAÇÕES MÉTRICAS NO HEXAEDRO
REGULAR DE ARESTA A 03. Um poliedro convexo possui 10 faces triangulares, 10 faces
quadrangulares e 1 face decagonal. O número de vértices deste
poliedro é
a) 20
b) 21
c) 22
d) 23
e) 24

PROMILITARES.COM.BR 237

PM_BOOK05_MAT.indb 237 01/11/2021 13:50:28


GEOMETRIA ESPACIAL: CONCEITOS, PARALELEPÍPEDO E CUBO

Resolução: B Resolução: B
10 triângulos → 10x3 = 30 lados.
10 quadriláteros → 10x4 = 40 lados.
1 decágono → 1x10 = 10 lados
total de lados 80
Total de lados = 80 → A = →A= → A = 40
2 2
F = 10 + 10 + 1 → F = 21 Vpiscina = a ⋅ b ⋅ c → Vpiscina = 10 ⋅ 15 ⋅ 1,5 →
V + F = A + 2 → V + 21 = 40 + 2 → V = 21
Vpiscina = 225 m³ → Vpiscina = 225000 litros.
4500 litros → 1 pacot e
225000 → x pacot es
Exercício Resolvido
=
4500 ⋅ x 225000= → x 50 pacot es
04. O triângulo ACF tem vértices coincidindo com três dos vértices
de um cubo de aresta a, como mostra a figura abaixo.
EXERCÍCIOS DE

FIXAÇÃO
L D O 01.
PUmaRpessoa
O pede informação na recepção de um prédio comercial

I A de como chegar
Fa uma sala, e recebe as seguintes instruções: suba a

R
escada em forma
E seguida vire à direita e siga pelo corredor. Ao
de U à frente, ao final dela vire à esquerda, siga um
pouco à frente e em
final do corredor, vire à direita. Uma possível projeção vertical dessa

S
TE

trajetória no plano da base do prédio é:

SO
MA

A área de ACF em função de a.

R
a2 3 a)
a)
2

b) a2 3
3
R
a2 3
MA

c)
4
SO

a2 2 b)
d)
2
T

R
E

Resolução: A
IA F
(a 2 ) L DO PRO
2
3 a2 3
AC =CF =AF =a 2 ⇒ SACF = =
4 2
c)

Exercício Resolvido

05. Uma piscina em forma de um paralelepípedo retângulo de


10,0 m × 15,0 m e fundo horizontal está com água até a altura de
1,5 m. Um produto químico em pó deve ser misturado à água à
razão de um pacote para cada 4500 litros. O número de pacotes d)
a serem usados é:
a) 25
b) 50
c) 100
d) 250
e) 500
e)

238 PROMILITARES.COM.BR

PM_BOOK05_MAT.indb 238 01/11/2021 13:50:29


GEOMETRIA ESPACIAL: CONCEITOS, PARALELEPÍPEDO E CUBO

02. Uma lagartixa está no interior de um quarto e começa a se


deslocar. Esse quarto, apresentando o formato de um paralelepípedo
retangular, é representado pela figura.

Após a armação das hastes, um dos escoteiros observou um inseto


deslocar-se sobre elas, partindo do vértice A em direção ao vértice B,
A lagartixa parte do ponto B e vai até o ponto A. A seguir, de A ela deste em direção ao vértice E e, finalmente, fez o trajeto do vértice
se desloca, pela parede, até o ponto M, que é o ponto médio do E ao C. Considere que todos esses deslocamentos foram feitos
segmento EF. Finalmente, pelo teto, ela vai do ponto M até o ponto H. pelo caminho de menor distância entre os pontos. A projeção do
Considere que todos esses deslocamentos foram feitos pelo caminho deslocamento do inseto no plano que contém a base ABCD é dada
de menor distância entre os respectivos pontos envolvidos. A projeção por
ortogonal desses deslocamentos no plano que contém o chão do
quarto é dado por:
a) d)
a)

b)
d)

L DO PRO
I A FE
e) R b) e)

S
TE

c)

SO
MA

c)

R
03. Os alunos de uma escola utilizaram cadeiras iguais às da figura
para uma aula ao ar livre. A professora, ao final da aula, solicitou que
os alunos fechassem as cadeiras para guardá-las. Depois de guardadas,
os alunos fizeram um esboço da vista lateral da cadeira fechada. 05. A figura a seguir representa uma cadeira onde o assento é um
paralelogramo perpendicular ao encosto.
R
MA

SO
T

R
E

IA F
Qual é o esboço obtido pelos alunos?
L DO PRO
A partir dos pontos dados, é correto afirmar que os segmentos de retas
a) CD e EF são paralelos. c) AC e CD são coincidentes.
a) c) e) b) BD e FJ são concorrentes. d) AB e EI são perpendiculares.

06. (EEAR) Se as dimensões de um paralelepípedo retângulo medem,


em cm, “a”, “a + 3” e “a + 5”, então a soma das medidas de todas
as arestas desse paralelepípedo é maior que 48 cm, se “a” for maior
que _____ cm.
a) 4/3. b) 5/4. c) 3/4. d) 4/5.
b) d)
07. (EEAR) O número de poliedros regulares que têm faces triangulares é
a) 1. b) 2. c) 3. d) 4.

08. (EEAR) Seja V o volume de um cubo de aresta “a”. Constrói-se um


04. Um grupo de escoteiros mirins, numa atividade no parque da prisma quadrangular de volume V’ e de vértices nos pontos médios
cidade onde moram, montou uma barraca conforme a foto da Figura das arestas das bases do cubo. O volume V’ desse prisma é igual a
1. A Figura 2 mostra o esquema da estrutura dessa barraca, em forma V. V. V.
de um prisma reto, em que foram usadas hastes metálicas. a) b) V. c) d)
2 3 4

PROMILITARES.COM.BR 239

PM_BOOK05_MAT.indb 239 01/11/2021 13:50:30


GEOMETRIA ESPACIAL: CONCEITOS, PARALELEPÍPEDO E CUBO

09. (EEAR) Um cubo tem 216 cm² de área total. A medida, em cm, d) Apenas as afirmações II e IV são verdadeiras.
de sua diagonal é e) Apenas as afirmações III e IV são verdadeiras.
a) 6 2 b) 6 3 c) 2 6 d) 2 2
05. As afirmações seguintes podem ser verdadeiras ou falsas.
10. (EEAR) A diagonal de um cubo mede 3 cm. O volume desse cubo, I. A projeção ortogonal de uma reta num plano é uma reta.
em cm³, é II. Distância entre duas retas reversas é a perpendicular comum a
a) 9 b) 6 c) 3 3 d) 2 6 essas retas.
III. A distância entre dois planos só é definida se esses planos são paralelos.
É correto afirmar que SOMENTE
EXERCÍCIOS DE a) II é verdadeira. d) I e III são verdadeiras.

TREINAMENTO b) III é verdadeira.


c) I e II são verdadeiras.
e) II e III são verdadeiras.

06. Considere uma reta s, contida em um plano α, e uma reta r


01. (EEAR) Um cubo tem 3 cm de altura, e um paralelepípedo perpendicular a s. Então, necessariamente:
retângulo tem dimensões 1 cm, 2 cm e 3 cm. A razão entre os volumes
do cubo e do paralelepípedo é a) r é perpendicular a α.
a) 3/2 b) 4/3 c) 9/2 d) 8/3 b) r e s são coplanares.
c) r é paralela a α.

D O e) PTodas
02. (EEAR) Quatro cubos idênticos são dispostos como na figura a d) r está contida em α.

L
seguir, formando um único sólido. Considerando que a diagonal de R asOretas paralelas a r interceptam s.
A
cada cubo mede 10 3 , a diagonal desse sólido é, em cm, igual a
I F plano e um ponto P qualquer do espaço. Se por
07. Considere um E
R P traçarmos a reta perpendicular a α, a intersecção dessa reta com
α é um ponto chamado projeção ortogonal do ponto P sobre α. No

S
TE

caso de uma figura F do espaço, a projeção ortogonal de F sobre α é

SO
definida pelo conjunto das projeções ortogonais de seus pontos. Com
MA

relação a um plano á qualquer fixado, pode-se dizer que:

R
a) a projeção ortogonal de um segmento de reta pode resultar numa
semi-reta.
b) a projeção ortogonal de uma reta sempre resulta numa reta.
a) 30 3 b) 40 3 c) 20 d) 30
c) a projeção ortogonal de uma parábola pode resultar num
segmento de reta.
03. (EEAR) Assinale V (verdadeiro) ou F (falso), considerando a
geometria de posição espacial e plana. d) a projeção ortogonal de um triângulo pode resultar num
R
quadrilátero.
MA

( ) A condição r ∩ s = ∅ é necessário para que as retas r e s sejam


SO

paralelas distintas. e) a projeção ortogonal de uma circunferência pode resultar num


segmento de reta.
( ) Duas retas que formam um ângulo reto são necessariamente
perpendiculares.
T

08. (AFA) A intersecção de 3 superfícies esféricas distintas pode ser,


S

R
( ) Se duas retas têm um único ponto em comum, então elas são
somente, ou
E

concorrentes.
IA F
a) 1 ponto, ou vazia, ou 1 circunferência.
( ) A condição r ∩ s = ∅ é suficiente para que as retas r e s sejam
reversas. O
L D Oc) 1Psegmento
R de reta, ou vazia, ou 1 circunferência.
b) 1 ponto, ou vazia, ou 2 circunferências.
A sequência correta é:
a) V – V – V – V c) F–V–F–V d) 2 pontos, ou 1 ponto, ou vazia, ou 1 circunferência.
b) V – F – V – F d) F – F – F – F
09. (AFA) Qual das afirmações abaixo é verdadeira?
04. Observe e classifique as afirmações abaixo como sendo verdadeiras a) Por uma reta dada pode-se conduzir um plano paralelo a um
ou falsas: plano dado.
I. Se um plano intercepta dois outros planos paralelos, então as b) Se uma reta é paralela a dois planos, então esses planos são paralelos.
interseções são retas paralelas. c) Por um ponto qualquer é possível traçar uma reta que intercepta
II. Se dois planos são paralelos, qualquer reta de um deles é paralela duas retas reversas dadas.
a qualquer reta do outro. d) Se duas retas concorrentes de um plano são, respectivamente,
III. Se uma reta é paralela a dois planos, então esses planos são paralelas a duas retas de outro plano, então estes planos são paralelos.
paralelos.
IV. Se dois planos são paralelos, uma reta de um deles pode ser 10. (AFA) A reta r é paralela ao plano α; o plano β contém r e intercepta
reversa a uma reta do outro. o plano α segundo a reta s. O que se pode afirmar sobre as retas r e s?

Marque a alternativa CORRETA: a) são perpendiculares

a) Apenas as afirmações I e II são verdadeiras. b) são reversas

b) Apenas as afirmações I e III são verdadeiras. c) são paralelas

c) Apenas as afirmações I e IV são verdadeiras. d) podem ser paralelas ou reversas

240 PROMILITARES.COM.BR

PM_BOOK05_MAT.indb 240 01/11/2021 13:50:31


GEOMETRIA ESPACIAL: CONCEITOS, PARALELEPÍPEDO E CUBO

EXERCÍCIOS DE d) se r e s forem paralelas, então elas definem um plano perpendicular

COMBATE a α e β.
e) o plano definido por r e t é necessariamente paralelo a s.

08. (ESPCEX) Considere as seguintes afirmações:


01. A reta r é a intersecção dos planos α e β, perpendiculares entre si.
I. Se uma reta r é perpendicular a um plano α, então todas as retas
A reta s, contida em α, intercepta r no ponto P. A reta t, perpendicular
de α são perpendiculares ou ortogonais a r;
a β, intercepta-o no ponto Q, não pertencente a r. Nessas condições,
é verdade que as retas II. Se a medida da projeção ortogonal de um segmento AB sobre um
plano α é a metade da medida do segmento AB, então a reta AB
a) r e s são perpendiculares entre si.
faz com α um ângulo de 60°;
b) s e t são paralelas entre si.
III. Dados dois planos paralelos α e β, se um terceiro plano γ intercepta
c) r e t são concorrentes. α e β, as interseções entre esses planos serão retas reversas;
d) s e t são reversas. IV. Se α e β são dois planos secantes, todas as retas de α também
e) r e t são ortogonais. interceptam β.
Estão corretas as afirmações
02. (AFA) Se a soma das arestas de um cubo é igual a 72, então o a) apenas I e II c) I, II e III e) II, III e IV
volume do cubo será igual a:
b) apenas II e III d) I, II e IV
a) 40 b) 100 c) 144 d) 216
09. (AFA) Considere as proposições a seguir:
03. (EFOMM) Duas caixas cúbicas e retangulares perfeitas, têm seis
faces de quadrados perfeitos. As faces da primeira caixa tem 3 m² de
D O I. PSedelesRdoisé paralela
planos são paralelos, então toda reta que é paralela a um
L
área, e cada face da segunda caixa tem 9 m² de área. A razão entre o
A O ou está contida no outro.
II. Se uma retaFé paralela a um plano, então é paralela a todas as
volume da primeira caixa e o volume da segunda é:
a) 31 2 c) 3− 3 2
I
e)R 3−2 3 retas do plano.E
III. Se uma reta possui dois pontos distintos num plano, então ela
−1 2

S
TE

32
b) 3 d) 3 está contida no plano.

SO
IV. Se dois planos são secantes, toda reta de um, sempre intercepta
04. (EN) A altura de um paralelepípedo retângulo mede 60 cm e sua
MA

o outro plano.
base é um quadrado. A diagonal do paralelepípedo forma um ângulo
Pode-se afirmar que as proposições verdadeiras são

R
de 60° com o plano da base. O volume do paralelepípedo retângulo
é em cm³ a) I e IV b) II e III c) I e III d) II e IV
a) 12000 b) 18000 c) 24000 d) 36000
10. (ESPCEX) Considere as seguintes proposições:
05. (ESPCEX) Uma piscina em forma de paralelepípedo retângulo tem I. Toda reta paralela a um plano é paralela a qualquer reta desse plano.
largura de 6 metros, diagonal do fundo com 10 metros e diagonal da II. Uma reta e um ponto determinam sempre um único plano.
R
face que contém o comprimento igual a 4 5 metros. Para enchê-
III. Se uma reta é perpendicular a duas retas concorrentes de um
MA

la com água será utilizado um caminhão tanque com capacidade de


plano, então ela é perpendicular a esse plano.
SO

6000 litros. O número de cargas completas, desse mesmo caminhão,


necessárias para que a piscina fique completamente cheia é: Pode-se afirmar que:
a) 24 b) 28 c) 32 d) 54 e) 80 a) Só I é verdadeira. d) Só III é falsa.
T

b) Só III é verdadeira. e) Só I e III são falsas.


R
E

06. (ESPCEX) Considere as afirmações abaixo


IA F c) Só I e III são verdadeiras.
I. Se um plano encontra outros dois planos paralelos, então as
L O
II.
intersecções são retas paralelas.
Uma reta perpendicular a uma reta de um plano e ortogonalD PR
a O RESOLUÇÃO EM VÍDEO
outra reta desse plano é perpendicular ao plano. Abra o ProApp, leia o QR Code, assista à resolução
de cada exercício e AVANCE NOS ESTUDOS!
III. Se a intersecção de uma reta r com um plano é o ponto P, reta essa
não perpendicular ao plano, então existe uma única reta s contida
nesse plano que é perpendicular à reta r passando por P.
Pode-se afirmar GABARITO
a) que todas são verdadeiras. EXERCÍCIOS DE FIXAÇÃO
b) apenas I e II são verdadeiras. 01. B 04. E 07. C 10. C
c) apenas I e III são verdadeiras. 02. B 05. A 08. A
d) apenas II e III são verdadeiras. 03. C 06. A 09. B
e) todas são falsas. EXERCÍCIOS DE TREINAMENTO
01. C 04. C 07. E 10. C
07. (ESPCEX) Considere dois planos α e β perpendiculares e três retas
02. D 05. B 08. D
distintas r, s e t tais que r ⊂ α, s ⊂ β e t = α ∩ β. Sobre essas retas e os
planos é correto afirmar que 03. B 06. B 09. D
a) as retas r e s somente definirão um plano se forem concorrentes EXERCÍCIOS DE COMBATE
com t em um único ponto. 01. E 04. D 07. B 10. B
b) as retas r e s podem definir um plano paralelo à reta t. 02. D 05. C 08. A
c) as retas r e s são necessariamente concorrentes. 03. C 06. C 09. C

PROMILITARES.COM.BR 241

PM_BOOK05_MAT.indb 241 01/11/2021 13:50:31


GEOMETRIA ESPACIAL: CONCEITOS, PARALELEPÍPEDO E CUBO

ANOTAÇÕES

L DO PRO
I A FE
R

S
TE

SO
MA

R
R
MA

SO
T

R
E

IA F
L DO PRO

242 PROMILITARES.COM.BR

PM_BOOK05_MAT.indb 242 01/11/2021 13:50:31


GEOMETRIA ESPACIAL: PRISMAS,
PIRÂMIDES E CILINDRO

PRISMAS CLASSIFICAÇÃO
Dados um polígono ABC…MN situado num plano α e outro Podemos classificar um prisma de acordo com o número de lados
polígono A’B’C’...M’N’ congruente ao primeiro e situado num plano das duas bases.
paralelo β (β ≠ α), chama-se prisma o sólido formado pela reunião • Prisma triangular: bases: triângulos
de todos os segmentos de reta com uma extremidade num ponto de
• Prisma quadrangular: bases: quadriláteros
ABC…MN ou em sua região interna e outra num ponto de A’B’C’…
M’N’ ou em sua região interna. • Prisma pentagonal: bases: pentágonos
• Prisma hexagonal: bases: hexágonos
Se as bases são polígonos regulares, o prisma é chamado regular.

L DO PRO
I A FE
R

S
TE

SO
MA

Exemplo: Prisma hexagonal

R
R
MA

SO

Um prisma é reto se as arestas laterais forem perpendiculares às


bases; caso contrário, o prisma é dito oblíquo.
T

R • Se as arestas laterais são perpendiculares aos planos das


E

IA F
bases, o prisma e reto. Exemplo:

L DO PRO
ELEMENTOS PRINCIPAIS
• Bases - formada por polígonos
• Arestas das bases - lados das bases
• Faces laterais - formadas por paralelogramos
• Altura - distância H entre os planos das bases
• Superfície lateral - conjunto de todas as faces laterais
• Se as arestas laterais são obliquas aos planos das bases, o
• Superfície total - união da superfície lateral com as duas bases
prisma e dito oblíquo.

PROMILITARES.COM.BR 243

PM_BOOK05_MAT.indb 243 01/11/2021 13:50:32


GEOMETRIA ESPACIAL: PRISMAS, PIRÂMIDES E CILINDRO

• O prisma será regular se for reto e sua base for um polígono PRISMA HEXAGONAL REGULAR
regular.

Altura do prisma e a distância entre os planos das bases.


2 3 3 2 3
• Área da base (SB) Área da base: SB = 6⋅ =
4 2
É a área de uma das bases do prisma. Área lateral: Sl = 6 ·  · H
• Área lateral (SL) 3² 3
É soma das áreas das faces laterais. Área total: ST= 6H + 2 ⋅ = 6H + 3² 3
2
• Área total (ST) 3 2 3
=
Volume: V ⋅H
É a soma das áreas de todas as faces do prisma. 2
ST = SL + 2SB
L DO PRO
• Volume (V)
A pela altura: PIRÂMIDE
I base F E sólido delimitado por faces planas. Sua base é
A pirâmide é um
R
O volume do prisma é dado pelo produto da área da
um polígono e suas faces laterais são triângulos.
V = SB · H

S
TE

Observe as figuras seguintes:

SO
MA

PRINCIPAIS PRISMAS

R
Em qualquer prisma reto as faces laterais serão retângulos e dessa
forma a área lateral será dada por todos esses retângulos. Chamando
a aresta da base de  e sendo a altura do prisma H teremos que a área
de cada retângulo da face lateral será  · H.
ELEMENTOS PRINCIPAIS
PRISMA TRIANGULAR REGULAR • Base: formada por polígono
R
• Vértice: ponto V
MA

SO

• Arestas da Base: lados do polígono da base


• Faces laterais: formada por triângulos
2 3 • Arestas laterais: lados dos triângulos das faces laterais, com
T

Área da base: SB =
R
4 exceção dos lados do polígono da base
E

Área lateral: Sl = 3 ⋅  ⋅ H
IA •
F
Altura: distância H do ponto V ao plano da base

Área total: ST= 3  H + 2 ⋅


 2
3
= 3 H +
 2
3 O
L D O •P Superfície
• Superfície lateral: conjunto de todas as faces
R total: união da superfície lateral com a base
4 2
2 3
Volume:
= V ⋅H
4

PRISMA QUADRANGULAR REGULAR

Área da base: SB = ² CLASSIFICAÇÃO


Área lateral: Sl = 4 ·  · H Podemos classificar uma pirâmide de acordo com o tipo de
Área total: ST = 4H + 2² polígono que constitui a sua base.
Volume: V = ² · H • Pirâmide triangular – base é um triângulo
• Pirâmide quadrangular – base é um quadrilátero
• Pirâmide pentagonal – base é um pentágono
• Pirâmide hexagonal – base é um hexágono

244 PROMILITARES.COM.BR

PM_BOOK05_MAT.indb 244 01/11/2021 13:50:33


GEOMETRIA ESPACIAL: PRISMAS, PIRÂMIDES E CILINDRO

Indicando por SB, SL e ST, respectivamente, as áreas da base, da


superfície lateral e da superfície total de uma pirâmide, temos:

S T = SB + SL

SB = (dependerá do polígono da base)


SL = soma das áreas das faces laterais
Lembrando que as principais bases serão a triangular, que teremos
2 3
SB = , a base quadrada onde SB = 2 e a base hexagonal que é
4
2 3 32 3
formada por 6 triângulos equiláteros, assim SB =
6⋅ = .
4 2

Se a base é um polígono regular, a pirâmide é chamada regular.


As arestas laterais são congruentes entre si e as faces laterais são VOLUME
triângulos isósceles congruentes entre si. O volume de uma pirâmide é a terça parte do volume de um
prisma de base e altura iguais às da pirâmide. Assim temos
DIMENSÕES LINEARES DA PIRÂMIDE
SB ⋅ H
Na figura a seguir, temos uma pirâmide regular, na qual vamos V=
destacar alguns segmentos importantes. A medida de cada um estará 3
sendo representada por uma letra.
L DO PRO
IA TETRAEDRO F EREGULAR
R Chama-se tetraedro regular o tetraedro que possui as seis arestas
congruentes entre si. Nesse caso, todas as faces são triângulos

S
TE

equiláteros. O tetraedro é uma pirâmide triangular.

SO
MA

R
• Aresta da base (b)
• Apótema da base (m)
• Raio da base (r) R
MA

• Altura (h)
SO

• Aresta lateral (a)


• Apótema da pirâmide (g) ou (M)
Para o cálculo da área total, da altura e do volume de um tetraedro
T

regular, utilizamos:
S

Chama-se apótema de uma pirâmide regular cada uma das alturas


R
E

de suas faces laterais, relativas às arestas da base.


E

IA
Os triângulos VOM, VOB e VMB são retângulos. Aplicando-se o =ST a=
2
F
3 H
a 6
= V
a3 2

L DO PRO
3 12
teorema de Pitágoras, obtemos algumas relações importantes entre as
dimensões lineares citadas anteriormente. Vejamos:
• No triângulo VOM: g2 = H2 + m2
RELAÇÕES MÉTRICAS NO OCTAEDRO
• No triângulo VOB: a2 = H2 + r2
2
REGULAR DE ARESTA A
No triângulo VMB: a2 = g2 +  
b

 2

ÁREAS
Superfície Lateral é a reunião das faces laterais. Já a Superfície
Total é a reunião das faces laterais com a base.

BCDE, ABFD e ACFE são quadrados de lado a.


Diagonal do octaedro: AF = BD = CE = a 2.
Centro do octaedro: Ponto O, interseção das três diagonais. O
ponto O equidista dos vértices, equidista das faces e equidista das
arestas do octaedro.

PROMILITARES.COM.BR 245

PM_BOOK05_MAT.indb 245 01/11/2021 13:50:34


GEOMETRIA ESPACIAL: PRISMAS, PIRÂMIDES E CILINDRO

CILINDRO CIRCULAR S=
T SL + 2SB
Dado um círculo de centro O e raio R situado num plano α, e 2 r (h + r )
ST =π
um segmento de reta PQ, não nulo, não paralelo e não contido em
α, chama-se cilindro circular ou cilindro a reunião dos segmentos
congruentes e paralelos a PQ, que tem uma extremidade no círculo e
VOLUME
situados num mesmo semiespaço dos determinados por α.
O volume de um cilindro e o produto da área da base pela medida
da altura.
V = πr²h

CILINDRO EQUILÁTERO
O cilindro que possui as seções meridianas quadradas é chamado
de cilindro equilátero. No cilindro equilátero a altura é igual ao
diâmetro da base.

ELEMENTOS, DENOMINAÇÃO E L DO PRO


CLASSIFICAÇÃO I A FE
• R
Os círculos congruentes situados em planos paralelos são as
bases do cilindro;

S
TE

h = 2r
• Geratriz g e todo segmento com uma extremidade

SO
MA

• em um ponto da circunferência de centro O e raio r e outra no Exercício Resolvido


ponto correspondente da circunferência de centro O’ e raio r;

R
• A altura h de um cilindro e a distância entre os planos das 01. Considere o cubo da figura e as linhas nele traçadas. É
bases; INCORRETO afirmar que:
• Se as geratrizes são obliquas aos planos das bases, o cilindro
e dito cilindro circular oblíquo (figura do exemplo), mas se
são perpendiculares aos planos das bases, temos um cilindro
circular reto ou de revolução. R
MA

ÁREAS
SO

ÁREA DA BASE - SB
T

E a área do círculo da base do cilindro.


R
E

SB = πr²
IA F
L DO PR O
ÁREA LATERAL - S L
Superfície lateral e a reunião das geratrizes. A área dessa superfície a) triângulo GAH é retângulo.
e chamada área lateral (SL) é dada por: b) as medidas das áreas dos triângulos GAH e CAH são iguais.
SL = 2πrh c) os ângulos GÂH e CÂH são coplanares.
2
d) seno do ângulo BÂC é .
2
2
e) a tangente do ângulo GÂH é .
2
Resolução: C
ˆ = 90º
a) Verdadeira: AGH
b) Verdadeira: ∆GAH = ∆CAH
c) Falsa: plano (G,A,H) ≠ plano (C,A,H)

ˆ= BC
d) Verdadeira: sen(BAC) = =
a 2
AC a 2 2
ÁREA TOTAL - ST
ˆ= GH
e) Verdadeira: tg(GAH) = =
a 2
Superfície total e a reunião da superfície lateral com os círculos das GA a 2 2
bases. A área dessa superfície e denominada área total e é dada por:

246 PROMILITARES.COM.BR

PM_BOOK05_MAT.indb 246 01/11/2021 13:50:35


GEOMETRIA ESPACIAL: PRISMAS, PIRÂMIDES E CILINDRO

Exercício Resolvido Resolução: C


02. O triângulo ACF tem vértices coincidindo com três dos vértices A área da base da pirâmide é a área de seis triângulos equiláteros
de um cubo de aresta a, como mostra a figura abaixo. com lado igual à aresta do hexágono. Calculando essa área e o
volume, temos:
 Abase ⋅ h
V(pirâmide) = 3


Abase =
L 3 
6 ⋅ 
2

 =
(
3 ⋅ 
)
 2 3 2 3
=  (12) ⋅ ( 3) 
3⋅  =
18 3

  
 4   2   2 
  
Abase ⋅ h 18 3 ⋅ 8
⇒ V(pirâmide) = = =
48 3
3 3

A área de ACF em função de a.


a2 3 Exercício Resolvido
a)
2 05. Considere um paralelepípedo retângulo, cujas arestas têm
a2 3 comprimento 6 cm, 8 cm e 10 cm, e um triângulo cujos vértices são
b) os centros (intersecção das diagonais) de três faces de dimensões
3
distintas, como ilustra a figura a seguir.

DO PRO
a2 3
c)
4
A L
d)
a2
2
I FE
2
R

S
TE

Resolução: A

SO
(a 2 )
2
3 a2 3
MA

AC =CF =AF =a 2 ⇒ SACF = =


4 2

R
O perímetro P desse triângulo é tal que
a) P < 14 cm.
Exercício Resolvido b) 14 cm < P < 16 cm.
c) 16 cm < P < 18 cm.
03. Um prisma de altura H e uma pirâmide têm bases com a
mesma área. Se o volume do prisma é a metade do volume da d) P > 18 cm. R
pirâmide, a altura da pirâmide é:
MA

SO

a) H/6 Resolução: C
b) H/3 Considere a figura.
c) 2H
T

d) 3H
R
E

e) 6H
IA F
Resolução: E L DO PRO
Escrevendo a fórmula dos volumes e considerando H’ a altura da
pirâmide, temos:
Abase ⋅ H'
V(pirâmide) 3 A ⋅ H' H'
V(prisma) = ⇒ Abase ⋅ H = ⇒ 2 ⋅ Abase ⋅ H = base ⇒ 2H = ⇒ H' = 6 ⋅ H
Abase ⋅ H' 2 2 3 3
ide) 3 Abase ⋅ H' H' Pelo Teorema de Pitágoras, segue que AB = 5cm, BC = 34 cm
⇒ Abase ⋅ H = ⇒ 2 ⋅ Abase ⋅ H = ⇒ 2H = ⇒ H' = 6 ⋅ H
2 3 3 AC = 41cm.
e Logo, como 5 < 34 < 6 e 6 < 41 < 7, temos
16 cm < P < 18 cm.

Exercício Resolvido
Exercício Resolvido
04. Uma pirâmide regular de base hexagonal é tal que a altura
06. Uma piscina em forma de um paralelepípedo retângulo de
mede 8 cm e a aresta da base mede 2 3 cm. O volume dessa
10,0 m × 15,0 m e fundo horizontal está com água até a altura de
pirâmide, em centímetros cúbicos, é:
1,5 m. Um produto químico em pó deve ser misturado à água à
razão de um pacote para cada 4500 litros. O número de pacotes
a) 24 3 d) 72 3 a serem usados é:
b) 36 3 e) 144 3 a) 25 c) 100 e) 500
c) 48 3 b) 50 d) 250

PROMILITARES.COM.BR 247

PM_BOOK05_MAT.indb 247 01/11/2021 13:50:36


GEOMETRIA ESPACIAL: PRISMAS, PIRÂMIDES E CILINDRO

03. (EEAR) Um prisma reto é regular quando suas bases


Resolução: B
a) são paralelas.
b) têm a mesma área.
c) têm arestas congruentes.
d) são polígonos regulares.

04. (EEAR) Um pedaço de queijo, em forma de prisma triangular


regular, tem 6 cm de altura e possui como base um triângulo de 10
Vpiscina = a ⋅ b ⋅ c → Vpiscina = 10 ⋅ 15 ⋅ 1,5 → cm de lado. O volume desse pedaço de queijo é ____ 3 cm3.
Vpiscina = 225 m³ → Vpiscina = 225000 litros. a) 150 c) 185
4500 litros → 1pacot e b) 165 d) 200
225000 → x pacot es
= 225000=
4500.x → x 50 pacot es 05. (EEAR) A base de um prisma regular é um hexágono inscrito num
círculo de raio R. Se o prisma é equivalente ao cubo, cuja base está ins-
crita no mesmo círculo, então a altura do prisma hexagonal, em cm, é
a) 2R 2R 6 4R 6 4R 6
Exercício Resolvido b) c) d)
3 3 9
07. Um prisma reto de base hexagonal regular tem a mesma altura
de um prisma cuja base é um triângulo equilátero. Considere h a 06. (EEAR) Um prisma regular de base triangular tem altura igual ao

DO PRO
medida da aresta da base do prisma hexagonal e t a medida da lado da base e volume igual a 16 3 cm³. A área lateral desse prisma,
aresta da base do prisma triangular. Se ambos os prismas têm o

A L em cm², é
mesmo volume, então a razão
h
t
vale
I
a) 24.
FE c) 4.

a)
1
6
. R b) 8. d) 48.

S
TE

1 07. (EEAR) Um vaso tem formato de um cilindro reto, de 16 cm de


b) .

SO
1
6 altura interna e 6 cm de diâmetro interno. Ele contém água até de
MA

c) 1. 3
sua altura. Acrescentando-se uma quantidade de água equivalente ao
d) 6.

R
volume de uma esfera de 6 cm de diâmetro, o nível da água subirá
e) 6. a) 3 cm. c) 5 cm.
b) 4 cm. d) 6 cm.
Resolução: A
Calculando: 08. (EEAR) Num cilindro circular reto, o diâmetro da base mede 8 cm
H = altura dos prismas e a geratriz, 10 cm. A área lateral desse cilindro, em cm2, é
R
h2 ⋅ 3
MA

a) 160π. c) 80.
Vh = 6⋅ ⋅H
SO

4 b) 80π. d) 40.
t2 ⋅ 3
=Vt ⋅H
4 09. (EEAR) Considere:
T

h2 ⋅ 3 t2 ⋅ 3 I. No prisma reto,
R
h 1
Vh = Vt ⇒ 6 ⋅ ⋅H = ⋅ H ⇒ 6h2 = t2 ⇒ =
E

4 4 t
IA 6 II.
F
No prisma oblíquo,

L D OA. asParestas O
III. No prisma regular,
R laterais não são perpendiculares ao plano da base.
EXERCÍCIOS DE

FIXAÇÃO
B. as bases são polígonos regulares.
C. as faces laterais são quadriláteros cujos ângulos são retos.
D. as arestas laterais são perpendiculares ao plano da base.
E. as faces laterais são losangos ou paralelogramos propriamente
01. (EEAR) Se uma pirâmide tem 9 faces, então essa pirâmide é ditos.
a) eneagonal. F. as bases são polígonos regulares ou não.
b) octogonal.
O número de afirmações corretas que se pode fazer, iniciando-se com
c) heptagonal. I, II ou III e completando-se com A, B, C, D, E, ou F, é
d) hexagonal. a) 9. c) 7.
b) 8. d) 6.
02. (EEAR) A medida da altura de um prisma triangular regular é igual
à medida da aresta de sua base. Se a área lateral desse prisma é 10 m2,
10. (EEAR) Um prisma quadrangular regular está circunscrito a um
então sua altura mede, em m,
cilindro equilátero. Se a aresta da base do prisma mede 4 cm, então o
15 15 volume do cilindro, em cm³, é
a) c)
2
a) 16π. c) 8π.
b) 30 30 b) 12π. d) 4π.
d)
3

248 PROMILITARES.COM.BR

PM_BOOK05_MAT.indb 248 01/11/2021 13:50:37


GEOMETRIA ESPACIAL: PRISMAS, PIRÂMIDES E CILINDRO

EXERCÍCIOS DE

TREINAMENTO
01. (EEAR) Se a aresta da base de um tetraedro regular mede 3 cm,
então sua altura, em cm, é
a) 3 c) 2 6
b) 2 3 d) 6
Considerando π = 3, o valor que mais se aproxima da altura atingida
02. (EEAR) Uma pirâmide regular de base hexagonal tem 20 cm de pela água no segundo cilindro é
altura e 10 cm de aresta da base. O apótema dessa pirâmide mede,
em cm, a) 14 cm c) 20 cm
b) 6 cm d) 24 cm
a) 5 3 c) 5 19
b) 5 17 d) 5 23
EXERCÍCIOS DE

COMBATE
03. (EEAR) O perímetro da base de um tetraedro regular é 9 m. A
medida da altura desse tetraedro, em m, é

DO PRO
6
a) c) 3 6
2
L
IA F E tetraedro, em cm, é
3 6 6 01. (EEAR) Se o apótema de um tetraedro regular mede 5 3 cm,
b) d)
então, a altura desse
2
R a) 5 3

S
TE

04. (EEAR) O raio da base de um cilindro equilátero e a aresta de um


cubo são congruentes. A razão entre as áreas totais do cilindro e do b) 10 2

SO
cubo é 10 6
MA

c)
a) 2. c) π. 3

R
b) 4. d) 2π. 10 3
d)
3
05. (EEAR) Um cilindro de cobre tem volume V, raio da base R = 50 cm
e altura H = 40 cm. Este cilindro será derretido para fazer cilindros de e) 6
volume v, raio r = R/5 e altura h = H/4. Dessa forma, V/v =
02. (EEAR) Um barril, cuja forma é a de um cilindro reto, está repleto
a) 50. c) 150. R
de vinho. Este vinho deve ser distribuído em copos cilíndricos de altura
MA

b) 100. d) 200. igual a 1/8 da altura do barril, e de diâmetro da base igual a 1/5 do
SO

diâmetro da base do barril. A quantidade de copos necessária para


06. (EEAR) Um retângulo, de lados 2 m e 5 m, gira 360º em torno de distribuir todo o vinho é
seu maior lado. A área lateral do sólido obtido, em m², é a) 400 b) 300 c) 200 d) 100
T

a) 10. c) 10π.
R
E

b) 20. d) 20π.
IA F
03. (EEAR) A figura abaixo é a planificação de um poliedro convexo
( A ≡ B ≡ C ≡ D ; E ≡ F ) . O volume desse poliedro, em unidades de
L DO PRO
07. (EEAR) O perímetro da base de uma pirâmide quadrangular regular
é 80 cm. Se a altura dessa pirâmide é 15 cm, seu volume, em cm³, é
volume, é

a) 2300. c) 1200.
b) 2000. d) 1000.

08. (EEAR) A diagonal da secção meridiana de um cilindro equilátero


mede 10 2 cm. A área lateral desse cilindro, em cm², é
a) 250π. c) 100π.
b) 200π. d) 50π.

2
09. (EEAR) Um cilindro circular reto, de altura igual a do raio da base
3
e de 12π cm² de área lateral, possui raio da base igual a _____ cm.
a) 5 c) 3
b) 4 d) 2
425 850
10. (EEAR) Um cilindro de 18 cm de altura e raio da base igual a 5 cm a) c)
2 3
contém água até a metade de sua altura. Por algum motivo, houve
necessidade de despejar essa água em outro cilindro com 40 cm de 425 850
b) d)
altura, cujo raio da base mede 4 cm. 3 2

PROMILITARES.COM.BR 249

PM_BOOK05_MAT.indb 249 01/11/2021 13:50:38


GEOMETRIA ESPACIAL: PRISMAS, PIRÂMIDES E CILINDRO

04. (ESA) A pirâmide de Quéops, em Gizé, no Egito, tem


aproximadamente 90 2 metros de altura, possui uma base quadrada
e suas faces laterais são triângulos equiláteros. Nessas condições,
RESOLUÇÃO EM VÍDEO
pode-se afirmar que, em metros, cada uma de suas arestas mede: Abra o ProApp, leia o QR Code, assista à resolução
de cada exercício e AVANCE NOS ESTUDOS!
a) 90 c) 160 e) 200
b) 120 d) 180
GABARITO
05. (ESA) Em uma pirâmide reta de base quadrada, de 4 m de altura,
EXERCÍCIOS DE FIXAÇÃO
uma aresta da base mede 6 m. A área total dessa pirâmide, em m² é
01. B 04. A 07. B 10. A
a) 144 d) 72
02. D 05. D 08. B
b) 84 e) 96
03. D 06. D 09. A
c) 48
EXERCÍCIOS DE TREINAMENTO
06. Sejam duas pirâmides quadrangulares regulares de bases 01. D 04. C 07. B 10. A
congruentes, cujas alturas são 4 cm e 3 cm, e cujo apótema da base 02. C 05. B 08. C
mede 4 cm. Unindo-se essas pirâmides pelas bases, de forma que suas
03. D 06. D 09. C
arestas coincidam, obtém-se um octaedro cuja área total, em cm², é
igual a EXERCÍCIOS DE COMBATE

a) (
8 5+ 2 ) 01. C
02. C
04. D
05. E
07. E
08. C
10. A

b) 8 (5 + 4 2 )
L D O 03.PC R O 06. D 09. C
16 (5 + 2 2 )
c)
16 (5 + 4 2 ) I A ANOTAÇÕES FE
d)
R

S
TE

07. (ESPCEX) Aumentando-se em 10% as arestas da base e a altura

SO
de uma pirâmide regular, seu volume será aumentado de:
MA

a) 10 % d) 30%
b) 20% e) 33,1%

R
c) 21%

08. (ESPCEX) A área da base de uma pirâmide quadrangular regular


é 36 m². Se a altura da pirâmide mede 4 m, sua área total, em m², é
igual a:
a) 48 c) 96 e) 144
R
MA

b) 54 d) 120
SO

09. (ESPCEX) Uma pirâmide quadrangular regular tem a por aresta


da base e 2a por aresta lateral. A altura e o volume dessa pirâmide
T

medem, respectivamente:
R
E

a)
a 15 a3 15
e IA F
L DO PRO
2 3

a 3 a3 3
b) e
2 6

a 14 a3 14
c) e
2 6

a 12 a3 12
d) e
2 3

a 10 a3 10
e) e
2 3

10. (AFA) A área total da pirâmide regular de apótema A2, onde A1 e


2p são, respectivamente, apótema e perímetro de sua base, é
a) p(A1 + A2)
p
b) (A1 + A2)
2
c) 2p(A1 + A2)
 A 
d) p  A1 + 2 
 2 

250 PROMILITARES.COM.BR

PM_BOOK05_MAT.indb 250 01/11/2021 13:50:39


GEOMETRIA ESPACIAL: CONE, ESFERA,
INSCRIÇÃO E CIRCUNSCRIÇÃO DE SÓLIDOS

CONE

L DO PRO
I A FE
R

S
TE

• Secção transversal de um cone é qualquer interseção não

SO
vazia do cone com um plano paralelo à base (desde que este
MA

não passe pelo vértice); trata-se de um círculo.

R
CLASSIFICAÇÃO
CONCEITO Um cone pode ser classificado conforme a inclinação da reta VO
Consideremos um círculo de centro O e raio r, situado num plano sendo ‘O’ o centro da base, em relação ao plano da base:
α, e um ponto V fora de α. Chama-se cone circular, ou cone, a reunião
dos segmentos com uma extremidade em V e a outra em um ponto
do círculo. R
MA

ELEMENTOS
SO

Considerando o cone representado a seguir, temos:


T

R
E

IA F
L DO PRO

• O cone circular é obliquo quando a reta VO é oblíqua à base;


• O cone circular é reto quando a reta VO é perpendicular à
base

• O ponto V é o vértice do cone


• O círculo de raio r é a base do cone
• Os segmentos com uma extremidade em V e a outra nos
pontos da circunferência da base são as geratrizes do cone
• A distância do vértice ao plano da base é a altura do cone

PROMILITARES.COM.BR 251

PM_BOOK05_MAT.indb 251 01/11/2021 13:50:40


GEOMETRIA ESPACIAL: CONE, ESFERA, INSCRIÇÃO E CIRCUNSCRIÇÃO DE SÓLIDOS

ÁREAS ÁREA TOTAL - ST


Superfície total de um cone é a reunião da superfície lateral com o
ÁREA DA BASE - SB círculo da base. A área dessa superfície é chamada área total.
A área da base de um cone é a área de um círculo de raio r. S=
T SL + SB

ST = πrg + πr²

ST =πr(g + r)

VOLUME
O volume de um cone vale um terço do produto da área da base
pela altura:
1 2
V= πr h
3

SEÇÃO MERIDIANA E CONE EQUILÁTERO


Seção meridiana de um cone é a intersecção dele com um plano

L D O que
Pcontém
R Oo eixo. A seção meridiana de um cone reto é um triângulo
equilátero.

IA FE
R

S
TE

SO
MA

SB = πr²

R
ÁREA LATERAL - SL
A planificação da superfície lateral (ou a reunião das geratrizes)
de um cone nos dá um setor circular com as seguintes características:
• Raio: g (geratriz do cone) R
MA

• Comprimento do arco: 2πr (perímetro da base)


SO

A área lateral do cone é dada por:


T

R
E

IA F
L DO PRO

Cone equilátero é um cone cuja seção meridiana é um triângulo


equilátero. Assim
g = 2r

ESFERA
Chama-se esfera a um sólido limitado por uma superfície em que
todos os pontos de um ponto interior chamado de centro.
A superfície que delimita a esfera, denomina-se: superfície
esféricae qualquer segmento OP que une o centro O com um ponto P
da superfície esférica denomina-se raio da esfera.

SL = πrg

252 PROMILITARES.COM.BR

PM_BOOK05_MAT.indb 252 01/11/2021 13:50:41


GEOMETRIA ESPACIAL: CONE, ESFERA, INSCRIÇÃO E CIRCUNSCRIÇÃO DE SÓLIDOS

VOLUME DO SEGMENTO ESFÉRICO


De modo análogo ao volume da esfera, podemos calcular o
volume do segmento esférico de altura h subtraindo um tronco de
cone de altura h, de um cilindro de raio R e também altura h.
ÁREA DA SUPERFÍCIE ESFÉRICA DE RAIO R
A área da superfície esférica é igual ao quádruplo da área do
círculo máximo da esfera que tem área igual a πR². Portanto a área
da superfície esférica é:
S = 4πR²

L DO PRO
VOLUME DA ESFERA I A FE
R de 43
O Volume da esfera de raio R é igual ao produto do raio

S
TE

pela área do círculo de área máxima logo seu volume é dado pela

SO
fórmula: Volume do segmento esférico de altura h em uma esfera de raio R:
MA

4 3 πh2 (
V= πR V= 3R − h)
3 segmento
3

R
Área da calota esférica de altura h em uma esfera de raio R:
FUSO E CUNHA ESFÉRICA Vcalota = 2πRh
Denomina-se fuso esférico a superfície gerada pela rotação de
uma circunferência que gira um ângulo α menor que 2π radianos ao
redor do seu diâmetro. TRONCOS DE SÓLIDOS R
MA

Chama-se cunha esférica ao sólido gerado pela rotação, de um


SO

ângulo α, menor que 2π radianos, de um semicírculo, ao redor do SÓLIDOS HOMOGÊNEOS


seu diâmetro. Nosso estudo irá focar nos cortes de sólidos, cortes esses feitos
por planos paralelos as bases.
T

R Primeiramente vamos pensar o seguinte: por que vemos tantas


E

IA F
questões que envolvem apenas troncos de pirâmides ou cones?

L DO PRO
A resposta acaba sendo simples, esses tipos de sólidos são aqueles
que quando seccionados por um plano paralelo a base geram
maiores investigações. Sabemos, do dia a dia inclusive, que quando
temos sólidos homogêneos, cubos, paralelepípedos, prismas retos
ou cilindros retos, que quando queremos tomar uma porção de seu
volume basta tomarmos a mesma fração da altura que queremos de
volume. Veja abaixo.

A área do fuso esférico e o volume da cunha esférica são


proporcionais ao ângulo do fuso (α).
Área do fuso esférico de ângulo α radianos: Sfuso = 2αR2
2 3
Volume da cunha esférica de ângulo α radianos: Vcunha= αR
3

SEGMENTO E CALOTA ESFÉRICA


Quando seccionamos uma esfera por um plano e um outro plano
tangencia a esfera, o plano secante divide a esfera em dois segmentos
esféricos, e a superfície esférica em duas calotas esféricas.

PROMILITARES.COM.BR 253

PM_BOOK05_MAT.indb 253 01/11/2021 13:50:41


GEOMETRIA ESPACIAL: CONE, ESFERA, INSCRIÇÃO E CIRCUNSCRIÇÃO DE SÓLIDOS

Temos um cilindro circular reto e vamos fazer um corte utilizando ProBizu


um plano paralelo as bases exatamente na metade da sua altura.
Nos sólidos homogêneos os cortes em diagonal também geram
metade do volume.

H Cada sólido resultante tem metade do volume do sólido original.


Com isso geramos 2 novos cilindros retos, mas com alturas

L DO PRO
porém, pelo fato de ser um sólido homogêneo, qualquer corte por
2

IA F EasCONES
um plano paralelo as bases irá gerar o mesmo polígono das bases e
PIRÂMIDES
O estudo para E
de mesma área S.
R pirâmides retas e cones retos se dá da mesma
maneira quando seccionados por planos paralelos a suas bases.

S
TE

Utilizando a mesma ideia dos sólidos homogêneos fica fácil

SO
perceber que após um corte por um plano paralelo a base os sólidos
MA

resultantes não terão seus volumes ligados diretamente a proporção


de suas alturas. Dessa forma temos de recorrer a semelhança entre os

R
sólidos. Tomemos o cone reto abaixo.

R
MA

SO
T

R
E

Todos os círculos, de centros O1, O2 e O3 possuem as mesmas


IA
áreas πR², fazendo com que seus volumes dependam somente das F
L DO PRO
suas alturas. Assim a fração da altura que se efetua o corte define a
fração do volume. Assim no nosso exemplo, cada sólido resultante do
corte terá metade do volume do sólido inicial.
Não é à toa que se uma receita pede que seja utilizado metade
do volume de uma lata de leite condensado nós utilizamos o leite Vamos fazer uma secção por um plano paralelo a base de centro
condensado até a metade da lata. Se enchermos um cubo até um O do cone.
terço de sua altura teremos um terço do volume total e assim para
qualquer sólido homogêneo.

254 PROMILITARES.COM.BR

PM_BOOK05_MAT.indb 254 01/11/2021 13:50:42


GEOMETRIA ESPACIAL: CONE, ESFERA, INSCRIÇÃO E CIRCUNSCRIÇÃO DE SÓLIDOS

Vamos “ver” de frente os sólidos e ver os triângulos que temos.


Quando fazemos a semelhança de medidas lineares, medidas de
Vemos que o novo cone gerado, VCD de centro O’, também tem comprimento, apenas uma dimensão, dizemos que essa razão de
como base um círculo, porém de raio diferente do cone VAB de centro semelhança é k. Assim para os triângulos da figura teríamos
O. Dessa forma seu volume não fica dependendo somente de sua r h VD
k= = = =…
D O P R OVD g
altura h como também passa a depender desse seu novo raio da base
r, que é definido através de uma semelhança.
R H VA
L
IA F =EG
*Para o cone
VA
R Sabemos que quando fazemos razões entre áreas de figuras
planas nossa razão passa a ser k². Agora quando fizermos razões entre

S
TE

volumes teremos a razão k³.

SO
Dessa forma podemos fazer razões entre os volumes dos sólidos
MA

VCD e VAB.
VVCD

R
= k3
VVAB

Assim para encontrarmos k basta utilizarmos a razão que nos for


r h
mais convenientes seja ou ou qualquer outra razão linear.
R H
Depois de encontrado o volume do sólido “menor” para
R
encontrarmos o volume do tronco basta efetuar a subtração entre os
MA

volumes dos cones “maior” e “menor”.


SO

Exemplo:
Tronco é o sólido resultante da retirada do sólido superior ao plano Um cone reto de volume igual a 64 cm3 é seccionado na metade
T

de corte (sólido que é semelhante ao sólido original). Vale lembrar que


S

de sua altura por um plano paralelo a sua base. Calcular o volume do


R
os troncos eles não são sólidos semelhantes com os sólidos originais tronco resultante.
E

IA
ou os retirados, utilizá-los na semelhança é um erro muito comum.
F
L DO PRO

A figura acima ilustra um tronco de cone reto, resultado da


remoção do cone menor do cone maior.
Chegaremos ao seu volume utilizando semelhança entre os
sólidos encontrando os volumes do sólido maior e do menor e no final
fazendo a diferença entre seus volumes.
Ao utilizarmos a subtração teremos

PROMILITARES.COM.BR 255

PM_BOOK05_MAT.indb 255 01/11/2021 13:50:42


GEOMETRIA ESPACIAL: CONE, ESFERA, INSCRIÇÃO E CIRCUNSCRIÇÃO DE SÓLIDOS

II. Tronco de pirâmide

* A fórmula serve para troncos de pirâmides retas de qualquer base, a


figura utilizou a quadrangular só para exemplificar.
h r
k= =
H R Vtronco=
H
3
(
S + S⋅s + s )
H
S → Área da base maior
H
Onde é mais conveniente utilizarmos h =
2
e assim =k
L D O Ps →RÁreaOda base menor
2= 1 .
H 2

I A 1= r ⇒r = R FE
H H → Altura do tronco
r
H
r 1 r
* A semelhança também nos permite concluir que 2 = ⇒
2 = ⇒ = ⇒r = R H R R 2 R 2
INSCRIÇÃO E CIRCUNSCRIÇÃO DE

S
TE

H R 2 R 2
SÓLIDOS

SO
Assim fazendo a semelhança entre os volumes dos 2 cones
MA

Vmenor
=
V  1
k 3 ⇒ menor =
3
Vmenor 1 REVISÃO – PRINCIPAIS POLÍGONOS
  ⇒ =⇒ Vmenor =
8

R
Vmaior 64  2 64 8 REGULARES INSCRITOS
Assim o volume do cone “menor” é 8 cm3 e consequentemente o
volume do tronco gerado será Vtronco = 64 − 8 = 56 cm3.
Existem 2 fórmulas que calculam direto os volumes do tronco de
cone e de pirâmide quando são conhecidas as medidas das 2 bases e R
da altura do tronco.
MA

SO

I. Tronco de cone
T

R
E

IA F
L DO PRO
Figura 1

πH 2
=
Vtronco
3
(R + Rr + r2 )
R → Raio da base maior
r → Raio da base menor
H → Altura do tronco

Figura 2

256 PROMILITARES.COM.BR

PM_BOOK05_MAT.indb 256 01/11/2021 13:50:43


GEOMETRIA ESPACIAL: CONE, ESFERA, INSCRIÇÃO E CIRCUNSCRIÇÃO DE SÓLIDOS

PIRÂMIDE REGULAR INSCRITA EM UM


PRISMA REGULAR DE BASE DIFERENTE

Podemos perceber que ambos os sólidos, pirâmide e prisma,


Figura 3
possuem a mesma altura (H). Vamos chamar de Sprisma a área da base
do prisma e de Spir a área da base da pirâmide.
PIRÂMIDE REGULAR INSCRITA EM UM Sendo assim o volume do prisma será Vprisma = Sprisma ⋅ H enquanto
PRISMA REGULAR DE MESMA BASE
L DO PRO o volume da pirâmide será Vpir=
.
1
3
⋅ Spir ⋅ H

IA Fazendo a F
razão entre os volumes do prisma e da pirâmide
E V S ⋅H S
R teremos:
prisma
= prisma
=3 prisma

S
TE

Vpir 1 Spir
⋅ Spir ⋅ H
3

SO
MA

Ou seja, a razão entre os volumes será o triplo da razão entre as


áreas das bases.

R
Vamos mostrar alguns casos
I. Pirâmide triangular regular inscrita num prisma
hexagonal regular

R
MA

SO

Podemos perceber que ambos os sólidos, pirâmide e prisma,


possuem a mesma altura (H) e a mesma área da base. Vamos chamar
T

de Sprisma a área da base do prisma e de Spir a área da base da pirâmide.


R
E

IA
Sendo assim o volume do prisma será Vprisma = Sprisma ⋅ H enquanto o
F
volume da pirâmide será Vpir=
1
3
⋅ Spir ⋅ H .
Fazendo a razão entre os volumes do prisma e daLpirâmide R O
teremos:
DO P
Vprisma Sprisma.H S
= = 3 prisma
Vpir 1 Spir
.Spir .H
3 Chamando o lado do hexágono regular ABCDEF de I teremos
Como o prisma e a pirâmide possuem como bases os mesmos que o lado do triângulo equilátero ACE será l 3. Fazendo a área do
polígonos regulares, teremos que Sprisma = Spir e assim hexágono teremos
Vprisma Sprisma  l2 3  3l2 3
= 3= 3 = =
SABCDEF 6.  
Vpir Spir  4  2

A mesma razão é válida para um cone inscrito num cilindro. Teremos a área do triângulo equilátero
Vcilindro
Vcone
=3
=
 l 3 2 3
SACE =
2
 3l 3 ( )
4  4
 
 
Assim vemos que a área do hexágono regular é o dobro da área
do triângulo equilátero. Sendo o hexágono regular a base do prisma
regular e o triângulo equilátero a base da pirâmide regular teremos que
Vprisma Sprisma Vprisma S
=3 como Sprisma = 2Spir então =
3 ⋅ ABCDE ==
3.2 6
Vpir Spir Vpir SACE

PROMILITARES.COM.BR 257

PM_BOOK05_MAT.indb 257 01/11/2021 13:50:44


GEOMETRIA ESPACIAL: CONE, ESFERA, INSCRIÇÃO E CIRCUNSCRIÇÃO DE SÓLIDOS

II. Pirâmide quadrangular regular inscrita num prisma Assim a razão entre os volumes do prisma e da pirâmide será
triangular regular Vprisma Sprisma l2 3
Vpir
=3
Spir
= S=
como Sprisma ABC
4
2 e=
Spir S=
MNPQ (
3l2 7 − 4 3 )
l2 3
EntãoVprisma =
3⋅
SABC
3⋅ 2 4
= =
3

7+4 3
=
7 3 + 4.3 (
12 + 7
=
)
l2 3
Vpir SMNPQ 3l 7 − 4 3 4 7−4 3( 7+4 3 ) ( ) (
4 ( 49 − 16.3) 4 )
Vprisma
=
3⋅
SABC
3⋅ 2 4
= =
3

7+4 3
=
7 3 + 4.3 12 + 7 3
=
( )
Vpir SMNPQ 3l 7 − 4 3 4 7−4 3 (7+4 3 )
4 ( 49 − 16.3) (4 ) ( )

PRISMA REGULAR INSCRITO EM UM


CILINDRO RETO

L DO PRO
I A FE
R

S
TE

SO
A razão entre os volumes do cilindro reto e do prisma regular
MA

inscrito em sua base dar-se-á por


Vcilindro Scilindro ⋅ H Scilindro

R
= =
Vprisma Sprisma ⋅ H Sprisma

Teremos então que a razão entre os volumes será dada pela razão
entre as áreas das bases, que com a ajuda da revisão dos principais
polígonos regulares inscritos poderemos definir para os principais
prismas regulares retos. R
I. Prisma triangular regular inscrito no cilindro reto
MA

SO

Scilindro = πR2

A relação entre o lado do triângulo inscrito e o raio do círculo


Sendo o lado do triângulo equilátero ABC igual a l e o lado do
T

circunscrito é dado por (figura 1)


S

quadrado MNPQ igual a x montaremos as razões entre os triângulos


R
E

semelhantes AMN e ABC.


IA F l=R 3
AH BC
L D O Assim O 3 (R 3 ) =
2

P RS= l=
= 2 3 3 3R2
AO PQ prisma , logo
4 4 4
l 3
2 l x 3 l 3 x 3 l 3  3  l 3 Vcilindro Scilindro πR2 4π 4 3π
= ⇒ = −x⇒ +x= ⇒ x  + 1 = = = = =
l 3 x 2 2 2 2  2  2 Vprisma Sprisma 3 3R2 3 3 9
−x
2 4
l 3
x= 2 =
l 3

2− 3
=
l 3 2− 3 ( ) ( ) II. Prisma quadrangular regular inscrito no cilindro reto
3 +2 2+ 3 2− 3 4−3 ( ) Scilindro = πR2
2
A relação entre o lado do quadrado inscrito e o raio do círculo
(
x= 2l 3 − 3l= l 2 3 − 3 ) circunscrito é dado por (figura 2)
l=R 2
Assim a área do quadrado MNPQ será
(R 2 ) =
2

(l ( 2 )) ( ) Assim Sprisma= l2= 2R2 , logo


2
SMNPQ= 3 − 3 = l2 4 ⋅ 3 − 12 3 + 9
Vcilindro Scilindro πR2 π
( ) ((
SMNPQ =l2 21 − 12 3 =l2 3 7 − 4 3 )) =3l (7 − 4 3 )
2 = = =
Vprisma Sprisma 2R2 2

l2 3
Enquanto a área do triângulo equilátero será dada por SABC = III. Prisma triangular regular inscrito no cilindro reto
4
Scilindro = πR2

258 PROMILITARES.COM.BR

PM_BOOK05_MAT.indb 258 01/11/2021 13:50:45


GEOMETRIA ESPACIAL: CONE, ESFERA, INSCRIÇÃO E CIRCUNSCRIÇÃO DE SÓLIDOS

A relação entre o lado do hexágono inscrito e o raio do círculo Podemos perceber que ambos os sólidos, pirâmide e cone,
circunscrito é dado por (figura 3) possuem a mesma altura (H). Vamos chamar de Scone a área da base
l=R do cone e de Spir a área da base da pirâmide.
1
 l2 3   R2 3  3 3R2 Sendo assim o volume do cone será Vcone = ·Scone ·H enquanto o
=
Assim Sprisma 6=
  6 =  , logo 1 3
 4   4  2 volume da pirâmide será Vpir= 3 ⋅ Spir ⋅ H .
Vcilindro Scilindro πR2 2π 2 3π Fazendo a razão entre os volumes do cone e da pirâmide:
= = = =
Vprisma Sprisma 3 3R2 3 3 9
1
2 ⋅S ⋅H
Vcone 3 cone S
= = cone
Vpir 1 Spir
⋅ Spir ⋅ H
3
PIRÂMIDE REGULAR INSCRITA EM UM
CILINDRO RETO
A área da base do cone é dada por Scone = π·R e utilizando os
2

polígonos regulares das principais pirâmides regulares teremos os


mesmos resultados que os encontrados nos prismas regulares inscritos
nos cilindros retos.
I. Pirâmide triangular regular inscrita no cone reto
Vcone 4 3π
=
Vpir 9

L D O PII. RPirâmide
O quadrangular regular inscrita no cone reto
I A FE V
=
π
R
cone
V 2 pir

S
TE

III. Prisma triangular regular inscrita no cone reto

SO
De maneira análoga ao item anterior teremos Vcone 2 3π
MA

Vcilindro S ⋅H S =
= cilindro = 3 ⋅ cilindro Vpir 9
1

R
Vpir ⋅ Sprisma ⋅ H Sprisma
3

Então vemos que temos o triplo das razões anteriores, assim


CONE RETO INSCRITO
I. Pirâmide triangular regular inscrita no cilindro reto EM UMA PIRÂMIDE REGULAR
Vcilindro V 4 3π 4 3π
=
3 ⋅ cilindro =
3⋅ = R
Vpir Vprisma 9 3
MA

SO

II. Pirâmide quadrangular regular inscrita no cilindro reto


Vcilindro V π 3π
=3 ⋅ cilindro =3 ⋅ =
Vpir Vprisma 2 2
T

R
E

III. Pirâmide triangular regular inscrita no cilindro reto


IA F
L DO PRO
Vcilindro V 2 3π 2 3π
=
3 ⋅ cilindro =
3⋅ =
Vpir Vprisma 9 3

PIRÂMIDE REGULAR INSCRITA


EM UM CONE RETO
Podemos perceber que ambos os sólidos, pirâmide e cone,
possuem a mesma altura (H). Vamos chamar de Scone a área da base
do cone e de Spir a área da base da pirâmide.

1
Sendo assim o volume do cone será Vcone= 3 ⋅ Scone ⋅ H enquanto o
1
volume da pirâmide será Vpir= 3 ⋅ Spir ⋅ H .

Fazendo a razão entre os volumes da pirâmide e do cone teremos:

1
Vpir ·Spir ·H Spir
= = 3
Vcone 1·S ·H Scone
cone
3

PROMILITARES.COM.BR 259

PM_BOOK05_MAT.indb 259 01/11/2021 13:50:47


GEOMETRIA ESPACIAL: CONE, ESFERA, INSCRIÇÃO E CIRCUNSCRIÇÃO DE SÓLIDOS

a
A área da base do cone é dada por Scone = π·R enquanto, que
2
É fácil ver que h = , assim
para a área da pirâmide podemos utilizar a fórmula da área de um 2
polígono regular Spir = p·m , onde m é o apótema da base e p é o 2
2  a 2  a 2 a2 ⋅ 2 a a3
semiperímetro do polígono. Como neste caso o círculo da base do Voctaedro = ⋅  ⋅ = ⋅ ⋅ =
cone está inscrito no polígono da base da pirâmide teremos que o 3  2  2 3 4 2 6
apótema do polígono será igual ao próprio raio do círculo, assim
m = R. Assim 1
Isso quer dizer que o volume do octaedro regular é do volume
6
Vpir Spir p·m p·R p do cubo.
= = = =
Vcone Scone π·R2 π·R2 πR

O que nos faz concluir que a razão entre os volumes é a razão CONE RETO INSCRITO NA ESFERA
entre os perímetros das bases.

OCTAEDRO FORMADO PELOS CENTROS


DAS FACES DE UM CUBO

L DO PRO
I A FE
R

S
TE

SO
MA

R
Podemos fazer a assimilação do triângulo inscrito no círculo.
Vamos ligar 2 centros de faces laterais consecutivas do cubo.

R
MA

SO
T

R
E

IA F
L DO PRO

Agora temos o triângulo retângulo indicado na figura e dessa


forma podemos calcular a aresta do octaedro regular em função da
Utilizaremos a fórmula da área do triângulo em função do raio
aresta a do cubo. Vamos chamar a aresta do octaedro regular de x.
abc
2 2
a2 a2 a2 a2
do círculo circunscrito S∆ = , onde poderemos calcular a área do
a a a a 2 4R
x2 =   +   = + = ⇒x= = ⇒x=
 2  2 4 4 2 2 2 2 2r ⋅ H
triângulo como S∆ = 2 . No caso analisado os lados do triângulo
Dessa forma o volume do octaedro regular será o dobro do volume VAB serão g, g e 2r. Assim
de uma das pirâmides quadrangulares regulares que o formam.
2r ⋅ g ⋅ g 2r ⋅ H 2rg2 g2
1  S∆ = ⇒ = ⇒ 2RH = g2 ⇒ R =
= 2  ⋅ Squadrado ⋅ h 
Voctaedro 4R 2 4R 2H
3 

260 PROMILITARES.COM.BR

PM_BOOK05_MAT.indb 260 01/11/2021 13:50:47


GEOMETRIA ESPACIAL: CONE, ESFERA, INSCRIÇÃO E CIRCUNSCRIÇÃO DE SÓLIDOS

ProBizu ProBizu
No caso do cone equilátero podemos imaginar o caso do No caso do cilindro equilátero podemos imaginar o caso do
triângulo equilátero inscrito no círculo de raio R, onde quadrado inscrito no círculo de raio R, onde teremos:
2 2  2r 3  2r 3 H =2r ⇒ 4R2 =4r2 + ( 2r ) ⇒ 4R2 =
2
4r2 + 4r2 ⇒ 4R2 =
8r2
R= H=   ⇒ R=
3 3  2  3
=R2 2r=
2
⇒R r 2

CILINDRO RETO INSCRITO NA ESFERA ESFERA INSCRITA NO CONE RETO

L DO PRO
I A FE
R

S
TE

SO
MA

R
Neste caso podemos fazer a vista frontal e utilizarmos a ideia de
um retângulo inscrito num círculo de raio R. Podemos fazer a assimilação do triângulo circunscrito ao círculo.

R
MA

SO
T

R
E

IA F
L DO PRO

Assim podemos relacionar nossos elementos aplicando o Teorema


de Pitágoras no triângulo retângulo BCD.
Utilizaremos a fórmula da área do triângulo em função do raio
( 2R ) ( 2r )
2 2
= + H2 do círculo inscrito S∆= p ⋅ R , onde poderemos calcular a área do
=
4R 2
4r2 + H2 2r ⋅ H
triângulo como S∆ = . No caso analisado os lados do triângulo
H= 4R − 4r =
2 2
4 (R − r
2 2
) 2
VAB serão g, g e 2r. Assim
=H 2 (R 2 − r 2 ) ( g + g + 2r ) ⋅ R ⇒ 2r ⋅ H = rH
S∆ = (r + g) ⋅ R ⇒ R =
2 2 r+g

PROMILITARES.COM.BR 261

PM_BOOK05_MAT.indb 261 01/11/2021 13:50:48


GEOMETRIA ESPACIAL: CONE, ESFERA, INSCRIÇÃO E CIRCUNSCRIÇÃO DE SÓLIDOS

ProBizu a
O diâmetro da esfera é igual a aresta do cubo, assim 2R = a ⇒ R = .
2
No caso do cone equilátero podemos imaginar o caso do triângulo
equilátero circunscrito ao círculo de raio R, onde g = 2r e

=H
2r 3
=
= r 3 , assim R =
rH r. r 3
= =
( )
r2 3 r 3
ESFERA INSCRITA NO TETRAEDRO REGULAR
2 r+g r + 2r 3r 3 Seja uma esfera de centro O inscrita em um tetraedro regular
VABC como na figura.

ProBizu
A esfera só pode ser inscrita no cilindro equilátero.

L DO PRO
I A FE
H
No cilindro equilátero H = 2r , assim R= = r
R Ao traçarmos as distâncias do centro O da esfera as 4 faces VAB,
VAC, VBC e ABC do tetraedro regular teremos todas essas 4 distâncias

S
2
TE

iguais ao raio da esfera (r).

SO
MA

CUBO INSCRITO NA ESFERA

R
R
MA

SO
T

R
E

IA F
L DO PR O
A diagonal do cubo é o diâmetro da esfera, assim Ao ligarmos o centro O da esfera aos 4 vértices do tetraedro
a 3 formaremos 4 tetraedros OVAB, OVBC, OVCA e OABC todos de
2R= a 3 ⇒ R= .
2 alturas relativas as bases VAB, VBC, VCA e ABC iguais a r.

ESFERA INSCRITA NO CUBO

262 PROMILITARES.COM.BR

PM_BOOK05_MAT.indb 262 01/11/2021 13:50:49


GEOMETRIA ESPACIAL: CONE, ESFERA, INSCRIÇÃO E CIRCUNSCRIÇÃO DE SÓLIDOS

As bases VAB, VBC, VCA e ABC são todas de mesma área, pois Exercício Resolvido
são os mesmos triângulos equiláteros que formam o tetraedro regular
VABC. 02. Um torneiro mecânico construiu uma peça retirando, de um
S=VAB S=
VBC S=
VCA SABC cilindro metálico maciço, uma forma cônica, de acordo com a
figura 01 a seguir:
Assim podemos concluir que o volume do tetraedro VABC pode
Considere π ≅ 3.
ser decomposto no volume dos tetraedros OVAB, OVBC, OVCA e
OABC.
VVABC = VOABC + VOAB + VOVBC
1 1 1 1 1
⋅ SABC ⋅ H = ⋅ SABC ⋅ r + ⋅ SVAB ⋅ r + ⋅ SVAC ⋅ r + ⋅ SVBC ⋅ r =
3 3 3 3 3
r
(SABC + SVAB + SVAC + SVBC )
3

Como S=
VAB S=
VBC S=
VCA SABC teremos que
1 r H
⋅ SABC ⋅ H = ( SABC + SVAB + SVAC + SVBC ) ⇒ H = 4r ⇒ r =
3 3 4
H Qual é o volume aproximado da peça em milímetros cúbicos?
r=
4 a) 2,16 × 105
b) 7,2 × 104
D O c)P 2,8R×O
Perceba que se tivéssemos um ponto P qualquer interior ao
10
L
5
tetraedro teríamos as distâncias em relação as 4 faces: d1, d2, d3 e d4.
Assim teríamos
1 1 1 1 I
1 A d) 8,32 × 10
e) 3,14 × 105
FE
4

3 3 3 3 R
⋅ SABC ⋅ H = ⋅ SABC ⋅ d1 + ⋅ SVAB ⋅ d2 + ⋅ SVAC ⋅ d3 + ⋅ SVBC ⋅ d4
3

S
Resolução: A
TE

Procedendo de maneira análoga teremos que

SO
H = d1 + d2 + d3 + d4 1
O volume do cone retirado é dado por ⋅ π ⋅ 32 ⋅ 6 ≅ 54 cm3 , enquanto
MA

3
que o volume do cilindro é π ⋅ 3² ⋅ 10 ≅ 270 cm³. Portanto, o volume da
Exercício Resolvido

R
aproximado da peça é igual a 270 – 54 = 216 cm³ = 2,16 ⋅ 105 mm³.
01. Um artesão resolveu fabricar uma ampulheta de volume total
V constituída de uma semiesfera de raio 4 cm e de um cone reto,
Exercício Resolvido
com raio e altura 4 cm, comunicando-se pelo vértice do cone, de
acordo com a figura abaixo. 03. Um cone circular reto, cuja medida do raio da base é R, é
cortado por um plano paralelo a sua base, resultando dois sólidos
R
de volumes iguais. Um destes sólidos é um cone circular reto, cuja
MA

SO

medida do raio da base é r. A relação existente entre R e r é:


a) R³ = 3r³.
b) R² = 2r².
T

R
c) R³ = 2r³.
E

IA F
d) R² = 3r².

L D O Resolução: O
PR C
Sejam v e 2v, respectivamente, o volume do cone de raio r e o
volume do cone de raio R.
Portanto, como os cones são semelhantes, temos
3
Para seu funcionamento, o artesão depositará na ampulheta areia v r
que corresponda a 25% de V. Portanto o volume de areia, em =   ⇔ R3= 2r3.
2v  R 
cm3, é
a) 16π.
64 π Exercício Resolvido
b) .
3
04. Uma indústria de bebidas criou um brinde para seus clientes
c) 32π.
com a forma exata da garrafa de um de seus produtos, mas com
128π medidas reduzidas a 20% das originais. Se em cada garrafinha
d) .
3 brinde cabem 7 ml de bebida, podemos concluir que a capacidade
e) 64π. da garrafa original é de:
a) 875 ml
Resolução: A b) 938 ml
O resultado pedido é dado por c) 742 ml
 1 4π 3 1  1
0,25 ⋅  ⋅ ⋅ 4 + ⋅ π ⋅ 42 ⋅ 4 = ⋅ 64 π= 16π cm3 . d) 693 ml
2 3 3  4
e) 567 ml

PROMILITARES.COM.BR 263

PM_BOOK05_MAT.indb 263 01/11/2021 13:50:50


GEOMETRIA ESPACIAL: CONE, ESFERA, INSCRIÇÃO E CIRCUNSCRIÇÃO DE SÓLIDOS

02. (EEAR) A área lateral de um cone circular reto é 24π cm². Se o


Resolução: A raio da base desse cone mede 4 cm, então sua altura, em cm, mede
Seja C a capacidade da garrafa original, em mililitros. Como os
a) 5 2 . b) 5 3. c) 2 5. d) 3 5 .
sólidos são semelhantes, tem-se que
3
c  1  03. (EEAR) Considere as afirmações:
=  ⇔
= c 875mL.
7  0,2 
I. A esfera é um sólido gerado pela rotação de uma semicircunferência
em torno de seu diâmetro.
II. A esfera é um sólido gerado pela rotação de um semicírculo em
torno de seu diâmetro.
Exercício Resolvido
III. Nem toda secção plana de uma esfera é um círculo.
05. Uma esfera metálica de 3 cm de raio é colocada em um IV. Toda secção plana de uma esfera é um círculo.
congelador e, após algum tempo, acumula uma camada de gelo
São FALSAS as afirmações
de 3 cm de espessura, mantendo a forma esférica. Então, o volume
do gelo acumulado é a) I e IV. b) I e III. c) II e III. d) II e IV.
a) 198π cm³
04. (EEAR) A base de um cone circular reto está inscrita num triângulo
b) 215π cm³
equilátero de área 9 3 cm². Se as alturas do cone e do triângulo são
c) 252π cm³ congruentes, então o volume do cone, em cm³, é
d) 207π cm³
a) 3π 6 b) 3π 3 c) 6π 3 d) 6π 6
e) 225π cm³

L D O 05.P(EEAR)
R SeUmo cilindro
equilátero. O
equilátero é equivalente a um cone, também
Resolução: C

IA F cm,
raio da base do cone mede 3cm, o raio da base do

R
cilindro mede, em
E 12 3
6 3
a) 3 b) c) d) 6

S
TE

2 2

SO
06. (EEAR) Considere duas esferas: a primeira com 16π cm² de área, e
MA

a segunda com raio igual a 5/2 do raio da primeira. A área da segunda

R
esfera, em cm², é
a) 100π. b) 50π. c) 40π. d) 20π.

07. (EEAR) Uma esfera tem 9π cm2 de área. Para que a área passe a
100π cm2, o raio deve ter sua medida aumentada em
4 70 70 700 700
Volume da esfera maior: V= π ⋅ 63= 288π cm3 a) % b) %
R c) % d) %
3 9 3 9 3
MA

4
SO

Volume da esfera menor: v= π ⋅ 33= 36π cm3


3
08. (EEAR) Um escultor irá pintar completamente a superfície de
Volume da camada de gelo: V – v = 252π cm³
uma esfera de 6 m de diâmetro, utilizando uma tinta que, para essa
T

superfície, rende 3 m² por litro. Para essa tarefa, o escultor gastará, no


S

R mínimo, _____ litros de tinta. (Considere π ≅ 3)


E

b)F 24
EXERCÍCIOS DE IA a) 18
O
c) 36 d) 48

FIXAÇÃO L D O09. (EEAR)


P RUma esfera está inscrita num cilindro equilátero cuja área
lateral mede 16π cm². O volume da esfera inscrita é
32 256
a) 8π b) 16π c) π d) π
3 3
01. (EEAR) No tronco de cone reto, as bases são paralelas. Se o raio
da base maior mede 5 cm e a distância entre as duas bases, 4 3 cm,
10. (CFOE) A geratriz de um cone de revolução mede 6 cm e o ângulo
então o volume desse tronco de cone, em cm³, é
da geratriz com a altura do cone é de 30º. O volume desse cone, em
cm³, é
a) 9π c) 9π 3 e) 6π

b) 3π 3 d) 27π 3

EXERCÍCIOS DE

TREINAMENTO
124 π 3 c) 96π 3
a) 01. (CFOE) Qual a razão entre o volume e a altura de um tronco de
3 3
pirâmide cujas bases são quadrados de lados 2 m e 5 m?
b) 125π 3 d) 124 π 3 a) 9 b) 11 c) 13 d) 15

264 PROMILITARES.COM.BR

PM_BOOK05_MAT.indb 264 01/11/2021 13:50:51


GEOMETRIA ESPACIAL: CONE, ESFERA, INSCRIÇÃO E CIRCUNSCRIÇÃO DE SÓLIDOS

02. Um tanque cilíndrico com água tem raio da base R. Mergulha-se 02. (EEAR) Uma esfera tem 36π m³ de volume. A medida de sua
9 superfície, em m², é
nesse tanque uma esfera de aço e o nível da água sobe R . O raio
da esfera é 16 a) 72π b) 56π c) 48π d) 36π
3 9 3 R
a) R b) R c) R d) 03. (EEAR) Um reservatório, com volume igual a 144π m³, tem a
4 16 5 2
forma de uma semiesfera. Para aumentar seu volume em 342π m³, é
preciso aumentar o raio do reservatório em
03. (EEAR) A geratriz de um cone de revolução forma com o eixo do
cone um ângulo de 45º. A área lateral, em dm², desse cone, sabendo- a) 12m b) 9m c) 6m d) 3m
se que a área de sua secção meridiana é 18 dm², é
a) 18π 2 b) 9π 2 c) 18π d) 18π ( )
2 +1 04. Ao triplicarmos o raio e tomarmos a terça parte de uma esfera, ela
possuirá, em relação à esfera original, um volume

04. (EEAR) A razão entre os volumes de dois cones equiláteros de a) 2 vezes maior
alturas h e 2h é b) 3 vezes maior
a) 1/2 b) 1/4 c) 1/6 d) 1/8 c) 9 vezes maior
d) 12 vezes maior
05. (EEAR) Se um cilindro reto está circunscrito a uma esfera de raio e) 20 vezes maior
“R”, então a razão entre a área da superfície esférica e a área total
do cilindro é
05. (EEAR) Uma “casquinha de sorvete” tem a forma de um cone
1 2 4
a) 1 b) c) d) circular reto cujas medidas internas são 12 cm de altura e 5 cm de
2 3 5
DO PRO
diâmetro da base. O volume de sorvete que enche completamente
essa casquinha é _________ π cm³.
L
06. (ESA) Dobrando o raio da base de um cone e reduzindo sua altura
A
à metade, seu volume
I
a) 30
Fb)E 25 c) 20 d) 15
a) dobra.
b) quadruplica.
R 06. (EEAR) A superfície lateral de um cone, ao ser planificada, gera um
setor circular cujo raio mede 10 cm e cujo comprimento do arco mede

S
TE

c) não se altera. 10π cm. O raio da base do cone, em cm, mede

SO
d) reduz-se à metade do volume original. a) 5 b) 10 c) 5π d) 10π
MA

e) reduz-se a um quarto do volume original.


07. (EEAR) O setor circular da figura representa a superfície lateral

R
07. (ESA) O volume de um tronco de pirâmide de 4 dm de altura e deum cone circular reto. Considerando π = 3, a geratriz e o raio da
cujas áreas das bases são iguais a 36 dm² e 144 dm² vale base do cone medem, em cm, respectivamente,
a) 330 m² c) 360 dm³ e) 336 dm³
b) 330 cm² d) 720 dm³
R
08. (ESA) Um tanque subterrâneo tem a forma de um cone invertido.
MA

SO

Esse tanque está completamente cheio com 8 dm³ de água e 56 dm³


de petróleo. Petróleo e água não se misturam, ficando o petróleo na
parte superior do tanque e a água na parte inferior. Sabendo que o
a) 5 e 2 b) 5 e 3 c) 3e5 d) 4 e 5
tanque tem 12 m de profundidade, a altura da camada de petróleo é
T

a) 10 m. b) 9 m. c) 8 m.
R
d) 7 m. e) 6 m.
E

IA F
08. (EEAR) Uma esfera inscrita em um cubo de diagonal 2 3 m tem
09. (EEAR) Um chapéu de festa, feito de cartolina, tem a forma de um
cone de 1 dm de raio e 5 dm de geratriz. Para fazer 20 chapéus, são
O
o volume igual a
L D Oa) πPm³ R b) 2π m³ 4π 32π
c) m³ d) m³
necessários, no mínimo, ________ dm² de cartolina. 3 3 3 3
Considere π = 3,14.
a) 157 b) 225 c) 314 d) 426 09. (EEAR) Uma esfera de raio R = 3 cm foi cortada ao meio, gerando
duas semiesferas. A área da superfície de cada semiesfera é _____ π cm2.
10. (EEAR) Um cone tem 3 cm de altura 8π cm³ de volume. O raio da
base desse cone, em cm, é
a) 4 b) 6 c) 3 2 d) 2 2

EXERCÍCIOS DE

COMBATE
a) 20 b) 22 c) 25 d) 27
01. (ESA) Duas esferas de aço de raio 4 cm e 3 61 cm fundem-se
para formar uma esfera maior. Considerando que não houve perda de 10. (EEAR) Se um cone equilátero tem 50π cm2 de área lateral, então
material das esferas durante o processo de fundição, a medida do raio a soma das medidas de sua geratriz e do raio de sua base, em cm, é
da nova esfera é de: igual a
a) 5 cm b) 5,5 cm c) 4,5 cm d) 6 cm e) 7 cm a) 10. b) 15. c) 20. d) 25.

PROMILITARES.COM.BR 265

PM_BOOK05_MAT.indb 265 01/11/2021 13:50:52


GEOMETRIA ESPACIAL: CONE, ESFERA, INSCRIÇÃO E CIRCUNSCRIÇÃO DE SÓLIDOS

RESOLUÇÃO EM VÍDEO
Abra o ProApp, leia o QR Code, assista à resolução
de cada exercício e AVANCE NOS ESTUDOS!

GABARITO
EXERCÍCIOS DE FIXAÇÃO
01. A 04. B 07. D 10. C
02. C 05. B 08. C
03. B 06. A 09. C
EXERCÍCIOS DE TREINAMENTO
01. C 04. D 07. E 10. D
02. A 05. C 08. E
03. A 06. A 09. C
EXERCÍCIOS DE COMBATE
01. A 04. C 07. A 10. B
02. D 05. B 08. C
L DO PRO
03. D 06. A 09. D
I A FE
ANOTAÇÕES R

S
TE

SO
MA

R
R
MA

SO
T

R
E

IA F
L DO PRO

266 PROMILITARES.COM.BR

PM_BOOK05_MAT.indb 266 01/11/2021 13:50:53


GEOMETRIA ESPACIAL: CONE, ESFERA, INSCRIÇÃO E CIRCUNSCRIÇÃO DE SÓLIDOS

ANOTAÇÕES

L DO PRO
I A FE
R

S
TE

SO
MA

R
R
MA

SO
T

R
E

IA F
L DO PRO

PROMILITARES.COM.BR 267

PM_BOOK05_MAT.indb 267 01/11/2021 13:50:53


GEOMETRIA ESPACIAL: CONE, ESFERA, INSCRIÇÃO E CIRCUNSCRIÇÃO DE SÓLIDOS

ANOTAÇÕES

L DO PRO
I A FE
R

S
TE

SO
MA

R
R
MA

SO
T

R
E

IA F
L DO PRO

268 PROMILITARES.COM.BR

PM_BOOK05_MAT.indb 268 01/11/2021 13:50:53

Você também pode gostar